Case Files Pediatrics [6 ed.] 9781260474961, 1260474968, 9781260474954, 126047495X

328 158 19MB

English Pages [641] Year 2021

Report DMCA / Copyright

DOWNLOAD FILE

Polecaj historie

Case Files Pediatrics [6 ed.]
 9781260474961, 1260474968, 9781260474954, 126047495X

Table of contents :
Cover
Title Page
Copyright Page
Dedication
Contents
Reviewers
Contributors
Preface
Acknowledgments
Introduction
Listing of Cases
Section I How to Approach Clinical Problems
Part 1. Approach to the Patient
Part 2. Approach to Clinical Problem Solving
Part 3. Approach to Reading
Section II Clinical Cases
Sixty Case Scenarios
Section III Review Questions
Index

Citation preview

SIXTH EDITION

CASE FILES®

Pediatrics

Eugene C. Toy, MD Assistant Dean for Educational Programs Director of Doctoring Courses Professor and Vice Chair of Medical Education Department of Obstetrics and Gynecology McGovern Medical School at The University of Texas Health Science Center at Houston (UTHealth) Houston, Texas Mark D. Hormann, MD Professor of Pediatrics Vice Chair for Education and Training Division of Community and General Pediatrics Department of Pediatrics McGovern Medical School at The University of Texas Health Science Center at Houston (UTHealth) Houston, Texas Robert J. Yetman, MD Professor of Pediatrics Vice Chair for Clinical Operations Director, Division of Community and General Pediatrics Department of Pediatrics McGovern Medical School at The University of Texas Health Science Center at Houston (UTHealth) Houston, Texas Margaret C. McNeese, MD Ransom Lummis Family Professor of Pediatrics Vice Dean for Admissions and Student Affairs Division of Community and General Pediatrics Department of Pediatrics

McGovern Medical School at The University of Texas Health Science Center at Houston (UTHealth) Houston, Texas Sheela L. Lahoti, MD Professor of Pediatrics Associate Dean for Admissions and Student Affairs Division of Child Safety and Integrated Care Department of Pediatrics McGovern Medical School at The University of Texas Health Science Center at Houston (UTHealth) Houston, Texas Emma A. Omoruyi, MD, MPH Associate Professor of Pediatrics Division of Community and General Pediatrics Department of Pediatrics McGovern Medical School at The University of Texas Health Science Center at Houston (UTHealth) Houston, Texas Abby M. Geltemeyer, MD Assistant Professor of Pediatrics Division of Community and General Pediatrics Department of Pediatrics McGovern Medical School at The University of Texas Health Science Center at Houston (UTHealth) Houston, Texas

New York Chicago San Francisco Athens London Madrid Mexico City Milan New Delhi Singapore Sydney Toronto

00_Toy-Pediatrics_FM_p000i-xxiv.indd 1

23/07/21 6:34 PM

Copyright © 2022 by McGraw Hill. All rights reserved. Except as permitted under the United States Copyright Act of 1976, no part of this publication may be reproduced or distributed in any form or by any means, or stored in a database or retrieval system, without the prior written permission of the publisher. ISBN: 978-1-26-047496-1 MHID: 1-26-047496-8 The material in this eBook also appears in the print version of this title: ISBN: 978-1-26-047495-4, MHID: 1-26-047495-X. eBook conversion by codeMantra Version 1.0 All trademarks are trademarks of their respective owners. Rather than put a trademark symbol after every occurrence of a trademarked name, we use names in an editorial fashion only, and to the benefit of the trademark owner, with no intention of infringement of the trademark. Where such designations appear in this book, they have been printed with initial caps. McGraw-Hill Education eBooks are available at special quantity discounts to use as premiums and sales promotions or for use in corporate training programs. To contact a representative, please visit the Contact Us page at www.mhprofessional.com. Notice Medicine is an ever-changing science. As new research and clinical experience broaden our knowledge, changes in treatment and drug therapy are required. The authors and the publisher of this work have checked with sources believed to be reliable in their efforts to provide information that is complete and generally in accord with the standards accepted at the time of publication. However, in view of the possibility of human error or changes in medical sciences, neither the authors nor the publisher nor any other party who has been involved in the preparation or publication of this work warrants that the information contained herein is in every respect accurate or complete, and they disclaim all responsibility for any errors or omissions or for the results obtained from use of the information contained in this work. Readers are encouraged to confirm the information contained herein with other sources. For example and in particular, readers are advised to check the product information sheet included in the package of each drug they plan to administer to be certain that the information contained in this work is accurate and that changes have not been made in the recommended dose or in the contraindications for administration. This recommendation is of particular importance in connection with new or infrequently used drugs. TERMS OF USE This is a copyrighted work and McGraw-Hill Education and its licensors reserve all rights in and to the work. Use of this work is subject to these terms. Except as permitted under the Copyright Act of 1976 and the right to store and retrieve one copy of the work, you may not decompile, disassemble, reverse engineer, reproduce, modify, create derivative works based upon, transmit, distribute, disseminate, sell, publish or sublicense the work or any part of it without McGraw-Hill Education’s prior consent. You may use the work for your own noncommercial and personal use; any other use of the work is strictly prohibited. Your right to use the work may be terminated if you fail to comply with these terms. THE WORK IS PROVIDED “AS IS.” McGRAW-HILL EDUCATION AND ITS LICENSORS MAKE NO GUARANTEES OR WARRANTIES AS TO THE ACCURACY, ADEQUACY OR COMPLETENESS OF OR RESULTS TO BE OBTAINED FROM USING THE WORK, INCLUDING ANY INFORMATION THAT CAN BE ACCESSED THROUGH THE WORK VIA HYPERLINK OR OTHERWISE, AND EXPRESSLY DISCLAIM ANY WARRANTY, EXPRESS OR IMPLIED, INCLUDING BUT NOT LIMITED TO IMPLIED WARRANTIES OF MERCHANTABILITY OR FITNESS FOR A PARTICULAR PURPOSE. McGraw-Hill Education and its licensors do not warrant or guarantee that the functions contained in the work will meet your requirements or that its operation will be uninterrupted or error free. Neither McGraw-Hill Education nor its licensors shall be liable to you or anyone else for any inaccuracy, error or omission, regardless of cause, in the work or for any damages resulting therefrom. McGraw-Hill Education has no responsibility for the content of any information accessed through the work. Under no circumstances shall McGraw-Hill Education and/or its licensors be liable for any indirect, incidental, special, punitive, consequential or similar damages that result from the use of or inability to use the work, even if any of them has been advised of the possibility of such damages. This limitation of liability shall apply to any claim or cause whatsoever whether such claim or cause arises in contract, tort or otherwise.

DEDICATION

To pediatricians everywhere, who are well known as “the nicest doctors”, and especially to my pediatrician Dr. Patrick Robert Robbie at Kaiser Sunset in Los Angeles, who passed away in 2013. Dr. Robbie’s gentle manner, expertise, and sense of humor made such an impression on me as “the kindest and smartest person I had ever encountered”, made each patient feel special, and inspired me to pursue medicine. —Eugene C. Toy

00_Toy-Pediatrics_FM_p000i-xxiv.indd 3

23/07/21 6:34 PM

This page intentionally left blank

CONTENTS

Reviewers / vii Contributors / ix Preface / xv Acknowledgments / xvii Introduction / xix Listing of Cases / xxi Section I How to Approach Clinical Problems....................................................................1 Part 1. Approach to the Patient.................................................................................................. 3 Part 2. Approach to Clinical Problem Solving....................................................................... 9 Part 3. Approach to Reading.....................................................................................................11 Section II Clinical Cases.......................................................................................................15 Sixty Case Scenarios.....................................................................................................................17 Section III Review Questions ...................................................................................................................... 567 Index / 601

v

00_Toy-Pediatrics_FM_p000i-xxiv.indd 5

23/07/21 6:34 PM

This page intentionally left blank

REVIEWERS

Heba A. Ahmad, MS4 Medical Student McGovern Medical School at the University of Texas Houston Health Science Center at Houston (UTHealth) Houston, Texas Manuscript Reviewer Joy M. Davis, MS4 Medical Student McGovern Medical School at the University of Texas Houston Health Science Center at Houston (UTHealth) Houston, Texas Manuscript Reviewer Allison L. Toy, RN Waco, Texas Senior Medical Writer Principal Manuscript Reviewer

vii

00_Toy-Pediatrics_FM_p000i-xxiv.indd 7

23/07/21 6:34 PM

This page intentionally left blank

CONTRIBUTORS

Kristopher Ahn, MD Internal Medicine-Pediatrics Resident McGovern Medical School at the University of Texas Houston Health Science Center at Houston (UTHealth) Houston, Texas Patent Ductus Arteriosus Mohammad Alnoor, MD Pediatric Cardiology Fellow Doernbecher Children’s Hospital Portland, Oregon Bacterial Enteritis Craig Authenment, MD Pediatric Resident McGovern Medical School at the University of Texas Houston Health Science Center at Houston (UTHealth) Houston, Texas Acute Otitis Media Adolescent Substance Use Disorder Diabetic Ketoacidosis Andrea Carlo-Angleró, MD Pediatric Resident McGovern Medical School at the University of Texas Houston Health Science Center at Houston (UTHealth) Houston, Texas Atopic and Contact Dermatitis Attention Deficit Hyperactivity Disorder Immunodeficiency and Human Immunodeficiency Virus Cathy Chang, MD Pediatric Hospitalist Texas Children’s Hospital Houston, Texas Cystic Fibrosis Wan-Hsuan Chen, DO Pediatric Hospitalist Memorial Hermann Hospital Sugar Land Sugar Land, Texas Inflammatory Bowel Disease Subdural Hematoma Sudden Infant Death Syndrome

ix

00_Toy-Pediatrics_FM_p000i-xxiv.indd 9

23/07/21 6:34 PM

x

CONTRIBUTORS

R. William Chong, MD Internal Medicine-Pediatrics Resident McGovern Medical School at the University of Texas Houston Health Science Center at Houston (UTHealth) Houston, Texas Neonatal Hyperbilirubinemia Nursemaid’s Elbow (Subluxation of Radial Head) Slipped Femoral Capital Epiphysis Emily Hopkins, MD Pediatric Chief Resident McGovern Medical School at the University of Texas Houston Health Science Center at Houston (UTHealth) Houston, Texas Trisomy 21 Julie Hwang, DO Pediatric Hospitalist Wise Health System Decatur, Texas Stevens-Johnson Syndrome Jill Jacoby, MD Internal Medicine-Pediatrics Resident McGovern Medical School at the University of Texas Houston Health Science Center at Houston (UTHealth) Houston, Texas Anemia in the Pediatric Patient Tarun Jain, MD Internal Medicine-Pediatrics Resident McGovern Medical School at the University of Texas Houston Health Science Center at Houston (UTHealth) Houston, Texas Pneumonia and Tuberculosis Precocious Puberty Retropharyngeal Abscess Monica Kodakandla, MD Pediatric Hospitalist Memorial Hermann Hospital Memorial City Houston, Texas Transient Tachypnea of the Newborn Patent Ductus Arteriosus Charlyn Laserna, MD Neonatal Hospitalist Mount Sinai West Hospital New York, New York Turner Syndrome

00_Toy-Pediatrics_FM_p000i-xxiv.indd 10

23/07/21 6:34 PM

CONTRIBUTORS

xi

Ryan Le, MD Internal Medicine-Pediatrics Resident McGovern Medical School at the University of Texas Houston Health Science Center at Houston (UTHealth) Houston, Texas Pediatric Eye Problems Child Abuse Esophageal Atresia Jeffrey Lofgran, MD Internal Medicine-Pediatrics Hospitalist Utah Valley and American Fork Hospitals Provo, Utah Measles Jason Meschin, MD Pediatric Resident McGovern Medical School at the University of Texas Houston Health Science Center at Houston (UTHealth) Houston, Texas Epstein-Barr Virus (Infectious Mononucleosis) Meningitis Truncus Arteriosus Shilpa Mohan, MD Pediatric Nephrology Fellow UCLA Mattel Children’s Hospital Los Angeles, California Postinfectious Glomerulonephritis Posterior Urethral Valves Rickets Fernando Najar, DO Neonatal-Perinatal Fellow Baylor Scott and White Medical Center Temple, Texas Obstructive Sleep Apnea Syndrome Nicolas A. Ortiz, MD Pediatric Cardiology Fellow Nicklaus Children’s Hospital Miami, Florida Failure to Thrive Sepsis and Group B Streptococcal Infections Growth Hormone Deficiency

00_Toy-Pediatrics_FM_p000i-xxiv.indd 11

23/07/21 6:34 PM

xii

CONTRIBUTORS

Tuong Phan, MD Internal Medicine-Pediatrics Resident McGovern Medical School at the University of Texas Houston Health Science Center at Houston (UTHealth) Houston, Texas Febrile Seizures Lead Ingestion Neonatal Herpes Simplex Virus Infection Syeda Hiba Rizvi, MD Pediatric Resident McGovern Medical School at the University of Texas Houston Health Science Center at Houston (UTHealth) Houston, Texas Pelvic Inflammatory Disease Mauro Rodriguez, DO Pediatric Resident McGovern Medical School at the University of Texas Houston Health Science Center at Houston (UTHealth)

Houston, Texas Appendicitis Asthma Exacerbation Concussion Carly Rosemore, MD

Pediatric Hematology/Oncology Fellow Memorial Sloan Kettering Cancer Center New York, New York Acute Lymphoblastic Leukemia Immune Thrombocytopenic Purpura Neuroblastoma Zeina Saleh, MD

Pediatric Resident

McGovern Medical School at the University of Texas Houston Health Science Center at Houston (UTHealth) Houston, Texas Anaphylactoid Purpura Ekta Shah, DO Child Neurology Resident McGovern Medical School at the University of Texas Houston Health Science Center at Houston (UTHealth) Houston, Texas Myasthenia Gravis

00_Toy-Pediatrics_FM_p000i-xxiv.indd 12

23/07/21 6:34 PM

CONTRIBUTORS

xiii

Irtiza Sheikh, DO Pediatric Hematology/Oncology Fellow University of Texas MD Anderson Cancer Center Houston, Texas Infant of a Diabetic Mother Kawasaki Disease Rectal Bleeding Karli Silverberg, MD Pediatric Resident McGovern Medical School at the University of Texas Houston Health Science Center at Houston (UTHealth)

Houston, Texas Accidental Drug Ingestion

Leigh Anna Stubbs, MD, MDH

Pediatric Rheumatology Fellow Texas Children’s Hospital Houston, Texas Juvenile Idiopathic Arthritis Systemic Lupus Erythematosus Brian Townsend, DO

Private Pediatrician Blue Fish Pediatrics Cypress, Texas Infant Rashes Sophia Wang, MD

Pediatric Urgent Care Fellow Children’s National Washington, DC Headache in Children Intestinal Malrotation Muscular Dystrophy Sarah Lund Wilson, MD

Child Neurology Resident

McGovern Medical School at the University of Texas Houston Health Science Center at Houston (UTHealth) Houston, Texas Myasthenia Gravis Regina Yu, DO Pediatric Critical Care Fellow Harbor-UCLA Medical Center Los Angeles, California Sickle Cell Disease

00_Toy-Pediatrics_FM_p000i-xxiv.indd 13

23/07/21 6:34 PM

This page intentionally left blank

PREFACE

We appreciate all the kind remarks and suggestions from the many medical students over the past 3 years. Your positive reception has been an incredible encouragement since the inception of the Case Files® series. In this sixth edition of Case Files®: Pediatrics, the basic format of the book has been retained. Improvements were made in updating many of the sections, including grouping of the cases in a more logical order for students to more easily cross-reference cases. We have also used case correlations to assist further. We reviewed the clinical scenarios and revised several of them, keeping their “real-life” presentations patterned after actual clinical experience. We have written seven new cases, including Infant Rashes, Accidental Drug Ingestion, Anemia in the Pediatric Patient, Pelvic Inflammatory Disease, Patent Ductus Arteriosus, Myasthenia Gravis, and Juvenile Idiopathic Arthritis. We have used a bulleted format for the summary for easier reading and correlated entrustable professional activities (EPAs) for the learning objectives. The multiple-choice questions have been carefully reviewed and rewritten to ensure that they comply with the National Board and USMLE format, and the Review Questions (Section III) have been updated for the student to test their knowledge after reading the book. Through this sixth edition, we hope that the reader will continue to enjoy learning how to diagnose and manage patients through the simulated clinical cases. It certainly is a privilege to be teachers for so many students, and it is with humility that we present this edition. The Authors

xv

00_Toy-Pediatrics_FM_p000i-xxiv.indd 15

23/07/21 6:34 PM

This page intentionally left blank

ACKNOWLEDGMENTS

The clerkship curriculum that evolved into the ideas for this edition was inspired by two talented and forthright students, Philbert Yao and Chuck Rosipal, who have since graduated from medical school. It has been a tremendous joy to work with the excellent pediatricians at the McGovern Medical School at the University of Texas Health Science Center (UTHealth) at Houston. I am greatly indebted to my editor, Bob Boehringer, whose experience and vision helped to shape this series. I appreciate McGraw Hill’s believing in the concept of teaching through clinical cases, and I would like to especially acknowledge Catherine Saggese for her production expertise, Madison Tucky for her editorial guidance, and Revathi Viswanathan for her excellent production skills. At the McGovern Medical School at Houston, I appreciate Dr. Barbara Stoll, our recently retired Dean, for her support. Without the encouragement from my chairman Dr. Sean Blackwell, a wonderful clinician, administrator, scientist, and leader, and Dr. Patricia Butler, Vice Dean for Educational Programs, who inspires us all to be excellent educators, I could not have succeeded in this endeavor. Two medical students—Joy Davis and Heba Ahmad—were instrumental in providing excellent feedback and recommendations. I continue to marvel at my daughter Allison’s amazing intellectual, clinical, and writing ability, effortlessly elevating the quality of our Case Files manuscripts. Most of all, I appreciate my ever-loving wife Terri, and my four wonderful children Andy and his wife Anna, Michael and his wife Nadine, Allison, and Christina and her husband Andy, for their patience and understanding in the writing process. Eugene C. Toy, MD

xvii

00_Toy-Pediatrics_FM_p000i-xxiv.indd 17

23/07/21 6:34 PM

This page intentionally left blank

INTRODUCTION

Mastering the cognitive knowledge within a field such as pediatrics is a formidable task. It is even more difficult to draw on that knowledge, procure and filter through the clinical and laboratory data, develop a differential diagnosis, and finally form a rational treatment plan. To gain these skills, the student often learns best at the bedside, guided and instructed by experienced teachers, and inspired toward selfdirected, diligent reading. Clearly, there is no replacement for education at the bedside. Unfortunately, clinical situations usually do not encompass the breadth of the specialty. Perhaps the best alternative is a carefully crafted patient case designed to stimulate the clinical approach and decision making. In an attempt to achieve that goal, we have constructed a collection of clinical vignettes to teach diagnostic or therapeutic approaches relevant to pediatrics. Most importantly, the explanations for the cases emphasize the mechanisms and underlying principles, rather than merely rote questions and answers. This book is organized for versatility. It allows the student “in a rush” to go quickly through the scenarios and check the corresponding answers, while allowing the student who wants more thought-provoking explanations to go at a more measured pace. The answers are arranged from simple to complex: a summary of the pertinent points, the bare answers, an analysis of the case, an approach to the topic, comprehension questions at the end for reinforcement and emphasis, and a list of references for further reading. A listing of cases is included in Section III to aid the student who desires to test his or her knowledge of a specific area or who wants to review a topic. Finally, we intentionally did not primarily use a multiple-choice question format in our clinical case scenarios because clues (or distractions) are not available in the real world. Nevertheless, several multiple-choice comprehension questions are included at the end of each case discussion to reinforce concepts or introduce related topics.

HOW TO GET THE MOST OUT OF THIS BOOK Each case is designed to simulate a patient encounter with open-ended questions. At times, the patient’s complaint is different from the most concerning issue, and sometimes extraneous information is given. The answers are organized into four different parts:

PART I 1. Summary: The salient aspects of the case are identified, filtering out the extraneous information. Students should formulate their summary from the case before looking at the answers. These are now in bulleted form for easier reading. A comparison to the summation in the answer will help to improve their ability to focus on the important data while appropriately discarding the irrelevant information—a fundamental skill in clinical problem solving. 2. A straightforward Answer is given to each open-ended question. xix

00_Toy-Pediatrics_FM_p000i-xxiv.indd 19

23/07/21 6:34 PM

xx

INTRODUC TION

Table 1  •  SYNOPSIS OF ENTRUSTABLE PROFESSIONAL ACTIVITIES EPA 1

Gather a history and perform a physical examination

EPA 2

Prioritize a differential diagnosis following a clinical encounter

EPA 3

Recommend and interpret common diagnostic and screening tests

EPA 4

Enter and discuss orders and prescriptions

EPA 5

Document a clinical encounter in the patient record

EPA 6

Provide an oral presentation of a clinical encounter

EPA 7

Form clinical questions and retrieve evidence to advance patient care

EPA 8

Give or receive a patient handover to transition care responsibly

EPA 9

Collaborate as a member of a interprofessional team

EPA 10

Recognize a patient requiring urgent or emergent care and initiate evaluation and management

EPA 11

Obtain informed consent for tests and/or procedures

EPA 12

Perform general procedures as a physician

EPA 13

Identify system failures and contribute to a culture of safety and improvement

3. The Analysis of the case is composed of two parts: a. Objectives: A listing of the main principles that are crucial for a practitioner to manage the patient. Again, the students are challenged to make educated “guesses” about the objectives of the case upon initial review of the case scenario, which helps to sharpen their clinical and analytical skills. The objectives are linked to the entrustable professional activities (EPAs), which are listed in Table 1. b. Considerations: A discussion of the relevant points and brief approach to the specific patient.

PART II Approach to the disease process consists of two distinct parts: a. Definitions: Terminology pertinent to the disease process. b. Clinical Approach: A discussion of the approach to the clinical problem in general, including tables, figures, and algorithms.

PART III Comprehension Questions: Each case contains several multiple-choice questions, which reinforce the material or introduce new and related concepts. Questions about material not found in the text have explanations in the answers.

PART IV Clinical Pearls: Several clinically important points are reiterated as a summation of the text. This allows for easy review, such as before an examination.

00_Toy-Pediatrics_FM_p000i-xxiv.indd 20

23/07/21 6:34 PM

LISTING OF CASES

LISTING BY CASE NUMBER CASE NO.

DISEASE CASE PAGE

  1 Infant Rashes   2 Infant of a Diabetic Mother   3 Neonatal Hyperbilirubinemia   4 Sepsis and Group B Streptococcal Infections   5 Accidental Drug Ingestion   6 Neonatal Herpes Simplex Virus Infection   7 Esophageal Atresia   8 Transient Tachypnea of the Newborn   9 Pediatric Eye Problems 10 Failure to Thrive 11 Anemia in the Pediatric Patient 12 Rickets 13 Sickle Cell Disease With Vaso-Occlusive Crisis 14 Pneumonia and Tuberculosis 15 Rectal Bleeding 16 Acute Otitis Media 17 Pelvic Inflammatory Disease 18 Cystic Fibrosis 19 Acute Lymphoblastic Leukemia 20 Asthma Exacerbation 21 Sudden Infant Death Syndrome 22 Patent Ductus Arteriosus 23 Cyanotic Congenital Heart Disease 24 Measles 25 Lead Ingestion 26 Stevens-Johnson Syndrome 27 Bacterial Meningitis 28 Bacterial Enteritis 29 Subdural Hematoma 30 Febrile Seizures 31 Muscular Dystrophy 32 Atopic Dermatitis and Contact Dermatitis 33 Neuroblastoma 34 Intestinal Malrotation 35 Posterior Urethral Valves

17 27 35 45 55 61 71 79 87 97 107 117 129 139 151 159 169 177 185 193 203 211 219 229 237 247 257 265 273 283 291 299 311 319 327 xxi

00_Toy-Pediatrics_FM_p000i-xxiv.indd 21

23/07/21 6:34 PM

xxii

LISTING OF CASES

36 Nursemaid’s Elbow (Radial Head Subluxation) 37 Immune Thrombocytopenic Purpura 38 Child Abuse 39 Kawasaki Disease 40 Immunodeficiency and Human Immunodeficiency Virus 41 Trisomy 21 42 Diabetic Ketoacidosis 43 Obstructive Sleep Apnea Syndrome 44 Growth Hormone Deficiency 45 Precocious Puberty 46 Retropharyngeal Abscess 47 Slipped Capital Femoral Epiphysis 48 Headache in Children 49 Adolescent Substance Use Disorder 50 Turner Syndrome 51 Systemic Lupus Erythematosus 52 Acute Postinfectious Glomerulonephritis 53 Inflammatory Bowel Disease 54 Appendicitis 55 Acute Epstein-Barr Virus (Infectious Mononucleosis) 56 Myasthenia Gravis 57 Juvenile Idiopathic Arthritis 58 Anaphylactoid Purpura 59 Attention Deficit Hyperactivity Disorder 60 Concussion

335 343 351 359 369 379 387 395 403 413 425 435 445 455 471 479 489 497 505 515 523 533 543 551 559

LISTING BY DISORDER (ALPHABETICAL) CASE NO. DISEASE   5 Accidental Drug Ingestion 55 Acute Epstein-Barr Virus (Infectious Mononucleosis) 19 Acute Lymphoblastic Leukemia 16 Acute Otitis Media 52 Acute Postinfectious Glomerulonephritis 49 Adolescent Substance Use Disorder 58 Anaphylactoid Purpura 11 Anemia in the Pediatric Patient 54 Appendicitis 20 Asthma Exacerbation

00_Toy-Pediatrics_FM_p000i-xxiv.indd 22

CASE PAGE 55 515 185 159 489 455 543 107 505 193

23/07/21 6:34 PM

LISTING OF CASES

32 Atopic Dermatitis and Contact Dermatitis 59 Attention Deficit Hyperactivity Disorder 28 Bacterial Enteritis 27 Bacterial Meningitis 38 Child Abuse 60 Concussion 23 Cyanotic Congenital Heart Disease 18 Cystic Fibrosis 42 Diabetic Ketoacidosis   7 Esophageal Atresia 10 Failure to Thrive 30 Febrile Seizures 44 Growth Hormone Deficiency 48 Headache in Children 37 Immune Thrombocytopenic Purpura 40 Immunodeficiency and Human Immunodeficiency Virus   2 Infant of a Diabetic Mother   1 Infant Rashes 53 Inflammatory Bowel Disease 34 Intestinal Malrotation 57 Juvenile Idiopathic Arthritis 39 Kawasaki Disease 25 Lead Ingestion 24 Measles 31 Muscular Dystrophy 56 Myasthenia Gravis 33 Neuroblastoma   6 Neonatal Herpes Simplex Virus Infection   3 Neonatal Hyperbilirubinemia 36 Nursemaid’s Elbow (Radial Head Subluxation) 43 Obstructive Sleep Apnea Syndrome 22 Patent Ductus Arteriosus   9 Pediatric Eye Problems 17 Pelvic Inflammatory Disease 14 Pneumonia and Tuberculosis 35 Posterior Urethral Valves 45 Precocious Puberty 15 Rectal Bleeding 46 Retropharyngeal Abscess 12 Rickets   4 Sepsis and Group B Streptococcal Infections 13 Sickle Cell Disease With Vaso-Occlusive Crisis 47 Slipped Capital Femoral Epiphysis

00_Toy-Pediatrics_FM_p000i-xxiv.indd 23

xxiii

299 551 265 257 351 559 219 177 387 71 97 283 403 445 343 369 27 17 497 319 533 359 237 229 291 523 311 61 35 335 395 211 87 169 139 327 413 151 425 117 45 129 435

23/07/21 6:34 PM

xxiv

26 29 21 51   8 41 50

LISTING OF CASES

Stevens-Johnson Syndrome Subdural Hematoma Sudden Infant Death Syndrome Systemic Lupus Erythematosus Transient Tachypnea of the Newborn Trisomy 21 Turner Syndrome

00_Toy-Pediatrics_FM_p000i-xxiv.indd 24

247 273 203 479 79 379 471

23/07/21 6:34 PM

SECTION I

How to Approach Clinical Problems Part 1

Approach to the Patient

Part 2

Approach to Clinical Problem Solving

Part 3

Approach to Reading

01_Toy-Pediatrics_Sec01_p001-014.indd 1

10/03/21 8:22 AM

This page intentionally left blank

SECTION I: HOW TO APPROACH CLINICAL PROBLEMS

3

Part 1. Approach to the Patient The transition of information from the textbook or journal article to the clinical situation is perhaps the most challenging in medicine. Retention of information is difficult; organization of the facts and recall of myriad data to apply to the patient are crucial. This text aids in the process. The first step is gathering information, otherwise known as establishing the database. This consists of taking the history (asking questions), performing the physical examination, and obtaining selective laboratory and/or imaging tests. The history is the single most important method of establishing a diagnosis. Depending on the age of the child, the information may be gathered solely from the parent, from both the parent and the child, or solely from the adolescent. The student should remember not to be misled by the diagnosis of another clinician or by a family member. A statement such as “Johnnie has pneumonia and needs antibiotics” may or may not be correct; an astute clinician will keep an open mind and consider other possibilities, such as upper respiratory tract infection, aspirated foreign body, reactive airway disease, or even cystic fibrosis. The art of seeking the information in a nonjudgmental, sensitive, and thorough method cannot be overemphasized.

HISTORY 1. Basic information: a. Age, gender, and ethnicity are important because some childhood illnesses occur with increased regularity at various ages, with higher frequency in one gender or more commonly in one ethnic group. For instance, anorexia nervosa is more common in White adolescent females, whereas complications of sickle cell anemia are more common in African American children of both genders. 2. Chief complaint: This is usually the response that the patient or the patient’s family member gives to the question: “Why are you seeing the doctor today?” 3. History of present illness: The onset, duration, and intensity of the primary complaint, as well as associated symptoms, exacerbating and relieving factors, and previous attempts at therapy, should be determined. For children, especially adolescents, a hidden agenda must be considered; it is not uncommon for the adolescent to actually have questions about sexuality when the stated reason for the office visit is totally unrelated. Both positive findings (the stool was loose, voluminous, and foul smelling) and negative findings (without blood or mucus) are appropriate. 4. Past history: a. Pregnancy and delivery: The age of the mother, the number of pregnancies, the route of delivery, and the gestational age of the infant often can provide clues as to the etiology of pediatric conditions. For instance, a large, full-term infant born by cesarean delivery who then develops an increased respiratory

01_Toy-Pediatrics_Sec01_p001-014.indd 3

10/03/21 8:22 AM

4

CASE FILES: PEDIATRICS

rate and streakiness on a chest radiograph is more likely to have transient tachypnea of the newborn than is an infant born vaginally at 28 weeks’ gestation with similar symptoms where a diagnosis of surfactant deficiency is the more likely cause of respiratory symptoms. A history of drug use (including over-the-counter, prescription, and illicit drugs) or infections during pregnancy should also be obtained. b. Neonatal history: Any problems identified in the neonatal period, such as severe jaundice, infections, feeding difficulties, and prolonged hospitalization, should be reviewed, especially for the younger pediatric patients in whom residua of these problems may remain. c. Surgical history: When, where, and for what reason the surgery was performed should be explored. Complications should be noted. d. Medical history: Whereas minor illnesses (such as occasional upper respiratory infections) can be reviewed quickly, more serious illnesses (such as diabetes mellitus) should be investigated fully. The age at diagnosis, treatments prescribed, and response to therapies can be reviewed. The number and nature of hospitalizations and complications are often important. For instance, a diabetic patient with frequent hospitalizations for ketoacidosis may indicate a lack of education of the family or underlying psychosocial issues complicating therapy. A child with a history of frequent, serious accidents should alert the health care provider of possible child abuse. e. Developmental history: For preschool children, a few questions about language and fine motor, gross motor, and psychosocial skills will provide good clues about development. For school-aged children, school performance (grades) and areas of strength and weaknesses are helpful. 5. Allergies: Reactions to medications should be recorded, including severity and temporal relationship to medications. 6. Immunizations: Dates for primary and booster series of immunizations should be recorded, preferably by reviewing the immunization cards or accessing the state’s immunization registry. If the child is in school, a presumption about state laws regarding immunization completion can be made while the immunization card is being retrieved. 7. Medications: List the names of current medications, dosages, routes of administration and frequency, and durations of use. Prescription, over-the-counter, and herbal remedies are relevant. 8. Sexual history of adolescents: Details of an adolescent’s sexual habits, contraceptive use, pregnancies, and sexually transmitted infections should be determined.

CLINICAL PEARL »»

The adolescent must be treated with sensitivity, respect, and confidentiality to foster the optimal environment for medical care.

01_Toy-Pediatrics_Sec01_p001-014.indd 4

10/03/21 8:22 AM

SECTION I: HOW TO APPROACH CLINICAL PROBLEMS

5

9. Family history: Because many conditions are inherited, the ages and health of siblings, parents, grandparents, and other family members can provide important diagnostic clues. For instance, an obese child with a family history of adult-onset diabetes is at high risk for developing diabetes; early intervention is warranted. 10. Social history: Living arrangements, economic situations, types of insurance, and religious affiliations may provide important clues to a puzzling diagnostic case or suggest important information about the acceptability of therapeutic options. 11. Review of systems: A few questions about each of the major body systems allows the practitioner to ensure that no problems are overlooked and to obtain crucial history about related and unrelated medical conditions.

PHYSICAL EXAMINATION 1. General appearance: Well versus poorly nourished; evidence of toxemia, including lethargy (defined as poor or absent eye contact and refusal to interact with environment), signs of poor perfusion, hypo- or hyperventilation, and cyanosis; or stigmata of syndromes (such as Down or Turner). 2. Skin: In smaller children, checking the color of the skin for evidence of pallor, plethora, jaundice, or cyanosis is important. Abnormalities such as capillary hemangiomas (eg, “stork bites” in a newborn), café-au-lait spots, pigmented nevi (eg, congenital dermal melanocytosis [also known as “Mongolian spots”]), erythema toxicum, or pustular melanosis can be identified. In older children, macules, papules, vesicles, pustules, wheals, and petechiae or purpura should be described, and evidence of excoriation, crust formation, desquamation, hyperpigmentation, ulceration, scar formation, or atrophy should be identified. 3. Vital signs: Temperature, blood pressure (generally begin routine measurement after 3 years), heart rate, respiratory rate, height, weight, and head circumference (generally measured until age 3 years). Measurements are plotted and compared to normals for age. 4. Head, eyes, ears, nose, mouth, and throat: a. Head: For the neonate, the size of fontanelles and presence of overriding sutures, caput succedaneum (superficial edema or hematoma that crosses suture lines, usually located over crown), or cephalohematoma (hematoma that does not cross suture lines) should be noted. For the older child, the size and shape of the head as well as abnormalities such as swellings, depressions, or abnormal hair quality or distribution may be identified. b. Eyes: For infants, abnormalities in the size, shape, and position of the orbits, the color of the sclerae (blue sclerae, for instance, may indicate osteogenesis imperfecta), conjunctival hemorrhages, or the presence of iris defects (such as coloboma) may be found. The visual acuity of older children should be determined.

01_Toy-Pediatrics_Sec01_p001-014.indd 5

10/03/21 8:22 AM

6

CASE FILES: PEDIATRICS

c. Ears: For all children, abnormalities in the size, shape, and position of the ears can provide important diagnostic clues. Whereas tympanic membranes are difficult to assess in newborns, their integrity should be assessed in older children. For all children, the quality and character of discharge from the ear canal should be documented. d. Nose: The size, shape, and position of the nose (in relation to the face and mouth) can provide diagnostic clues for various syndromes, such as a small nose in Down syndrome. Patency of the nostrils, especially in neonates who are obligate nose breathers, is imperative. Abnormalities of the nasal bridge or septum, integrity of the mucosa, and the presence of foreign bodies should be noted. A butterfly rash across the nasal bridge can be associated with systemic lupus erythematosus (SLE), and a transverse crease across the anterior portion of the nose is seen with allergic rhinitis. e. Mouth and throat: The size, shape, and position of the mouth and lips in relation to other facial structures should be evaluated. In infants, common findings of the mouth include disruption of the palate (cleft palate syndrome), Epstein pearls (a tiny white papule in the center of the palate), and short frenulum (“tongue-tied”). For all children, the size, shape, and position of the tongue and uvula must be considered. The number and quality of teeth for age should be assessed, and the buccal mucosa and pharynx should be examined for color, rashes, exudate, size of tonsils, and symmetry. 5. Neck: The neck in infants usually is short and sometimes hard to evaluate. Nonetheless, the size, shape, and preferred position of the neck can be evaluated for all children. The range of motion can be evaluated by gentle movement. Symmetry of the muscles, thyroid gland, veins, and arteries is important. An abnormal mass, such as a thyroglossal duct cyst (midline above the level of the thyroid) or brachial cleft cyst (along the sternomastoid muscle), or unusual findings, such as webbing in Turner syndrome, can be identified. 6. Chest: General examination of the chest should include an evaluation of the size and shape of the structures along with identification of obvious abnormalities (such as supernumerary nipples) or movement with respirations. Respiratory rate varies according to age and ranges from 40 to 60 breaths per minute in the neonate to 12 to 14 breaths per minute in the toddler. The degree of respiratory distress can be stratified, with increasing distress noted when the child moves from subcostal to intercostal to supraclavicular to suprasternal retractions. Palpation of the chest should confirm the integrity of the ribs and clavicles and identify any swelling or tenderness in the joints. Percussion in older children may reveal abnormalities, especially if asymmetry is noted. The chest should be auscultated for air movement, vocal resonance, rales, rhonchi, wheezes, and rubs. In adolescent girls, symmetry of breast development and presence of masses or nipple discharge should be evaluated. 7. Cardiovascular: The precordium should be inspected for abnormal movements. The chest should be palpated for the location and quality of the cardiac impulse, as well as to determine if a thrill is present. The presence and quality

01_Toy-Pediatrics_Sec01_p001-014.indd 6

10/03/21 8:22 AM

SECTION I: HOW TO APPROACH CLINICAL PROBLEMS

7

of the first and second heart sounds, including splitting with respirations, should be noted. Murmurs, clicks, rubs, and abnormalities in the heart rate (which vary by age) or rhythm should be identified. The peripheral perfusion, pulses, and color should be assessed. 8. Abdominal examination: The abdomen should be inspected to determine whether it is flat or protuberant, if masses or lesions such as striae are obvious, or if pulsations are present. In older children, the abdomen usually is flat, but in the neonate, a very flat abdomen (scaphoid) in conjunction with respiratory distress may indicate diaphragmatic hernia. The umbilicus, especially for neonates, should be evaluated for defects, drainage, or masses; a small umbilical hernia often is present and is normal. In the newborn, one umbilical vein and two umbilical arteries are normal. In the neonate, palpation of the abdomen may reveal a liver edge about 2 cm below the costal margin, a spleen tip, and using deep pressure, kidneys. In older children, these structures are not usually palpable except in pathology. Depending on the history, other masses must be viewed with suspicion for a variety of conditions. Bowel sounds are usually heard throughout the abdomen except in pathology. In adolescent girls, the lower abdomen should be palpated for uterine enlargement (pregnancy). 9. Genitalia: Examination of the male for the size and shape of the penis, testicles, and scrotum is important. The position of the urethral opening should be assessed. In newborn girls, the labia majora usually is large and completely encloses the labia minora; the genitalia usually is highly pigmented and swollen with an especially prominent clitoris. A white discharge is usually present in the first days of life, and occasionally, blood-tinged fluid is also seen. In toddlers, examination of the genitalia can be challenging. Placing the toddler in a frog-leg position while the toddler sits in the parent’s lap (or on the examination table) often allows successful viewing of external genitalia. In older girls, the knee-chest position affords an excellent view of the external genitalia. In girls outside the newborn period, the labia minora are smaller compared to the remainder of the external genitalia, and the vaginal mucosa is red and appears thin. The hymen, which is just inside the introitus, should be inspected. Abnormalities of the hymen, such as imperforation or tags, vaginal discharge, foreign bodies, and labial adhesions, should be noted. A speculum examination should be performed for sexually active adolescent girls. Tanner staging for pubertal development should be done for both boys and girls. Inguinal hernias should be identified; normalcy of the anus should be confirmed. 10. Extremities: For all children, the size, shape, and symmetry of the extremities should be considered; muscle strength should be evaluated. Joints may be investigated for range of motion, warmth, tenderness, and redness. Normalcy of gait for age should be reviewed. For infants, recognition of dislocated hips is of critical importance since lifelong growth abnormalities may result. For adolescents, identification of significant scoliosis is important to prevent the debilitating complications of that condition. Athletes require evaluation of the integrity of their joints, especially those joints that will be used in sporting activities.

01_Toy-Pediatrics_Sec01_p001-014.indd 7

10/03/21 8:22 AM

8

CASE FILES: PEDIATRICS

11. Neurologic: Neurologic evaluation of the older child is similar to that in adults. Consciousness level and orientation are determined as a starting point. The cranial nerves should be assessed. The motor system should be evaluated (including strength, tone, coordination, and involuntary movements). Superficial and deep sensory systems, as well as deep tendon reflexes, should be reviewed. In younger infants, a variety of normal primitive reflexes (Moro, parachute, suck, grasp) can be found, and ensuring that these reflexes have extinguished by the appropriate age is equally important.

LABORATORY ASSESSMENT The American Academy of Pediatrics recommends a few laboratory screening tests for pediatric patients. These tests vary according to the child’s age and risk factors. 1. Newborn metabolic screening is done in all states, usually after 24 hours of age, but the exact tests performed vary by state. Conditions commonly screened for include hypothyroidism, phenylketonuria, galactosemia, hemoglobin type, and adrenal hyperplasia. Other conditions that may be assessed include maple syrup urine disease, homocystinuria, biotinidase deficiency, cystic fibrosis, tyrosinemia, and toxoplasmosis. Some states require a second newborn screen be performed after 7 days of age. 2. Measurement of oxygen saturation in all newborn infants is accomplished to assess for critical congenital heart defects. 3. Hemoglobin or hematocrit levels are recommended for high-risk infants (especially premature infants and those with low birth weight), at about 12 months of age, and as needed yearly if the risk of blood loss (such as in menstruating adolescents) is high. 4. Lead screening is done, especially in high-risk areas, at 9 to 12 months of age and again at 2 years of age. 5. Cholesterol screening is performed in high-risk patients (those with positive family histories) older than 24 months. 6. Sexually transmitted infection screening is performed yearly on all sexually active patients. Other specialized testing is accomplished depending on the child’s age, risk factors, chief complaint, and conditions included in the differential diagnosis.

IMAGING PROCEDURES 1. Plain radiographs offer the advantage of inexpensive testing that reveals global views of the anatomy. Unfortunately, fine organ detail sometimes is not revealed and further radiographic study is required. Bone films for fracture, chest films for pneumonia, and abdomen films for ileus are common uses of this modality.

01_Toy-Pediatrics_Sec01_p001-014.indd 8

10/03/21 8:22 AM

SECTION I: HOW TO APPROACH CLINICAL PROBLEMS

9

2. Ultrasonography is a fairly inexpensive modality that requires little or no sedation and has no radiation risks. It offers good organ and anatomic detail, but it can be operator dependent. Not all organs are accessible to sonography. Common examinations include the head for intraventricular hemorrhage (IVH) in the premature infant, the abdomen for conditions such as pyloric stenosis or appendicitis, and the kidneys for abnormal structure. 3. Computed tomography (CT) provides good organ and anatomic detail and is quick, but it is fairly expensive, may require contrast, and does involve radiation. Some children require sedation to complete the procedure. This test is often performed on the abdomen or head in trauma victims. 4. Magnetic resonance imaging (MRI) is expensive but does not involve radiation. Because it is a slow procedure, sedation is often needed for younger children, and contrast is sometimes required. It allows for superb tissue contrast in multiple planes and excellent anatomic and functional imaging. It is frequently used to provide detail of the brain in patients with seizures or developmental delay or to provide tissue detail on masses located virtually anywhere in the body. 5. Nuclear scan is moderately expensive and invasive. It provides functional information (usually organ specific) but poor anatomic detail. Radiation is involved. Common uses include bone scans for infection and renal scans for function.

Part 2. Approach to Clinical Problem Solving There are generally four steps to the systematic solving of clinical problems: 1. Make the diagnosis 2. Assess the severity of the disease 3. Render a treatment based on the stage of the disease 4. Follow the response to the treatment

MAKING THE DIAGNOSIS This is achieved with careful sifting of the database, analysis based on the risk factors present, and development of a list of possibilities (the differential diagnosis). The process includes knowing which pieces of information are more meaningful and which can be discarded. Experience and knowledge from reading help to guide the clinician to key in on the most important concerns. A good clinician also knows how to ask the same question in several different ways and using different terminology because patients at times will deny having been treated for asthma but will answer affirmatively to being hospitalized for wheezing. A diagnosis can be reached by systematically reviewing each possible cause and reading about each disease. The patient’s presentation is then matched up against each of these possibilities and either placed higher up on the list as a potential etiology or lower down because

01_Toy-Pediatrics_Sec01_p001-014.indd 9

10/03/21 8:22 AM

10

CASE FILES: PEDIATRICS

of the disease frequency, the patient’s presentation, or other clues. A patient’s risk factors may influence the probability of a diagnosis. Usually, a long list of possible diagnoses can be pared down to two or three top suspicions, based on key laboratory or imaging tests. For example, an adolescent presenting with a fever as the chief complaint can have an extensive differential diagnosis reduced to far fewer possibilities when the history reveals an uncle in the home with cough that contains blood, weight loss, and night sweats and the physical examination shows an increased respiratory rate, lymphadenopathy, and right lower lobe lung crackles. In this case, the patient likely has tuberculosis.

ASSESSING THE SEVERITY OF THE DISEASE The next step is to characterize the severity of the disease process. In asthma, this is done formally based on guidelines promulgated by the National Heart, Lung, and Blood Institute (NHLBI). Asthma categories range from mild intermittent (least severe) to severe persistent (most severe). For some conditions, such as syphilis, the staging depends on the length of time and follows along the natural history of the infection (ie, primary, secondary, or tertiary syphilis).

RENDERING TREATMENT BASED ON THE STAGE OF THE DISEASE Many illnesses are stratified according to severity because prognosis and treatment vary based on the severity. If neither the prognosis nor the treatment is affected by the stage of the disease process, it does not make much sense to subcategorize something as mild or severe. As an example, mild intermittent asthma poses less danger than does severe persistent asthma (particularly if the patient has been intubated for asthma in the past). Accordingly, with mild intermittent asthma, the management would be intermittent short-acting beta-agonist therapy while watching for any worsening of the disease into more serious categories (more severe disease). In contrast, a patient with severe persistent asthma would generally require short-acting beta-agonist medications as well as long-acting beta-agonists, inhaled steroids, and potentially oral steroids. Group A beta-hemolytic streptococcal pharyngeal infection (“strep throat”) is associated with complications including poststreptococcal glomerulonephritis and rheumatic fever. The presence of group A beta-hemolytic Streptococcus confers an increased risk of problems, but neither the prognosis nor the treatment is affected by “more” group A beta-hemolytic Streptococcus or “less” group A betahemolytic Streptococcus. Hence, the student should approach new disease by learning the mechanism, clinical presentation, how it is staged, and how the treatment varies based on stage.

FOLLOWING THE RESPONSE TO TREATMENT The final step in the approach to disease is to follow the patient’s response to the therapy. Whatever the “measure” of response, it should be recorded and monitored. Some responses are clinical, such as a change in the patient’s pain level or temperature or results of pulmonary examination. Obviously, the student must work on

01_Toy-Pediatrics_Sec01_p001-014.indd 10

10/03/21 8:22 AM

SECTION I: HOW TO APPROACH CLINICAL PROBLEMS

11

being more skilled in eliciting the data in an unbiased and standardized manner. Other patients may be followed by imaging, such as CT scan of a retroperitoneal (RP) node size in a patient receiving chemotherapy for neuroblastoma or a marker such as the platelet count in a patient recovering from Kawasaki syndrome. For syphilis, it may be the nonspecific treponemal antibody test rapid plasma reagin (RPR) titer every month. The student must know what to do if the measured marker does not respond according to the expected. Is the next step to treat further, or to repeat the metastatic workup, or to follow up with another more specific test?

Part 3. Approach to Reading The student must approach reading differently than the classic “systematic” review of a particular disease entity. Patients rarely arrive to their health care provider with a clear diagnosis; hence, the student must become skilled in applying the textbook information to the clinical setting. Everyone retains more when the reading is performed with a purpose. Experience teaches that with reading, there are several crucial questions to consider thinking clinically. They are as follows: 1. What is the most likely diagnosis? 2. What should be your next step? 3. What is the most likely mechanism for this process? 4. What are the risk factors for this condition? 5. What are the complications associated with this disease? 6. What is the best therapy?

WHAT IS THE MOST LIKELY DIAGNOSIS? Establishing the diagnosis was discussed in the previous part. This is a difficult task to give to the medical student; however, it is the basic problem that will confront clinicians for the rest of their careers. One way of attacking this problem is to develop standard “approaches” to common clinical problems. It is helpful to memorize the most common causes of various presentations, such as “the most common cause of mild respiratory distress in a term infant born by cesarean section is retained amniotic fluid (transient tachypnea of the newborn).” The clinical scenario would entail something such as the following: “A 3-hour-old infant is noted to have a mildly increased respiratory rate and slight subcostal retractions. The infant is term, is large for gestational age, and was born by repeat cesarean section. The pregnancy was uncomplicated. What is the most likely diagnosis?” With no other information to go on, the student would note that this baby has respiratory distress. Using the “most common cause” information, the student would guess transient tachypnea of the newborn. If, instead, the gestational age “term” is changed to “preterm at 30 weeks’ gestation,” a phrase can be added, such as the following:

01_Toy-Pediatrics_Sec01_p001-014.indd 11

10/03/21 8:22 AM

12

CASE FILES: PEDIATRICS

“The mother did not receive prophylactic steroids prior to birth.” Now, the student would use the “most common cause of respiratory distress in a preterm child whose mother did not receive prenatal steroids” is surfactant deficiency (respiratory distress syndrome).

WHAT SHOULD BE YOUR NEXT STEP? In many ways, this question is even more difficult than the question about the most likely diagnosis because there are many more possible answers. For example, insufficient information may be available to make a diagnosis and the next step may be to pursue more diagnostic information. Another possibility is that the diagnosis is clear, but the subsequent step is the staging of the disease. Finally, the next step may be to treat. Hence, from clinical data, a judgment needs to be rendered regarding how far along one is on the road of: Make diagnosis → Stage disease → Treat based on the stage → Follow response In particular, the student is accustomed to regurgitating the same information that someone has written about a particular disease but is not skilled at giving the next step. This talent is optimally learned at the bedside, in a supportive environment, with freedom to take educated guesses, and with constructive feedback. The student assessing a child in the hospital should go through the following thinking process: 1. Based on the information I have, I believe that Cedric Johnson (a 3-month-old child with a positive respiratory syncytial virus nasal washing) has bronchiolitis. 2. I don’t believe that this is a severe disease (such as significant oxygen requirement, severe retractions, or carbon dioxide retention on blood gas analysis). A chest radiograph shows no lobar consolidation (I believe this is important because a lobar consolidation would suggest a bacterial etiology). 3. Therefore, the treatment is supportive care with supplemental oxygen and intravenous fluids as needed. 4. I want to follow the treatment by assessing Cedric’s respiratory status (I will follow the oxygen saturation and degree of retractions), his temperature, and his ability to maintain his hydration orally without intravenous fluids. Also, if in the next few days Cedric does not get better or if he worsens, I think he will need a repeat chest radiograph to assess whether he has an evolving bacterial pneumonia. In a similar patient, when the clinical presentation is not so clear, perhaps the best “next step” may be diagnostic in nature such as blood cultures to determine if bacteremia is present. This information is sometimes tested by the dictum, “the gold standard for the diagnosis and treatment of a bacterial infection is a culture.”

WHAT IS THE MOST LIKELY MECHANISM FOR THIS PROCESS? This question goes further than requiring the student to make the diagnosis; it also requires the student to understand the underlying mechanism for the process. For example, a clinical scenario may describe a 5-year-old child with Henoch-Schönlein

01_Toy-Pediatrics_Sec01_p001-014.indd 12

10/03/21 8:22 AM

SECTION I: HOW TO APPROACH CLINICAL PROBLEMS

13

purpura (HSP) who develops abdominal pain and heme-positive stools a week after diagnosis. The student first must diagnose the heme-positive stools associated with HSP, which occur in approximately 50% of patients. Then, the student must understand that the edema and damage to the vasculature of the gastrointestinal (GI) tract can cause bleeding along with colicky abdominal pain, sometimes progressing to intussusception. The mechanism of the pain and bleeding is, therefore, vasculitis causing enlarged mesenteric lymph nodes, bowel edema, and hemorrhage into the bowel. Answers that a student may speculate, but would not be as likely, include appendicitis, bacterial gastroenteritis, or volvulus. The student is advised to learn the mechanisms for each disease process and not merely to memorize a constellation of symptoms. In other words, rather than trying to commit to memory the classic presentation of HSP (typical rash, abdominal pain, and arthritis), the student should also understand that vasculitis of the small vessels is the culprit. The vasculitis causes edema, mainly in the dependent areas, that precedes the palpable purpura. This vasculitis is responsible not only for edema in the joints (mainly in dependent areas such as the knees and ankles) causing the arthritis found in approximately two-thirds of patients, but also for damage to the vasculature of the GI tract leading to the intermittent, colicky abdominal pain that can manifest as heme-positive stools or even intussusception.

WHAT ARE THE RISK FACTORS FOR THIS CONDITION? Understanding the risk factors helps to establish the diagnosis and interpret test results. For example, a risk factor analysis may help to manage a 1-year-old child with anemia found on routine screening. If the child had no risk factors for lead poisoning or thalassemia, the practitioner may choose to treat with supplemental iron because the likelihood for more serious pathology is low. On the other hand, if the same 1-year-old child was a recent emigrant from an endemic area, lived in an older home with peeling paint, had a father who worked at a battery smelting plant, and ate meals from unglazed pottery, a practitioner should presumptively diagnose lead poisoning until proven otherwise. The clinician may want to obtain a serum lead level and a complete blood count with differential (looking for basophilic stippling and microcytosis) and thoroughly evaluate the child for developmental delay. Thus, the number of risk factors helps to categorize the likelihood of a disease process.

WHAT ARE THE COMPLICATIONS ASSOCIATED WITH THIS DISEASE? A clinician must understand the complications of a disease so that the patient can be monitored. Sometimes, students will have to make the diagnosis from clinical clues and then apply their knowledge of the sequelae of the pathologic process. For example, a child diagnosed with high fever, rash, lymphadenopathy, and oral and conjunctival changes is diagnosed with Kawasaki syndrome. Complications of this condition include arthritis, vasculitis of the medium-sized arteries, hydrops of the gallbladder, urethritis, and aseptic meningitis. Understanding the types of complications helps the clinician to assess the patient. For example, one life-threatening

01_Toy-Pediatrics_Sec01_p001-014.indd 13

10/03/21 8:22 AM

14

CASE FILES: PEDIATRICS

complication of Kawasaki syndrome is coronary artery aneurysm and thrombosis. The clinical presentation in the subacute phase is desquamation, thrombocytosis, and the development of coronary aneurysms with a high risk of sudden death. The appropriate therapy is intravenous immunoglobulin in the acute phase and high-dose aspirin as soon as possible after the diagnosis is made. Nonrecognition of the risk of coronary artery aneurysm and appropriate therapy for thrombosis can lead to the patient’s death. Students apply this information when they see on rounds a patient with Kawasaki syndrome and monitor for new murmurs, thrombocytosis, myocarditis, and development of coronary artery aneurysms. The clinician communicates to the team to watch the patient for any of these signs or symptoms so that appropriate therapy can be considered.

WHAT IS THE BEST THERAPY? This is perhaps the most difficult question, not only because the clinician needs to reach the correct diagnosis and assess the severity of the condition, but also because he or she must weigh the situation to reach the appropriate intervention. The student does not necessarily need to memorize exact dosages, but the medication, route of delivery, and possible complications are important. It is important for the student to verbalize the diagnosis and the rationale for the therapy. A common error is for the student to “jump to a treatment,” almost like a random guess, and therefore be given “right or wrong” feedback. In fact, the student’s guess may be correct but for the wrong reason; conversely, the answer may be a very reasonable one, with only one small error in thinking. It is crucial instead to give the steps so that feedback can be given for each step. For example, what is the best therapy for a 15-year-old sexually active girl with severe, cystic acne? The incorrect manner of response is for the student to blurt out “isotretinoin (Accutane).” Rather, the student should reason it as follows: “Severe, cystic acne can be treated with a variety of modalities. Side effects of the medications must be considered in a sexually active teenager who is statistically at high risk for pregnancy. Isotretinoin (Accutane) causes severe birth defects and is absolutely contraindicated in pregnancy. Therefore, the best treatment for this adolescent may be a combination of oral antibiotics and topical medications that present a much lower chance of devastating side effects.”

REFERENCES Bickley LS. Pocket Guide to Physical Examination and History Taking. 7th ed. Philadelphia, PA: Lippincott Williams & Wilkins; 2018. Blickman JG, Parker BR, Barnes PD. Pediatric Radiology: The Requisites. 4th ed. Philadelphia, PA: Mosby Publishers; 2016. Levine DA. Growth and development. In: Marcdante K, Kliegman RM, eds. Nelson Essentials of Pediatrics. 8th ed. Philadelphia, PA: Elsevier Publishers; 2018:11-28. Zitelli BI, McIntire SC, Nowalk AJ. Pediatric Physical Diagnosis. 8th ed. Philadelphia, PA: Elsevier Publishers; 2020.

01_Toy-Pediatrics_Sec01_p001-014.indd 14

10/03/21 8:22 AM

SECTION II

Clinical Cases

02_Toy-Pediatrics_Case01_p015-026.indd 15

07/07/21 11:31 AM

This page intentionally left blank

CASE 1 A 3-day-old African American male infant is seen for his early hospital follow-up appointment. His hospital discharge summary shows that he was born via spontaneous vaginal delivery (SVD) at 39 weeks’ gestation to a 22-year-old gravida 1, para 0 (G1P0) mother. The pregnancy was uncomplicated, and the mother had routine prenatal care. Maternal labs were significant for unknown group B streptococcal (GBS) status; the mother received 2 doses of penicillin, the last of which was 2 hours prior to delivery. Growth parameters were appropriate for gestational age, including a weight of 3.255 kg (31%), length of 49 cm (36%), and head circumference of 35 cm (34%). The postnatal hospital stay was unremarkable. He was discharged on day of life 2. Since discharge, he has been breastfeeding well and acting normally. His mother is concerned today, stating that he has “pimples” on his body. She first noticed them on hospital day 2, but she was told they were not a problem. She reports that some of them have “popped” and revealed darkened skin underneath, while more have “spread” to include his “whole body.” She reports no other problems. On examination, he is 3.10 kg (decreased 4.8% from birth weight) and has a heart rate of 136 beats per minute, a respiratory rate of 42 breaths per minute, and a temperature of 36.6 °C (97.8 °F). The physical examination is normal except for the skin, which shows many 2- to 3-mm white papules, mostly on his trunk but also involving all extremities including his palms and soles, and multiple 2- to 3-mm hyperpigmented macules in the same distribution. ▶▶ ▶▶ ▶▶

What is the most likely diagnosis? What treatment would you recommend? What anticipatory guidance should you give the mother?

02_Toy-Pediatrics_Case01_p015-026.indd 17

07/07/21 11:31 AM

18

CASE FILES: PEDIATRICS

ANSWERS TO CASE 1: Infant Rashes Summary: A 3-day-old African American boy presents with šš šš

šš

History of being born at term via SVD after an uneventful pregnancy Appropriate birth growth parameters for gestational age and an unremarkable postnatal hospital stay Development of a pustular rash on his body on day of life 2, with unroofing of lesions revealing hyperpigmented macules

Most likely diagnosis: Pustular melanosis. Recommended treatment: Reassurance. Anticipatory guidance: The rash is self-limited and will resolve on its own without treatment. The pustular phase resolves by about 10 days of age, and the hyperpigmented lesions resolve by 3 to 4 months of age. The mother should not manipulate the lesions but rather use nonirritating lotions and soaps on the infant’s skin.

ANALYSIS Objectives 1. Recognize and be able to describe common infant rashes. (EPA 1, 2, 6) 2. Know how to manage infant rashes. (EPA 3, 4, 5, 8) 3. Know what anticipatory guidance to provide to families regarding infant rashes. (EPA 12)

Considerations This is a 3-day-old African American neonate born at term whose mother is concerned because of a pustular rash over the body beginning on day of life 2. The prenatal and delivery history of this child is nonconcerning. The child was born to a GBS-positive mother, but prophylactic antibiotic therapy during labor was complete. The mother has no history of herpes simplex virus (HSV), thus reducing the chance of an infectious etiology. The infant’s postdelivery course has been normal, suggesting a benign condition. The physical examination is normal except for the skin showing many 2- to 3-mm white papules, mostly on his trunk but also involving all extremities including his palms and soles, and multiple 2- to 3-mm hyperpigmented macules in the same distribution. This is classic for pustular melanosis, which is a benign and harmless transient skin condition of newborns. About 5% of Black infants are affected (vs 1% of White babies). The diagnosis is made clinically. Other conditions in the differential include HSV, miliaria (Candida), Staphylococcus aureus infection, and pediatric erythema toxicum.

02_Toy-Pediatrics_Case01_p015-026.indd 18

07/07/21 11:31 AM

SECTION II: CLINICAL CASES

19

APPROACH TO: Infant Rashes DEFINITIONS RASH: A change of the skin that affects its color, appearance, or texture. MACULE: A distinct, flat lesion less than 1 cm. PAPULE: A distinct, raised lesion less than 1 cm. PATCH: A distinct, flat lesion greater than 1 cm. PLAQUE: A distinct, raised lesion greater than 1 cm. PUSTULE: A vesicle containing pus. VESICLE: A fluid-filled collection less than 1 cm.

CLINICAL APPROACH A critical component of the infant physical examination is a thorough evaluation of the skin. A careful history including maternal and birth history precedes the physical examination to determine whether concern for nonbenign skin lesions, including infectious, vascular, rheumatologic, or environmental etiologies, is present. Many infant rashes are self-limited and only require reassurance.

Erythema Toxicum One of the most common newborn skin finding is erythema toxicum, a transient, self-limited rash of unknown etiology that may occur in up to 50% of full-term infants, most commonly in White infants. It is characterized by an eruption of yellow-white papules or pustules with surrounding erythema that may occur anywhere on the body, commonly on the chest, back, and face and usually sparing the palms and soles. Histologically, a lesion content smear shows eosinophils and no organisms. Erythema toxicum occurs in the first 2 to 3 days of life and typically resolves after the first week of life without intervention, and the skin then returns to baseline pigmentation. See Figure 1–1.

Pustular Melanosis Pustular melanosis is a transient, self-limited, multiphase process of unknown etiology that occurs in Black infants (Figure 1–2). It is characterized by pustules in a widespread distribution, including the palms and soles, that may be present at the time of birth. These pustules tend to rupture and leave behind hyperpigmented macules surrounded by a “collarette” of scaly skin. Histologically, the pustules are filled with neutrophils, no organisms are seen on Gram stain, and the hyperpigmented macule epidermal cells demonstrate increased melanization. The pustular phase typically lasts 2 to 3 days; the macular phase may last 3 to 4 months before resolution. No treatment is required.

02_Toy-Pediatrics_Case01_p015-026.indd 19

07/07/21 11:31 AM

20

CASE FILES: PEDIATRICS

Figure 1–1.  Erythema toxicum. Newborn infant with diffuse macular rash of erythema toxicum. (Reproduced with permission, from Knoop K, Stack L, Storrow A, et al. The Atlas of Emergency Medicine. 5th ed. 2021. Copyright © McGraw Hill LLC. All rights reserved. https://accessmedicine.mhmedical.com.)

Milia Milia are 1- to 2-mm epidermal inclusion cysts that are white in color and filled with keratin. Milia occur in up to 40% of newborns and are found concentrated on the nose and face (Figure 1–3). Milia have intraoral counterparts called Epstein pearls located on the palate, which are typically larger and rarely can be misidentified as natal teeth. Milia rupture spontaneously and do not require treatment.

Congenital Dermal Melanocytosis Congenital dermal melanocytosis (CDM), formerly known as Mongolian spots, presents as sharply demarcated macules or patches of highly varying sizes and shapes. They may be small and isolated or very large and numerous. CDMs are blue-black in color and very commonly located in the sacral region, buttocks, legs, or back (Figure 1–4). CDMs have the highest incidence in African American, Hispanic, Asian, and Native American infants. Histologically, the lesions are characterized by melanin-containing melanocytes in the dermis. Over several years of life, the lesions typically fade owing to darkening of the surrounding skin; however, some may persist into adult life. They are benign and do not require treatment. Recognition and documentation of CDM at birth is imperative, as they can be mistaken for bruises if a question of nonaccidental trauma arises.

02_Toy-Pediatrics_Case01_p015-026.indd 20

07/07/21 11:31 AM

SECTION II: CLINICAL CASES

21

A

B Figure 1–2.  Pustular melanosis. A: A newborn with congenital, thin-walled pustules that rupture easily. B: Hyperpigmented macules appeared by 10 hours of age. (Reproduced with permission, from Kang S, Amagai M, Bruckner AL, et al., eds. Fitzpatrick’s Dermatology. 9th ed. 2019. Copyright © McGraw Hill LLC. All rights reserved. https://accessmedicine.mhmedical.com.)

02_Toy-Pediatrics_Case01_p015-026.indd 21

07/07/21 11:31 AM

22

CASE FILES: PEDIATRICS

Figure 1–3.  Milia. Milia on the face of a 2-week-old infant with greatest number of milia on the nose. (Reproduced with permission, from Usatine RP, Smith MA, Mayeaux, Jr. EJ, et al., eds. The Color Atlas and Synopsis of Family Medicine. 3rd ed. 2019. Copyright © McGraw Hill LLC. All rights reserved. https://accessmedicine.mhmedical.com.)

Miliaria Miliaria is caused by obstruction of the skin sweat glands and may produce differing clinical presentations depending on the depth of obstruction. Superficial epidermal obstruction will present with 1- to 2-mm grouped vesicles without erythema distributed over the chest and neck. This is described as miliaria crystalline (Figure 1–5). More commonly, sweat gland obstruction deeper in the epidermis will cause papules or pustules with surrounding erythema in the same distribution and is called miliaria rubra. These lesions are influenced by environmental factors; namely lesions worsen clinically with higher temperatures and high humidity, which predisposes the infant to an increase in sweat gland obstruction. Treatment includes removal from the predisposing environment to a cool environment.

02_Toy-Pediatrics_Case01_p015-026.indd 22

07/07/21 11:31 AM

SECTION II: CLINICAL CASES

23

Figure 1–4.  Congenital dermal melanocytosis. Congenital dermal melanocytosis (Mongolian spots) on the back of a 1-year-old Black child. (Reproduced with permission, from Usatine RP, Smith MA, Mayeaux, Jr. EJ, et al., eds. The Color Atlas and Synopsis of Family Medicine. 3rd ed. 2019. Copyright © McGraw Hill LLC. All rights reserved. https://accessmedicine.mhmedical.com.)

Figure 1–5.  Miliaria. An example of miliaria crystalline denoted by vesicular eruption over the trunk of this patient. Miliaria rubra can simulate folliculitis. (Reproduced with permission, from Papadakis MA, McPhee SJ, Rabow MW, eds. Current Medical Diagnosis & Treatment 2021. 2021. Copyright © McGraw Hill LLC. All rights reserved. https://accessmedicine.mhmedical.com.)

02_Toy-Pediatrics_Case01_p015-026.indd 23

07/07/21 11:31 AM

24

CASE FILES: PEDIATRICS

CASE CORRELATION šš

See also Case 6 (Neonatal Herpes Simplex Virus Infection), Case 24 (Measles), and Case 26 (Stevens-Johnson Syndrome).

COMPREHENSION QUESTIONS 1.1 A term male infant is delivered by uncomplicated spontaneous vaginal delivery (SVD) at 40 weeks’ gestation. On day of life 1, his mother raises concerns about a “rash on his chest and face.” Upon examination, you note that he has a splotchy, maculopapular, erythematous rash in the mentioned areas. What would you expect to see on the differential of a complete blood count obtained on this patient? A. Neutropenia B. Bandemia C. Lymphocytosis D. Eosinophilia E. Increased white blood cells with toxic granules 1.2 While on rounds in the newborn nursery, you are asked by the mother of a healthy 1-day-old Hispanic female infant about the congenital dermal melanocytosis (CDM) that you identified yesterday on the child’s left posterior thigh. She anxiously states that she has a sister who had melanoma requiring excision and chemotherapy. She asks you if the infant is at increased risk of skin cancer. You reply that the infant has: A. No increased risk of melanoma or other type of skin cancer B. A two-fold higher risk of melanoma and/or other skin cancers C. A four-fold higher risk of melanoma and/or other skin cancers D. No increased risk of melanoma, but she is at 30% risk of her CDM becoming a different type of skin cancer in the future E. A 30% risk of her CDM becoming melanoma in the future, but no increased risk of other types of skin cancer 1.3 Additionally, the mother in the previous question asks you if the infant’s CDM will always look like how it does now. You reply that it typically fades over the course of: A. 1 to 2 days B. 1 to 2 weeks C. 1 to 2 months D. 5 to 6 months E. Several years

02_Toy-Pediatrics_Case01_p015-026.indd 24

07/07/21 11:31 AM

SECTION II: CLINICAL CASES

25

1.4 The mother of a 3-week-old infant brings him to the pediatrician’s office with complaint of facial redness. The infant was born at term at a birthing center; birth documents are not available. The mother states her pregnancy and delivery were uncomplicated, although her third trimester prenatal care was spotty due to transportation issues. The infant had been doing well until 1 day prior, when he developed redness with small bumps on the left side of his face. The previous evening, he began crying inconsolably and began refusing bottles. He finally took a bottle this morning, and his mother managed to get him to go to sleep prior to bringing him to the appointment. On examination, he is sleeping in his mother’s arms and cries loudly when transitioned to the examination table. He has erythema with small vesicles over his left jaw angle that is warm to the touch. What should be your next step in management? A. Reassurance B. Suggest lowering the temperature of the family’s house C. Prescribe clindamycin and discharge home D. Admit to the hospital for intravenous antibiotics E. Obtain jaw plain films

ANSWERS 1.1 D. Eosinophilia. The description is that of a newborn with erythema toxicum. Peripheral blood count may demonstrate eosinophilia in up to 20% of infants with this condition. There is not increased white cells (answer E), lymphocytes (answer C), bands (answer B), or neutrophils (answer A). 1.2 A. No increased risk of melanoma or other type of skin cancer. CDM is a benign lesion without increased risk of malignant transformation (answer E). It is not associated with other types of skin cancer (answers B-D). 1.3 E. Several years. Typically, CDM lesions fade over the course of years (not days to months, as in answers A-D) because of darkening of the surrounding skin; however, some may persist into adult life. 1.4 D. Admit to the hospital for intravenous antibiotics. This patient has symptoms concerning late-onset sepsis, possibly with group B streptococcal infection or herpes simplex virus. Unknown maternal and birth history should heighten your suspicion for sepsis, along with findings such as fever or hypothermia, irritability, poor feeding, lethargy, rash and skin color changes, lymphadenopathy, and seizures. Reassurance (answer A), discharging home (answer C), or treating for miliaria (answer B) would not be appropriate given his age and clinical presentation. His cellulitis requires further evaluation with a complete blood count, blood cultures, lumbar puncture, and intravenous antibiotics. An x-ray (answer E) may be considered if there is concern for osteomyelitis of his jaw; however, this should not be done before initiating treatment for sepsis.

02_Toy-Pediatrics_Case01_p015-026.indd 25

07/07/21 11:31 AM

26

CASE FILES: PEDIATRICS

CLINICAL PEARLS »»

A thorough history and physical examination are essential in diagnosing infant rashes.

»»

Many infant rashes are self-limited and do not require treatment. Offer reassurance to worried parents.

»»

Pustular melanosis lesions are filled with neutrophils, erythema toxicum lesions are filled with eosinophils, and milia are filled with keratin.

»»

It is important to recognize congenital dermal melanocytosis at birth and document it accordingly, as the lesions can mistakenly be identified as bruises in nonaccidental trauma cases.

REFERENCES Hulsmann AR, Oranje AP. Educational paper: neonatal skin lesions. Eur J Pediatr. 2014;173:557-566. Martin KL. Diseases of the neonate. In: Kliegman RM, Stanton BF, St. Geme III JW, Schor NF, Behrman RE, eds. Nelson Textbook of Pediatrics. 20th ed. Philadelphia, PA: Elsevier; 2016:111-123. Prok LD, Torres-Zegarra CX. Skin. In: Hay WW Jr, Levin MJ, Deterding RR, Abzug MJ, eds. Current Diagnosis & Treatment: Pediatrics. 24th ed. New York, NY: McGraw Hill; 2018.

02_Toy-Pediatrics_Case01_p015-026.indd 26

07/07/21 11:31 AM

CASE 2 You are called to the nursery by the postpartum nurse to evaluate a 3-hourold female infant with tachypnea. She was born at 36 weeks’ gestation to a 38-year-old woman whose pregnancy was complicated by type 2 diabetes. Initially, the diabetes was treated with metformin; however, due to inadequate glycemic control, insulin was added during the second trimester. The mother was nonadherent with blood glucose monitoring and insulin therapy, and her hemoglobin A1C at delivery was 12%. Labor began spontaneously, and rupture of membranes occurred 2 hours before delivery. The infant is on the warmer, weighs 4200 g, has a pulse of 140 beats per minute, and has respirations of 72 breaths per minute with intercostal retractions and nasal flaring. She is jittery and plethoric. A capillary glucose measured with the bedside glucometer is 30 mg/dL. ▶▶ ▶▶ ▶▶

What is the next step in the evaluation of this infant? What is the treatment for this infant? What are other possible causes of this infant’s tachypnea?

03_Toy-Pediatrics_Case02_p027-034.indd 27

07/07/21 11:32 AM

28

CASE FILES: PEDIATRICS

ANSWERS TO CASE 2: Infant of a Diabetic Mother Summary: A 3-hour-old female infant presents with šš

Macrosomia

šš

A diabetic mother with poor glucose control

šš

Symptomatic hypoglycemia, which is a medical emergency

Next step: Send a stat serum glucose to confirm the presence of hypoglycemia. Bedside glucometers measure the glucose in whole blood and tend to be 10% lower than serum values. The range of difference is greater at lower glucose values. Treatment: Administer intravenous (IV) glucose because the infant has a glucose less than 40 mg/dL and is symptomatic. If the infant were asymptomatic, feeding should be initiated. Other possible causes of tachypnea: Respiratory distress syndrome (RDS), hypertrophic cardiomyopathy, hypocalcemia, polycythemia, and clavicle fracture.

ANALYSIS Objectives 1. Recognize the clinical features that may occur in the infant of a diabetic mother (IDM). (EPA 1, 2) 2. Know the management of complications occurring in the IDM, the most common of which is neonatal hypoglycemia. (EPA 4)

Considerations Maternal hyperglycemia very early in gestation can cause significant birth defects, including neural tube defects and congenital heart disease. Later, in response to poorly controlled maternal hyperglycemia, fetal hyperinsulinism begins in the second trimester, resulting in fetal macrosomia and increased fetal oxygen requirements. Fetal insulin production leads to increased glycogen production, which is deposited in the fetal liver, heart, kidneys, and skeletal muscle. The large shoulders and abdomen of such infants make delivery difficult, and the infant may sustain shoulder dystocia, clavicle fracture, or brachial plexus injury. Hypertrophic cardiomyopathy results from glycogen deposition in the myocardium. Increased fetal oxygen requirements cause polycythemia. Insulin appears to interfere with cortisol’s ability to induce surfactant production, which predisposes the neonate to RDS. After delivery and removal from the high-sugar in utero environment, the infant’s hyperinsulinism can cause hypoglycemia, which must be managed immediately. If the hypoglycemia is left untreated, seizures, obtundation, and respiratory arrest can occur.

03_Toy-Pediatrics_Case02_p027-034.indd 28

07/07/21 11:32 AM

SECTION II: CLINICAL CASES

29

APPROACH TO: Infant of a Diabetic Mother DEFINITIONS GESTATIONAL DIABETES MELLITUS (GDM): Persistent hyperglycemia during pregnancy, with serum glucose levels above the thresholds for the oral glucose tolerance test. HYPOGLYCEMIA: A blood glucose level less than 40 mg/dL is the usual definition, although other definitions exist. Symptoms include lethargy, listlessness, poor feeding, temperature instability, apnea, cyanosis, jitteriness, tremors, seizure activity, and respiratory distress. MACROSOMIA: Larger than normal baby with the birth weight exceeding the 90th percentile for gestational age, or any birth weight more than 4 kg.

CLINICAL APPROACH Diabetes in Pregnancy Diabetes affects an average of 7% of pregnancies. For most women, the condition is transient, occurring during pregnancy and disappearing after delivery. Women are generally screened for gestational diabetes between 24 and 28 weeks of pregnancy but can be screened earlier if considered high risk. If hyperglycemia is present in the first trimester, an increased risk for congenital anomalies of the central nervous system, heart, kidneys, and the skeletal system (such as caudal regression syndrome [hypoplasia of the sacrum and lower extremities]) is noted (Figure 2–1). Women who require insulin therapy are at higher risk for a poor perinatal outcome as compared to those whose carbohydrate intolerance can be managed by diet alone. With better glycemic control, the rates of malformations, macrosomia, and hypoglycemia are lower.

Hypoglycemia Hypoglycemia develops in about 25% to 50% of infants born to mothers with pregestational diabetes and in about 25% of infants born to mothers with gestational diabetes. šš

šš šš

šš

Therefore, all IDMs should be fed an unlimited quantity within the first hour of life. Bedside glucose measurement approximately 30 minutes after the feed. A blood glucose level of less than 40 mg/dL with any symptom of hypoglycemia requires IV glucose. If symptoms of hypoglycemia are absent, the infant is refed and the glucose is remeasured 30 minutes later.

03_Toy-Pediatrics_Case02_p027-034.indd 29

07/07/21 11:32 AM

30

CASE FILES: PEDIATRICS

Hypoglycemia (seizure, jittery, apnea) Spina bifida/Neural tube defect Shoulder dystocia, brachial plexus injury Hyaline membrane disease, respiratory distress syndrome

Hypertrophic cardiomyopathy Polycythemia, hyperbilirubinemia, hypocalcemia, hypomagnesia

Renal vein thrombosis Caudal regression syndrome

Small left colon syndrome

Figure 2–1.  Findings in infants of diabetic mothers.

Glucose levels in the first 4 hours of life that do not increase above 25 mg/dL or are persistently less than 40 mg/dL despite feeding will also require IV glucose. Hyperinsulinemia usually resolves after 1 to 2 days. Monitoring of glucose typically continues for the first 12 hours of life and until three consecutive preprandial measurements are normal. Feeding patterns are monitored closely because poor feeding in the IDM can represent a metabolic or cardiac abnormality.

Other Complications Hypocalcemia is another metabolic abnormality commonly seen in IDM and presents as irritability, sweating, or seizures. Symptomatic infants will require IV calcium replacement. Infants who are large for gestational age should be examined closely for signs of birth trauma as a result of their size, such as cephalohematoma, clavicle fracture, and brachial plexus injury. Macrosomia in utero creates an increase in the intrauterine oxygen requirement, and the relative placental insufficiency leads to increased production of erythropoietin. The resultant polycythemia, defined as a central hematocrit greater than 65% in a neonate, may give a ruddy or plethoric hue to the infant’s skin. Polycythemia contributes to elevated bilirubin levels and can cause hyperviscosity syndrome with resultant venous thrombosis in the renal veins, cerebral venous sinus, or mesenteric veins. Polycythemia is treated with increased hydration and, in rare instances, partial exchange transfusion.

03_Toy-Pediatrics_Case02_p027-034.indd 30

07/07/21 11:32 AM

SECTION II: CLINICAL CASES

31

CASE CORRELATION šš

See also Case 3 (Neonatal Hyperbilirubinemia) and Case 9 (Pediatric Eye Problems).

COMPREHENSION QUESTIONS 2.1 A 36-week gestation infant is delivered via cesarean section because of macrosomia and fetal distress. The mother has class D pregestational diabetes (insulin dependent, with vascular disease); her hemoglobin A1C is 15% (normal 7%). This infant is at risk for hypocalcemia, cardiomyopathy, polycythemia, and which of the following? A. Congenital hip dislocation B. Dacryostenosis C. Respiratory distress syndrome D. Hyperglycemia E. Pneumothorax 2.2 A term infant weighing 4530 g is born without complication to a mother with gestational diabetes. At 12 hours of life, he appears mildly jaundiced. Vital signs are stable, he is eating well, and his blood type is the same as his mother’s blood type. Which of the following serum laboratory tests are most likely to help you diagnose the cause of this infant’s jaundice? A. Total protein, serum albumin, and liver transaminases B. Total and direct bilirubin, liver transaminases, and a hepatitis panel C. Total bilirubin and a hematocrit D. Total bilirubin and a glucose E. Serum calcium 2.3 A term boy born to a mother with insulin-dependent pregestational diabetes has a bedside capillary glucose of 32 mg/dL at 1 hour of life. He is awake and has not fed yet. The vital signs are normal. Which of the following is the most appropriate next step in management? A. Instruct the mother to breastfeed him and recheck the glucose in 30 minutes. B. Place an IV and administer glucose. C. Recheck the glucose in 1 hour. D. Measure his serum insulin level. E. Take him to the nursery for observation.

03_Toy-Pediatrics_Case02_p027-034.indd 31

07/07/21 11:32 AM

32

CASE FILES: PEDIATRICS

ANSWERS 2.1 C. Respiratory distress syndrome. Infants born to mothers with poorly controlled diabetes are at risk for respiratory distress syndrome (due to surfactant deficiency) even at near-term gestational ages. The maternal hyperglycemia stimulates fetal hyperinsulinism, which has an antagonistic effect on surfactant production. These newborns are prone to severe hypoglycemia (not hyperglycemia, as in answer D). The other complications listed (answer A, congenital hip dislocation; answer B, dacryostenosis; and answer E, pneumothorax) are not associated with IDM. 2.2 C. Total bilirubin and a hematocrit. This baby most likely has hyperbilirubinemia due to polycythemia. Total bilirubin and hematocrit should be measured to determine if phototherapy is needed. The polycythemia warrants close monitoring of the infant’s hydration in order to avoid worsening the hyperviscosity. Total protein, serum albumin, and liver transaminases (answer A) would suggest a liver problem. Total and direct bilirubin, liver transaminases, and hepatitis panel (answer B) would indicate a viral hepatitis. Because the infant has no signs of hypoglycemia, it is not likely that the jaundice is due to diabetes (answer D). 2.3 A. Instruct the mother to breastfeed him and recheck the glucose in 30 minutes. Infants of diabetic mothers are at risk for hypoglycemia and should feed within the first hour of birth, with glucose measured 30 minutes later. In this infant’s case, he has not been fed yet, so he should breastfeed and have the glucose remeasured. IV glucose (answer B) would be needed at this time only if he had symptoms of hypoglycemia. Rechecking the glucose in an hour without feeding (answer C) is withholding treatment and not appropriate; the latter choices (answer D, measure serum insulin level; and answer E, observe in the nursery) also do not include providing the appropriate treatment.

CLINICAL PEARLS »»

Infants of diabetic mothers are at risk for congenital malformations and perinatal complications, including hypoglycemia, polycythemia, hyperbilirubinemia, hypocalcemia, and birth trauma.

»»

Hypoglycemia manifests with nonspecific symptoms; any alteration in the infant’s vital signs or general state should prompt immediate glucose measurement.

»»

Neonatal hyperbilirubinemia can be seen in the infant born to the diabetic mother, as polycythemia is a common complication resulting in excessive red blood cell breakdown.

»»

Congenital cataracts can present as a complication in infants of diabetic mothers.

03_Toy-Pediatrics_Case02_p027-034.indd 32

07/07/21 11:32 AM

SECTION II: CLINICAL CASES

33

REFERENCES Adamkin DH; Committee on Fetus and Newborn. Postnatal glucose homeostasis in late-preterm and term infants. Pediatrics. 2011;127:575-579. American College of Obstetricians and Gynecologists. Gestational diabetes mellitus. ACOG Practice Bulletin No. 190. Obstet Gynecol. 2018;131:349-364. French HM, Simmons RA. Infant of a diabetic mother. In: Rudolph CD, Rudolph AM, Lister GE, First LR, Gershon AA, eds. Rudolph’s Pediatrics. 22nd ed. New York, NY: McGraw Hill; 2011; 195-198. Sheanon NM, Muglia LJ. Infants of diabetic mothers. In: Kleigman RM, St. Geme JW, Blum NJ, Shah SS, Tasker RC, Wilson KM, Behrman RE, eds. Nelson Textbook of Pediatrics. 21st ed. Philadelphia, PA: Elsevier; 2020:983-985.

03_Toy-Pediatrics_Case02_p027-034.indd 33

07/07/21 11:32 AM

This page intentionally left blank

CASE 3 A mother presents to the pediatrician with concerns that her 4-day-old son’s face and chest are turning yellow. This Asian infant was delivered vaginally after an uncomplicated term pregnancy to a 25-year-old gravida 2, para 2 (G2P2) mother. The mother received routine prenatal care and is in good health. The mother states that when she had her first child 3 years ago, he had to have “light therapy” for a few days after he was born. The infant’s vital signs are within normal limits for a neonate. The temperature is 99.1 °F rectally. With the exception of a large cephalohematoma, his physical examination is normal. He is breastfeeding well and shows no signs of illness. ▶▶ ▶▶

What is the most likely diagnosis? What is the next step in evaluating this patient?

04_Toy-Pediatrics_Case03_p035-044.indd 35

07/07/21 11:36 AM

36

CASE FILES: PEDIATRICS

ANSWERS TO CASE 3: Neonatal Hyperbilirubinemia Summary: A healthy, 4-day-old Asian male infant presents with šš

Yellowing of his skin (jaundice)

šš

Breastfeeding without problems

šš

Unremarkable delivery and normal vital signs

šš

šš

History of a sibling who needed “light therapy” (likely phototherapy for jaundice) in infancy A cephalohematoma on physical exam

Most likely diagnosis: Neonatal hyperbilirubinemia. Next step: Serum or transcutaneous bilirubin level.

ANALYSIS Objectives 1. Understand the etiology of physiologic neonatal jaundice. (EPA 1, 2) 2. Identify the causes of pathologic jaundice in a newborn. (EPA 1, 2, 10) 3. Know the treatment for neonatal jaundice. (EPA 4)

Considerations Neonatal hyperbilirubinemia (Table 3–1) results from (1) higher rates of bilirubin production with red blood cells (RBCs) turned over at too rapid a rate, and (2) a limited ability to excrete bilirubin with interrupted transmission of unconjugated bilirubin to the liver or liver enzymatic deficiencies that preclude appropriate metabolism of the unconjugated material. This infant has several risk factors for neonatal physiologic jaundice: male gender, cephalohematoma, Asian ancestry, and breastfeeding. Other risk factors to be considered for neonatal jaundice are maternal diabetes, prematurity, polycythemia, trisomy 21, delayed bowel movement, upper gastrointestinal obstruction, hypothyroidism, swallowed maternal blood, and a sibling with physiologic jaundice.

04_Toy-Pediatrics_Case03_p035-044.indd 36

07/07/21 11:36 AM

SECTION II: CLINICAL CASES

37

Table 3–1  •  DIFFERENTIAL DIAGNOSIS OF NEONATAL HYPERBILIRUBINEMIA Unconjugated or Indirect Hyperbilirubinemia • Hemolytic disease • ABO incompatibility • Rh incompatibility • Other minor blood group incompatibility • Structural or metabolic abnormalities of RBCs • Hereditary spherocytosis • G6PD deficiency • Hereditary defects in bilirubin conjugation • Crigler-Najjar—types I and II • Gilbert disease • Bacterial sepsis • Breast milk jaundice • Physiologic jaundice Conjugated or Direct Hyperbilirubinemia • Biliary atresia • Extrahepatic biliary obstruction • Neonatal hepatitis • Bacterial • Viral • Nonspecific • TPN related • Short bowel related • Inspissated bile syndrome • Postasphyxia • Alpha-1-Antitrypsin deficiency • Neonatal hemosiderosis Abbreviations: G6PD, glucose-6-phosphate dehydrogenase; RBCs, red blood cells; TPN, total parenteral nutrition. Revised and adapted from McMillan JA, DeAngelis CD, Feigin RD, et al., eds. Oski’s Pediatrics: Principles and Practice. 4th ed. 2006. Copyright © Lippincott Williams & Wilkins. All rights reserved.

APPROACH TO: Neonatal Hyperbilirubinemia DEFINITIONS CONJUGATED (DIRECT) BILIRUBIN: Bilirubin chemically attached to a glucuronide by an enzymatic process in the liver; elevated levels are not neurotoxic (do not cross blood-brain barrier) but may be indicative of a more serious underlying illness. ERYTHROBLASTOSIS FETALIS (ALSO KNOWN AS HEMOLYTIC DISEASE OF THE NEWBORN): Increased RBC destruction due to transplacental maternal antibody passage active against the infant’s RBC antigens. Common causes are ABO incompatibility and maternal-fetal Rh antigen incompatibility.

04_Toy-Pediatrics_Case03_p035-044.indd 37

07/07/21 11:36 AM

38

CASE FILES: PEDIATRICS

KERNICTERUS: A neurologic syndrome resulting from unconjugated bilirubin deposition in brain cells, especially the basal ganglia, globus pallidus, putamen, and caudate nuclei. Less mature or sick infants have greater susceptibility. Lethargy, poor feeding, and loss of Moro reflex are common initial signs. POLYCYTHEMIA: A central hematocrit of 65% or higher, which can lead to blood hyperviscosity. TRANSCUTANEOUS BILIRUBINOMETER: A device that is placed on the infant’s skin and produces light at several wavelengths that then measures its reflection after interacting with the serum bilirubin in the microcirculation beneath the skin. The device is used to measure serum bilirubin noninvasively as an alternative to drawing blood to measure serum bilirubin. UNCONJUGATED (INDIRECT) BILIRUBIN: Bilirubin yet to be enzymatically attached to a glucuronide in the liver; elevated levels can cause neurotoxicity.

CLINICAL APPROACH Physiologic Jaundice Physiologic jaundice is unconjugated hyperbilirubinemia that is observed during the first week of life; it is seen in approximately 60% of full-term infants and 80% of preterm infants. The diagnosis of physiologic jaundice is established by precluding known jaundice causes through a thorough history, along with clinical and laboratory findings. Newborn infants have a limited ability to conjugate bilirubin and cannot readily excrete unconjugated bilirubin. Jaundice usually begins on the face and then progresses to the chest, abdomen, and feet; the total bilirubin roughly correlates with the extent of spread of jaundice. Full-term newborns with physiologic jaundice usually have peak bilirubin concentrations of 5 to 6 mg/dL between the second and fourth days of life. Parents can be reassured, but close outpatient follow-up is required to ensure that the infant is feeding and voiding well and that the bilirubin level does not reach unsafe levels. Approximately 2% of breastfed full-term infants develop significant unconjugated bilirubin elevations (breast milk jaundice) after the seventh day of life; concentrations up to 30 mg/dL during the second to third week can be seen. If breastfeeding is continued, the levels gradually decrease. Formula substitution for breast milk for 12 to 24 hours results in a rapid bilirubin level decrease; breastfeeding can be resumed without return of hyperbilirubinemia.

Nonphysiologic Jaundice Findings suggestive of nonphysiologic jaundice include: (1) appearance in the first 24 to 36 hours of life (2) bilirubin rate of rise greater than 5 mg/dL/24 h (3) bilirubin greater than 12 mg/dL in a full-term infant without other physiologic jaundice risk factors listed or (4) jaundice that persists after 10 to 14 days of life

04_Toy-Pediatrics_Case03_p035-044.indd 38

07/07/21 11:36 AM

SECTION II: CLINICAL CASES

39

Nonphysiologic etiologies are commonly diagnosed in a jaundiced infant who has a family history of hemolytic disease or in an infant with concomitant pallor, hepatomegaly, splenomegaly, failure of phototherapy to lower bilirubin, vomiting, lethargy, poor feeding, excessive weight loss, apnea, or bradycardia. Causes of nonphysiologic jaundice include septicemia, biliary atresia, hepatitis, galactosemia, hypothyroidism, cystic fibrosis, congenital hemolytic anemia (eg, spherocytosis, maternal Rh, or blood type sensitization), Lucey-Driscoll syndrome (transient familial hyperbilirubinemia of poorly understood etiology), or drug-induced hemolytic anemia. Crigler-Najjar syndrome is a problem with conjugation of bilirubin and leads to jaundice from day of life 3 or 4, progressively worsening by the second week of life.

Jaundice Within 24 Hours of Life Jaundice presenting within the first 24 hours of life is pathological and requires immediate attention; causes include erythroblastosis fetalis, hemorrhage, sepsis, cytomegalic inclusion disease, rubella, and congenital toxoplasmosis. Alloimmune hemolysis, such as ABO incompatibility, is seen in newborns with blood type A or B antigens whose mothers are blood type O with anti-A and anti-B immunoglobulins. Isoimmune hemolysis with Rh antigen incompatibility is seen in Rh-positive infants born to Rh-negative mothers. Unconjugated hyperbilirubinemia can cause kernicterus, the signs of which mimic sepsis, asphyxia, hypoglycemia, and intracranial hemorrhage. Lethargy and poor feeding are common initial signs, followed by a gravely ill appearance with respiratory distress and diminished tendon reflexes.

Assessing Bilirubin Levels The American Academy of Pediatrics recommends establishing protocols to assess the risk of severe hyperbilirubinemia in all newborns prior to their discharge home. This assessment can be done by measuring total serum bilirubin (TsB) levels or noninvasive transcutaneous bilirubin (TcB) levels. The TcB bilirubin measured at the newborn’s sternum correlates with serum levels and is reliable in newborns of different ethnicities and at different gestational ages. The TcB measurements are not reliable after the infant has undergone phototherapy. Any concerning or inconsistent TcB measurement should be confirmed with TsB. The infant’s TsB or TcB should be charted on a bilirubin nomogram, which plots bilirubin level versus hour of life, to assess the patient’s risk of developing severe hyperbilirubinemia. The nomogram categorizes the infant’s bilirubin levels as low risk, low-intermediate risk, high-intermediate risk, and high risk to estimate likelihood of bilirubin toxicity and the need for further evaluation or intervention. Online nomograms, such as BiliTool (www.bilitool.org), offer ease of use for risk designation and minimizing medical errors while also considering other risk factors that lower the threshold for initiating phototherapy. Significant hyperbilirubinemia requires a diagnostic evaluation, including the measurement of indirect and direct bilirubin concentrations, hemoglobin level, reticulocyte count, blood type, Coombs test, and peripheral blood smear examination.

04_Toy-Pediatrics_Case03_p035-044.indd 39

07/07/21 11:36 AM

40

CASE FILES: PEDIATRICS

Treatment Phototherapy is often used to treat unconjugated hyperbilirubinemia, with the unclothed infant placed under a bank of phototherapy lights, the eyes shielded, and hydration maintained. The phototherapy light converts unconjugated bilirubin into a less toxic, water-soluble form that can be easily excreted. Exchange transfusion is needed in a small number of jaundiced infants who do not respond to phototherapy or have total bilirubin greater than 25 mg/dL. Small aliquots of the infant’s blood are removed via a blood vessel catheter and replaced with similar aliquots of donor blood. Risks of this procedure include air embolus, volume imbalance, arrhythmias, acidosis, respiratory distress, electrolyte imbalance, anemia or polycythemia, blood pressure fluctuation, infection, and necrotizing enterocolitis.

CASE CORRELATION šš

See Case 2 (Infant of a Diabetic Mother).

COMPREHENSION QUESTIONS 3.1 Which of the following decreases the risk of neurologic damage in a jaundiced newborn? A. Acidosis B. Displacement of bilirubin from binding sites by drugs such as sulfisoxazole C. Hypoalbuminemia D. Sepsis E. Maternal ingestion of phenobarbital during pregnancy 3.2 An 8-day-old infant continues to have jaundice, which was first noted on the second day of life; his latest total and direct bilirubin levels are 12.5 mg/dL and 0.9 mg/dL, respectively. The baby and the mother have type O positive blood, the direct and indirect Coombs tests are negative, the infant’s reticulocyte count is 15%, hemoglobin is 17 g/dL, hematocrit is 52%, and a smear of his blood reveals no abnormally shaped cells. He is bottle-feeding well, produces normal stools and urine, and has gained weight well. Which of the following remains in the differential diagnosis? A. Gilbert syndrome B. Disseminated intravascular coagulation (DIC) C. Spherocytosis D. Polycythemia E. An undiagnosed blood group isoimmunization

04_Toy-Pediatrics_Case03_p035-044.indd 40

07/07/21 11:36 AM

SECTION II: CLINICAL CASES

41

3.3 Hyperbilirubinemia associated with Crigler-Najjar syndrome type I is caused by which of the following? A. Increased production of bilirubin B. Increased conjugation of bilirubin C. Deficient hepatic uptake of bilirubin D. Severe deficiency of uridine diphosphate glucuronosyltransferase E. Glucose-6-phosphate dehydrogenase deficiency 3.4 A 30-hour-old full-term infant has facial and chest jaundice. He is breastfeeding well and has an otherwise normal examination. His bilirubin level is 15.5 mg/dL. Which of the following is the most appropriate course of action? A. Recommend cessation of breastfeeding for 48 hours and supplement with formula B. Start phototherapy C. Wait for 6 hours and retest the serum bilirubin level D. Start an exchange transfusion E. No action is needed 3.5 A 12-day-old male infant presents to clinic for well-child check. His mother is concerned because she has noticed that “his eyes are turning yellow.” She reports that he is a “good eater” and is proud to report that he is exclusively breastfed. The infant has gained weight since birth and is voiding and stooling appropriately. What is the most likely cause of his jaundice? A. Physiologic jaundice B. Crigler-Najjar syndrome C. Breastfeeding failure D. Breast milk jaundice E. TORCH (toxoplasmosis, other, rubella, cytomegalovirus, herpes) infection

ANSWERS 3.1 E. Maternal ingestion of phenobarbital during pregnancy. Administration of phenobarbital induces glucuronyl transferase, thus reducing neonatal jaundice. Sepsis (answer D) and acidosis (answer A) increase the risk of neurologic damage by increasing the blood-brain barrier’s permeability to bilirubin. Hypoalbuminemia (answer C) reduces the infant’s ability to transport unconjugated bilirubin to the liver, and similarly, drugs that displace bilirubin from albumin (answer B) elevate free levels of unconjugated bilirubin in the serum. 3.2 A. Gilbert syndrome. Gilbert syndrome would present with a negative Coombs test, a normal (or low) hemoglobin, a normal (or slightly elevated) reticulocyte count, and prolonged hyperbilirubinemia. Red cell morphology would be abnormal in DIC (answer B) and spherocytosis (answer C). Polycythemia

04_Toy-Pediatrics_Case03_p035-044.indd 41

07/07/21 11:36 AM

42

CASE FILES: PEDIATRICS

(answer D) would present with an elevated hemoglobin level (that which is listed is normal for a newborn), and blood group isoimmunization (answer E) would present with a positive Coombs test. 3.3 D. Severe deficiency of uridine diphosphate glucuronosyltransferase. Although all infants are relatively deficient in uridine diphosphate glucuronosyltransferase, those with Crigler-Najjar syndrome type I have a severe deficiency, causing high bilirubin levels and encephalopathy. Treatment is phototherapy. Encephalopathy is rare with Crigler-Najjar syndrome type II, in which bilirubin levels infrequently exceed 20 mg/dL. The other mechanisms listed (answer A, increased production of bilirubin; answer B, increased conjugation of bilirubin; answer C, deficient hepatic uptake of bilirubin; and answer E, glucose6-phosphate dehydrogenase deficiency) are caused by other conditions. 3.4 B. Start phototherapy. A bilirubin of 15.5 mg/dL at 30 hours of life is considered high risk for severe hyperbilirubinemia based on the phototherapy nomograms. Although the etiology of the hyperbilirubinemia must be investigated, phototherapy should be started. For that reason, the option of no therapy (answer E) is incorrect. Bilirubin should still be monitored regularly while on phototherapy. If the level does not decrease appropriately, exchange transfusion (answer D) should be considered. Breastfeeding (answer A) would not be the cause of jaundice at this age and should not be discontinued. Waiting and retesting the bilirubin in 6 hours (answer C) may be associated with a dangerous bilirubin level. 3.5 D. Breast milk jaundice. Physiologic jaundice (answer A) usually presents during the first week of life, whereas this patient is 12 days old. The mechanism of breastfeeding jaundice is adequate milk intake, with reduced bowel movements and increased enterohepatic circulation of bilirubin. The patient is above birth weight and feeding well, so breastfeeding failure is unlikely (answer C). The infant is exclusively breastfed, and jaundice starting during the second week of life makes breast milk jaundice most likely; the mechanism is thought to be due to an inhibitor to bilirubin conjugation in the breast milk in some women, which leads to transient hyperbilirubinemia that does not require treatment. There is no stigmata of TORCH infection (answer E). Crigler-Najjar syndrome (answer B) is rare but should be considered if the bilirubin level is high or does not resolve.

04_Toy-Pediatrics_Case03_p035-044.indd 42

07/07/21 11:36 AM

SECTION II: CLINICAL CASES

43

CLINICAL PEARLS »»

Physiologic jaundice, observed during the first week of life in the majority of infants, results from higher bilirubin production rates and a limited ability of excretion. The diagnosis is established by precluding known causes of jaundice based on history and clinical and laboratory findings.

»»

Nonphysiologic jaundice is caused by septicemia, biliary atresia, hepatitis, galactosemia, hypothyroidism, cystic fibrosis, congenital hemolytic anemia, drug-induced hemolytic anemia, or antibodies directed at the fetal RBC.

»»

High levels of unconjugated bilirubin may lead to kernicterus, an irreversible neurologic syndrome resulting from brain cell bilirubin deposition, especially in the basal ganglia, globus pallidus, putamen, and caudate nuclei. Less mature or sick infants are at greater risk. The signs and symptoms of kernicterus may be subtle and similar to those of sepsis, asphyxia, hypoglycemia, and intracranial hemorrhage.

»»

It is important to plot the patient’s bilirubin on a nomogram that plots hours of age against bilirubin level to categorize the patient’s risk for severe hyperbilirubinemia and to determine the need for phototherapy or exchange transfusion.

»»

One of the complications of an infant born to a mother who has diabetes is polycythemia, a known cause of hyperbilirubinemia because of the excessive red cell breakdown.

REFERENCES American Academy of Pediatrics. Management of hyperbilirubinemia in the newborn infant 35 or more weeks of gestation. Pediatrics. 2004;114:297-316. Cashore WJ. Neonatal hyperbilirubinemia. In: McMillan JA, DeAngelis CD, Feigin RD, Jones MD, eds. Oski’s Pediatrics: Principles and Practice. 4th ed. Philadelphia, PA: Lippincott Williams & Wilkins; 2006:235-245. Pammi M, Lee HC, Suresh GK. Neonatal jaundice. In: Kline MW, Blaney SM, Giardino AP, Orange JS, Penny DJ, Schutze GE, Shekerdemian LS, eds. Rudolph’s Pediatrics. 23rd ed. New York, NY: McGraw Hill; 2018:261-264. Pan DH, Rivas Y. Jaundice: newborn to age 2 months. Pediatr Rev. 2017;38:499-508. Shaughnessy EE, Goyal NK. Jaundice and hyperbilirubinemia in the newborn. In: Kleigman RM, St. Geme JW, Blum NJ, Shah SS, Tasker RC, Wilson KM, Behrman RE, eds. Nelson Textbook of Pediatrics. 21st ed. Philadelphia, PA: Elsevier; 2020:953-957.

04_Toy-Pediatrics_Case03_p035-044.indd 43

07/07/21 11:36 AM

This page intentionally left blank

CASE 4 A 2800-g male infant is born at 36 weeks of gestation to a 19-year-old mother through vaginal delivery. Delivery occurred 19 hours after membranes ruptured. The mother’s pregnancy was uncomplicated, but her prenatal records are not available at delivery. The mother calls the nurse when the infant is at 6 hours of age because he is “breathing hard” and refusing to breastfeed. His respiratory rate is 60 breaths per minute with grunting. His temperature is 96.5 °F (35.8 °C), and his blood pressure is 42 mm Hg systolic (normally greater than 48 mm Hg). Upon assessment, the infant is in respiratory distress, and his perfusion is poor. A complete blood count (CBC) shows a white blood cell (WBC) count of 2500  cells/mm3 with 80% bands. A radiograph is performed and is shown in Figure 4–1 below.

Figure 4–1.  Chest x-ray. ▶▶ ▶▶

What is the most likely diagnosis? What is the best therapy?

05_Toy-Pediatrics_Case04_p045-054.indd 45

29/07/21 3:48 PM

46

CASE FILES: PEDIATRICS

ANSWERS TO CASE 4: Sepsis and Group B Streptococcal Infections Summary: A 6-hour-old male infant presents with šš

Vaginal delivery at 36 weeks’ gestation

šš

Weight of 2800 g

šš

Poor feeding, tachypnea, hypothermia, and poor perfusion

šš

WBC count of 2500 cells/mm3 with 80% bands

Most likely diagnosis: Early-onset sepsis due to group B Streptococcus (GBS). Best therapy: Intravenous (IV) antibiotics (after addressing the ABCs of resuscitation [airway, breathing, and circulation]).

ANALYSIS Objectives 1. Understand the common presentations of neonatal sepsis. (EPA 1, 2) 2. Understand the maternal risk factors for neonatal GBS infection. (EPA 12) 3. Recognize the variety of organisms responsible for neonatal infections. (EPA 12) 4. Learn treatment options for the common neonatal infections. (EPA 4)

Considerations The rapid symptom onset, the low WBC count with left shift, and the chest x-ray findings are typical for GBS pneumonia. At this point, management would include immediate application of the ABCs of resuscitation, followed by rapid institution of appropriate antibiotics once cultures are obtained. Despite these measures, morbidity and mortality are high. The case-fatality rate is approximately 20% for early-onset GBS preterm infants and about 1% to 3% for full-term neonates. The case-fatality rate for late-onset GBS disease is about 5%. Greater than 50% of infants at hospital discharge will be neurologically impaired due to involvement of the central nervous system.

05_Toy-Pediatrics_Case04_p045-054.indd 46

29/07/21 3:48 PM

SECTION II: CLINICAL CASES

47

APPROACH TO: Sepsis and Group B Streptococcal Infections DEFINITIONS EARLY-ONSET NEONATAL SEPSIS: Neonatal sepsis occurring in the first 6 days of life. The majority of infections (~85%) occur in the first 24 hours of life, an additional 5% by approximately 48 hours, and the remainder throughout the next 4 days. The infection source usually is microorganism acquisition from the mother’s genitourinary tract during delivery. GROUP B STREPTOCOCCUS (GBS) COLONIZATION: Infection with GBS limited to mucous membrane sites in a healthy adult; the gastrointestinal (GI) tract is the most common colonization reservoir. Approximately 10% to 30% of pregnant women are colonized with GBS. INTRAPARTUM ANTIBIOTIC PROPHYLAXIS: IV penicillin or ampicillin given to the mother during labor to prevent early-onset GBS disease. If the mother has penicillin allergy, cefazolin, clindamycin (if the strain is known to be susceptible), and vancomycin are valid alternatives. LATE-ONSET NEONATAL SEPSIS: Neonatal sepsis occurring between 7 and 89 days of life, typically in the third to the fourth week of life. Transmission is not well understood. Early GBS colonization may play a role, but other theories include contamination of human milk and other environmental exposures such as from caregivers and health care workers.

CLINICAL APPROACH TO SEPSIS Pathophysiology The organisms that commonly cause early-onset sepsis colonize the mother’s genitourinary tract and are acquired transplacentally, from an ascending infection, or as the infant passes through the birth canal. Specific organisms seen in early-onset disease include GBS, Escherichia coli, Haemophilus influenzae, and Listeria monocytogenes. Late-onset disease occurs when the infant becomes infected in the postnatal environment, such as from the skin, respiratory tract, conjunctivae, GI tract, and umbilicus. For the hospitalized infant, bacteria sources include vascular or urinary catheters or contact with health care workers. Organisms that commonly cause late-onset disease include coagulase-negative staphylococci, Staphylococcus aureus, E. coli, Klebsiella sp, Pseudomonas sp, Enterobacter sp, Candida, GBS, Serratia sp, Acinetobacter sp, and anaerobes.

Clinical Presentation of Sepsis The signs and symptoms of neonatal sepsis can be subtle and nonspecific, often overlapping with findings in other conditions, such as respiratory distress syndrome, metabolic disorders, intracranial hemorrhages, and traumatic deliveries. Temperature instability, tachypnea, hypotension, bradycardia, lethargy, irritability, and poor

05_Toy-Pediatrics_Case04_p045-054.indd 47

29/07/21 3:48 PM

48

CASE FILES: PEDIATRICS

feeding are common findings in sepsis and meningitis. Overwhelming shock is manifested as pallor and poor capillary refill. Neurologic findings of impaired level of consciousness, coma, seizures, bulging anterior fontanelle, focal cranial nerve signs, and nuchal rigidity are unusual; however, when they are present, they hint at meningitis, a condition more commonly seen in late-onset disease. Examination findings seen frequently with pneumonia (more commonly seen in early-onset disease) include tachypnea, grunting, nasal flaring, retractions (intercostal, supra- or substernal), decreased breath sounds, and cyanosis.

Evaluation of the Potentially Septic Child Complete Blood Count. Some neonatal sepsis laboratory findings can be nonspecific, including hypoglycemia, metabolic acidosis, and jaundice. The CBC often is used to help guide therapy, although the sensitivity and specificity of this test are low. Evidence of infection on CBC includes the following: šš

Markedly elevated or low WBC counts

šš

Increased neutrophil count or neutropenia (neutropenia is more specific)

šš

šš

Increased immature to total neutrophil (I/T) ratio (equal to or exceeding 0.2 has the best sensitivity on the CBC) Thrombocytopenia with platelet counts less than 50,000/mm3

C-Reactive Protein. The C-reactive protein (CRP; an acute phase protein increased with tissue injury) can be elevated in septic infants; its level can be useful in assessing for neonatal sepsis and guiding the duration of antibiotics. Procalcitonin. Procalcitonin (PCT; the precursor of calcitonin) increases in response to bacterial toxins and elevates early in an infection (starts at 4 hours and peaks at 6-8 hours). It is also helpful to guide the duration of therapy. Cultures. A blood culture is crucial for patients with suspected sepsis and has a sensitivity of 90%. Lumbar punctures, especially for young patients in whom the clinical manifestations might be limited, are also important. The results will be helpful when clinical or laboratory findings are concerning for sepsis, when a patient has a positive blood culture, or when a patient when a patient has clinical worsening despite being on broad antibiotic coverage. Urine cultures usually are included for late-onset disease evaluation. Urinary tract infection is uncommon in the first few days of life, and urinalysis or culture usually is not included in early-onset disease workup. Imaging. Chest radiologic findings include segmental, lobar, or diffuse reticulogranular patterns; the latter are easily confused with respiratory distress syndrome (lack of surfactant) in the premature newborn.

Treatment Initial measures for any septic infant are supportive; symptomatic care includes cardiopulmonary monitoring, as well as prevention and management of hypoglycemia, metabolic acidosis, and fluid and electrolytic abnormalities. Early broad-spectrum antibiotic administration is directed at the most common pathogens previously

05_Toy-Pediatrics_Case04_p045-054.indd 48

29/07/21 3:48 PM

SECTION II: CLINICAL CASES

49

listed, often consisting of a combination of IV aminoglycosides (often gentamicin or tobramycin) and penicillin (usually ampicillin). When GBS is identified by culture as the sole causative organism, antibiotic coverage can be narrowed to penicillin G if the patient has improved and is clinically stable. Antibiotic therapy duration varies depending on the presentation of the sepsis (early vs late onset), admission background (admitted from the community vs hospitalized since birth), other special circumstances (suspicion of meningitis, pneumonia, GI source of infection, or soft tissue/skin/joint/bone infection), and patterns of antibiotic resistance in the community and hospital. Antibiotics can be discontinued after 48 hours if cultures are negative and the infant is clinically well appearing. For infants presenting with convincing signs and symptoms of sepsis, antibiotics may be continued even with negative cultures. Close observation for signs of antibiotic toxicity is important for all infants.

CLINICAL APPROACH TO GROUP B STREPTOCOCCUS Epidemiology Risk Factors. Factors associated with an increased risk for early-onset GBS disease are: šš

Rupture of membranes more than 18 hours before delivery

šš

Chorioamnionitis

šš

Intrapartum temperature greater than 100.4 °F (38 °C)

šš

Previous infant with GBS infection

šš

Mother younger than 20 years

šš

Low birth weight or prematurity (less than 37 weeks of gestation)

Incidence and Screening. The incidence of early-onset GBS infection decreased from 1.7 per 1000 live births in 1993 to 0.23 per 1000 live births by 2015. The decline is largely attributed to the widespread use of GBS risk–reduction guidelines. These guidelines recommend screening women at 35 to 37 weeks of gestation and offering intrapartum antibiotic prophylaxis to those with risk factors or positive GBS cultures at 35 to 37 weeks of gestation. Infants born at less than 35 weeks of gestation or born to women who received inadequate intrapartum prophylaxis sometimes undergo a limited evaluation that might include a CBC and blood culture. Intrapartum antibiotic prophylaxis does not prevent late-onset GBS disease. The association of early antibiotic use with increased risk of late-onset serious bacterial infections remains under study.

Pathophysiology and Clinical Presentation GBS is the most common cause of neonatal sepsis from birth to 3 months of age. Approximately 80% of cases occur as early-onset disease (septicemia, pneumonia, and meningitis) resulting from vertical transmission from mother to infant during labor and delivery.

05_Toy-Pediatrics_Case04_p045-054.indd 49

29/07/21 3:48 PM

50

CASE FILES: PEDIATRICS

Respiratory signs (apnea, grunting respirations, tachypnea, or cyanosis) are the initial clinical findings in more than 80% of neonates, regardless of the site of involvement, whereas hypotension is an initial finding in approximately 25% of cases. Other signs are similar to those associated with the bacterial infections described earlier. Early-Onset GBS. Neonates with GBS meningitis rarely have seizures as a presenting sign, yet 50% develop seizures within 24 hours of infection. The median age at diagnosis of early-onset GBS infection is 13 hours, earlier than for the other bacterial infections described previously. Clinical history and findings suggestive of early-onset GBS disease (rather than of a noninfectious etiology for pulmonary findings) include prolonged rupture of membranes, apnea, hypotension in the first 24 hours of life, a 1-minute Apgar score less than 5, and rapid progression of pulmonary disease. Late-Onset GBS. Late-onset GBS disease is often subtler in presentation than early-onset disease. Symptoms often occur between 7 and 30 days of life but can occur up to 3 to 4 months of age. Most commonly, late-onset GBS disease presents as bacteremia without a focus. Meningitis occurs in 20% to 30% of late-onset GBS cases, and the presenting symptom may be seizures. Other manifestations of late-onset GBS disease include focal infections such as pneumonia, septic arthritis, osteomyelitis, or cellulitis. The major risk factor for late-onset GBS disease is prematurity. Diagnostic testing for late-onset GBS disease should include CBC, blood and urine cultures, and cerebrospinal fluid culture if the patient has symptoms concerning for meningitis and for the febrile neonate less than 28 days of life. Mortality from GBS disease is close to 10%. Major neurologic sequelae (cortical blindness, spasticity, and global mental retardation) occur in 12% to 30% of infants who survive meningitis. Treatment is with IV penicillin G.

CASE CORRELATION šš

See also Case 3 (Neonatal Hyperbilirubinemia).

COMPREHENSION QUESTIONS 4.1 An infant was born at 36 weeks of gestation to a 30-year-old gravida 3, para 2 (G3P2) mother via spontaneous vaginal delivery. Rupture of membranes occurred 15 hours prior to delivery. Birth weight is 4000 g, and Apgar scores were 6 and 9 at 1 and 5 minutes, respectively. Which of the following factors places this infant at greatest risk for sepsis? A. Maternal age B. Birth weight C. Apgar score D. Length of time membranes were ruptured E. Gestational age

05_Toy-Pediatrics_Case04_p045-054.indd 50

29/07/21 3:48 PM

SECTION II: CLINICAL CASES

51

4.2 A 25-day-old female infant is brought to the emergency department for fever of 101 °F (38.3 °C) at home. The baby was born vaginally at full term and was appropriate for gestational age. Maternal group B Streptococcus (GBS) was negative. Apgar scores were 8 and 9 at 1 and 5 minutes, respectively. The mother noticed the baby has had decreased feeding over the previous few days and has been sleeping more. Which of the following is the most appropriate initial choice of antibiotics for this infant? A. Oral amoxicillin B. Intravenous vancomycin C. Intravenous ampicillin D. Intravenous ampicillin and cefotaxime E. Intravenous ampicillin and gentamicin 4.3 You are called by the nurse to the intermediate newborn nursery to evaluate a term infant who is having some respiratory issues. The infant is 12 hours old and the product of a normal vaginal delivery. He is noted to be feeding poorly and becomes tachypneic with grunting during feeds. Which of the following initial tests has the lowest diagnostic yield? A. Chest radiograph B. CBC C. Urine culture D. Blood culture E. Glucose level 4.4 A 7-day-old infant is seen in the emergency department for fever and poor feeding. The baby was delivered vaginally 2 hours after the mother arrived at the hospital. The delivery was at 36 weeks of gestation, and the birth weight was 2900 g. Maternal laboratory test results were negative. The most likely organism causing this patient’s symptoms is: A. Group B Streptococcus B. Listeria monocytogenes C. Staphylococcus aureus D. Streptococcus pneumoniae E. Haemophilus influenzae

ANSWERS 4.1 E. Gestational age. Prematurity places this baby at greater risk for sepsis. Young maternal age (answer A), low birth weight (answer B), rupture of membranes greater than 18 hours (answer D), initial Apgar score less than 5 (answer C), and maternal fever are additional risk factors for sepsis. 4.2 D. Ampicillin and cefotaxime. This patient may have late-onset bacterial infection, likely GBS but also possibly coagulase negative Staphylococcus or E. coli;

05_Toy-Pediatrics_Case04_p045-054.indd 51

29/07/21 3:48 PM

52

CASE FILES: PEDIATRICS

she should be admitted for sepsis evaluation and IV antibiotics. The best initial treatment in this age group is broad-spectrum antibiotics such as ampicillin and cefotaxime. If cultures are positive for GBS, antibiotic therapy can be narrowed to penicillin G. Oral amoxicillin (answer A) is inadequate therapy for neonatal sepsis. Vancomycin (answer B) would cover gram-positive organisms but miss possible gram-negative bacteria. Ampicillin and gentamicin (answer E) are appropriate antibiotic coverage for early-onset neonatal sepsis but usually inadequate for late-onset infection. 4.3 C. Urine culture. Urine cultures are not usually obtained in the workup of early-onset sepsis. Urinary tract infections are rare in the first few days of life. The other tests (answer A, chest radiograph; answer B, CBC; answer D, blood culture, and answer E, glucose level) are high yield and often performed in the sepsis workup. 4.4 A. GBS. GBS is the most common pathogen to cause neonatal sepsis in infants aged 0 to 3 months, although in some settings, E. coli is more commonly isolated. The other organisms listed (answer B, Listeria monocytogenes; answer C, Staphylococcus aureus; answer D, Streptococcus pneumoniae; and answer E, Haemophilus influenzae) are not as frequently responsible for neonatal sepsis.

CLINICAL PEARLS »»

Sepsis in the neonate can present with nonspecific findings of temperature instability, tachypnea, poor feeding, bradycardia, hypotension, and hypoglycemia.

»»

Early-onset neonatal infection (occurring in the first 6 days of life) usually is caused by organisms of the maternal genitourinary system, including GBS, E. coli, H. influenzae, and L. monocytogenes. Pneumonia and sepsis are common presentations; GBS is the leading cause.

»»

Late-onset neonatal infection (occurring between 7 and 90 days of life) is often caused by organisms found in the infant’s environment, including coagulase-negative staphylococci, S. aureus, E. coli, Klebsiella sp, Pseudomonas sp, Enterobacter sp, Candida, GBS, Serratia sp, Acinetobacter sp, and anaerobic bacteria.

»»

Treatment of early-onset neonatal infection usually includes penicillin (such as ampicillin) and an aminoglycoside, whereas the treatment of late-onset disease consists of a beta-lactamase–resistant antibiotic (such as vancomycin) and often a third-generation cephalosporin.

»»

The incidence of early-onset GBS infection is decreasing, likely because of the widespread implementation of GBS risk–reduction guidelines.

»»

One of the signs of infection in the newborn population is hyperbilirubinemia, along with other findings such as temperature instability, poor feeding, and lethargy.

05_Toy-Pediatrics_Case04_p045-054.indd 52

29/07/21 3:48 PM

SECTION II: CLINICAL CASES

53

REFERENCES American College of Obstetricians and Gynecologists. Prevention of group B streptococcal early-onset disease in newborns. ACOG Committee Opinion No. 797. Obstet Gynecol. 2020;135:e51-e72. Edwards MS. Group B streptococcal infections. In: Kline MW, Blaney SM, Giardino AP, Orange JS, Penny DJ, Schutze GE, Shekerdemian LS, eds. Rudolph’s Pediatrics. 23rd ed. New York, NY: McGraw Hill; 2018:1296-1298. Gaensbauer J, Nomura Y, Ogle JW, Anderson MS. Group B streptococcal infections. In: Hay WW Jr, Levin MJ, Deterding RR, Abzug MJ, eds. Current Diagnosis & Treatment: Pediatrics. 24th ed. New York, NY: McGraw Hill; 2018. Guinn AG, Gao ZW. Red eye. In: Kline MW, Blaney SM, Giardino AP, Orange JS, Penny DJ, Schutze GE, Shekerdemian LS, eds. Rudolph’s Pediatrics. 23rd ed. New York, NY: McGraw Hill; 2018:2782-2787. Hayes EV. Human immunodeficiency virus and acquired immunodeficiency syndrome. In: Kleigman RM, St. Geme JW, Blum NJ, Shah SS, Tasker RC, Wilson KM, Behrman RE, eds. Nelson Textbook of Pediatrics. 21st ed. Philadelphia, PA: Elsevier; 2020:1778-1805. Jung J, Braverman R. Ophthalmia neonatorum. In: Hay WW Jr, Levin MJ, Deterding RR, Abzug MJ, eds. Current Diagnosis & Treatment: Pediatrics. 24th ed. New York, NY: McGraw Hill; 2018. Lachenauer CS, Wessels MR. Group B Streptococcus. In: Kleigman RM, St. Geme JW, Blum NJ, Shah SS, Tasker RC, Wilson KM, Behrman RE, eds. Nelson Textbook of Pediatrics. 21st ed. Philadelphia, PA: Elsevier; 2020:1450-1455. Libster R, Edwards KM, Levent F, et al. Long-term outcomes of group B streptococcal meningitis. Pediatrics. 2012;130(1):e8-e15. McFarland EJ. Human immunodeficiency virus infection. In: Hay WW Jr, Levin MJ, Deterding RR, Abzug MJ, eds. Current Diagnosis & Treatment: Pediatrics. 24th ed. New York, NY: McGraw Hill; 2018. Olitsky SE, Marsh JD. Disorders of the conjunctiva. In: Kleigman RM, St. Geme JW, Blum NJ, Shah SS, Tasker RC, Wilson KM, Behrman RE, eds. Nelson Textbook of Pediatrics. 21st ed. Philadelphia, PA: Elsevier; 2020:3364-3368. Puopolo KM, Lynfield R, Cummings JJ. Management of infants at risk for group B streptococcal Disease. Committee on Fetus and Newborn, Committee on Infectious Diseases. Pediatrics. 2019;144(2):e20191881. Smith D, Grover T. The newborn infant. In: Hay WW Jr, Levin MJ, Deterding RR, Abzug MJ, eds. Current Diagnosis & Treatment: Pediatrics. 24th ed. New York, NY: McGraw Hill; 2018. Traboulsi EI. Ophthalmia neonatorum. In: McMillan JA, Feigin RD, DeAngelis CD, Jones MD, eds. Oski’s Pediatrics: Principles and Practice. 4th ed. Philadelphia, PA: Lippincott Williams & Wilkins; 2006:811-812.

05_Toy-Pediatrics_Case04_p045-054.indd 53

29/07/21 3:48 PM

This page intentionally left blank

CASE 5 A 4-year-old boy is brought to the emergency department for acute mental status change. His parents report that their son was sleepy after spending a busy day at his grandparents, and when they arrived home, he did not wake or respond when they removed him from his car seat. He has had no significant illnesses and takes no medications. Physical examination demonstrates a well-developed, well-nourished sleeping 4-year-old who is difficult to arouse. His temperature is 98.6 °F (37 °C), heart rate is 60 beats per minute, respiratory rate is 15 breaths per minute, and blood pressure is 72/57 mm Hg. He has no bony deformity, bruising, or rashes; his pupils are constricted. Emergent computed tomographic (CT) scan of the head is normal. The electrocardiogram (ECG) shows normal sinus rhythm with a QTc interval of 476 milliseconds. Upon further questioning, the parents report that the grandparents have provided daycare for the past 2 years and that they are healthy without any significant disease burden. They comment that the grandfather did recently have a total hip replacement due to degenerative changes. The grandparents report the child last ate 1 hour before they picked him up, having had a meal of chicken nuggets and rice. ▶▶ ▶▶ ▶▶

What is the most likely diagnosis? What is the best treatment? What are the possible complications of this condition?

06_Toy-Pediatrics_Case05_p055-060.indd 55

12/03/21 6:11 PM

56

CASE FILES: PEDIATRICS

ANSWERS TO CASE 5: Accidental Ingestion of Opioids Summary: A 4-year-old boy presents with šš

An acute change in mental status

šš

Depressed respiratory drive and being difficult to arouse

šš

Miosis and bradycardia

šš

ECG showing mildly prolonged QTc interval

šš

A grandfather who recently had hip surgery and provides daycare for the child

Most likely diagnosis: Accidental opioid intoxication. Best treatment: Naloxone administration and observation. Possible complications: The most serious immediate complication is related to respiratory depression. Impaired oxygen delivery can lead to ischemic injury to the brain and heart. Opiates also can cause QTc prolongation, which can lead to sudden death.

ANALYSIS Objectives 1. Know the common presenting signs of opioid intoxication. (EPA 1) 2. Understand risk factors for accidental drug ingestion. (EPA 1, 2) 3. Recognize the potential complications and treatment of opioid overdose. (EPA 1-4, 12)

Considerations This 4-year-old child was in his normal state of health when he quickly became lethargic and unarousable. Some possible causes include central nervous system (CNS) or general infection, head trauma, CNS bleed or tumor, prolonged postictal state, and profound electrolyte or metabolic derangement. His lack of fever or other signs of illness, including trauma, suggest broadening the differential diagnosis. Accidental ingestion in a toddler in such a situation is high on the differential. When accidental drug ingestion is suspected, child abuse or neglect must be considered. To prevent future accidents, safety measures (eg, childproof medication bottles, keeping all medications locked and out of reach of children) should be reviewed with the family.

06_Toy-Pediatrics_Case05_p055-060.indd 56

12/03/21 6:11 PM

SECTION II: CLINICAL CASES

57

APPROACH TO: Accidental Drug Ingestion DEFINITIONS MIOSIS: Constriction of the pupil. OPIATE: Subclass of opioids composed of alkaloid compounds extracted from opium (eg, heroin, morphine, codeine). OPIOID: Natural and synthetic substances (eg, oxycodone, hydrocodone) that share morphine-like activity.

CLINICAL APPROACH Epidemiology Accidental ingestion is the leading cause of injury-related mortality in the United  States. Among children less than 6 years of age, opioid exposure is most commonly accidental, resulting in significant morbidity and mortality. The incidence of emergency department visits for opioid ingestion is epidemic, increasing by 101% from 2001 to 2008. Although hydrocodone accounts for most emergency department visits, methadone and buprenorphine are more likely to result in lifethreatening complications. These more potent formulations can cause severe respiratory depression in children after ingestion of a fraction of a pill. In children younger than 6 years, about 45% of ingestion cases result in hospitalization and 20% of children will have serious medical outcomes. Up to 25% of adolescents report either prescription or recreational opioid exposure; unintentional injury is a common complication in this population. Some studies have shown that up to 10% of adolescent trauma victims in the emergency department had a urine drug screen positive for opioids. Thus, opioid ingestion should be considered in such situations.

Clinical Presentation Children ingest myriad substances. Opioid intoxication results in obtundation, bradycardia, hypotension, miosis, and possibly QTc prolongation. Alcohol ingestion may produce drowsiness, bradycardia, and hypotension but not miosis. Clonidine, an alpha-agonist, causes obtundation, miosis, and bradycardia but not QTc prolongation. Anticholinesterases (ie, organophosphates) produce miosis and altered mental status but also parasympathetic activation (ie, vomiting, sweating, diarrhea). The evaluation of a child with possible ingestion of any kind should include a spot glucose to eliminate severe hypo- or hyperglycemia as a cause of obtundation. Patients with respiratory depression require pulse oximeter monitoring and possibly a blood gas. For intentional or accidental ingestions of unknown etiology, acetaminophen and alcohol levels may be indicated. Urine drug screens are helpful for unknown ingestions. The ECG can evaluate for QTc prolongation.

06_Toy-Pediatrics_Case05_p055-060.indd 57

12/03/21 6:11 PM

58

CASE FILES: PEDIATRICS

Treatment Treatment of opioid and opiate intoxication includes supportive care (including possible bag-mask ventilation or intubation) and the administration of naloxone (Narcan). Naloxone is weight-based in children with repeated administration every 1 to 2 minutes for a maximum dose of 10 mg. For patients who present within 1 hour of a known enteral opioid ingestion, oral or nasogastric administration of activated charcoal will help bind the consumed drug. Hospitalization is appropriate for patients who require naloxone (to monitor for reaction or rebound), for those in whom intentional poisoning is suspected, for those in whom child abuse is considered, and for patients for whom suicide is a possibility.

CASE CORRELATION šš

See Case 38 (Child Abuse).

COMPREHENSION QUESTIONS 5.1 The classic clinical signs of opioid toxicity include depressed mentation, decreased respiratory rate, decreased bowel sounds, miotic pupils, and which of the following pulmonary findings? A. Increased tidal volume B. Decreased tidal volume C. Increased residual volume D. Decreased residual volume 5.2 A 7-year-old boy presents to the emergency department with decreased respirations and is arousable only to painful stimuli. In addition to the urine drug screen, which of the following is the most appropriate next test? A. Hemoglobin A1C B. Thyroid-stimulating hormone (TSH) and free thyroxine (T4) C. Spot glucose check D. Lead level E. Electroencephalogram (EEG) 5.3 A 17-year-old girl is admitted to the hospital after being found unresponsive at a local gas station. Her blood pressure is 100/60 mm Hg, and her heart rate is 62 beats per minute. After initial evaluation including stabilizing for airway, breathing, and circulation concerns, which of the following should be given? A. 1-L bolus of normal saline B. Oxygen, nitrate, aspirin, beta-blocker C. Naloxone D. Intramuscular (IM) glucagon E. Intravenous (IV) norepinephrine

06_Toy-Pediatrics_Case05_p055-060.indd 58

12/03/21 6:11 PM

SECTION II: CLINICAL CASES

59

5.4 A 6-year-old boy is brought to the emergency department after having been found by the baby sitter with a half-empty bottle of his father’s hydrocodone/ paracetamol. The emergency medical services (EMS) administered naloxone en route. One hour later, the child is awake and responsive. Which of the following is the next best step? A. Administer lorazepam B. Perform a spot glucose check C. Order an EEG D. Draw an acetaminophen level E. Perform a lumbar puncture

ANSWERS 5.1 B. Decreased tidal volume. In an acute opioid overdose, tidal volume is decreased (not increased, as in answer A) due to depressed respiratory drive. The residual volume (answers C and D) does not change with opioid toxicity. 5.2 C. Spot glucose check. Severe hypoglycemia is a common cause of altered mental status that is easily corrected; however, if uncorrected, it can lead to serious organ dysfunction. Evaluation for this condition is indicated before moving to other diagnostic possibilities. A hemoglobin A1C (answer A) only provides an average glucose over the past 90 days and would not provide information for immediate intervention. Alterations in the TSH level (answer B) would not lead to the patient’s current state of altered mental status. A toxic lead level (answer D) likely would present as more chronic cognitive deficits and anemia. An EEG (answer E) would not be helpful in the acute management of this patient; there was no history of seizures. 5.3 C. Naloxone. Acute opioid overdose must be considered in the differential diagnosis of a patient found unresponsive. Naloxone should be administered following initial assessment and stabilization. The bolus of normal saline (answer A) would be indicated if the patient were hypotensive due to volume depletion. Oxygen, nitrate, aspirin, and beta-blocker (answer B) are given for acute coronary syndrome. IM glucagon (answer D) is given for suspected hypoglycemia. IV norepinephrine (answer E) is given for hypotension caused by vascular relaxation (such as sepsis). 5.4 D. Draw an acetaminophen level. The patient ingested not only an opioid, but also an acetaminophen-containing compound. Acetaminophen toxicity with necessary treatment must be considered. Lorazepam (answer A) is not necessary to administer because there is no seizure activity. A glucose level (answer B) is a reasonable lab test, but because the child is alert and awake, it will likely be of low yield and is less important than the acetaminophen level. An EEG (answer C) is not needed because the child did not have altered mental status or seizures. A lumbar puncture (answer E) is not needed because there is no sign of meningismus or fever.

06_Toy-Pediatrics_Case05_p055-060.indd 59

12/03/21 6:11 PM

60

CASE FILES: PEDIATRICS

CLINICAL PEARLS »»

The classic triad of opioid toxicity is miotic pupils, respiratory depression, and altered mental status.

»»

Respiratory depression is the most life-threatening complication of opioid toxicity.

»»

Opioids can prolong the QT interval, and an ECG may be helpful in suspected cases.

»»

Emergent naloxone is the treatment of choice if opioid intoxication is suspected.

»»

When accidental drug ingestion is suspected, child abuse or neglect must be considered.

REFERENCES Allen JD, Casavant MJ, Spiller HA, et al. Prescription opioid exposures among children and adolescents in the United States: 2000-2015. Pediatrics. 2017;139:e20163382. American Academy of Pediatrics. Acetaminophen toxicity in children. Pediatrics. 2001;108:1020-1024. Basco WT, Garner SS, Ebeling M, Hulsey TC, Simpson K. Potential acetaminophen and opioid overdoses in young children prescribed combination acetaminophen/opioid preparations. Pediatr Qual Saf. 2016;1:e007. Chamberlain JM, Klein BL. A comprehensive review of naloxone for the emergency physician. Am J Emerg Med. 1994;12:650-660. Gugelmann HM, Nelson LS. The prescription opioid epidemic: repercussions on pediatric emergency medicine. Clin Pediatr Emerg Med. 2012;13:260-268. Klein-Schwartz W. Trends and toxic effects from pediatric clonidine exposures. Arch Pediatr Adolesc Med. 2002;156:392-396. Loiselle JM, Baker MD, Templeton JM, Schwartz G, Drott H. Substance abuse in adolescent trauma. Ann Emerg Med. 1993;22:1530-1534. Martell BA, Arnsten JH, Krantz MJ, Gourevitch MN. Impact of methadone treatment on cardiac repolarization and conduction in opioid users. Am J Cardiol. 2005;95:915-918. Post S, Spiller HA, Casavant MJ, Chounthirath T, Smith GA. Buprenorphine exposures among children and adolescents reported to US poison control centers. Pediatrics. 2018;142:e20173652. Sachdeva DK, Stadnyk JM. Are one or two dangerous? Opioid exposure in toddlers. J Emerg Med. 2005;29:77-84. Salvucci A, Koenig K, Gausche-Hill M, et al. Altered mental status: current evidence-based recommendations for prehospital care. West J Emerg Med. 2018;19:527-541. Sidell FR. Clinical effects of organophosphorus cholinesterase inhibitors. J Appl Toxicol. 1994;14:111-113. Swartz MK. Opioids: a pediatric epidemic. J Pediatr Health Care. 2018;32:115-116.

06_Toy-Pediatrics_Case05_p055-060.indd 60

12/03/21 6:11 PM

CASE 6 A 6-day-old girl is brought to the emergency department (ED) by her mother for a 12-hour history of irritability, decreased oral intake, and fever of 100.4 °F (38 °C). She was delivered vaginally at 39 weeks’ gestation to a gravida 2, para 1 (G2P1) woman after an uncomplicated pregnancy with routine prenatal care. The mother denies any past medical history, illness, or infections during pregnancy; her only medications were prenatal vitamins. On examination, the infant has a temperature of 100.2 °F (37.9 °C), heart rate of 145 beats per minute, and respiratory rate of 64 breaths per minute. She is fussy with any movement despite being swaddled. The only findings on physical examination are about 10 small, 2-mm, fluid-filled lesions surrounded by an erythematous base on her right shoulder (Figure 6–1). Shortly after her blood, urine, and cerebrospinal fluid (CSF) studies are obtained, she exhibits shaking of the right side of her body that then generalizes. The episode lasts approximately 20 seconds, and afterward, she is somnolent. Lumbar puncture results show 850 white blood cells (WBCs) with 90% lymphocytes, 80 red blood cells (RBCs), and a protein of 200 mg/dL. Computed tomography (CT) of the head is normal. Her complete blood count (CBC) reveals a platelet count of 57,000/mm3, alanine transaminase (ALT) of 112 U/L, and aspartame transaminase (AST) of 108 U/L. Her chest x-ray has patchy bilateral ground-glass opacities.

07_Toy-Pediatrics_Case06_p061-070.indd 61

07/07/21 11:51 AM

62

CASE FILES: PEDIATRICS

Figure 6–1.  Vesicular eruption. (Reproduced with permission, from Wolff K, Johnson RA, Saavedra AP. Fitzpatrick’s Color Atlas and Synopsis of Clinical Dermatology. 7th ed. 2013. Copyright © McGraw Hill LLC. All rights reserved.)

▶▶ ▶▶

What is the most likely diagnosis? What is the next step in management?

07_Toy-Pediatrics_Case06_p061-070.indd 62

07/07/21 11:51 AM

SECTION II: CLINICAL CASES

63

ANSWERS TO CASE 6: Neonatal Herpes Simplex Virus Infection Summary: A 6-day-old previously healthy female infant presents with šš

Fever, irritability, decreased oral intake, and vesicles on her shoulder

šš

An episode of seizure-like activity

šš

Laboratory and radiologic studies revealing lymphocytic meningitis, thrombocytopenia, transaminitis, and pneumonitis

Most likely diagnosis: Neonatal herpes simplex virus (HSV) infection, disseminated. Next step in management: High-dose intravenous acyclovir along with empiric antibiotics must be administered until more definitive results are available. Even with early antiviral treatment, mortality is 20% in neonates with disseminated HSV disease.

ANALYSIS Objectives 1. Recognize the different presentations of neonatal herpes infection including its role as a “TORCH” (toxoplasmosis, other, rubella, cytomegalovirus [CMV], and HSV or human immunodeficiency virus [HIV]) infection. (EPA 1, 2) 2. Know how to diagnose neonatal herpes infection. (EPA 1-3) 3. Know the appropriate management of neonatal herpes infection. (EPA 4, 10, 12)

Considerations A young infant with fever and irritability is presumed to have a serious bacterial or viral infection. Bacterial causes in this age include group B Streptococcus, Listeria, and gram-negative pathogens such as Escherichia coli. The history in this patient of a fever, a focal seizure, the finding of vesicles on the infant’s shoulder, and the laboratory and radiologic abnormalities described make HSV the most likely pathogen. The absence of a maternal history of herpes is not unusual; only 15% to 20% of mothers of HSV-infected infants have a history of herpes, and only approximately 25% of these mothers have symptoms at delivery. The risk of maternal passage of HSV to the neonate is 25% to 60% in cases of primary herpes infection because the viral inoculum in the genital tract is high and protective antibody is not present. Conversely, the risk of transmission is less than 2% if the mother is having a reactivation of infection at the time of delivery. When a bacterial infection is suspected, blood, urine, and CSF specimens should be obtained for routine cultures. Investigation for an inborn error of metabolism would be needed if this patient did not have fever, vesicles, or meningoencephalitis. Workup for suspected neonatal HSV infection includes HSV surface cultures or polymerase chain reaction (PCR) obtained from the conjunctiva, nasopharynx, mouth, rectum, and any vesicular lesion. CSF and blood are tested by PCR for HSV DNA. A CBC, metabolic panel, and coagulation studies may reveal abnormalities

07_Toy-Pediatrics_Case06_p061-070.indd 63

07/07/21 11:51 AM

64

CASE FILES: PEDIATRICS

such as cytopenias, transaminitis, and disseminated intravascular coagulation (DIC). An electroencephalogram (EEG) is indicated because of the seizure. In HSV infection, the EEG is often abnormal early in the disease course; neuroimaging studies may not show abnormalities for several days.

APPROACH TO: Neonatal Herpes Simplex Virus Infection DEFINITIONS NEONATE: Infant who is 30 days old or younger. PRIMARY HERPES INFECTION: HSV infection in a previously seronegative host. Most primary infections are subclinical, but they can cause localized lesions or severe systemic symptoms. TORCH: An acronym to recall several of the intrauterine infections associated with birth anomalies. T = toxoplasmosis, O = other (syphilis, varicella), R = rubella, C = cytomegalovirus, H = herpes simplex virus and HIV. VESICLE: A fluid-filled elevation in the epidermis that measures less than 1 cm.

CLINICAL APPROACH Epidemiology Approximately 20% to 30% of American women of childbearing age have antibodies to HSV-2, with a higher rate in women of lower socioeconomic groups and those in crowded living conditions. The incidence of neonatal HSV infection in the United States is between 1 in 2000 and 1 in 3000 live births. Approximately 75% of neonatal herpes cases are caused by HSV-2, and HSV-2 is associated with greater morbidity among survivors than HSV-1. Neonatal infection is most commonly acquired during delivery but can also be acquired postnatally from an infected caregiver’s mouth or hand. Intrauterine infection is rare, but an affected infant would be expected to be born with skin vesicles (or their scars), chorioretinitis, and microcephaly.

Differential Diagnosis Other congenital infections have symptoms that overlap with those of HSV, but none will also exhibit vesicles. Toxoplasmosis may be characterized by chorioretinitis, seizure, CSF pleocytosis, and thrombocytopenia, but the CT scan will show diffuse intracranial calcifications with a predilection for the basal ganglia and obstructive hydrocephalus. Congenital rubella can also present with meningoencephalitis, microcephaly, seizure, and thrombocytopenia, but this disease process would also be characterized by cataracts and a purpuric rash (blue-gray nodules known as “blueberry muffin rash”). Congenital CMV may exhibit meningoencephalitis, chorioretinitis, microcephaly, seizure, pneumonitis, transaminitis, and

07_Toy-Pediatrics_Case06_p061-070.indd 64

07/07/21 11:51 AM

SECTION II: CLINICAL CASES

65

thrombocytopenia, but CT would demonstrate periventricular calcifications and the blueberry muffin rash. Congenital syphilis may have symptoms of chorioretinitis, aseptic meningitis, pneumonitis, transaminitis, thrombocytopenia, and fever; the rash is characteristically maculopapular.

Clinical Presentation The three presentations of neonatal HSV disease are (1) localized skin, eye, and mouth (SEM) involvement; (2) central nervous system (CNS) disease; or (3) disseminated disease. SEM.  SEM usually appears at 1 to 2 weeks of life; it requires intravenous acyclovir to prevent progression to one of the other presentations. Lumbar puncture for cell count and HSV PCR must be done at the time of diagnosis to exclude the presence of CNS disease. CNS.  CNS disease typically manifests at 2 to 3 weeks of life. Fever occurs in less than half of infants with CNS disease, and only 60% of cases will have vesicles. The infant will be lethargic, irritable, or have seizures. Recognition of the symptoms and laboratory findings is important; without treatment, 50% of neonates will die. Disseminated.  Disseminated disease has multiple signs and symptoms in the 1to 2-week-old neonate: fever, lethargy, irritability, apnea, a bulging fontanelle, or seizures (focal or generalized). Skin vesicles will be present in approximately twothirds of cases. Hepatitis, pneumonitis, shock, and DIC occur in severe cases. An ophthalmologic examination is necessary with any form of HSV infection to detect chorioretinitis.

Diagnostic Tests Viral surface cultures swabbed from conjunctivae, mouth, nasopharynx, and anus, in addition to PCR of CSF, are the most useful diagnostic tests. Any vesicle fluid should be sent for PCR testing; blood should also be sent for PCR. Serologic tests for herpes virus are not helpful in the acute setting because titers rise late in the infection’s course. Tzanck preparation of lesions and antigen detection methods applied to the specimens can aid in rapid diagnosis, but the sensitivity is low. Infected individuals often have moderate peripheral leukocytosis, elevated serum transaminase levels, and thrombocytopenia. When the CNS is involved, the CSF may contain an elevated number of RBCs, lymphocytes, and protein; CSF glucose usually is normal but may be reduced. EEG shows characteristic patterns in acutely affected infants. Magnetic resonance imaging (MRI) and CT of the brain will become abnormal as the disease progresses, with MRI findings usually appearing before CT findings.

Treatment Prevention.  HSV infection most commonly occurs during delivery, so if the woman has an outbreak of visible lesions or any symptoms consistent with HSV (paresthesia in a dermatomal pattern), then cesarean delivery is indicated. HSV surveillance

07_Toy-Pediatrics_Case06_p061-070.indd 65

07/07/21 11:51 AM

66

CASE FILES: PEDIATRICS

cultures are not recommended in pregnant women because those at greatest risk for infecting their infants do not have a history of prior infection. Suppressive antiviral therapy starting at 36 weeks of gestation can reduce how many symptomatic recurrences the mother has, but it does not completely eliminate viral shedding and does not prevent neonatal infection. Pharmacologic Therapy.  Parenteral acyclovir is the preferred treatment for any neonatal HSV infection. It can stop the viral replication at the site of inoculation (skin, mouth, nares, eyes). Without treatment, HSV can spread in the neonate to the respiratory tract, down neurons, or enter the bloodstream, allowing hematogenous infection of the liver, adrenals, and CNS. Treatment is for 14 days in SEM disease but a minimum of 21 days with CNS or disseminated disease. Ophthalmologic involvement warrants the use of topical ophthalmologic drops in addition to parenteral treatment. About 50% of neonates with HSV infection will have skin recurrences over the subsequent 6 months and require daily suppressive acyclovir.

CASE CORRELATION šš

See Case 3 (Neonatal Hyperbilirubinemia) and Case 4 (Sepsis and Group B Streptococcal Infections).

COMPREHENSION QUESTIONS 6.1 A 10-day-old infant presents to clinic with a painful, red vesicular rash in the diaper area. He is mildly fussy but afebrile, and he has good oral intake. Which of the following is the most appropriate management of this infant? A. Hospitalize the patient, obtain HSV surface and vesicle fluid cultures and CSF for HSV culture and PCR, and initiate intravenous acyclovir B. Order an EEG and brain MRI immediately C. Perform a Tzanck smear and send the patient home if it is negative D. Prescribe an antifungal cream and follow up by telephone in 24 hours E. Schedule an appointment with a pediatric dermatologist 6.2 A woman presents for her first prenatal visit at 9 weeks of gestation. She reports that she is generally healthy, except that she has an outbreak of genital herpes approximately once per year. To prevent transmission of the virus to her infant, her provider should do which of following? A. Prescribe her daily acyclovir B. Order titers to determine if the infection is HSV-1 or HSV-2 C. Perform weekly genital viral cultures starting at 36 weeks’ gestation D. Perform a cesarean delivery if herpetic lesions or prodromal symptoms are present when labor has begun E. No change in management is indicated; the risk of infant transmission is low even if she has an outbreak at delivery

07_Toy-Pediatrics_Case06_p061-070.indd 66

07/07/21 11:51 AM

SECTION II: CLINICAL CASES

67

6.3 An 18-day-old male infant is brought to the ED by his mother with an 8-hour history of poor feeding and decreased wet diapers. On examination, he is afebrile; he cries during venipuncture, urine catheterization, and lumbar puncture but otherwise sleeps through the examination. The remainder of his examination, including the skin, is normal. His CSF shows 78 WBCs, 80% lymphocytes, 40 RBCs, protein of 130 mg/dL, and no bacteria on Gram stain. The mother has a cold sore at her lower lip. Which of the following would be least useful for your management? A. Consult ophthalmology to assess for chorioretinitis B. C. D. E.

Evaluate HSV IgG and IgM titers Perform a surface swab for HSV PCR Order a CBC, ALT, and AST Evaluate CSF for HSV PCR

6.4 A male infant at 38 weeks’ gestation is precipitously delivered vaginally in the ED. Weight and head circumference are less than fifth percentile, and length is at the 10th percentile. He has hepatosplenomegaly and a petechial rash. Notable laboratory results include platelets of 22,000/mm3 and elevated total and direct bilirubin levels. CT of the head shows bilateral intracranial calcifications with several around the basal ganglia and obstructive hydrocephalus. Ophthalmologic examination reveals chorioretinitis. What is the most likely cause of the patient’s findings? A. Cytomegalovirus B. Rubella virus C. Toxoplasma gondii D. HSV E. Treponema pallidum

ANSWERS 6.1 A. Hospitalize the patient, obtain HSV surface and vesicle fluid cultures and CSF for HSV culture and PCR, and initiate intravenous acyclovir. In contrast to children and adults, neonates with suspected herpes skin lesions require parenteral antiviral therapy to prevent more serious sequelae, as well as CSF analysis to define the extent of disease. HSV PCR of the CSF is the standard for diagnosis. Tzanck smears (answer C) have low sensitivity, and EEG (answer B) may have nonspecific findings. The absence of fever does not indicate that the rash is from a benign etiology (answer D). A scheduled dermatology appointment (answer E) would delay diagnosis and treatment. 6.2 D. Perform a cesarean delivery if herpetic lesions or prodromal symptoms are present when labor has begun. Even though the viral transmission risk in the setting of a recurrent HSV outbreak is low (answer E), cesarean section is indicated if lesions are present at the time of delivery because of the severity of

07_Toy-Pediatrics_Case06_p061-070.indd 67

07/07/21 11:51 AM

68

CASE FILES: PEDIATRICS

neonatal HSV disease. Surveillance cultures (answer C) are not recommended; negative results a few days prior to delivery do not preclude a later outbreak, and results of analysis of a more recently obtained specimen may not be available. Either type 1 or type 2 HSV can cause neonatal infection and disease (answer B). Daily antiviral therapy starting at 9 weeks’ gestation (answer A) is not indicated; the American College of Obstetricians and Gynecologists (ACOG) recommends suppressive antiviral therapy be started at 36  weeks’ gestation in mothers with a history of recurrent HSV to decrease the chance of a lesion at delivery, which would necessitate a cesarean delivery. 6.3 B. Evaluate HSV IgG and IgM titers. Serology is not useful for the diagnosis of neonatal herpes infection. Neonatal herpes infection in any presentation should prompt an ophthalmology examination (answer A) because topical ophthalmic antiviral therapy will be needed in addition to intravenous acyclovir if chorioretinitis or keratitis is present. A surface swab (answer C) and CSF for HSV PCR (answer E) are key steps in diagnosing neonatal HSV infection. Abnormalities in the CBC, ALT, or AST (answer D) would suggest that he has disseminated disease. 6.4 C. Toxoplasma gondii. The triad of hydrocephalus, intracranial calcifications, and chorioretinitis is a classic presentation for congenital toxoplasmosis. With cytomegalovirus (answer A), the intracranial calcifications would be in a periventricular distribution, and the typical skin finding would be the blueberry muffin rash. Rubella (answer  B) would also be expected to present with the purpuric rash, along with cataracts and no CT findings. Herpes (answer D) would have either vesicles or scarring. Syphilis infection (answer E) would not have an abnormal CT.

CLINICAL PEARLS »»

Most infants with neonatal HSV are born to mothers without a prior history of HSV infection.

»»

The presenting signs and symptoms of neonatal HSV may be nonspecific, without any visible herpetic lesions.

»»

Neonates with suspected HSV infection should be hospitalized for testing and parenteral antiviral therapy pending test results.

»»

Neonates with HSV skin, eye, and mouth (SEM) disease generally have the best outcomes, whereas the majority of infants with CNS disease develop neurologic sequelae.

»»

Approximately 30% of infants with systemic infection die despite aggressive antiviral therapy.

07_Toy-Pediatrics_Case06_p061-070.indd 68

07/07/21 11:51 AM

SECTION II: CLINICAL CASES

69

REFERENCES ACOG Committee on Practice Bulletins. Management of herpes in pregnancy. ACOG Practice Bulletin. Clinical management guidelines for obstetrician-gynecologists. Obstet Gynecol. 2007;109(6):1489-1498. American Academy of Pediatrics. Herpes simplex. In: Kimberlin DW, Brady MT, Jackson, MT, and Long SS, eds. Red Book: 2018 Report of the Committee on Infectious Diseases. 31st ed. Elk Grove Village, IL: American Academy of Pediatrics; 2018:437-449. Genital HSV infection. In: Sexually Transmitted Diseases Treatment Guidelines. MMWR Recomm Rep. 2015;64(3):27-32. Poole CL, Kimberlin DW. Herpes simplex virus infections. In: Kline MW, Blaney SM, Giardino AP, et al, eds. Rudolph’s Pediatrics. 23rd ed. New York, NY: McGraw Hill; 2018:1366-1370. Stanberry LR. Herpes simplex virus. In: Kleigman RM, St. Geme JW, Blum NJ, et al, eds. Nelson Textbook of Pediatrics. 21st ed. Philadelphia, PA: Elsevier; 2020:1701-1708.

07_Toy-Pediatrics_Case06_p061-070.indd 69

07/07/21 11:51 AM

This page intentionally left blank

CASE 7 A term 3700-g male infant is born vaginally to a 27-year-old gravida 2, para 2 mother following an uncomplicated pregnancy. Shortly after birth, the infant begins to cough; this is followed by a choking episode, difficulty handling secretions, and cyanosis. During the resuscitation, placement of an orogastric tube meets resistance at 10 cm. He is transferred to the level II nursery for evaluation and management of respiratory distress. ▶▶ ▶▶

What is the most likely diagnosis? What is the best test for evaluation?

08_Toy-Pediatrics_Case07_p071-078.indd 71

29/07/21 3:54 PM

72

CASE FILES: PEDIATRICS

ANSWERS TO CASE 7: Esophageal Atresia Summary: A male newborn presents with šš

Vaginal delivery after uncomplicated pregnancy

šš

Cough, choking, and cyanosis shortly after birth

šš

Inability to undergo passage of an orogastric tube

Most likely diagnosis: Esophageal atresia, probably with a tracheoesophageal fistula (TEF). Best test for evaluation: A chest and abdomen radiograph will most commonly show the orogastric tube coiled in the esophageal blind pouch with or without air in the stomach.

ANALYSIS Objectives 1. Describe the clinical presentation of TEF. (EPA 1) 2. Describe the anatomic variants of TEF. (EPA 1) 3. Identify and apply the principles of emergency management of newborns with TEF. (EPA 1-4, 10)

Considerations In this newborn with choking and coughing, esophageal atresia is suspected when failure to pass the orogastric tube is noted. Infants with esophageal atresia frequently have excessive oral secretions and coughing from pooled secretions. They will often require constant esophageal pouch drainage to prevent aspiration. They are monitored in the neonatal intensive care unit (NICU) while awaiting surgical intervention.

APPROACH TO: Esophageal Atresia DEFINITIONS ASSOCIATION: Sporadic occurrence of two or more clinical features occurring together more commonly than would be expected, but without an identifiable cause. POLYHYDRAMNIOS: Diagnosis of an increased amount of amniotic fluid. SYNDROME: A constellation of features having a common cause (such as the features of Down syndrome being caused by trisomy 21).

08_Toy-Pediatrics_Case07_p071-078.indd 72

29/07/21 3:54 PM

SECTION II: CLINICAL CASES

73

CLINICAL APPROACH Epidemiology and Pathophysiology Esophageal atresia occurs in 1 in 2500 to 3000 live births, usually accompanied by TEF. Males have a slight predominance. Most are sporadic events and multifactorial in etiology; however, there is a higher association with aneuploidy (trisomy 18) and various genetic disorders. Prenatal ultrasound findings of polyhydramnios, absence of a fluid-filled stomach, and a distended esophageal pouch are nonspecific findings suggestive of esophageal atresia. Five different TEF anatomic variants occur; the most common (87%) includes proximal atresia (esophageal pouch) with a distal fistula (Figure 7–1A). A

B

C

Figure 7–1.  Types of esophageal atresia/tracheoesophageal fistula. A.  Proximal esophageal atresia with distal fistula (80%-90%). B.  Esophageal atresia (10%). C.  H-type tracheoesophageal fistula (3%-4%).

08_Toy-Pediatrics_Case07_p071-078.indd 73

29/07/21 3:54 PM

74

CASE FILES: PEDIATRICS

Clinical Presentation Infants with TEF usually present in the newborn period with excessive oral secretions and coughing, choking, and cyanosis secondary to aspirated secretions or with initial feeds. Infants with the “H-type” fistula (~4% of cases) often present later in life with recurrent aspiration pneumonia or feeding difficulty (Figure 7–1C). Other congenital anomalies occur in approximately 30% to 50% of TEF patients, and a search for these anomalies should be undertaken. The most common association is the VACTERL or VATER association (Vertebral abnormality, Anal imperforation, Cardiac abnormalities, TracheoEsophageal fistula, Radial, Renal and Limb anomalies). In addition to esophageal abnormalities, cardiac anomalies are the next most common malformation (~23%) seen with VACTERL or VATER association. Other conditions notable for TEF include CHARGE (Coloboma of the eye, Heart defects, Atresia of the nasal choanae, Retardation of growth and/or development, Genital and/or urinary abnormalities, and Ear abnormalities and deafness) association, DiGeorge syndrome (21q11.2 deletion associated with congenital heart defects, immunodeficiency, facial defects, and cleft palate), and trisomies 18, 21, and 13.

Treatment Neonates with TEF or esophageal atresia are at risk for respiratory compromise due to aspiration; thus, early identification and prevention of aspiration and pneumonia is critical, such as with constant suctioning of the esophageal pouch while awaiting surgery. Surgical repair is the main treatment with two options: primary (one step) vs. staged surgery. Primary surgery consists of ligating the fistula and anastomosis of the esophagus. Staged surgery is required if anatomic conditions preclude primary anastomosis. Postsurgical complications include leak and stenosis of anastomosis, fistula recurrence, and esophageal dysmotility; chronic gastroesophageal reflux is common.

CASE CORRELATION šš

See also Case 8 (Transient Tachypnea of the Newborn), Case 10 (Failure to Thrive), and Case 20 (Asthma Exacerbation).

COMPREHENSION QUESTIONS 7.1 A 2-hour-old term newborn male has coughing, choking, and cyanosis prior to feeding. A nasogastric tube is placed and meets resistance at 10 cm. Prenatal history is significant for polyhydramnios. Which of the following is most likely to be found in this infant? A. Congenital cataracts B. Gingival hyperplasia C. Hepatosplenomegaly D. Microcephaly E. Fusion of two lower thoracic vertebral bodies

08_Toy-Pediatrics_Case07_p071-078.indd 74

29/07/21 3:54 PM

SECTION II: CLINICAL CASES

75

7.2 An 8-month-old infant is hospitalized with his third bout of pneumonia. The patient undergoes bronchoscopy and is diagnosed with a transesophageal fistula (TEF). Which of the following statements is most likely to describe this patient’s condition? A. The infant most likely has an H-type TEF B. The infant most likely has proximal esophageal atresia with distal fistula C. The infant likely has a previously undetected, associated finding of imperforate anus D. The infant is unlikely to have gastroesophageal reflux E. The infant is likely to have cystic fibrosis 7.3 A 2-year-old girl is hospitalized with Pneumocystis carinii pneumonia. She has a history of esophageal atresia and a ventricular septal defect. Her immunodeficiency is likely a result of which of the following? A. Bruton agammaglobulinemia B. Chronic granulomatous disease C. DiGeorge syndrome D. Hyperimmunoglobulin E syndrome E. Severe combined immunodeficiency syndrome 7.4 A 2-year-old boy, living with new foster parents for 3 weeks, has become progressively short of breath. When he first arrived at their home, he was active and playful, but now he is too tired to play. They have few details, but they know that he had neonatal surgery for a problem with his “esophagus being connected to his lungs” and that he takes no medications. On examination, he is afebrile, diaphoretic, tachycardic, and tachypneic. His symptoms can most likely be attributed to which of the following? A. Adjustment disorder B. Heart failure secondary to ventricular septal defect C. Kawasaki disease D. Reactive airway disease E. Rheumatic heart disease

ANSWERS 7.1 E. Fusion of two lower thoracic vertebral bodies. The infant probably has esophageal atresia. VATER or VACTERL association, as described in the case, can have vertebral anomalies such as fused or bifid vertebral bodies. None of the other findings listed (answer A, congenital cataracts; answer B, gingival hyperplasia; answer C, hepatosplenomegaly; and answer D, microcephaly) is commonly associated with VATER or VACTERL. 7.2 A. The infant most likely has an H-type TEF. This infant likely has an H-type TEF, which is usually found later in infancy with recurrent pneumonias

08_Toy-Pediatrics_Case07_p071-078.indd 75

29/07/21 3:54 PM

76

CASE FILES: PEDIATRICS

and/or feeding difficulty. Patients with esophageal atresia and distal fistula (answer  B) are generally identified in the first hours of life because of their inability to swallow oropharyngeal secretions. All patients with TEF are at high risk for gastroesophageal reflux (answer D). Imperforate anus (answer C) would present within the first week of life. Cystic fibrosis (answer E) is a cause of recurrent pneumonia; however, this patient has a reason for recurrent lung infections with esophageal secretions entering into the trachea. 7.3 C. DiGeorge syndrome. DiGeorge syndrome (thymic hypoplasia) results from abnormal third and fourth pharyngeal pouch formation during fetal development and is associated with 22q11.2 deletion. Neighboring structures formed during the same fetal growth period are often affected. Associated conditions include anomalies of the great vessels, esophageal atresia, bifid uvula, congenital heart disease, short philtrum, hypertelorism, antimongoloid slant palpebrae, mandibular hypoplasia, and low-set, notched ears. The thymic hypoplasia leads to T-cell immunodeficiency, which predisposes to Pneumocystis pneumonia. DiGeorge syndrome may present in neonates with hypocalcemic seizures because of parathyroid hypoplasia. Children with Bruton agammaglobulinemia (answer  A) present with recurrent respiratory infections from encapsulated organisms such as Streptococcus pneumoniae; decreased IgG, IgA, and IgM levels are noted. Those with chronic granulomatous disease (answer B) have a defect of NADPH oxidase, rendering phagocytic leukocytes unable to kill pathogens. The presentation consists of recurrent infections from catalaseproducing organisms leading to skin infections, bloody diarrhea, and multiple abscesses. Hyperimmunoglobulin E syndrome (answer D) is associated with recurrent boils and eczema, pneumonia, eosinophilia, and high levels of IgE. Severe combined immunodeficiency syndrome (answer E) is the most severe form of immunodeficiency, with affected individuals presenting with failure to thrive and recurrent respiratory, skin, and gastrointestinal infections. Without treatment, most patients will die by the first year of life. 7.4 B. Heart failure secondary to ventricular septal defect. This child likely underwent TEF repair and has associated congenital heart disease with heart failure symptoms. Other congenital anomalies are present in 30% to 50% of patients with TEF. VATER or VACTERL associations include cardiac anomalies. A chest x-ray and echocardiogram would be confirmatory. Adjustment disorder (answer A) is a behavioral diagnosis, and medical disorders need to be ruled out before this diagnosis is entertained. Kawasaki disease (answer C) is associated with mucocutaneous inflammation and lymphadenopathy. Reactive airway disease (answer D) is not as likely in this scenario since there is no mention of wheezing. Rheumatic heart disease (answer E) will usually cause valvular dysfunction and requires many years for symptoms to develop.

08_Toy-Pediatrics_Case07_p071-078.indd 76

29/07/21 3:54 PM

77

SECTION II: CLINICAL CASES

CLINICAL PEARLS »»

VACTERL or VATER association stands for Vertebral abnormality, Anal imperforation, Cardiac abnormalities, TracheoEsophageal fistula, Radial, Renal and Limb anomalies. It is often seen in patients with TEF.

»»

Esophageal atresia is associated with CHARGE (Coloboma of the eye, Heart defects, Atresia of the nasal choanae, Retardation of growth and/or development, Genital and/or urinary abnormalities, and Ear abnormalities and deafness) association, DiGeorge syndrome, and trisomies 13, 18, and 21.

»»

The H-type TEF often presents later in infancy as recurrent pneumonitis and can be difficult to diagnose.

»»

Failure to thrive is a potential complication of patients with esophageal atresia.

»»

If a child presents with asthma that is difficult to control, along with other symptoms such as failure to thrive, a diagnosis of esophageal atresia should be considered.

REFERENCES Anderson SA, Chen MK. Anatomic disorders of the esophagus. In: Kline MW, Blaney SM, Giardino AP, et al, eds. Rudolph’s Pediatrics. 23rd ed. New York, NY: McGraw Hill; 2018:1726-1731. Katz A, Richardson W. Mediastinum and diaphragm. In: Zitelli BJ, McIntire SC, Nowalk AJ, eds. Zitelli and Davis’ Atlas of Pediatric Physical Diagnosis. Philadelphia, PA: Elsevier; 2018:623-624. Khan S, Matta, SKR. Esophageal atresia and tracheoesophageal fistula. In: Kleigman RM, St. Geme JW, Blum NJ, et al, eds. Nelson Textbook of Pediatrics. 21st ed. Philadelphia, PA: Elsevier; 2020: 1929-1931. McEvoy CF. Developmental disorders of gastrointestinal function. In: McMillan JA, Feigin RD, DeAngelis CD, Jones MD, eds. Oski’s Pediatrics: Principles and Practice. 4th ed. Philadelphia, PA: Lippincott Williams & Wilkins; 2006:369-370.

08_Toy-Pediatrics_Case07_p071-078.indd 77

29/07/21 3:54 PM

This page intentionally left blank

CASE 8 A 3200 g term male is born at 38 weeks of gestation by a scheduled repeat cesarean section prior to the onset of labor. The infant’s mother had good prenatal care, including vaginal cultures negative for group B Streptococcus. At the delivery, the amniotic fluid was clear and not foul-smelling. Apgar scores are 8 at 1 minute and 8 at 5 minutes. Within the first hour of birth, he has tachypnea, nasal flaring, and mild retractions. Chest auscultation reveals good air movement bilaterally; a few rales are noted. ▶▶ ▶▶

What is the most likely diagnosis? What is the best management for this condition?

09_Toy-Pediatrics_Case08_p079-086.indd 79

29/07/21 8:18 PM

80

CASE FILES: PEDIATRICS

ANSWERS TO CASE 8: Transient Tachypnea of the Newborn Summary: A 3200 g term newborn presents with šš šš

šš

Delivery by cesarean section Uncomplicated pregnancy, with mother receiving prenatal care and negative for group B Streptococcus (GBS) Respiratory distress and rales

Most likely diagnosis: Transient tachypnea of the newborn (TTN). Best management: Supportive care including supplemental oxygen, if necessary.

ANALYSIS Objectives 1. Know the presentation of TTN. (EPA 1) 2. Understand the medical care for TTN. (EPA 4) 3. Recognize common etiologies of tachypnea and respiratory distress in the newborn. (EPA 1, 2, 10)

Considerations This term infant presents soon after birth with mild respiratory distress following an uneventful pregnancy and cesarean delivery. The initial Apgar scores were 8 at 1 minute and 8 at 5 minutes. The immediate resuscitation includes ensuring good airway, air movement, and oxygenation; providing warmth (drying and providing radiant warmer if needed); and encouraging skin-to-skin contact with the mother. At this time, the newborn is displaying possible respiratory distress as manifested by tachypnea, nasal flaring, and mild retractions. Pulse oximetry is important to assess for adequate oxygenation. The possible etiologies include cardiac disease, pulmonary disease, sepsis, or metabolic derangement. The most common condition would be TTN, where residual pulmonary fluid remains in fetal lungs after delivery. TTN is self-limited; however, other more serious diseases should be ruled out. Evaluation of this infant begins with auscultation of the lungs and heart. A chest x-ray showing prominent perihilar interstitial markings and fluid in the fissure is common in TTN.

09_Toy-Pediatrics_Case08_p079-086.indd 80

29/07/21 8:18 PM

SECTION II: CLINICAL CASES

81

APPROACH TO: Transient Tachypnea of the Newborn and Respiratory Distress Syndrome DEFINITIONS CONGENITAL DIAPHRAGMATIC HERNIA (CDH): Herniation of abdominal contents through the posterolateral foramen of Bochdalek into the thoracic cavity, resulting in pulmonary hypoplasia. The incidence is approximately 1 in 5000 live births. Radiographic findings include bowel gas in the thoracic cavity. EXTRACORPOREAL MEMBRANE OXYGENATION (ECMO): A system using a modified heart-lung machine used in severe pulmonary failure. Cannulation of the carotid artery and jugular vein is required to link the neonate to the system. MECONIUM ASPIRATION SYNDROME: Aspiration of meconium during delivery resulting in respiratory distress. Radiographic findings include hyperinflation with patchy infiltrates. Because meconium may plug small airways, areas of air trapping are often present and may lead to the development of pneumothorax. Commonly associated with persistent pulmonary hypertension of the newborn. RESPIRATORY DISTRESS SYNDROME: A condition seen in premature infants resulting from surfactant deficiency. Radiographic findings include a characteristic reticulonodular “ground glass” pattern with air bronchograms and decreased aeration. TRANSIENT TACHYPNEA OF THE NEWBORN (TTN): Slow absorption of fetal lung fluid with resultant tachypnea. The condition is more commonly associated with cesarean section deliveries.

CLINICAL APPROACH Pathophysiology TTN is a self-limited condition that is frequently seen in a term infant. It is often noted after an uneventful cesarean section but can also occur after a precipitous vaginal birth. The etiology is most likely secondary to slow resorption of fetal alveolar fluid; this excess fluid decreases lung compliance in newborns.

Clinical Presentation Infants with TTN develop respiratory distress shortly after birth with tachypnea (exceeds 60 breaths per minute), mild retractions, and nasal flaring; severe cases (rare) may present with grunting and cyanosis. Hypoxemia and hypercapnia can result from perfusion of inadequately ventilated alveoli. Chest radiography reveals perihilar streaking and fluid in the interlobar fissures; lungs are symmetrically aerated (see Figure 8–1). Most infants with TTN have resolution of symptoms within 24 to 72 hours, and management is focused on supportive care.

09_Toy-Pediatrics_Case08_p079-086.indd 81

29/07/21 8:18 PM

82

CASE FILES: PEDIATRICS

Figure 8–1.  Frontal radiograph of the chest in a full-term infant with transient tachypnea of the newborn shows normal lung volumes with diffuse, bilateral prominent interstitial lung markings. (Reproduced with permission, from Elsayes KM, Oldham SA, eds. Introduction to Diagnostic Radiology. 2014. Copyright © McGraw Hill LLC. All rights reserved. https://accessmedicine.mhmedical.com.)

Treatment For some infants with TTN, oxygen saturations drop and supplemental oxygen is required. In the rare, more severe case of TTN, persistent pulmonary hypertension secondary to ongoing increased pulmonary vascular resistance should be considered. Infants with TTN do not require antimicrobial therapy; failure of the infant to follow the expected course of mild respiratory distress necessitates evaluating the child for a more serious pathology, such as sepsis.

Respiratory Distress Syndrome Clinical Presentation.  Infants with respiratory distress syndrome (RDS) are usually born prematurely (less than 34 weeks of gestational age) and are deficient in surfactant. Shortly after birth, they present with symptoms of respiratory distress, including poor oxygenation, grunting, retracting, and poor air movement. Radiographically, they have findings of a reticulonodular “ground glass” pattern with air bronchograms and decreased aeration of the lungs. The use of antenatal corticosteroids given to pregnant woman is helpful in reducing RDS. Treatment.  Supportive care includes supplemental oxygen as needed to maintain oxygen saturations greater than 90% and intravenous fluids or nasogastric feeding to maintain hydration (because the degree of tachypnea usually precludes oral feeding). Exogenous surfactant is available and is administered by the resuscitation team

09_Toy-Pediatrics_Case08_p079-086.indd 82

29/07/21 8:18 PM

SECTION II: CLINICAL CASES

83

in an effort to ameliorate the effects of surfactant deficiency; this agent is more effective when given shortly after birth. Prognosis.  Although more infants with RDS are surviving due to improved neonatal intensive care unit care and surfactant, there are possible long-term complications, such as bronchopulmonary dysplasia.

CASE CORRELATION šš

See also Case 2 (Infant of a Diabetic Mother), Case 4 (Sepsis and Group B Streptococcal Infections), Case 6 (Neonatal Herpes Simplex Virus Infection), and Case 7 (Esophageal Atresia).

COMPREHENSION QUESTIONS 8.1 A term male is born to a 33-year-old woman who had little prenatal care. Immediately after birth, he has cyanosis and respiratory distress. Chest auscultation in the delivery room reveals right-sided heart sounds and absent leftsided breath sounds. Which of the following is the most appropriate next step? A. Determine if the abdomen is distended or scaphoid B. Order a computed tomography of the chest C. Order ultrasonography of the chest D. Perform a needle thoracostomy for possible pneumothorax E. Prepare the infant for extracorporeal membrane oxygenation (ECMO) 8.2 A term male is born via repeat cesarean section to a 30-year-old woman. Immediately after birth, he is noted to have mild respiratory distress. Chest auscultation in the delivery room reveals clear breath sounds. Which of the following is the most appropriate next step? A. Perform endotracheal intubation with direct suction B. Begin intravenous antibiotic therapy C. Deliver surfactant therapy D. Observe and administer supplemental oxygen as needed E. Begin bag-mask ventilation 8.3 A term male is born vaginally to a 22-year-old primigravida woman at 38 weeks’ gestation; the pregnancy was uncomplicated. Just prior to delivery, fetal bradycardia was noted to the 60 beats per minute range, and thick meconium is noted at delivery. The infant has hypotonia and bradycardia in the 40 beats per minute range. Which of the following is the first step in resuscitation? A. Administration of epinephrine through endotracheal tube B. Bag-mask ventilation C. Endotracheal intubation with direct suction D. Oxygen delivered by cannula in close proximity to the nares E. Tracheostomy

09_Toy-Pediatrics_Case08_p079-086.indd 83

29/07/21 8:18 PM

84

CASE FILES: PEDIATRICS

8.4 A newborn female is delivered by cesarean section to a 23-year-old mother at 29 weeks’ gestation. She has poor respiratory effort at the time of delivery with cyanosis, requiring resuscitation and eventually intubation. On examination in the delivery room, the infant continues to have subcostal retractions and is difficult to ventilate after 10 minutes of life. What is the next best step in management? A. Obtain a chest x-ray B. Administer albuterol C. Administer surfactant D. Closely monitor clinically E. Obtain an echocardiogram

ANSWERS 8.1 A. Determine if the abdomen is distended or scaphoid. Evaluation of neonates born with respiratory distress and unilateral breath sounds includes an abdominal examination. With asymmetrical breath sounds, pneumothorax and congenital diaphragmatic hernia (CDH) should be considered. The typical physical examination of CDH in a newborn reveals a scaphoid abdomen and the presence of bowel sounds in the chest. Needle thoracostomy (answer D) is avoided because intestinal perforation may occur. The patient should be stabilized and the need for ECMO (answer E) ascertained after the infant’s initial response to therapy is evaluated. Many cases of CDH are diagnosed by prenatal ultrasound. Infants with CDH do not respond to the typical steps of neonatal resuscitation and often have worsening respiratory status with bagmask ventilation, ultimately requiring intubation. The diagnosis of CDH can be made on plain imaging (not computed tomography [answer B] or ultrasonography [answer C]) by locating the nasogastric feeding tube in the chest where the stomach has been displaced from the abdomen. 8.2 D. Observe and administer supplemental oxygen as needed. Because this infant most likely has transient tachypnea of the newborn, the next step is to observe and administer supplemental oxygen as needed. The medical record should be reviewed to ensure that an antenatal fetal complication was not suspected. A careful heart and lung examination is important. Endotracheal intubation (answer A) is not indicated because the infant is breathing, and presumably pink, and oxygenated. Intravenous antibiotics (answer B) are not indicated because there is no suspicion of sepsis. Surfactant therapy (answer C) is not indicated because this is a term infant. Bag-mask ventilation (answer E) is not needed because the patient is breathing spontaneously and oxygenating. 8.3 C. Endotracheal intubation with direct suction. Endotracheal intubation with direct suctioning of meconium below the vocal cords is performed in a depressed infant with thick meconium noted at delivery. Bag-mask ventilation (answer B) or endotracheal intubation without suction may increase the volume of meconium aspirated. A vigorous infant with a heart rate above 100 beats per minute,

09_Toy-Pediatrics_Case08_p079-086.indd 84

29/07/21 8:18 PM

85

SECTION II: CLINICAL CASES

strong respirations, and good muscle tone with meconium-stained fluids does not need to be suctioned immediately after birth. Epinephrine (answer A) is usually not indicated until more than a minute of positive-pressure ventilation is provided with the heart rate still persisting below 60 beats per minute. Oxygen to the nares (answer D) would not be effective in this depressed infant who is breathing poorly and would not help to prevent meconium aspiration. A tracheostomy (answer E) is only indicated when there are severe upper airway or oral deformities preventing an endotracheal tube in an infant requiring an airway. 8.4 C. Administer surfactant. This infant born at 29 weeks’ gestation likely has respiratory distress syndrome from surfactant deficiency. Difficulty in ventilating arises due to collapsed alveoli. Introduction of surfactant as a treatment in delivery rooms decreases alveoli surface tension, thus improving ventilation, and has increased the survival of premature infants. Echocardiography (answer E) and chest imaging (answer A) may be required if symptoms persist. Albuterol (answer  B) is rarely needed in neonatal resuscitation due to lack of reactive airway disease at this stage of life. Close monitoring (answer  D) without intervention is inappropriate because this infant is continuing to have respiratory difficulties.

CLINICAL PEARLS »»

Transient tachypnea of the newborn (TTN) is associated with birth by cesarean section in term infants.

»»

TTN is managed with supportive care and does not lead to chronic lung disease.

»»

The infant born to a diabetic mother has an increased incidence of polycythemia and hypoglycemia, both of which can result in tachypnea. In addition, these infants have a higher incidence of surfactant deficiency at later gestational ages, again resulting in tachypnea.

»»

GBS infection and neonatal herpes simplex virus infection are common infections in the newborn period; both can cause pneumonia that may present with an increased respiratory rate, among other symptoms.

»»

The child with tracheoesophageal atresia will have recurrent episodes of aspiration and clinical findings of tachypnea as a result.

REFERENCES Ahlfeld SK. Respiratory distress syndrome (hyaline membrane disease). In: Kleigman RM, St. Geme JW, Blum NJ, et al, eds. Nelson Textbook of Pediatrics. 21st ed. Philadelphia, PA: Elsevier; 2020:932-936. de Waal CG, van Kaam AH. Respiratory distress syndrome. In: Kline MW, Blaney SM, Giardino AP, et al, eds. Rudolph’s Pediatrics. 23rd ed. New York, NY: McGraw Hill; 2018:265-270.

09_Toy-Pediatrics_Case08_p079-086.indd 85

29/07/21 8:18 PM

86

CASE FILES: PEDIATRICS

Galarza MG, Sosenko IRS. Abnormalities of the lungs. In: Kline MW, Blaney SM, Giardino AP, et al, eds. Rudolph’s Pediatrics. 23rd ed. New York, NY: McGraw Hill; 2018:234-241. Gross I. Meconium aspiration syndrome. In: McMillan JA, Feigin RD, DeAngelis CD, Jones MD, eds. Oski’s Pediatrics: Principles and Practice. 4th ed. Philadelphia, PA: Lippincott Williams & Wilkins; 2006:315. Gross I. Transient tachypnea of the newborn. In: McMillan JA, Feigin RD, DeAngelis CD, Jones MD, eds. Oski’s Pediatrics: Principles and Practice. 4th ed. Philadelphia, PA: Lippincott Williams & Wilkins; 2006:311. Johnson KE. Transient tachypnea of the newborn. Garcia-Prats JA, ed. UpToDate. Waltham, MA: UpToDate Inc. https://www.uptodate.com. Accessed on February 2, 2020.

09_Toy-Pediatrics_Case08_p079-086.indd 86

29/07/21 8:18 PM

CASE 9 A 3300 g full-term infant is delivered vaginally after an uncomplicated pregnancy at 40 weeks’ gestation. On initial examination, the baby is noted to have cloudiness of both lenses, which obscures the red reflex. The family history is significant for the father having had eye surgery at a young age as an infant or child. The physical examination otherwise is unremarkable. ▶▶ ▶▶ ▶▶

What is the most likely diagnosis? What are the possible complications of this diagnosis? What is the next step?

10_Toy-Pediatrics_Case09_p087-096.indd 87

12/07/21 3:11 PM

88

CASE FILES: PEDIATRICS

ANSWERS TO CASE 9: Congenital Cataracts Summary: A 3300 g full-term infant presents with šš

A vaginal delivery after an uncomplicated pregnancy

šš

Bilateral lens cloudiness

šš

A family history of an ophthalmologic condition requiring surgery

Most likely diagnosis: Congenital cataracts. Possible complications: Severe visual deprivation accompanied by poor fixation and nystagmus. Next step: Ophthalmologic evaluation and complete evaluation for possible associated hereditary, chromosomal, metabolic, or infectious causes.

ANALYSIS Objectives 1. Understand the conditions associated with congenital cataracts. (EPA 12) 2. Understand the development of amblyopia. (EPA 12)

Considerations This newborn presents with an isolated eye finding consistent with cataracts and a positive family history of eye disease. The infant must be evaluated for common chromosomal, hereditary, metabolic, infectious or inflammatory entities associated with congenital cataracts. Because of the varying severity of opacification of the lens, early referral to a pediatric ophthalmologist for treatment and visual rehabilitation is mandatory. Unilateral cataracts can cause severe deprivation amblyopia and strabismus.

APPROACH TO: Pediatric Eye Problems DEFINITIONS AMBLYOPIA: Decrease or loss of vision caused by underuse of one eye (deprivation amblyopia) or lack of clear image projecting onto the retina (strabismic amblyopia). APHAKIA: Absence of the lens. CATARACT: Any opacity of the crystalline lens. Depending on the size and location, the cataract may be either partial or complete. The impact on vision depends on the location and characteristics of the cataract.

10_Toy-Pediatrics_Case09_p087-096.indd 88

12/07/21 3:11 PM

SECTION II: CLINICAL CASES

89

LEUKOCORIA: Abnormal white pupillary reflex. STRABISMUS: Misalignment of the visual axes. Strabismus can result in the loss of vision (amblyopia).

CLINICAL APPROACH Congenital Cataracts Epidemiology.  The incidence of congenital cataracts is between 2 and 13 per 10,000 live births. They are an isolated condition in 60% of cases, part of a syndrome in 20% to 25% of cases, and associated with unrelated major birth defects in the remainder. Cataracts can present either unilaterally or bilaterally. They are more common in low-birth-weight infants, with those who are at or below 2500 g having a three- to four-fold increased risk of developing infantile cataracts. Pathophysiology.  Many of the cases of isolated congenital cataracts are hereditary in origin, with most being transmitted through an autosomal dominance inheritance. Developmental cataracts may result from prenatal infections, such as the TORCH infections (toxoplasmosis, other [syphilis, varicella-zoster, parvovirus b19], rubella, cytomegalovirus, and herpes simplex virus), or may be secondary to metabolic diseases such as galactosemia, homocystinuria, galactokinase deficiency, abetalipoproteinemia, or Fabry, Hurlers, Niemann-Pick, or Wilson syndrome. The most common metabolic disorders causing congenital cataracts are hypoglycemia and hypocalcemia; infants born to diabetic mothers or those with hypoparathyroidism have a higher incidence of cataracts. Intraocular abnormalities, including retinopathy of prematurity, retinitis pigmentosa, uveitis, and retinal detachment, may lead to the development of cataracts. Chromosomal anomalies associated with cataracts include trisomies 13, 18, and 21; Turner syndrome; and various depletion and duplication syndromes. Evaluation.  All infants should be assessed for a red reflex such as using an ophthalmoscope in a darkened room. An abnormal red reflex (see Figure 9–1) such as opacity or white appearance may be due to a cataract or retinoblastoma. Evaluation of infants presenting with congenital cataracts includes a full history (with special attention to maternal and family history), physical examination, and an ophthalmologic examination by a pediatric ophthalmologist that might include ocular ultrasound if the lenses are found to be completely opaque. The need for TORCH titers and evaluation for galactosemia or other metabolic disorders should be considered, although many the metabolic disorders are now included in states’ newborn metabolic screens. Parents of infants with congenital cataracts may require an ophthalmologic evaluation. Treatment.  If visual disturbance is significant, surgical lens removal may be performed as early as 2 to 4 weeks after birth. The infant is then fitted with a refraction contact lens until intraocular lens placement is considered, usually at about 2 years of age. For lesser forms of visual disturbance, the cataract is monitored for changes and the child is monitored for the development of amblyopia. Infants with

10_Toy-Pediatrics_Case09_p087-096.indd 89

12/07/21 3:11 PM

90

CASE FILES: PEDIATRICS

Figure 9–1.  White pupillary reflection (leukocoria) in each eye (more pronounced in the right eye) due to bilateral retinoblastoma. (Reproduced with permission, from Riordan-Eva P, Augsburger JJ, eds. Vaughan & Asbury’s General Ophthalmology. 19th ed. 2018. Copyright © McGraw Hill LLC. All rights reserved. https://accessmedicine.mhmedical.com.)

unilateral cataracts who do not require surgery may require patching of their good eye to prevent the development of deprivation amblyopia. The prognosis for congenital cataracts is dependent on multiple factors, including the nature of the cataract, age of onset, age at intervention, underlying disease, and presence of any other associated ocular abnormalities. Deprivation amblyopia is the most common cause of poor visual recovery following cataract surgery in children.

Amblyopia Background.  Amblyopia is a reduction of visual acuity in one or both eyes without a structural reason. The prevalence is estimated at 1% to 4%. It is the most common cause of monocular vision loss in children and young adults. Pathophysiology.  During infancy and the early childhood years, the eyes and brain must work together and in conjunction for vision to develop normally. The visual cortex will not properly develop without appropriate stimulation. The most common cause of amblyopia is strabismus (ocular malalignment). Other causes include deprivation amblyopia (opacity in the visual axis), ametropia (high refractive error in both eyes), and anisometropia (unequal vision between the eyes). For all of these lesions, the common cause of pathology for the child is interference with the development of clear images during the critical period of eye development in infancy and early childhood.

10_Toy-Pediatrics_Case09_p087-096.indd 90

12/07/21 3:11 PM

SECTION II: CLINICAL CASES

91

Clinical Presentation.  Difficulty seeing, squinting, or head tilting are the most common presenting symptoms. Early vision screening is the most important way to detect amblyopia. It is usually asymmetric and is diagnosed when an ophthalmologic examination demonstrates reduced acuity otherwise not explained by a structural etiology. Early detection of this condition is key; the younger the child is, the more likely the child will recover eye function. Treatment.  Treatment for amblyopia must include removing any opacity and ensuring well-focused retinal images are being produced in each eye; glasses may be necessary. Strengthening of the “weak” eye in order to stimulate appropriate visual development is accomplished by covering the “good” eye (occlusion therapy) or using atropine eye drops in the “good” eye (penalization therapy) to blur vision in this eye. Close monitoring by a pediatric ophthalmologist will ensure the treatment maximizes the benefits to the amblyopic eye while not causing amblyopia to develop in the nonaffected eye. Although it was previously thought that fulltime occlusion was the best way to treat amblyopia, recent studies have shown that many children are able to achieve similar results with less patching or through the use of atropine drops. Older children who were previously thought to be “visually mature” may also respond to therapy.

Congenital Nasolacrimal Duct Obstruction This is a common condition causing excessive tearing or mucoid discharge from the eyes. As many as 5% to 10% of children aged less than 1 year are affected, and the condition often resolves without surgery. The etiology is the persistence of membrane at the distal portion of the nasolacrimal duct. The best treatment is observation, massage of the lacrimal sac, and application of topical antibiotics for any infection that occurs. Most will resolve by age 1 year, and if persistent, then lacrimal duct probing is usually effective.

Pediatric Glaucoma Pediatric or congenital glaucoma is a rare condition (1 in 10,000 births) that often presents between the ages of 3 and 9 months. The symptoms are excessive tearing, light sensitivity, a large cloudy cornea, and unequal pupils. An early diagnosis is important to avoid permanent vision loss. The diagnosis is made by documented elevated intraocular pressure, which often requires general anesthesia in children. Optic nerve cupping may also be seen on fundoscopic exam. Most cases are treated surgically by opening the drainage canals for the aqueous fluid.

CASE CORRELATION šš

See Case 2 (Infant of a Diabetic Mother) and Case 6 (Neonatal Herpes Simplex Virus Infection).

10_Toy-Pediatrics_Case09_p087-096.indd 91

12/07/21 3:11 PM

92

CASE FILES: PEDIATRICS

COMPREHENSION QUESTIONS 9.1 A full-term, small for gestational age newborn girl presents with cataracts, petechiae, and a continuous machine-like murmur. Which of the following statements is accurate? A. This infant needs an audiology evaluation because sensorineural hearing loss is a common association. B. This infant needs a renal ultrasound because she is likely to have renal abnormalities. C. Treatment of her condition includes 14 days of intravenous penicillin after evaluation of her cerebrospinal fluid. D. The infant’s condition is likely to have occurred because of a maternal illness during the third trimester. E. Intravenous antiviral therapy should be initiated, and viral cultures should be obtained. 9.2 A healthy 2-week-old girl has yellow discharge from her left eye. Her mother had early prenatal care, the baby was delivered vaginally, and she was discharged at 48 hours of life. Within the first few days of life, the mother noted that the baby had increased tear production in her left eye, which now has yellow discharge. She has red reflexes bilaterally, her pupils are equal and reactive to light, and she has no scleral injection. She has left-sided mucous ocular discharge. What is the best next step in management? A. Administer intravenous antibiotic therapy B. Begin a course of oral antimicrobial treatment C. Begin a course of topical antimicrobial treatment, nasolacrimal massage, and warm water cleansing D. Incise and drain the area E. Refer the child for a stat outpatient ophthalmologic evaluation 9.3 A 4-month-old infant has excessive right-sided tearing. His mother states that he becomes irritable in bright light and calms in a darkened room. On examination, he has eye asymmetry, with the right eye appearing to be larger than the left. Which of the following is the most appropriate next step in management? A. Warm compresses and gentle massage B. Topical antibiotic therapy C. Referral to genetic counselor D. Systemic antibiotic therapy E. Immediate referral to a pediatric ophthalmologist

10_Toy-Pediatrics_Case09_p087-096.indd 92

12/07/21 3:11 PM

SECTION II: CLINICAL CASES

93

9.4 While examining a term neonate in the newborn nursery, the red reflex is noted to be markedly less obvious in the right eye. The remainder of the newborn examination is normal, including all growth parameters. The mother has no history of infections during pregnancy. Which of the following is the correct course of action? A. Continue routine newborn care and reexamine the baby in 2 weeks B. Consult pediatric ophthalmology for immediate evaluation C. Order an ocular ultrasound D. Order complete blood count (CBC), comprehensive metabolic panel (CMP), rubella titers, and karyotype, as well as urine for reducing substances E. Order TORCH (toxoplasmosis, other, rubella, cytomegalovirus, and herpes simplex virus) titers

ANSWERS 9.1 A. This infant needs an audiology evaluation because sensorineural hearing loss is a common association. This infant has the classic features of congenital rubella syndrome, including low birth weight, heart defect (patent ductus arteriosus), and congenital cataracts. Other clinical findings associated with congenital rubella syndrome include purpura, hepatosplenomegaly, jaundice, retinopathy, glaucoma, pulmonary artery stenosis, meningoencephalitis, thrombocytopenia, and hemolytic anemia. Long-term sequelae of congenital rubella include sensorineural hearing loss, neurodevelopmental abnormalities, growth retardation, endocrine disease (diabetes mellitus, thyroid dysfunction), and hypogammaglobulinemia. Maternal infection may or may not be clinically apparent, and infection during the first month is most likely to result in fetal infection with resultant involvement of multiple organs. The infection is likely first trimester rather than third trimester (answer D). Penicillin therapy (answer C) would be the treatment for congenital syphilis. Antiviral therapy (answer E) would be indicated with HSV infection. Kidney problems (answer B) are not as common with congenital rubella infections. 9.2 C. Begin a course of topical antimicrobial treatment, nasolacrimal massage, and warm water cleansing. This infant had excessive tear production that later became a mucopurulent discharge but an otherwise normal ophthalmologic examination. The most likely cause is congenital nasolacrimal duct obstruction (CNLDO) or dacryostenosis. This condition is typically caused by failure of canalization of the cells that form the nasolacrimal duct. Infants with CNLDO are at risk of developing acute infection of the nasolacrimal sac (dacryocystitis) or, rarely, periorbital cellulitis. Of note, in this case, the conjunctiva is not inflamed and the cornea is not involved. Initial treatment includes topical antibiotic therapy and nasolacrimal duct massage two to three times daily with warm water eyelid cleansing. Most cases of CLDNO resolve spontaneously, usually before 1 year of age. Careful evaluation is warranted because of the risk

10_Toy-Pediatrics_Case09_p087-096.indd 93

12/07/21 3:11 PM

94

CASE FILES: PEDIATRICS

for infection and cellulitis. Neither oral (answer B) nor intravenous antibiotics (answer A) are indicated. Ophthalmological evaluation (answer E) is not required at this stage. 9.3 E. Immediate referral to a pediatric ophthalmologist. A history of excessive tearing and photophobia, along with examination findings of corneal enlargement, suggest an immediate need for the evaluation by an ophthalmologist for congenital glaucoma. Treatment likely is surgical. Infantile glaucoma occurs in 1 in 100,000 births with a classic triad of tearing, photophobia, and blepharospasm. It may be isolated (primary congenital glaucoma) or occur with various conditions, including congenital rubella, neurofibromatosis type 1, mucopolysaccharidosis type I, Lowe oculocerebrorenal syndrome, Sturge-Weber syndrome, Marfan syndrome, and several chromosomal abnormalities. The increased intraocular pressure can lead to expansion of the globe and corneal damage. The other answer choices (answer A, warm compress and massage; answer B, topical antibiotic therapy; answer C, referral to genetic counselor; and answer D, systemic antibiotic therapy) would only delay proper treatment and may lead to vision loss. 9.4 B. Consult pediatric ophthalmology for immediate evaluation. A positive Bruckner test (also known as “red reflex” test) warrants immediate evaluation by pediatric ophthalmology because the presence of an opacity of the lens or congenital cataracts exists. Leukocoria or a white pupil may indicate the presence of a retinoblastoma. Management of congenital cataracts includes early pediatric ophthalmology involvement for possible cataract extraction, usually within the first 4 weeks to 3 months of life. Indications for surgery include opacity greater than 3 mm in diameter, decreased visual response, and onset of strabismus. Babies with unilateral cataracts are at increased risk of developing deprivation amblyopia, and adherence to a postoperative schedule of occlusion therapy with close follow-up with an ophthalmologist is essential to achieve good visual outcome. The other answer choices (answer A, continue care and reexamine in 2 weeks; answer C, ocular ultrasound; answer D, order CBC, CMP, rubella titers, karyotype, and urine; and answer E, order TORCH titers) would potentially delay appropriate diagnosis and treatment.

10_Toy-Pediatrics_Case09_p087-096.indd 94

12/07/21 3:11 PM

95

SECTION II: CLINICAL CASES

CLINICAL PEARLS »»

Galactosemia is associated with cataracts.

»»

Workup of an infant with congenital cataract includes TORCH titers. All can cause cataracts.

»»

Amblyopia, which is a reduction of visual acuity without a structural cause, is most commonly due to strabismus. Early diagnosis is important to avoid permanent vision loss.

»»

Amblyopia must be diagnosed at an early period so that occlusive or penalization therapy may be instituted on the unaffected eye to maximize improvement in vision of the affected eye.

»»

One of the complications of an infant born to a mother who has diabetes is cataracts.

»»

Congenital nasolacrimal duct obstruction, affecting up to 5% to 10% of infants, is associated with excessive tearing.

»»

Pediatric glaucoma presents with excessive tearing, photophobia, a large cloudy cornea, and unequal pupils.

REFERENCES Cazacu AC, Demmler GJ. Rubella (German measles). In: McMillan JA, Feigin RD, DeAngelis CD, Jones MD, eds. Oski’s Pediatrics: Principles and Practice. 4th ed. Philadelphia, PA: Lippincott Williams & Wilkins; 2006:1272-1275. Cheng KP. Ophthalmology. In: Zitelli BJ, McIntire S, Nowalk AJ, eds. Zitelli and Davis’ Atlas of Pediatric Physical Diagnosis. 6th ed. Philadelphia, PA: Saunders; 2018:691-732. Mason WH, Gans HA. Rubella. In: Kleigman RM, St. Geme JW, Blum NJ, et al, eds. Nelson Textbook of Pediatrics. 21st ed. Philadelphia, PA: Elsevier; 2020:1676-1680. Olitsky SE, Marsh JD. Abnormalities of the lens. In: Kleigman RM, St. Geme JW, Blum NJ, et al, eds. Nelson Textbook of Pediatrics. 21st ed. Philadelphia, PA: Elsevier; 2020:3372-3375. Olitsky SE, Marsh JD. Disorders of eye movement and alignment. In: Kleigman RM, St. Geme JW, Blum NJ, et al, eds. Nelson Textbook of Pediatrics. 21st ed. Philadelphia, PA: Elsevier; 2020:3353-3360. Olitsky SE, Marsh JD. Disorders of the lacrimal system. In: Kleigman RM, St. Geme JW, Blum NJ, et al, eds. Nelson Textbook of Pediatrics. 21st ed. Philadelphia, PA: Elsevier; 2020:3363-3364. Olitsky SE, Marsh JD. Disorders of vision. In: Kleigman RM, St. Geme JW, Blum NJ, et al, eds. Nelson Textbook of Pediatrics. 21st ed. Philadelphia, PA: Elsevier; 2020:3346-3349. Quinn AG, Gao ZW, Levin AV. Amblyopia. In: Kline MW, Blaney SM, Giardino AP, et al, eds. Rudolph’s Pediatrics. 23rd ed. New York, NY: McGraw Hill; 2018:2773-2775. Traboulski EI. Pediatric ophthalmology. In: McMillan JA, Feigin RD, DeAngelis CD, Jones MD, eds. Oski’s Pediatrics: Principles and Practice. 4th ed. Philadelphia, PA: Lippincott Williams & Wilkins; 2006:801-819. Yeung HH, Walton DS. Visual impairment in children. In: Kline MW, Blaney SM, Giardino AP, et al, eds. Rudolph’s Pediatrics. 23rd ed. New York, NY: McGraw Hill; 2018:2771-2773.

10_Toy-Pediatrics_Case09_p087-096.indd 95

12/07/21 3:11 PM

This page intentionally left blank

CASE 10 A mother brings her 18-month-old daughter to your clinic for a well-child visit. She is a new patient without available past medical records, but her mother declares no known health issues. Her diet is varied, but she is a picky eater with frequent tantrums when given anything beyond fried foods and juices. You immediately note the child to be small for her age. Her weight is below the fifth percentile on standardized growth curves (50th percentile for a 12-month-old), her length is at the 25th percentile, and her head circumference is at the 50th percentile. Her vital signs and her examination are otherwise normal. ▶▶ ▶▶

What is the most likely diagnosis? What is the next step in evaluation?

11_Toy-Pediatrics_Case10_p097-106.indd 97

27/03/21 9:06 AM

98

CASE FILES: PEDIATRICS

ANSWERS TO CASE 10: Failure to Thrive Summary: An 18-month-old girl presents with šš

Poor weight gain (weight below the fifth percentile)

šš

Length at the 25th percentile and head circumference at the 50th percentile

šš

Reportedly being a picky eater

šš

Normal vital signs and examination findings

Most likely diagnosis: Failure to thrive (FTT), most likely nonorganic (nonmedical) in etiology. Next step in evaluation: Gather more information, including birth, past medical, family, social, developmental, and dietary histories. Perform limited screening laboratory testing to identify organic causes of FTT, provide dietary counseling, and schedule frequent office visits to assess appropriate weight gain.

ANALYSIS Objectives 1. Apply historical clues to recognize organic and nonorganic FTT. (EPA 1, 2) 2. Describe the appropriate use of laboratory tests in an otherwise healthy child with FTT. (EPA 3) 3. Describe the treatment and follow-up of a child with nonorganic FTT. (EPA 4, 12) 4. Identify some of the common etiologies for FTT. (EPA 2)

Considerations FTT is broadly divided into organic (due to an underlying medical cause) and nonorganic (frequently psychosocial). This patient’s growth pattern (inadequate weight gain, potentially modest length retardation, head circumference sparing) suggests FTT, most likely nonorganic given that the physical examination is normal. A nonorganic FTT diagnosis is made after organic etiologies are excluded and, after adequate nutrition and an adequate environment are assured, growth resumes normally after catch-up growth is demonstrated. Diagnostic and therapeutic maneuvers aimed at medical causes are appropriate when supported by the history (eg, prematurity, maternal infection) or examination (eg, enlarged spleen, significant developmental delay). Although organic and nonorganic FTT can occur simultaneously, attempts to differentiate the two forms are helpful because the evaluation, treatment, and follow-up may be different. In this case scenario, had the same practitioner followed this patient since birth or had records from the previous health care provider, earlier detection of FTT and its potential etiology might have occurred, thus allowing for more rapid

11_Toy-Pediatrics_Case10_p097-106.indd 98

27/03/21 9:06 AM

SECTION II: CLINICAL CASES

99

intervention. For instance, patients with poor caloric intake usually fail to gain weight but maintain length and head circumference. As nutrition remains poor, length becomes affected next, and then ultimately head circumference.

APPROACH TO: Failure to Thrive DEFINITIONS FAILURE TO THRIVE (FTT): A physical sign, not a final diagnosis. A clear consensus on FTT criteria does not exist, but in general terms, it should be suspected in a child when: šš

Growth is below the fifth percentile for gender and corrected age

šš

Weight for length is below the fifth percentile

šš

Body mass index (BMI) is below the fifth percentile

šš

In a child whose growth crosses more than two major growth percentiles (90th, 75th, 25th, 10th, and fifth) in a short time frame.

Other definitions include a child younger than 2 years of age with growth repeatedly below the third or the fifth percentile for age or whose weight is less than 80% of the ideal weight for age. NONORGANIC (PSYCHOSOCIAL) FTT: Poor growth without a medical etiology; nonorganic FTT often is related to poverty or poor caregiver-child interaction; constitutes one-third to one-half of FTT cases identified in tertiary care settings and nearly all cases in primary care settings. ORGANIC FTT: Poor growth caused by an underlying medical condition, such as inflammatory bowel disease, renal disease, or congenital heart conditions.

CLINICAL APPROACH The goals of the history, physical examination, and laboratory testing are to establish whether the child’s caregiver is supplying enough calories and whether the child is consuming enough calories and is able to use them for growth. Identification of which factor (medical, nutritional, psychosocial, and developmental) is the likely source of the problem then helps guide management.

Pathophysiology Definition.  The history and physical examination are the most important tools in an FTT evaluation. Of note, properly defining FTT involves the use of adjusted growth curves in select populations; to standardize growth, the World Health Organization (WHO) growth charts are recommended for children up to 2 years of age and the Centers for Disease Control and Prevention (CDC) charts are recommended for patients from 2 to 20 years of age. Using other specialized growth

11_Toy-Pediatrics_Case10_p097-106.indd 99

27/03/21 9:06 AM

100

CASE FILES: PEDIATRICS

charts for patients with conditions such as prematurity, Turner syndrome, or trisomy 21 is appropriate. Diet.  A dietary history can offer important clues to identify an etiology. The type of milk (breast or formula) and frequency and quality of feeding, voiding, vomiting, and stooling should be recorded. The formula used (commercial vs homemade) and the mixing process should be reviewed to ensure appropriate dilution, as adding too much water to powdered formula results in inadequate nutrition. The amount and type of juices and solid foods should be noted for older children. Significant food aversions might suggest gastric distress due to malabsorption or metabolic conditions. A dietary history where the parent notes all foods offered and taken by the child can include a 24-hour recall or, more formally, a 3-day diary of 2 weekdays and 1 weekend day. Identification of associated symptoms of sweating, choking, cyanosis, or difficulty sucking can be useful. Familial practices including restrictions due to allergies or dietary beliefs, such as vegetarianism or veganism, should be identified. Pregnancy and Neonatal Histories.  Pregnancy and early neonatal histories may reveal maternal infection (toxoplasmosis, cytomegalovirus, rubella, syphilis), maternal depression, drug use, intrauterine growth restriction, prematurity, or other chronic neonatal conditions. When children suspected of having FTT are seen in families whose members are genetically small or with a slow growth history (constitutional delay), affected children are usually normal and do not require an exhaustive evaluation. In contrast, a family history of inheritable disease associated with poor growth (cystic fibrosis) should be evaluated more extensively. Social Etiologies.  Because nonorganic FTT is more commonly associated with poverty, a social history is often useful. The child’s living arrangements, including primary and secondary caregivers, housing type, caregiver’s financial and employment status, the family’s social supports, and unusual stresses (domestic abuse or neglect) should be reviewed. While gathering the history, the clinician can observe for unusual caregiver-child interactions that may suggest abuse.

Clinical Presentation History and Physical Examination.  All body organ systems potentially harbor a cause for organic FTT (Table 10–1). The developmental status (possibly delayed in organic and nonorganic FTT) needs evaluation. Children with nonorganic FTT may demonstrate an occipital bald spot from lying in a bed and failure to attain appropriate developmental milestones resulting from lack of parental stimulation; may be disinterested in their environment; may avoid eye contact, smiling, or vocalization; and may not respond well to maternal attempts of comforting. Children with some types of organic FTT (for instance renal tubular acidosis) and most nonorganic FTT show “catch-up” in developmental milestones with successful therapy. During the examination (especially of younger infants), the clinician can observe a feeding, which may give clues to maternal-child interaction bonding issues or to physical problems (cerebral palsy, oral motor or swallowing difficulties, cleft palate). Imaging and Laboratory Findings. The history or physical examination suggestive of organic FTT directs the laboratory and radiologic evaluation. In most

11_Toy-Pediatrics_Case10_p097-106.indd 100

27/03/21 9:06 AM

SECTION II: CLINICAL CASES

101

Table 10–1  •  MAJOR CAUSES OF POOR WEIGHT GAIN Inadequate Caloric Intake •  Inappropriate feeding technique including incorrect formula preparation •  Inappropriate nutrient intake •  Lack of appetite: depression (maternal or child), chronic disease •  Ingestion difficulties: feeding disorders, neurologic disorders (cerebral palsy), craniofacial anomalies, genetic syndromes, tracheoesophageal fistula •  Unavailability of food: neglect, inappropriate food for age, insufficient volume of food Altered Growth Potential •  Prenatal insult, chromosomal anomalies, endocrine disorders Caloric Wasting •  Emesis: intestinal tract disorders, drugs, toxins, CNS pathology •  Malabsorption: GI disease (biliary atresia, celiac disease), inflammatory bowel disease, infections, toxins, metabolic abnormalities •  Renal loses: diabetes, renal tubular acidosis Increased Caloric Requirements •  Increased metabolism: congenital heart disease, chronic respiratory diseases (eg, cystic fibrosis and chronic lung disease), neoplasms, chronic infection, hyperthyroidism •  Defective use of calories: metabolic disorders, renal tubular acidosis Abbreviations: CNS, central nervous system; GI, gastrointestinal.

cases, results of the newborn state screen are critical. A child with cystic fibrosis in the family requires sweat chloride or genetic testing, especially if this testing is not included on the newborn state screen. A child with a loud, harsh systolic murmur and bounding pulses deserves a chest radiograph, an electrocardiogram (ECG), and perhaps an echocardiogram and cardiology consult. However, most FTT children have few or no signs; thus, laboratory evaluation is usually limited to a few screening tests: a complete blood count (CBC), C-reactive protein (CRP), erythrocyte sedimentation rate (ESR), lead level (especially for patients in lower socioeconomic classes or in cities with a high lead prevalence), thyroid and liver function tests, urinalysis and culture, serum electrolyte levels (including calcium, phosphorus, magnesium, blood urea nitrogen [BUN], and creatinine), stool studies (including occult blood, leukocyte presence, culture, ova and parasites, and Giardia antigen), and a chest radiograph. Abnormalities in screening tests are pursued more extensively. Other conditions (eg, HIV or tuberculosis) can be tested accordingly if they are suspected (see Figure 10–1).

Treatment The treatment and follow-up for organic FTT are specific to the condition. If the malnutrition remains chronic, the child’s neurologic development may be irreversibly affected, reinforcing the importance of early diagnosis and appropriate intervention with a multidisciplinary team (gastroenterologist, nutritionist, social worker, and therapists including occupational, speech, and physical). Patients with nonorganic FTT are managed with improved dietary intake, close follow-up, and attention to psychosocial issues. Dietary Intake.  Healthy infants in the first year of life require approximately 100 kcal/kg/d of nutrition and about 80 kcal/kg/d thereafter; FTT children require an

11_Toy-Pediatrics_Case10_p097-106.indd 101

27/03/21 9:06 AM

102

CASE FILES: PEDIATRICS

History, Physical Exam, and Developmental History No

Explore social history and caloric intake

Yes Consider CBC, serum electrolytes, BUN, Cr, Urinalysis, UC, ESR, TFT, LFT, TORCH titers

As History, Physical, Initial Labs indicate

• Immune studies • CXR, ECG, echocardiogram • CF studies • HIV, hepatitis studies, TB studies • Stool studies • Chromosomal studies Figure 10–1.  Sample algorithm for evaluation of FTT. Abbreviations: BUN, blood urea nitrogen; CBC, complete blood count; CF, cystic fibrosis; Cr, serum creatinine; CXR, chest x-ray; ECG, electrocardiogram; ESR, erythrocyte sedimentation rate; HIV, human immunodeficiency virus; LFT, liver function test; TB, tuberculosis; TFT, thyroid function test; TORCH, toxoplasmosis, other (syphilis, varicella-zoster, parvovirus B19), rubella, cytomegalovirus, and herpes, UC, urine culture.

additional 50% to 100% to ensure adequate catch-up growth. A mealtime routine is important. Families should eat together in a nondistracting environment, with meals lasting between 20 and 30 minutes. Solid foods should be offered before liquids; children should not be force-fed. Low-calorie drinks, juices, and water should be limited; age-appropriate, high-calorie foods (whole milk, cheese, dried fruits, peanut butter) should be encouraged. Formulas containing more than the standard 20 kcal/oz may be necessary for infants, and high-calorie supplementation (Pediasure, Ensure) may be required for older children. Frequent office or home health visits are indicated to ensure weight gain. In some instances, hospitalization of an FTT child is required; some hospitalized infants have rapid weight gain, supporting the diagnosis of nonorganic FTT. Psychosocial Issues.  Nonorganic FTT treatment requires not only the provision of increased calories, but also attention to contributing psychosocial issues. Referral to community services (Women, Infants, and Children [WIC] Program, Food Stamp Program, and local food banks) may be required. Caregiver help in the form of job training, substance and physical abuse prevention, parenting classes, and psychotherapy may be available through community programs. Older children and their families may benefit from Early Childhood Intervention (ECI) and Head Start programs.

11_Toy-Pediatrics_Case10_p097-106.indd 102

27/03/21 9:06 AM

SECTION II: CLINICAL CASES

103

Some children with organic FTT also have nonorganic FTT. For instance, a poorly growing special-needs premature infant is at increased risk for superimposed nonorganic FTT because of psychosocial issues, such as poor bonding with the family during a prolonged hospital stay. In such cases, care for the organic causes is coordinated with attempts to preclude nonorganic FTT.

CASE CORRELATION šš

See also Case 6 (Neonatal Herpes Simplex Infection), Case 7 (Esophageal Atresia), and Case 9 (Pediatric Eye Problems).

COMPREHENSION QUESTIONS 10.1 The parents of a 6-month-old boy bring their son to the office for a well-child visit. He is symmetrically less than the fifth percentile for height, weight, and head circumference on routine growth curves. He was born at 30 weeks of gestation and weighed 1000 g. He was a planned pregnancy, and his mother’s prenatal course was uneventful until an automobile accident initiated the labor. He was ventilated for 3 days in the neonatal intensive care unit (NICU) but otherwise did well without ongoing problems. He was discharged at 8 weeks of life. The examination appears otherwise normal. Which of the following is the mostly likely explanation for his small size? A. Chromosomal abnormality B. Protein-calorie malnutrition C. Normal ex-premature infant growth D. Intestinal malabsorption E. Congenital hypothyroidism 10.2 A 13-month-old child is noted to be at the 25th percentile for weight, the 10th percentile for height, and less than the fifth percentile for head circumference. She was born at term but was noted to have a small head circumference at birth and has been developmentally delayed throughout her life. She required cataract surgery shortly after birth. The child currently takes phenobarbital for seizures. Caloric intake has been deemed appropriate by history, and neither frequent emesis nor excessive stooling is reported. Her examination is remarkable for a small head and liver enlargement on abdominal palpation. Which of the following would most likely explain this child’s small size? A. Congenital infection B. Chromosomal abnormality C. Metabolic disorder D. Gastrointestinal dysmotility E. Increased intracranial pressure

11_Toy-Pediatrics_Case10_p097-106.indd 103

27/03/21 9:06 AM

104

CASE FILES: PEDIATRICS

10.3 A 2-year-old boy had been slightly less than the 50th percentile for weight, height, and head circumference, but in the past 6 months, he has fallen to slightly less than the 25th percentile for weight. The pregnancy was normal, his development is as expected, and the family reports no psychosocial problems. The mother says that he is now a picky eater (wants only macaroni and cheese at all meals), but she insists that he eat a variety of foods. The meals are marked by much frustration for everyone. His examination is normal. Which of the following is the best next step in his care? A. Sweat chloride testing B. Ophthalmologic examination for retinal hemorrhages C. Reassurance and counseling for family about normal childhood development D. Testing of stool for parasites E. Magnetic resonance imaging (MRI) of the brain 10.4 A 10-month-old child is seen in follow-up for poor weight gain. Her current length is at the 10th percentile, head circumference is at the 50th percentile, and weight is less than the fifth percentile. The examination is unremarkable except for small size. She was sent to the pediatric gastroenterologist about a month prior by a colleague after her weight was noted to have dropped from the 50th percentile on her 6-month-old visit to less than the fifth percentile on her 9-month-old visit. Perinatal history is unremarkable. Feeding is via breast and bottle with a standard milk-based formula; the quantity of feeds reported seems sufficient, and no excessive spit-up is reported. Various table foods are eaten and reportedly tolerated well. The child has had no recent or recurring illness. At the gastroenterology visit, a comprehensive array of laboratory and imaging studies were performed, as outlined in the referral letter, and were unhelpful in diagnosing the cause of the failure to thrive (FTT). The pediatric gastroenterologist’s recommendations at that time were to begin a 1-month trial of a 24 kcal/oz formula and follow-up with you for a weight check. Which of the following is the best next step in her care? A. Commence caloric supplementation with 27 kcal/oz formula B. Counsel family regarding diet and schedule follow-up weight check in 6 months C. Commence growth hormone administration D. Refer to dietician E. Admit to the hospital

ANSWERS 10.1 C. Normal ex-premature infant growth. The expected weight versus age must be modified for a preterm infant. Similarly, growth for children with Down or Turner syndrome varies from that of other children. Thus, use of an appropriate growth curve is paramount. For the child in the question, weight gain

11_Toy-Pediatrics_Case10_p097-106.indd 104

27/03/21 9:06 AM

SECTION II: CLINICAL CASES

105

should follow or exceed that of term infants. For this premature infant, when his parameters are plotted on a premature infant growth chart, normal growth is revealed. Chromosomal abnormality (answer A) is less likely because the physical exam appears normal. Malnutrition (answer B) or intestinal malabsorption (answer D) would most likely lead to head sparing or asymmetric growth issues. Congenital hypothyroidism (answer E) is a possible etiology but is less likely than the obvious preterm gestation. 10.2 A. Congenital infection. Developmental delay, intrauterine growth restriction (including microcephaly), cataracts, seizures, hepatosplenomegaly, prolonged neonatal jaundice, and purpura at birth are consistent with congenital cytomegalovirus (CMV) or toxoplasmosis infection. Calcified brain densities of CMV typically are found in a periventricular pattern; in toxoplasmosis, they are found scattered throughout the cortex. Children with chromosomal anomalies (answer B) have some of the aforementioned findings such as cataracts and developmental delay and microcephaly, but not hepatosplenomegaly. A patient with a metabolic disorder (answer C) would usually present later in infancy and not immediately with microcephaly. A patient with increased intracranial pressure (answer E) would present with an enlarged head circumference. 10.3 C. Reassurance and counseling for family about normal childhood development. Between 18 and 30 months of age, children often become “picky eaters.” Their growth rate can plateau, and this period can be distressing for families. Of note, this patient’s growth decline has not crossed two major growth percentiles to define FTT. Counseling parents on how to provide optimal nutrition, avoid force-feeding, and avoid providing snacks is usually effective. Close follow-up is required. Further evaluation for malnutrition, such as answer A (sweat chloride test) or answer D (testing stool for oval and parasites), is not indicated at this time. Evaluation for possible retinal hemorrhages (answer B) is done when there is suspicion of child abuse, but currently, there are no red flags for abuse. MRI of the brain (answer E) would be indicated for neurologic conditions, but the development and neurologic exams are normal in this patient. 10.4 E. Admit to the hospital. Without evident abnormality on examination and with a presumed thorough outpatient FTT workup, this infant probably has nonorganic FTT. Of note, the infant is already on a calorie-fortified formula (answer A) in addition to breast milk. Although it might be appropriate to solicit advice from a dietician (answer D), the best response would be to consider admission to the hospital for a multidisciplinary assessment of her FTT. A 2- to 3-day stay to watch intake (and perform a calorie count) and daily weights, while selected services comment on the patient and her family (eg, social worker, dietician, pediatric gastroenterologist), might uncover a nonorganic cause for her FTT (parental neglect). Growth hormone administration (answer C) and further counseling regarding diet (answer B) without close follow-up are not standard of care.

11_Toy-Pediatrics_Case10_p097-106.indd 105

27/03/21 9:06 AM

106

CASE FILES: PEDIATRICS

CLINICAL PEARLS »»

In the United States, psychosocial failure to thrive (FTT) is more common than organic FTT; it often is associated with poverty or poor parent-child interaction.

»»

Inexpensive laboratory screening tests, dietary counseling, and close observation of weight changes are appropriate first steps for most healthy-appearing infants with FTT.

»»

Organic FTT can be associated with abnormalities of any organ system. Clues in history, examination, and selected screening laboratory tests may help identify affected organ systems.

»»

Up to one-third of patients with psychosocial FTT have developmental delay, as well as social and emotional problems.

REFERENCES Bamba V, Kelly A. Assessment of growth. In: Kleigman RM, St. Geme JW, Blum NJ, et al, eds. Nelson Textbook of Pediatrics. 21st ed. Philadelphia, PA: Elsevier; 2020:151-157. Britt WJ. Cytomegalovirus. In: Kleigman RM, St. Geme JW, Blum NJ, et al, eds. Nelson Textbook of Pediatrics. 21st ed. Philadelphia, PA: Elsevier; 2020:1718-1723. Chiesa A, Sirotnak AP. Child abuse and neglect. In: Hay WW Jr, Levin MJ, Deterding RR, Abzug MJ. eds. Current Diagnosis & Treatment: Pediatrics. 24th ed. New York, NY: McGraw Hill; 2018. Homan GJ. Failure to thrive: a practical guide. Am Fam Physician. 2016;94(4):295-299. Kirkland RT. Failure to thrive. In: McMillan JA, Feigin RD, DeAngelis CD, Jones MD, eds. Oski’s Pediatrics: Principles and Practice. 4th ed. Philadelphia, PA: Lippincott Williams & Wilkins; 2006:900-906. Puls HT, Plencner L, Krager M, et al. The diagnostic accuracy of in-hospital weight gain for differentiating neglect from other failure to thrive etiologies. Hosp Pediatr. 2018;8(10):620-627. Raszka WV. Neonatal toxoplasmosis. In: McMillan JA, Feigin RD, DeAngelis CD, Jones MD, eds. Oski’s Pediatrics: Principles and Practice. 4th ed. Philadelphia, PA: Lippincott Williams & Wilkins; 2006:530-532. Redel CA. Feeding difficulties in infants and children. In: Kline MW, Blaney SM, Giardino AP, et al, eds. Rudolph’s Pediatrics. 23rd ed. New York, NY: McGraw Hill; 2018:101-106. Shaw JS, Palfrey JS. Health maintenance issues. In: Kline MW, Blaney SM, Giardino AP, et al, eds. Rudolph’s Pediatrics. 23rd ed. New York, NY: McGraw Hill; 2018:39-45. Treitz M, Nicklas D, Bunik M, Fox D. Ambulatory and office pediatrics. In: Hay WW Jr, Levin MJ, Deterding RR, Abzug MJ, eds. Current Diagnosis & Treatment: Pediatrics. 24th ed. New York, NY: McGraw Hill; 2018.

11_Toy-Pediatrics_Case10_p097-106.indd 106

27/03/21 9:06 AM

CASE 11 A 9-month-old boy presents to the office for his well-child visit. His mother reports that he has been in good health but notes that he enjoys eating rocks and dirt that he finds outside. He was born vaginally at 36 weeks’ gestation after an uncomplicated pregnancy. He was breastfed for the first 4 months of life and was then switched to cow’s milk and a variety of baby foods. She reports that he has an excellent appetite. He has had no illnesses and has never required medications or supplements. The family history is unremarkable. Vital signs include temperature of 98.6 °F (37 °C), heart rate of 145 beats per minute, and respiratory rate of 18 breaths per minute. The conjunctivae are pale, but he has no dysmorphic features, generalized jaundice, scleral icterus, or organomegaly. His complete blood count (CBC) shows a hemoglobin of 10 g/dL and mean corpuscular volume (MCV) of 65 fL. ▶▶ ▶▶ ▶▶ ▶▶

What is the most likely cause of this child’s behavior? What are the patient’s preventable risk factors? What is the next step in evaluation? How is this condition treated?

12_Toy-Pediatrics_Case11_p107-116.indd 107

29/07/21 6:51 PM

108

CASE FILES: PEDIATRICS

ANSWERS TO CASE 11: Anemia in the Pediatric Patient Summary: A 9-month-old boy presents with šš

A history of being a late preterm infant born vaginally after an uncomplicated pregnancy

šš

A history of breastfeeding without vitamin supplementation

šš

An early introduction to cow’s milk

šš

A propensity for eating dirt and rocks

šš

A hemoglobin of 10 g/dL and a low MCV

Most likely cause: Iron deficiency anemia secondary to excessive cow’s milk intake in an infant under 12 months old. Preventable risk factors: Avoid unmodified (nonformula) cow’s milk in infants under 1 year of age. Iron supplementation should be provided to exclusively breastfed patients after 4 months of age for term infants. Next step in evaluation: No further workup is needed; this is a typical presentation of iron deficiency anemia. Treatment: Begin treatment with 3 to 6 mg/kg daily of elemental iron. Recheck hemoglobin and hematocrit in 4 weeks.

ANALYSIS Objectives 1. Understand the indications for anemia workup. (EPA 1, 2) 2. Describe the typical findings in iron deficiency anemia. (EPA 1, 3) 3. Learn the differential for iron deficiency anemia. (EPA 2)

Considerations Anemia, which is defined as a low hemoglobin level or low red blood cell (RBC) mass, is common in children. About 20% of US children will be diagnosed with anemia at some point. For this reason, the American Academy of Pediatrics and the World Health Organization recommend universal screening for anemia at 12 months of age. The evaluation of anemia should focus on diet, possible blood loss, vitamin deficiency, genetic disorders, RBC disorders, and lead poisoning. Anemia can be divided into three general categories: impaired production, increased destruction, and blood loss. A full evaluation for anemia includes a thorough medical history (including diet, exposures, and personal medical and family histories) and a comprehensive physical examination.

12_Toy-Pediatrics_Case11_p107-116.indd 108

29/07/21 6:51 PM

SECTION II: CLINICAL CASES

109

APPROACH TO: Anemia in the Pediatric Patient DEFINITIONS ANEMIA: Hemoglobin concentration more than two standard deviations below the mean for age. Typical values are less than 13.5 g/dL from birth to 1 month; less than 11 g/dL from age 1 month to 12 years; and less than 12 g/dL in females age 12 years or older and less than 13 g/dL in males 12 years or older. ANISOCYTOSIS: Variation in RBC size. Indicated by elevated RBC distribution width (RDW). MEAN CORPUSCULAR HEMOGLOBIN CONCENTRATION (MCHC): Average hemoglobin concentration per RBC. Hypochromia is seen with low MCHC. MEAN CORPUSCULAR VOLUME: Average RBC size, with normal values defined by standard deviations from mean. Used to subdivide anemia into microcytic (small cells), normocytic, and macrocytic (large cells). Normal ranges are dependent on age, race, and gender. PICA: Desire to ingest nonfood items, such as dirt, rocks, or chalk, that is seen in iron deficiency state. A subtype of pica is pagophagia, the desire to eat ice, which is particularly specific for iron deficiency. RETICULOCYTE COUNT: Immature RBCs that are 1 to 3 days old expressed as a percentage of all RBCs. Normal reticulocyte percentage in nonanemic patient is 1.5%.

CLINICAL APPROACH Iron Deficiency Anemia Epidemiology and Pathophysiology.  Iron deficiency anemia (IDA) is one of the most common causes of anemia. IDA can be seen at any age, although infancy and adolescence are two of the more common pediatric populations affected. Infants require high levels of iron due to rapid growth. Maternal iron stores, which are primarily transferred late in pregnancy, are typically sufficient in the early newborn period for term infants. Thus, premature infants are at increased for IDA. Clinical Presentation.  Symptoms may include fatigue, poor appetite, pica, behavioral problems, dizziness, and frequent infections; signs of IDA are pallor, tachycardia, glossitis and spoon nails. For patients with established risk factors, such as a teenager with heavy menses or an infant with intake of unfortified cow’s milk, a CBC consistent with IDA (low MCV, low RBC, high RDW) is sufficient for a diagnosis of presumed iron deficiency; no further testing is indicated. Labs should be repeated 1 month after treatment is begun.

12_Toy-Pediatrics_Case11_p107-116.indd 109

29/07/21 6:51 PM

110

CASE FILES: PEDIATRICS

Prevention.  Iron supplementation for non-anemic patients is based on severity, age, whether or not the child is breastfed, and birth history. šš

šš

šš

šš

šš

For the exclusively breastfed premature infant, begin supplementation with 2 mg/kg elemental iron at 2 weeks of age. For the exclusively breastfed term baby, begin 1 mg/kg elemental iron daily at 4 months of age if solid foods containing iron are not being introduced until later. For both groups, continue supplementation until the infant is receiving an equivalent dose of iron through their diet, typically occurring at transition to iron-containing infant foods. Cow’s milk has low iron content. Infants under 12 months of age should not drink cow’s milk, and toddlers should be limited to 24 oz daily. IDA in adolescent females is most commonly associated with blood loss from menses.

Treatment.  After a presumptive diagnosis of IDA, begin treatment with 3-6 mg/kg of oral elemental iron daily (depending on severity) and repeat labs in 1 month. If the hemoglobin does not increase more than 1 g/dL after 1 month, other etiologies of anemia are considered such as noncompliance, concurrent vitamin B12 or folic acid deficiency, or malabsorption syndromes. In these instances, assessment with a reticulocyte count, iron panel, ferritin, and peripheral smear is warranted.

Other Microcytic Anemias Microcytic anemia can be seen with many conditions, the most common of which are alpha- and beta-thalassemia minor, anemia of chronic disease, lead poisoning, and sideroblastic anemia (Table 11–1). Table 11–1  •  DIFFERENTIATING CHARACTERISTICS OF COMMON MICROCYTIC ANEMIAS Microcytic Anemia

Iron Anemia of Deficiency Chronic Anemia Disease

Sideroblastic AlphaBetaAnemias Thalassemia Thalassemia

MCV



—/↓

Variablea





RDW



—/↑

↑↑





Serum iron

↓↓



↑↑

—/↑



Serum ferritin







—/↑

—/↑

TIBC





—/↓



Hemoglobin electrophoresis

Normal

Normal

Normal

Normal

— b

↑ HbA2 ± ↑ HbF

Abbreviations: HbA2, hemoglobin alpha 2; HbF, fetal hemoglobin; MCV, mean corpuscular volume; RDW, red cell distribution width; TIBC, total iron-binding capacity. a Elevated, low, or normal MCV seen in sideroblastic anemias. b Normal hemoglobin electrophoresis seen in alpha thalassemia minor (or alpha-thalassemia trait) after the newborn period. Abnormal results seen shortly after birth due to elevated Hb Barts (3%-8%).

12_Toy-Pediatrics_Case11_p107-116.indd 110

29/07/21 6:51 PM

SECTION II: CLINICAL CASES

111

Lead Poisoning.  Risk factors include residing in housing built prior to 1978 due to lead-based paint, pica, toys painted with lead-based paint, or lead-contaminated soil. Lead poisoning does not improve with iron treatment. Pica, which can be a manifestation of iron deficiency, can expose children to lead through ingestion of contaminated soil, dust, and paint chips. Capillary blood lead levels are used to test for lead poisoning; confirmation is with venous sampling. Chelation is initiated for venous blood levels of 45 mcg/dL or greater. Alpha-Thalassemia and Beta-Thalassemia Minor.  Alpha-thalassemia and betathalassemia minor generally have a mild presentation and are often initially mistaken for IDA. Patients with thalassemia are usually of Mediterranean, African, or Southeast Asian descent. Suspect alpha-thalassemia if hypochromic, microcytic anemia is present in an infant under 6 months old. Neither beta-thalassemia minor nor IDA (in term infants) typically presents until after 6 months of age. Anemia of Chronic Disease.  Anemia of chronic disease (ACD) is a common cause of microcytosis, although it can also cause normocytic anemia. It is associated with inflammatory conditions such as rheumatologic disorders, malignancy, and chronic infection. Treatment of mild ACD is focused on management of the underlying condition. Sideroblastic Anemias.  Sideroblastic anemias are characterized by defects in mitochondrial iron utilization, which lead to impaired heme synthesis and subsequent accumulation of iron within the mitochondria. The most common is X-linked sideroblastic anemia (XLSA). Vitamin B6 (pyridoxine) is an essential cofactor of the affected enzyme; XLSA responds well to vitamin B6 supplementation. Although uncommon, sideroblastic anemia can be acquired, most frequently in children being treated with isoniazid for tuberculosis.

Macrocytic Anemia Similar to microcytic anemia, the initial evaluation beyond history and physical examination includes a reticulocyte count and peripheral smear. Common causes of macrocytic anemia with low reticulocyte count are vitamin deficiencies (eg, vitamin B12 and folic acid), hypothyroidism, trisomy 21, congenital anemias (eg, Fanconi and Diamond-Blackfan), and medications. Hemolysis results in a high reticulocyte count with macrocytosis because reticulocytes are larger than mature RBCs. Megaloblastic Anemia.  Megaloblastic anemia is a macrocytic anemia that occurs due to dysfunctional RBC maturation. Vitamin B12 and folic acid are particularly significant in RBC development due to rapid division of precursor RBCs. Impaired development leads to relatively large nuclei in megaloblastic RBCs. An associated finding with megaloblastic anemia is hypersegmented neutrophils. Vitamin B12 and folate deficiencies can be the result of inadequate intake, inborn error of metabolism (eg, homocystinuria and hyperhomocysteinemia), impaired absorption, or drug interactions. Inadequate B12 intake is uncommon in pediatric patients in developed countries except in infants. In older children, B12 stores last 3 to 5 years, so low levels are not seen without prolonged deficiency. Breastfed infants

12_Toy-Pediatrics_Case11_p107-116.indd 111

29/07/21 6:51 PM

112

CASE FILES: PEDIATRICS

of mothers with low vitamin B12 levels are also at risk for vitamin B12 deficiency. Individuals with a vegan diet or gastric malabsorption are at risk for low vitamin B12. In contrast to B12, folic acid stores can be depleted after only 2 to 3 months of a folatedeficient diet. Folic acid deficiency is seen in infants who drink large amounts of goat milk or have a diet devoid of green leafy vegetables or fortified grains foods. Deficiency can also develop in the setting of hemolysis and increased cell turnover (ie, sickle cell disease); folic acid supplementation is often recommended.

CASE CORRELATION šš

See also Case 3 (Neonatal Hyperbilirubinemia) and Case 13 (Sickle Cell Disease).

COMPREHENSION QUESTIONS 11.1 An 11-month-old African American girl presents for follow-up of presumed IDA. She was diagnosed a month ago after her parents noticed regular ingestion of dirt found near her house built in the 1950s. The initial evaluation demonstrated hemoglobin of 10 g/dL and MCV of 70 fL, and iron treatment was initiated. Repeat labs show no change in the hemoglobin level. Additional parental concerns now include absence of babbling and verbal communication and persistently low energy levels. Which of the following is the most appropriate next step? A. Begin daily folic acid supplementation B. Check a blood lead level C. Continue oral iron treatment and recheck hemoglobin in 3 months D. Obtain long bone radiographs E. Stop iron treatment and begin vitamin B12 11.2 A 4-month-old boy is seen in the clinic for pallor and a gradual decrease in energy level. Physical examination of the infant is notable for microcephaly and a broad, flat nasal bridge. There is delayed capillary refill. The cardiac exam reveals a holosystolic murmur at the left sternal border. Exam of the hands shows triphalangeal thumbs. No recent infections are reported. The CBC shows a hemoglobin of 4.5 g/dL, MCV of 110 fL, white blood cell count of 9000 cells/mm3, and 2% reticulocytes. Family history is significant for the patient’s father requiring regular blood transfusions, although his mother is unsure why. Which of the following would be expected to be seen on a peripheral smear of this patient? A. Hypersegmented neutrophils B. Microcytic RBCs C. Normal platelet count D. Pica bodies E. Target cells

12_Toy-Pediatrics_Case11_p107-116.indd 112

29/07/21 6:51 PM

SECTION II: CLINICAL CASES

113

11.3 A previously healthy 12-year-old girl presents to the clinic for a well-child visit. Recent history is notable for decreased oral intake due to a sore, red tongue. The patient and her family adhere to a strict vegan diet. She has not started her menses yet. Laboratory evaluation shows a hemoglobin of 10 g/dL and MCV of 110 fL. Peripheral smear reveals large RBCs and neutrophils with four to six lobes each. Which of the following is most consistent with this patient’s likely diagnosis? A. High methylmalonic acid (MMA), high homocysteine, high cobalamin B. High MMA, high homocysteine, low cobalamin C. Low iron, low ferritin, high total iron-binding capacity (TIBC) D. Low iron, high ferritin, low TIBC E. Normal MMA, high homocysteine, high cobalamin 11.4 A 4-month-old former term infant presents for follow-up of anemia, diagnosed on laboratory data obtained at an outside pediatrician. Dietary history is negative for goat’s milk or cow’s milk intake. Both of her parents are from the same town in Greece. Current laboratory evaluations show a hemoglobin of 9.5 g/dL, MCV of 60 fL, target cells, and normal hemoglobin electrophoresis. RDW, serum iron, serum ferritin, and TIBC are normal. Which of the following is the best next step in management? A. Check blood lead level B. Provide preconception counseling for parents C. Repeat hemoglobin electrophoresis in 2 weeks D. Send globin gene testing E. Start pyridoxine 100 mg/d orally

ANSWERS 11.1 B. Check a blood lead level. This patient’s presentation is concerning for lead poisoning due to her history of pica and exposure to a house built before the 1960s. Continued iron therapy (assuming adherence) without further evaluation (answer  C) would be inappropriate. The majority of patients with lead poisoning will initially be asymptomatic, but a lack of babbling (language delay) and neurocognitive effects (impaired verbal abilities and speech) are some of the first changes seen even with low lead concentrations. Long bone radiographs (answer D) are no longer recommended as part of lead exposure workup. Daily folic acid supplementation (answer A) or vitamin B12 supplementation (answer E) would not be effective in this patient’s condition. 11.2 C. Normal platelet count. This infant has Diamond-Blackfan anemia (also known as Blackfan-Diamond or congenital hypoplastic anemia), an autosomal dominant syndrome that typically presents before 6 months of age with severe macrocytic anemia, reticulocytopenia, and multisystemic congenital

12_Toy-Pediatrics_Case11_p107-116.indd 113

29/07/21 6:51 PM

114

CASE FILES: PEDIATRICS

anomalies. Unlike many inherited anemias, platelet and white blood cell counts are normal or mildly increased. Associated anomalies include high-arched or cleft palate, “snub nose” (broad, flat nasal bridge), triphalangeal thumbs (three bones in thumb instead of the normal two), shield chest, and cardiac defects such as ventricular septal defect, atrial septal defect, and coarctation of the aorta. It is distinguished from transient erythroblastopenia of childhood by elevated erythrocytic adenosine deaminase (eADA) activity and persistence of fetal hemoglobin (HbF). Steroids and transfusions are the mainstay of treatment, although steroid initiation is often deferred until at least 6 months of age. Hypersegmented neutrophils (answer A) are seen with megaloblastic anemia (eg, vitamin B12 or folic acid deficiency). Microcytic cells (answer B) are associated with low MCV, not high MCV. Target cells (answer E) are seen with hemoglobinopathies and liver disease. 11.3 B. High MMA, high homocysteine, low cobalamin. This patient has vitamin B12 deficiency as a result of a strict vegan diet. Vitamin B12 deficiency presents as a macrocytic anemia with megaloblastic changes on peripheral smear. Patients may present with glossitis (vs oral ulcers in folic acid deficiency), ataxia, or fatigue, or they may be completely asymptomatic. Deficiency can be diagnosed by a low vitamin B12 level. Serum vitamin B12 levels have low sensitivity and specificity for deficiency, so homocysteine and MMA levels are also used. Elevated levels of both MMA and homocysteine are indicative of vitamin B12 deficiency. Low iron, low ferritin, and high TIBC (answer C) are consistent with IDA, typically due to menstruation-related blood loss. Low iron, high ferritin, and low TIBC (answer D) are consistent with anemia of chronic disease; patients who are otherwise healthy typically presenting with microcytic or normocytic anemia, not macrocytic. Normal MMA, high homocysteine, and low cobalamin (answer E) are consistent with folic acid deficiency. 11.4 D. Send globin gene testing. This patient likely has alpha-thalassemia minor, as suggested by her Mediterranean heritage, presentation before 6 months of age, microcytic anemia with normal RDW, and target cells on peripheral smear. Iron studies can be normal or demonstrate high iron and high ferritin. This is diagnosed by globin gene testing. Hemoglobin electrophoresis (answer C) outside of the newborn period is normal; repeating would not assist in diagnosis or management. Preconception counseling (answer B) is important after the diagnosis. Alpha-thalassemia minor occurs when two out of four alpha-globin genes are deleted. Deletion of all alpha-globin genes is incompatible with life, and deletion of three alpha-globin genes leads to alpha-thalassemia major. Pyridoxine (answer E) is appropriate in X-linked sideroblastic anemia (XLSA), a type of sideroblastic anemia. Although the peripheral smear of thalassemia and sideroblastic anemias can have shared findings such as basophilic stippling, target cells are not associated with XLSA. A blood lead level is not indicated (answer A); the patient does not have risk factors, and the degree of microcytosis described would be unusual for lead poisoning.

12_Toy-Pediatrics_Case11_p107-116.indd 114

29/07/21 6:51 PM

SECTION II: CLINICAL CASES

115

CLINICAL PEARLS »»

Iron deficiency anemia, which causes a microcytic anemia, can be due to an excess or early intake of cow’s milk.

»»

Folic acid deficiency, which leads to a macrocytic and megaloblastic anemia, can be the result of excess goat’s milk in infants. Labs demonstrate high homocysteine and normal methylmalonic acid levels.

»»

Iron prophylaxis is begun for premature infants at 2 weeks of age, and term infants at 4 months of age.

»»

Treatment of IDA consists of 3-6 mg/kg/d of elemental iron. The hemoglobin should rise by at least 1 g/dL within 1 month.

»»

Serum ferritin less than 12 ng/mL in a patient under 5 years old and less than 15 ng/mL in a patient over 5 years old is diagnostic of IDA.

»»

Microcytic anemia before 6 months old in a term infant is unlikely to be due to nutritional iron deficiency or beta-thalassemia minor.

»»

Congenital hemolytic anemia and drug-induced hemolytic anemia can present with neonatal hyperbilirubinemia.

»»

Sickle cell disease patients, by definition, are anemic due to the reduced life span of RBCs. It is important to establish a baseline hemoglobin in sickle cell disease patients.

REFERENCES Bunn H, Heeney MM. Iron homeostasis: deficiency and overload. In: Aster JC, Bunn H, eds. Pathophysiology of Blood Disorders. 2nd ed. New York, NY: McGraw Hill; 2017. Bunn H, Sankaran VG. Thalassemia. In: Aster JC, Bunn H, eds. Pathophysiology of Blood Disorders. 2nd ed. New York, NY: McGraw Hill; 2017. Cembrowski GS, Chan J, Cheng C. NHANES 1999-2000 data used to create comprehensive healthassociated race-, sex- and age-stratified pediatric reference intervals for the Coulter MAXM. Laboratory Hematol. 2004;10:245. Markowitz M. Lead poisoning. In: Kliegman RM, St Geme JW, Blum NJ, Shah SS, Tasker RC, Wilson KM, eds. Nelson Textbook of Pediatrics. 21st ed. Philadelphia, PA: Elsevier; 2020:3797-3802. Powers JM, Heeney MM. Anemia. In: Kline MW, Blaney SM, Giardino AP, et al, eds. Rudolph’s Pediatrics. 23rd ed. New York, NY: McGraw Hill; 2018:1569. Rothman JA. Iron-deficiency anemia. In: Kliegman RM, St Geme JW, Blum NJ, Shah SS, Tasker RC, Wilson KM, eds. Nelson Textbook of Pediatrics. 21st ed. Philadelphia, PA: Elsevier; 2020:2522-2526. Smith-Whitley K, Kwiatkowski JL. Hemoglobinopathies. In: Kliegman RM, St Geme JW, Blum NJ, Shah SS, Tasker RC, Wilson KM, eds. Nelson Textbook of Pediatrics. 21st ed. Philadelphia, PA: Elsevier; 2020:2540-2558. Thornburg CD. Megaloblastic anemias. In: Kliegman RM, St Geme JW, Blum NJ, Shah SS, Tasker RC, Wilson KM, eds. Nelson Textbook of Pediatrics. 21st ed. Philadelphia, PA: Elsevier; 2020:2517-2521.

12_Toy-Pediatrics_Case11_p107-116.indd 115

29/07/21 6:51 PM

This page intentionally left blank

CASE 12 An 8-month-old child has a 24-hour history of increased crying when she moves her right leg. She has a prominent bulge over the mid-right thigh, where she had received an immunization the previous day. She has not had fever or a change in appetite, and she seems upset only when the leg is disturbed. The child underwent a Kasai procedure for biliary atresia and is awaiting a liver transplant. A radiograph of the leg demonstrates a mid-shaft fracture and poor mineralization. ▶▶ ▶▶

What is the mechanism for this condition? What are the best diagnostic tests to diagnose this condition?

13_Toy-Pediatrics_Case12_p117-128.indd 117

29/07/21 6:50 PM

118

CASE FILES: PEDIATRICS

ANSWERS TO CASE 12: Rickets Summary: An 8-month-old child presents with šš

A 24-hour history of increased crying when moving her right leg

šš

A prominent bulge over the mid-right thigh after receiving an immunization

šš

A chronic medical condition, including biliary atresia

šš

Poor bone mineralization

šš

A mid-shaft fracture

Mechanism: Malabsorption of vitamin D (among other fat-soluble vitamins) due to lack of intestinal secretion of bile salts, resulting in rickets. Best diagnostic tests: Serum 25-hydroxyvitamin D [25(OH)D], calcium, phosphorus, and alkaline phosphatase levels. Radiographs demonstrate poor bone mineralization.

ANALYSIS Objectives 1. Become familiar with the clinical presentation of rickets. (EPA 1) 2. Understand the pathophysiology behind nutritional and nonnutritional rickets. (EPA 11, 12) 3. Appreciate some of the other causes of childhood fractures. (EPA 2)

Considerations This child has biliary atresia and underwent a Kasai procedure. Metabolic aberrations are expected while this child awaits liver transplantation. A review of her medications and adherence in receiving them is warranted. Because of the brittle nature of her bones, her leg was fractured while receiving immunizations.

APPROACH TO: Rickets DEFINITIONS BILIARY ATRESIA: A congenital condition affecting approximately 1 in 16,000 live births in which the liver’s bile ducts become blocked and fibrotic, resulting in reduced bile flow into the bowel. CRANIOTABES: Thinning of the bones of the skull.

13_Toy-Pediatrics_Case12_p117-128.indd 118

29/07/21 6:50 PM

119

SECTION II: CLINICAL CASES

GENU VALGUM: “Knock” knees, with knee misalignment that turns both knees inward. GENU VARUM: “Bowed” legs. KASAI PROCEDURE: An operative procedure in which a bowel loop forms a duct to allow bile to drain from a liver with biliary atresia. RACHITIC ROSARY: Enlarged costochondral junction along anterior part of chest wall. RICKETS: Poor mineralization of growing bone or of osteoid tissue.

CLINICAL APPROACH Pathophysiology Rickets refers to a “softening of the bones.” Calcium-deficient rickets is due to insufficient intake or metabolism of vitamin D or calcium. Phosphate-deficient rickets is due to renal phosphate wasting. The predominant cause for rickets is vitamin D deficiency. Vitamin D is synthesized in the skin after exposure to ultraviolet (UV) radiation. It then binds to vitamin D–binding protein (DBP), undergoes 25-hydroxylation in the liver, and then further undergoes 1-hydroxylation in the kidney, thereby converting to the metabolically active form. The active form is essential for intestinal calcium absorption. The level of 25(OH)D is the best indicator of vitamin D status and stores. These levels can be categorized as vitamin D sufficiency, insufficiency, or deficiency, defined as follows. šš

Vitamin D sufficiency: greater than 20 ng/mL

šš

Vitamin D insufficiency: 15-20 ng/mL

šš

Vitamin D deficiency: less than 15 ng/mL

Alkaline phosphatase is also a good screening tool; if the result is greater than the 95th percentile for age, 25(OH)D, calcium, phosphorus, and parathyroid hormone (PTH) levels should be checked.

Risk Factors Populations at risk for vitamin D deficiency include children who: šš

Are exclusively breastfed

šš

Are born to vitamin D–deficient mothers

šš

Have darker skin tones

šš

Are born prematurely

šš

Have limited sun exposure (especially during the winter season)

šš

Partake in vegetarian or vegan diets

šš

Take anticonvulsant or antiretroviral medications

13_Toy-Pediatrics_Case12_p117-128.indd 119

29/07/21 6:50 PM

120

šš šš

šš

CASE FILES: PEDIATRICS

Are obese Have conditions associated with malabsorption (celiac disease, inflammatory bowel disease [IBD], cystic fibrosis, gut resection) Have liver or kidney disease

Vitamin D deficiency is uncommon in formula-fed infants but can occur if the mother was vitamin D deficient during pregnancy or if the quantity of formula intake or vitamin D content of the formula is insufficient to compensate.

Calcipenic Rickets Stage 1 calcipenic rickets presents with hypocalcemia. Stage 2 presents with normal calcium but low phosphorus and increased alkaline phosphatase because of increased PTH secretion. Stage 3 presents with low calcium because PTH compensation reaches its limit. Symptoms of severe hypocalcemia include seizures, tetany (neuromuscular excitability leading to muscle contractions), poor feeding, vomiting, apneic spells, stridor, wheezing, hypotonia, lethargy, hyperreflexia, and arrhythmias. In addition to the stages of calcipenic rickets, there are four categories of etiologies, as follows: 1. Nutritional rickets: Inadequate intake of vitamin D and/or calcium (Figure 12–1). 2. Vitamin D–dependent rickets type I: Defective conversion of 25(OH)D to 1,25(OH)2D, which leads to early onset of skeletal disease, enamel hypoplasia, and severe hypocalcemia. 3. Hereditary vitamin D–resistant rickets (vitamin D-dependent rickets type II): End-organ resistance to vitamin D secondary to a mutation in the vitamin D receptor (autosomal recessive disorder). 4. Defects in vitamin D metabolism (liver or kidney disease), defects in absorption of calcium or vitamin D, and biochemical abnormalities in calcium or phosphorus metabolism (Table 12–1).

Phosphopenic Rickets Renal phosphate wasting causes phosphopenic rickets; low serum phosphorus combined with normal PTH concentrations is the most defining symptom. Causes may include renal tubular disorders (Fanconi syndrome), X-linked hypophosphatemic rickets, tumor-induced osteomalacia, and hereditary hypophosphatemic rickets with hypercalciuria. X-linked hypophosphatemic rickets is the most common cause of isolated renal phosphate loss and is caused by mutations in the PHEX gene. This congenital condition becomes clinically apparent when the child begins to walk. In affected children, phosphate reabsorption is defective, and conversion of 25(OH)D to 1,25(OH)2D in the proximal tubules of the kidneys is also decreased. Children at the age of walking present with smooth lower-extremity bowing (as compared to angular

13_Toy-Pediatrics_Case12_p117-128.indd 120

29/07/21 6:50 PM

SECTION II: CLINICAL CASES

Sunlight

121

280-305 nm 24

CH3

CH3 25

20

CH3

Cholesterol 7-dehydrocholesterol H

Previtamin D3

19

CH3

Vitamin D3 3

1

HO

Vitamin D3 or cholecalciferol

DBP

Liver 25-dehydroxylase

25-OHD3



25-hydroxyvitamin D3

PTH

+

↓PO4

↓Ca

24,25(OH)2D3

25-OHD31-hydroxylase

↑PO4



↑Ca

1,25(OH)2D3

Kidney

1,25-dihydroxyvitamin D3 25-OHD324-hydroxylase

↑PO4

+

↑Ca

1,25(OH)2D3 –

24,25(OH)2D3 24R,25-dihydroxyvitamin D3

Figure 12–1.  Vitamin D metabolism.

bowing of calcium-deficient rickets), a waddling gait, genu varum, genu valgum, short stature, craniostenosis, and spontaneous dental abscesses.

Premature Infants This population (less than 37 gestational weeks) is at risk for bone disease because substantial mineralization occurs between 32 and 36 weeks of gestation; 80% of calcium and phosphorus is acquired through placental transfer during the third trimester.

13_Toy-Pediatrics_Case12_p117-128.indd 121

29/07/21 6:50 PM

122

13_Toy-Pediatrics_Case12_p117-128.indd 122

Table 12–1  •  COMMON CAUSES OF ABNORMAL METABOLISM OF CALCIUM AND PHOSPHORUS Serum Calcium

Serum Phosphorus

Serum Alkaline Phosphatase

Urine Amino Acids Comments

Calcium deficiency with secondary hyperparathyroidism [vitamin D deficiency, decreased conversion to 1,25(OH)2D or renal osteodystrophy] Lack of vitamin D (lack of exposure to sunlight; dietary deficiency, congenital)

N or ↓







Unusual except in dark-skinned infants without vitamin D supplementation or in exclusively breastfed infants without exposure to sunlight

Malabsorption of vitamin D

N or ↓







Such as in celiac disease, cystic fibrosis, or steatorrhea

Hepatic disease

N or ↓







See discussion of case

Anticonvulsive drugs

N or ↓







Usually phenobarbital and phenytoin; patients have reduced 25(OH)D levels, possibly because of increased cytochrome P450 activity; treatment is with vitamin D2 and adequate dietary calcium

Renal osteodystrophy

N or ↓





Variable

Decreased renal excretion of phosphate results in hypocalcemia that then stimulates parathyroid secretion and enhanced bone turnover; in addition, diminished conversion of 25(OH)D to 1,25(OH)2D occurs as renal damage progresses

Vitamin D–dependent type I



N or ↓





Autosomal recessive; believed to be reduced activity of 25(OH)D1, 1 alpha-hydroxylase; responds to massive doses of vitamin D2 or lowdose 1,25(OH)2D



N

X-linked dominant; most common form of nonnutritional rickets (see text)

Phosphate deficiency without secondary hyperparathyroidism X-linked hypophosphatemic rickets

N



29/07/21 6:50 PM

13_Toy-Pediatrics_Case12_p117-128.indd 123

Fanconi syndrome

N







Includes cystinosis, tyrosinosis, Lowe syndrome, and acquired forms; cystinosis and tyrosinosis are autosomal recessive, Lowe syndrome is X-linked recessive

Renal tubular acidosis, type II (proximal)

N





N

Bicarbonaturia, hyperkaluria, hypercalciuria, hypophosphatemia, and phosphaturia are common; rickets may result from leaching of bone calcium bicarbonate in an attempt to buffer retained hydrogen ions seen in this condition

Oncogenic hypophosphatemia

N





Usually N

Caused by tumor secretion of a phosphateregulating gene product (PEX), which results in phosphaturia and impaired conversion of 25(OH) D to 1,25(OH)2D; the tumors are often hard to detect but are found in the small bones of the hands and feet, abdominal sheath, nasal antrum, and pharynx; resolution occurs after tumor removal

Phosphate deficiency or malabsorption

N





N

Caused by parenteral hyperalimentation or low phosphate intake



↓ or N





Autosomal recessive; mutation in gene encoding 1,25(OH)2D receptor or overexpression of a protein that interferes with the actions of 1,25(OH)2D

End-organ resistance to 1,25(OH)2D3 Vitamin D–dependent type II

Abbreviation: N, normal.

123

29/07/21 6:50 PM

124

CASE FILES: PEDIATRICS

Parenteral nutrition vitamin D supplementation is weight based. Therefore, infants receiving parenteral nutrition alone do not receive 400 IU/d of vitamin D supplementation until reaching a weight of 2.5 kg. Infants taking preterm infant formula also may receive inadequate vitamin D supplementation, depending on weight and enteral intake. Vitamin D supplementation in this population is dependent on weight and is adjusted accordingly.

Clinical Presentation Symptoms and signs of rickets include bone pain, motor delays, muscle weakness, failure to thrive, delayed closure of fontanelles, craniotabes, frontal bossing, dental abnormalities, widening of wrists and ankles, genu valgum, genu varus, and the “rachitic rosary.” Infants can also demonstrate increased respiratory distress (due to softening of the ribs) and increased susceptibility to respiratory infections. Deformities of the forearms are more common in infants, whereas angular bowing of the legs is more common in toddlers. The changes of rickets are best visualized at the growth plate of rapidly growing bones; the best sites to examine to find clinical evidence are the distal ulna and the metaphyses of the knees. On radiographs, it is typical to see widened distal ends of long bones with cupping and fraying, osteopenia, and deformities of the long bone shafts. In severe rickets, pathological fractures and Looser zones (pseudofractures, fissures, or radiolucent lines) can be present.

Treatment Treatment for vitamin D deficiency consists of supplementation with 2000 IU of vitamin D daily. Treatment continues for an average of 3 months or until there is chemical or radiographic evidence of healing. Concurrent calcium supplementation is also recommended. Supplementation.  In November 2008, the American Academy of Pediatrics (AAP) recommended an increase for vitamin D daily requirements. All infants younger than 12 months old should receive 400 IU daily beginning within the first few days after birth. Healthy children 1 to 18 years of age should receive 600 IU daily. Formula-fed infants and older children who consume at least 33 oz of formula or fortified beverages daily meet the current AAP standards. However, many children fail to consume the recommended levels and should also receive supplementation. Vitamin D fortification is found in many foods, especially milk, dairy products, orange juice, bread, and cereals. To optimize an infant’s vitamin D status, pregnant mothers should maintain sufficient vitamin D intake throughout pregnancy; supplementation with 600 IU/d is recommended. Limited sunlight exposure and outdoor activities should be encouraged in older infants and children, while maintaining an emphasis on sun safety. Direct sunlight exposure generally is not recommended in infants younger than 6 months.

CASE CORRELATION šš

See also Case 10 (Failure to Thrive) and Case 38 (Child Abuse).

13_Toy-Pediatrics_Case12_p117-128.indd 124

29/07/21 6:50 PM

SECTION II: CLINICAL CASES

125

COMPREHENSION QUESTIONS 12.1 An 18-month-old infant is seen in the pediatrician’s office as a new patient for a well-child visit after moving from Minnesota to Texas. A comparison of new to previous growth parameters suggest he has dropped from the 30th percentile to less than the fifth percentile for height, weight, and head circumference. Physical examination findings are significant for lower-extremity bowing, wrist enlargement, frontal bossing, and some hypotonia. The mother is concerned because he does not pull to stand and is not walking. She reports that he was exclusively breastfed until he was 9 to 10 months old, currently drinks about 24 oz of whole milk daily, and has never taken medications or dietary supplements. The mother also has a question about whether he needs allergy testing because he has a history of frequent upper respiratory infections. Which of the following laboratory findings would be expected? A. Low calcium, normal phosphorus, elevated alkaline phosphatase, elevated PTH B. Low calcium, low phosphorus, low alkaline phosphatase, elevated PTH C. Low calcium, low phosphorus, elevated alkaline phosphatase, elevated PTH D. Low calcium, high phosphorus, elevated alkaline phosphatase, elevated PTH E. Low calcium, low phosphorus, low alkaline phosphatase, low PTH 12.2 A 3-year-old child is being seen in the office for his well-child visit. The family has recently emigrated from Central Africa to the United States. Due to the clinical findings and height and weight, rickets is suspected. Which of the following is the most likely primary factor leading to this child’s condition? A. Intestinal malabsorption B. Dietary C. Lack of exposure to sunlight D. Hyperparathyroidism 12.3 A 12-month-old patient is seen in the clinic. His mother reports that she exclusively breastfed the child until he was 6 months of age. He still breastfeeds approximately four to five times daily. The mother reports that he also drinks 2 to 3 oz of water daily and 8 to 10 oz of juice daily, and he has been doing well eating table foods. What dosing of vitamin D would be the most appropriate for this child? A. 200 IU daily B. 400 IU daily C. 600 IU daily D. 800 IU daily E. 1000 IU daily

13_Toy-Pediatrics_Case12_p117-128.indd 125

29/07/21 6:50 PM

126

CASE FILES: PEDIATRICS

12.4 A 2-year-old child is being seen in the office. There are multiple extremity deformities noted, and the provider is suspicious of possible bony problems. Which of the following is a potentially distinguishing feature of osteogenesis imperfecta versus rickets? A. Poor muscle tone B. Easily fractured long bones C. Increased susceptibility to respiratory infections D. Small stature E. Blue sclerae

ANSWERS 12.1 C. Low calcium, low phosphorus, elevated alkaline phosphatase, elevated PTH. Rickets secondary to vitamin D deficiency frequently presents with low to normal serum calcium (depending on the duration of the disease), elevated serum PTH (to compensate for the calcium deficiency), elevated serum alkaline phosphatase, and low serum phosphorous. PTH mobilizes calcium from the bone and subsequently increases renal phosphate loss. Answers A, B, and D all have elevated PTH, but the other serum markers and incorrect. Answer E has a low alkaline phosphatase. 12.2 B. Dietary. Rickets throughout the world is most commonly due to vitamin D deficiency from lack of dietary calcium or vitamin D deficiency. Intestinal malabsorption (answer A) is a common cause in Western or developed countries. Lack of sunlight exposure (answer C) may be noted in some Middle Eastern countries. Hyperparathyroidism (answer D) is not as common and may present as abdominal pain or kidney stones. 12.3 C. 600 IU/day. All infants younger than 12 months old should receive 400 IU daily (answer B) beginning within the first few days after birth. Healthy children 1 to 18 years of age should receive 600 IU daily. Children with liver disease, kidney disease, or malabsorption (ie, secondary to cystic fibrosis, celiac disease, IBD, etc), as well as premature infants, need even higher doses to account for the limitations in vitamin D metabolism or absorption brought on by these health issues. 12.4 E. Blue sclerae. Osteogenesis imperfecta is a congenital bone disorder that is caused by defective connective tissue formation secondary to collagen deficiency. Diagnosis is confirmed via collagen or DNA testing. Inheritance can be either autosomal dominant or autosomal recessive (depending on the type). There are eight different types of osteogenesis imperfecta with varying degrees of disability, but many of the features bear similarities to rickets. A few exceptions to this, however, are bluish sclerae, triangular facies, and degrees of hearing loss. The other clinical findings (answer A, poor muscle tone; answer B, easily fractured bones; answer C, increased susceptibility to respiratory infections, and answer D, small stature) may be seen in both conditions.

13_Toy-Pediatrics_Case12_p117-128.indd 126

29/07/21 6:50 PM

127

SECTION II: CLINICAL CASES

CLINICAL PEARLS »»

Nutritional rickets (inadequate dietary vitamin D or sunlight exposure) is rare in healthy children in industrialized countries. Medical conditions (liver or renal failure) or abnormalities in calcium and phosphorus metabolism usually are responsible.

»»

Primary hypophosphatemia (X-linked dominant) is the most common cause of nonnutritional rickets; proximal kidney tubule defects in phosphate reabsorption and conversion of 25(OH)D to 1,25(OH)2D are seen. Findings include low normal serum calcium, moderately low serum phosphate, elevated serum alkaline phosphatase, low serum 1,25(OH)2D levels, hyperphosphaturia, and no evidence of hyperparathyroidism.

»»

Poor dietary intake of vitamin D may result in nutritional rickets.

»»

In cases of unexpected fractures in children, rickets and child abuse should both be investigated.

REFERENCES Abrams S. Dietary guidelines for calcium and vitamin D: a new era. Pediatrics. 2011;127:566-568. Bartz S, Barker JM, Kappy MS, et al. Disorders of calcium and phosphorus metabolism. In: Hay WW Jr, Levin MJ, Deterding RR, Abzug MJ, eds. Current Diagnosis & Treatment: Pediatrics. 24th ed. New York, NY: McGraw Hill; 2018. Beary J, Chines A. Osteogenesis imperfecta: clinical features and diagnosis. UpToDate. http://www .uptodate.com/home/index.html. Accessed February 9, 2020. Brewer ED. Pan-proximal tubular dysfunction (Fanconi syndrome). In: McMillan JA, Feigin RD, DeAngelis CD, Jones MD, eds. Oski’s Pediatrics: Principles and Practice. 4th ed. Philadelphia, PA: Lippincott Williams & Wilkins; 2006:1892-1897. Cadnapaphornchai MA, Lum GM. Chronic renal failure. In: Hay WW Jr, Levin MJ, Deterding RR, Abzug MJ, eds. Current Diagnosis & Treatment: Pediatrics. 24th ed. New York, NY: McGraw Hill; 2018. Carpenter T. Etiology and treatment of calcipenic rickets in children. UpToDate. http://www.uptodate .com/home/index.html. Accessed February 11, 2020. Carpenter T. Overview of rickets in children. UpToDate. http://www.uptodate.com/home/index.html. Accessed February 11, 2020. Chiang ML. Disorders of renal phosphate transport. In: McMillan JA, Feigin RD, DeAngelis CD, Jones MD, eds. Oski’s Pediatrics: Principles and Practice. 4th ed. Philadelphia, PA: Lippincott Williams & Wilkins; 2006:1898-1901. Devarajan P. Toxic nephropathy. In: Kleigman RM, St. Geme JW, Blum NJ, et al, eds. Nelson Textbook of Pediatrics. 21st ed. Philadelphia, PA: Elsevier; 2020:2734-2736. Egan M, Schechter MS, Voynow JA. Cystic fibrosis. In: Kleigman RM, St. Geme JW, Blum NJ, et al, eds. Nelson Textbook of Pediatrics. 21st ed. Philadelphia, PA: Elsevier; 2020:2282-2297. Federico MJ, Baker CD, Deboer EM, et al. Respiratory tract and mediastinum. In: Hay WW Jr, Levin MJ, Deterding RR, Abzug MJ, eds. Current Diagnosis & Treatment: Pediatrics. 24th ed. New York, NY: McGraw Hill; 2018. Golden NH, Abrams SA. Optimizing bone health in children and adolescents. Pediatrics. 2014;134(4):e1229-e1243.

13_Toy-Pediatrics_Case12_p117-128.indd 127

29/07/21 6:50 PM

128

CASE FILES: PEDIATRICS

Gordon CM. Metabolic bone disease. In: Kleigman RM, St. Geme JW, Blum NJ, et al, eds. Nelson Textbook of Pediatrics. 21st ed. Philadelphia, PA: Elsevier; 2020:3746-3748. Greenbaum LA. Vitamin D deficiency (rickets) and excess. In: Kleigman RM, St. Geme JW, Blum NJ, et al, eds. Nelson Textbook of Pediatrics. 21st ed. Philadelphia, PA: Elsevier; 2020:375-385. Hill LL, Chiang ML. Renal tubular acidosis. In: McMillan JA, Feigin RD, DeAngelis CD, Jones MD, eds. Oski’s Pediatrics: Principles and Practice. 4th ed. Philadelphia, PA: Lippincott Williams & Wilkins; 2006:1886-1892. Joiner T, Foster C, Shope T. The many face of vitamin deficiency rickets. Pediatr Rev. 2000;21:296-302. Kohaut EC. Chronic renal failure. In: McMillan JA, Feigin RD, DeAngelis CD, Jones MD, eds. Oski’s Pediatrics: Principles and Practice. 4th ed. Philadelphia, PA: Lippincott Williams & Wilkins; 2006:1841-1844. Lauer B, Spector N. Vitamins. Pediatr Rev. 2012;33:339-352. Linglart A, Biosse-Duplan M, Briot K, et al. Therapeutic management of hypophosphatemic rickets from infancy to adulthood. Endocr Connect. 2014;3:R13-R30. Madhusmita M. Vitamin D insufficiency and deficiency in children and adolescents. In: Rose BD, ed. UpToDate. http://www.uptodate.com/home/index.html. Accessed February 11, 2020. Merchant N, Root A, Mendiratta M, Bacino CA. Disorders of low bone mass due to impaired bone mineralization. In: Kline MW, Blaney SM, Giardino AP, et al, eds. Rudolph’s Pediatrics. 23rd ed. New York, NY: McGraw Hill; 2018:2605-2608. Rosenstein BJ. Cystic fibrosis. In: McMillan JA, Feigin RD, DeAngelis CD, Jones MD, eds. Oski’s Pediatrics: Principles and Practice. 4th ed. Philadelphia, PA: Lippincott Williams & Wilkins; 2006:1425-1438. Saddi V, Ooi CY, Jaffe A. Cystic fibrosis. In: Kline MW, Blaney SM, Giardino AP, et al, eds. Rudolph’s Pediatrics. 23rd ed. New York, NY: McGraw Hill; 2018:2244-2461. Sokol R, Narkewicz MR, Sundaram S, Mack CL. Biliary atresia. In: Hay WW Jr, Levin MJ, Deterding RR, Abzug MJ, eds. Current Diagnosis & Treatment: Pediatrics. 24th ed. New York, NY: McGraw Hill; 2018. Sokol R, Narkewicz MR, Sundaram S, Mack CL. Extrahepatic neonatal cholestasis. In: Hay WW Jr, Levin MJ, Deterding RR, Abzug MJ, eds. Current Diagnosis & Treatment: Pediatrics. 24th ed. New York, NY: McGraw Hill; 2018. Srivaths P. Chronic kidney disease. In: Kline MW, Blaney SM, Giardino AP, et al, eds. Rudolph’s Pediatrics. 23rd ed. New York, NY: McGraw Hill; 2018:2175-2182. Taylor S, Hollis B, Wagner C. Vitamin D needs of preterm infants. NeoReviews. 2009;10:e590-e599. Zhou P, Markowitz M. Hypocalcemia in infants and children. Pediatr Rev. 2009;30:190-192.

13_Toy-Pediatrics_Case12_p117-128.indd 128

29/07/21 6:50 PM

CASE 13 A 10-year-old child with known sickle cell disease (SCD) presents to the emergency department (ED) with a complaint of right thigh pain. His medical history includes two previous hospitalizations, once at 6 months for fever and another at 12 months for a swollen, painful left wrist. On examination, he is afebrile, and his heart rate is 130 beats per minute. You notice tenderness to palpation over the right femur with an otherwise benign physical examination. ▶▶ ▶▶ ▶▶

What is the most likely diagnosis? What is the next step in the care of this patient? What long-term strategies might be employed to prevent recurrence?

14_Toy-Pediatrics_Case13_p129-138.indd 129

09/06/21 3:13 PM

130

CASE FILES: PEDIATRICS

ANSWERS TO CASE 13: Sickle Cell Disease With Vaso-Occlusive Crisis Summary: An otherwise healthy 10-year-old boy presents with šš

Known SCD

šš

Tachycardia and no fever

šš

Pain localized to his right thigh

Most likely diagnosis: Vaso-occlusive pain crisis. Next step: Admit to the hospital to administer intravenous (IV) fluids and to provide pain management (likely with narcotics). Long-term strategy: Administration of hydroxyurea will increase the concentration of fetal hemoglobin, thus reducing the frequency of sickle cell crisis episodes.

ANALYSIS Objectives 1. Learn the common complications and treatment strategies for a child with SCD. (EPA 4, 12) 2. Become familiar with the goals of a routine well-child (or health supervision) session for a patient with SCD. (EPA 12)

Considerations This is a 10-year-old boy with known SCD who is being seen in the ED for right thigh pain. This is most likely due to vaso-occlusive complications of his disease. Vaso-occlusive crisis is the most commonly experienced complication of SCD in children. Risk factors of this condition include increasing age and high baseline hemoglobin level. Episodes can be triggered by infection, stress, dehydration, cold temperatures, or high altitude, but often no trigger can be identified. The site of pain may vary, but the most common sites are the extremities or back. For many children experiencing recurrent vaso-occlusive crises, the pain tends to occur in similar locations with repeat episodes. The severity of these episodes varies; some patients are successfully treated as outpatients. When oral medications are insufficient to control the pain, hospital admission for IV fluids and narcotic medication is required.

14_Toy-Pediatrics_Case13_p129-138.indd 130

09/06/21 3:13 PM

SECTION II: CLINICAL CASES

131

APPROACH TO: Sickle Cell Disease DEFINITIONS ACUTE CHEST SYNDROME (ACS): A new pulmonary infiltrate on chest x-ray involving at least one complete lung segment in addition to one of the following signs: fever greater than 38.5 °C, chest pain, shortness of breath, wheezing, cough, tachypnea, or lower than baseline oxygen saturations. APLASTIC CRISIS: Infection with parvovirus B19 (most commonly) can lead to temporary cessation of red blood cell (RBC) formation, which then leads to an abrupt decline in bone marrow RBC precursors as well as peripheral reticulocytes. Reduced life span of RBCs in SCD coupled with reduced production may result in profound anemia. DACTYLITIS: A form of vaso-occlusive crisis that involves painful swelling of the hands and/or feet. SICKLE CELL DISEASE (SCD): A group of disorders affecting at least one of the beta-globulin genes, resulting in some degree of sickling of the RBCs. VASO-OCCLUSIVE CRISIS: An episode of severe pain caused by increased sickling of RBCs, which can lead to bone marrow ischemia and infarction.

CLINICAL APPROACH Epidemiology SCD is a multisystem disorder and the most common genetic disease in the United  States, affecting 1 in 500 African Americans. About 1 in 12 African Americans carry the autosomal recessive mutation. Approximately 300,000 infants are born with sickle cell anemia annually. Most patients in the United States are diagnosed by prenatal or newborn screening.

Pathophysiology Sickle cell anemia is caused by homozygosity of the beta-S allele on chromosome 11, due to the replacement of a hydrophilic glutamic acid residue with a hydrophobic valine residue in the sixth position of the beta-globin chain. This mutation causes HbS polymerization and sickling of the RBCs, which can cause blood vessel occlusion.

Routine Care Children with SCD require multiple evaluations both by primary care providers and by hematologists per year, especially when young, and should receive screening for the various known complications of the disease. Anticipatory guidance aims to foster good health habits, prevent illness, and assist in family communication. For children with an SCD diagnosis, another important part of improving overall health is to ensure that they are linked to a comprehensive SCD program.

14_Toy-Pediatrics_Case13_p129-138.indd 131

09/06/21 3:13 PM

132

CASE FILES: PEDIATRICS

Routine care for children with SCD includes initiation of prophylactic penicillin as early as possible; the pneumococcal PCV13 series at 2, 4, 6, and 12 to 15 months; and the pneumococcal and meningococcal vaccines starting at 2 years of age. Frequent complete blood counts (CBCs) are performed, and renal, liver, and lung function are monitored annually beginning at 1 year of age. Transcranial Doppler ultrasounds to screen annually for stroke prevention should begin at 2 years of age. Screening for sickle cell retinopathy should begin by 10 years of age. Folic acid is no longer recommended in high-resource areas such as the United States.

Vaso-Occlusive Crises Diagnosis.  Vaso-occlusive crises (previously known as sickle cell crises) are the most common complication among children with SCD. Episodes can occur as early as 6 months of age. Pain is the most common presenting symptom, but patients may also have erythema, edema, or joint effusions near the site of the pain. Episodes may vary in severity. No specific laboratory work is diagnostic of the crisis, but affected patients may have decreased hemoglobin from baseline or increased white blood cells (WBCs). Treatment.  Children whose pain is inadequately controlled with home medication regimens must be evaluated because significant pain can co-occur with other potentially life-threatening SCD complications. Additional pain medications, such as morphine or hydromorphone, along with hydration, may be attempted in the outpatient setting. However, if more than one or two doses of these additional pain medications are required, inpatient hospitalization is required. Additional inpatient strategies include IV fluids (often given at higher than maintenance rates) and IV narcotics, with doses and frequencies titrated to control the patient’s severity of pain. RBC transfusions typically are not effective for simple vaso-occlusive crises and are reserved for severe anemia or complicated vasoocclusive episodes. Prevention.  Prevention of recurrent episodes of vaso-occlusive crises is attempted by avoidance of known triggers, as well as administration of hydroxyurea beginning at 9 months of age. This medication increases the concentration of fetal hemoglobin, thus decreasing sickling. L-glutamine can be given to children aged 5 years and older for prevention of acute vaso-occlusive events by repleting glutathione stores lacking in RBCs.

SCD Patients With Fever An SCD patient with fever (with or without vaso-occlusive crisis) may have a medical emergency. SCD causes functional asplenia, making patients susceptible to severe infections from invasive encapsulated organisms (typically pneumococcal disease) as well as viruses. Any child with SCD presenting with a fever greater than 38.5 °C (101.3 °F) should be evaluated emergently for both possible infection and potentially life-threatening complications of SCD. All ill-appearing SCD patients with fever should be hospitalized. In such situations, a CBC and blood cultures are warranted, and if the screening results are concerning for infection without source, empiric antibiotics typically should be initiated (usually with ceftriaxone).

14_Toy-Pediatrics_Case13_p129-138.indd 132

09/06/21 3:13 PM

SECTION II: CLINICAL CASES

133

Acute Chest Syndrome A variety of other complications can occur with SCD. Children with SCD who have significant respiratory symptoms, such as severe cough, shortness of breath, wheezing, or chest pain, may be exhibiting symptoms of ACS. Because of potentially fatal complications of ACS, immediate treatment is warranted; treatment includes empiric antibiotics, supplemental oxygen, incentive spirometry, bronchodilators (if there is history of prior reactive airway disease), pain medications, and IV fluids. Simple transfusion should be considered in patients with ACS to improve oxygenation. Exchange transfusion should be performed for ACS episodes that have progressed despite simple transfusion. Close inpatient observation for respiratory failure is warranted.

Acute Splenic Sequestration Parents of a child with SCD are taught to palpate the abdomen of their younger children to observe for splenic enlargement. A child who has abdominal pain, distension, or acute enlargement of the spleen likely has an acute splenic sequestration and requires hospitalization, possibly in the intensive care unit, to observe for cardiovascular collapse. Splenic sequestration can lead to a significant drop in hemoglobin levels due to pooling of RBCs in the spleen. Blood transfusions may be required and could potentially be lifesaving. Elective splenectomy is often performed to prevent recurrence after the first episode. As the child ages, the spleen usually auto-infarcts, becoming fibrotic and leading to impaired splenic function. Although lack of a functional spleen eliminates the complication of splenic sequestration, it also increases the odds of an infection with an encapsulated organism.

Stroke Diagnosis.  Approximately 10% of children with SCD have acute strokes, with a peak incidence between 4 and 8 years of age. Symptoms may include paresis, aphasia, seizures, cranial nerve palsy, headache, or coma; all children with such symptoms should be admitted to the hospital. Emergency neuroimaging via noncontrast computed tomography (CT) followed by magnetic resonance imaging/angiography (MRI/MRA) is warranted. Additionally, repeated neurologic examinations should be conducted, and partial or simple transfusions should be performed to reduce the percentage of sickled cells. Treatment.  If a stroke is confirmed, exchange transfusion should be performed as soon as possible. Thrombolytics and anticoagulants are not recommended. Physical therapy and rehabilitation should be provided as the patient recovers. Chronic transfusions (often monthly) should be instituted to reduce the risk of recurrence. As part of the routine well-child care of a patient with SCD, transcranial Doppler ultrasonography is often recommended annually to identify those with increased flow velocity in the large cerebral blood vessels and thus at high-risk for developing a first stroke. Routine chronic transfusion among these high-risk children has resulted in reduced risk of first stroke.

14_Toy-Pediatrics_Case13_p129-138.indd 133

09/06/21 3:13 PM

134

CASE FILES: PEDIATRICS

Aplastic Crisis A child with SCD who presents with a significant increase in pallor, fatigue, or lethargy may be exhibiting signs of an aplastic crisis, most often caused by infection with parvovirus B19. These children will have a hemoglobin level below their normal baseline and a low reticulocyte count. These children require hospitalization to observe for evidence of cardiovascular collapse, and blood transfusions may be required.

Priapism A boy with SCD who has a priapism episode persisting for more than 3 to 4 hours must be evaluated by a urologist, as this can lead to irreversible tissue damage. IV fluid hydration and pain control are provided; ice should not be used. The urologist may be required to aspirate and irrigate the corpora cavernosa to achieve detumescence. Failure of three or four aspirations in the outpatient setting requires more extensive inpatient management, including exchange blood transfusions, further pain control, and additional surgical intervention.

Emerging Treatments In 2019, the US Food and Drug Administration (FDA) approved voxelotor, a small molecule binding to hemoglobin, which increases the protein’s affinity to oxygen and inhibits sickle cell polymerization. Crizanlizumab is a monoclonal antibody that binds to the P-selectin protein of endothelial cells and reduces sickled RBCs from adhering to blood vessels; this agent was also approved by the FDA in 2019.

CASE CORRELATION šš

See Case 11 (Anemia in the Pediatric Patient) and Case 14 (Pneumonia and Tuberculosis).

COMPREHENSION QUESTIONS 13.1 A 2-week-old infant is being seen in the office with his parents. As you review the labs, you note that the newborn state screen is positive for sickle cell disease (SCD). Which of the following is the most important counseling to provide to the parents? A. Initiation of iron therapy B. Emergent genetic testing of both parents for hemoglobinopathy status C. Initiation of hydroxyurea therapy D. Purchase of an apnea monitor E. Enrollment in a comprehensive sickle cell program

14_Toy-Pediatrics_Case13_p129-138.indd 134

09/06/21 3:13 PM

SECTION II: CLINICAL CASES

135

13.2 A 14-year-old boy with known SCD has been admitted to the hospital for a presumed vaso-occlusive crisis. He has been receiving IV fluids and IV narcotics for the past 2 days. He seemed to be getting better, but the nurse has just called and stated his temperature is now 101.8 °F. On examination, you find his respiratory rate to be 32 breaths per minute, and his oxygen saturation is 90%. What is the next step in management? A. Order a spiral CT B. Initiate antibiotics and provide supplemental oxygen via nasal cannula C. Obtain stat hemoglobin level D. Initiate incentive spirometry E. Administer another dose of narcotics 13.3 A 9-year-old girl with known SCD is brought to the ED by her mother for right-sided weakness and slurring of speech of 2 hours’ duration. She denies any associated pain. Her vital signs are within normal limits for her age. Physical examination confirms right arm and right leg weakness and slurred speech. Which of the following screening tests might have helped to prevent this complication of SCD? A. Annual CT of head B. Annual MRI of head C. Biannual check of hemoglobin level D. Annual transcranial Doppler ultrasound E. Annual neurologic assessments during well-child checks 13.4 A 2-month-old infant girl with known SCD is being seen in the office for follow-up. There are no complaints, and the examination is unremarkable. Which of the following statements about routine procedures for a patient with SCD is accurate? A. Periodic CBC and reticulocyte measurement screenings begin at the age of 2 months. B. Polysaccharide pneumococcal 23 vaccines are administered at 2, 4, and 6 months of age. C. Chest radiographs at routine visits begin at about 12 months of age. D. Annual gallbladder ultrasounds begin at age 12 to identify the presence of stones. E. Human papillomavirus vaccines begin at the age of 18 years.

ANSWERS 13.1 E. Enrollment in a comprehensive sickle cell program. This child should be enrolled in a comprehensive SCD program to ensure the best possible outcome. At 2 weeks of age, the child has no reason to be iron deficient, and combined with future blood transfusions that may be required, iron therapy

14_Toy-Pediatrics_Case13_p129-138.indd 135

09/06/21 3:13 PM

136

CASE FILES: PEDIATRICS

(answer A) could result in iron overload. The newborn state screen has shown that the child has SCD and that both parents have at least a single sickle cell gene; further testing of the family may be warranted, but not as an emergency (answer  B). Hydroxyurea (answer C) is used to increase the levels of fetal hemoglobin beginning at 9 months of age. This child in the first month of life already has significant quantities of fetal hemoglobin still present. SCD is not an indication for an apnea monitor (answer D). 13.2 B. Initiate antibiotics and provide supplemental oxygen via nasal cannula. This patient was admitted for a vaso-occlusive crisis 2 days ago and recently has developed fever and tachypnea; he is at high risk of developing acute chest syndrome (ACS). Given that he currently has a fever, tachypnea, and decreased oxygen saturation, he meets ACS criteria. Prompt initiation of antibiotics and supplemental oxygen is imperative in preventing deadly complications of ACS. Pulmonary embolism (answer A) is in the differential for this patient, and an acute drop in hemoglobin (answer C) might cause tachypnea; nevertheless, initiation of antibiotics should be your first step in management. Incentive spirometry (answer D) is an important part of the prevention of ACS, but antibiotics are more important in its treatment. Although increased pain may result in tachypnea, it would not usually cause a decrease in the patient’s oxygen saturation; therefore, although treating pain (answer E) is important, it is not the first priority of care at this time. 13.3 D. Annual transcranial Doppler ultrasound. Annual transcranial Doppler ultrasound should initially be performed at 2 years of age. If it is normal, it should be repeated annually until the patient is 16 years old. If two ultrasounds are abnormal, transfusion therapy typically is initiated and continued indefinitely to help prevent stroke. CT (answer A) and MRI (answer B) without any signs of stroke are not indicated. A change in hemoglobin level (answer C) does not indicate potential stroke in a patient with SCD, but it may be concerning for infection or aplastic crisis. Although neurologic examinations (answer E) are an important part of any physical examination, changes in the examination would indicate an already evolving process, rather than help to predict the potential for future disease. 13.4 A. Periodic CBC and reticulocyte measurement screenings begin at the age of 2 months. Patients with SCD require baseline and periodic blood counts beginning at the age of 2 months. The 23-valent polysaccharide pneumococcal vaccine (answer B) is initiated at 2 years of age, whereas the 13-valent conjugate pneumococcal vaccine is administered at 2, 4, and 6 months of age. Chest radiographs (answer C) typically are obtained at approximately 2 years of age and periodically thereafter for screening purposes, for recent ACS, or if the child has chronic cardiac or pulmonary disease. Ultrasounds of the gallbladder (answer D) are reserved for patients who are symptomatic (abdominal pain). The human papillomavirus vaccine should be routinely administered between ages 11 and 12 years and as early as age 9 years (answer E).

14_Toy-Pediatrics_Case13_p129-138.indd 136

09/06/21 3:13 PM

SECTION II: CLINICAL CASES

137

CLINICAL PEARLS »»

Children with sickle cell disease (SCD) who have fever (risk of sepsis), pallor (aplastic crisis), abdominal pain or distension (splenic sequestration), pain crisis, evidence of lower respiratory disease (acute chest syndrome), priapism, new neurologic findings (stroke), or dehydration must be evaluated urgently.

»»

Additions to routine care required for all children include initiation of penicillin, as well as administration of meningococcal and polysaccharide vaccines at earlier than typical ages.

»»

A variety of screening tests, such as routine CBC and reticulocyte measurements, begin at 2 months of age or at diagnosis.

»»

Parvovirus infection is associated with aplastic anemia in patients with SCD.

»»

Pneumonia in a sickle cell patient can result in a life-threatening condition known as acute chest syndrome, which consists of a new pulmonary infiltrate on chest radiograph in addition to one of the following: fever, dyspnea, tachypnea, chest pain, or decreased oxygen saturations.

REFERENCES Ambruso DR, Nuss R, Wang M. Hematologic disorders. In: Hay WW Jr, Levin MJ, Deterding RR, Abzug MJ, eds. Current Diagnosis & Treatment: Pediatrics. 24th ed. New York, NY: McGraw Hill; 2018. Anders DT, Tang F, Ledneva T, et al. Hydroxyurea use in young children with sickle cell anemia in New York State. Am J Prev Med. 2016;51:S31-S38. Emond AM, Collis R, Darvill D, Higgs DR, Maude GH, Serjeant GR. Acute splenic sequestration in homozygous sickle cell disease: natural history and management. J Pediatr. 1985;107:201-206. George A. Hemoglobinopathies. In: Kline MW, Blaney SM, Giardino AP, et al, eds. Rudolph’s Pediatrics. 23rd ed. New York, NY: McGraw Hill; 2018:1147-1154. Lane PA, Buchanan GR, Hutter JJ, et al. Sickle cell disease in children and adolescents: diagnosis, guidelines for comprehensive care, and care paths and protocols for management of acute and chronic complications. Sickle Cell Disease Care Consortium. http://www.dshs.state.tx.us/WorkArea/ DownloadAsset.aspx?id=8589985663. Accessed March 1, 2020. Martin PL. Sickle cell disease and trait. In: McMillan JA, Feigin RD, DeAngelis CD, Jones MD, eds. Oski’s Pediatrics: Principles and Practice. 4th ed. Philadelphia, PA: Lippincott Williams & Wilkins; 2006:1696-1698. McCavit TL. Sickle cell disease. Pediatr Rev. 2012;33:195-206. National Institutes of Health. National Heart, Lung, and Blood Institute. Evidence-based management of sickle cell disease: expert panel report, 2014. https://www.nhlbi.nih.gov/sites/default/files/ media/docs/sickle-cell-disease-report%20020816_0.pdf. Accessed March 1, 2020.

14_Toy-Pediatrics_Case13_p129-138.indd 137

09/06/21 3:13 PM

138

CASE FILES: PEDIATRICS

Niihara Y, Miller ST, Kanter J, et al. A phase 3 trial of L-glutamine in sickle cell disease. N Engl J Med. 2018;379:226-235. Smith-Whitley K. Sickle cell disease. In: Kleigman RM, St. Geme JW, Blum NJ, et al, eds. Nelson Textbook of Pediatrics. 21st ed. Philadelphia, PA: Elsevier; 2020:2541-2550. Vichinsky EP, Neumayr LD, Earles AN, et al. Causes and outcomes of the acute chest syndrome in sickle cell disease. National Acute Chest Syndrome Study Group. N Engl J Med. 2000;342:1855-1865.

14_Toy-Pediatrics_Case13_p129-138.indd 138

09/06/21 3:13 PM

CASE 14 A 5-year-old boy is brought by his mother to the clinic due to a worsening cough and wheezing over the previous 4 days. He first developed a runny nose and sore throat about 7 days ago, and he has since developed a productive cough. He had been attending kindergarten as usual until today, when he was kept home from school due to fever. His mother also expresses concern for wheezing that she has not heard before. He has no known history of cardiorespiratory disease, and his immunizations are current. On examination, the patient looks uncomfortable but is cooperative. He is febrile to 103.2 °F (39.6 °C), with a heart rate of 105 beats per minute and a respiratory rate of 22 breaths per minute. He has congested nares with clear rhinorrhea. Auscultation of the lungs reveals coarse breath sounds in all lung fields and bibasilar end-expiratory wheezes and crackles. ▶▶ ▶▶

What is the most likely diagnosis? What is the next step in evaluation?

15_Toy-Pediatrics_Case14_p139-150.indd 139

12/07/21 2:53 PM

140

CASE FILES: PEDIATRICS

ANSWERS TO CASE 14: Pneumonia Summary: A 5-year-old boy presents with šš

A productive cough and fever

šš

Rhinorrhea and a sore throat that began 7 days ago

šš

Crackles on chest examination

šš

No prior medical history

Most likely diagnosis: Pneumonia. Next step in evaluation: Confirm cardiorespiratory stability. If established, a chest x-ray may be indicated to ascertain if radiographic changes support clinical findings. Laboratory tests (complete blood count [CBC], blood cultures, and nasal swab for respiratory viral polymerase chain reaction [PCR] or direct fluorescent antibody staining [DFA]) may help elucidate the etiology and extent of infection, as well as direct possible antimicrobial therapy.

ANALYSIS Objectives 1. Describe the etiologies of pneumonia and their age predilections. (EPA 1, 2) 2. Describe various clinical and radiographic findings in pneumonia and tuberculosis. (EPA 1, 3) 3. Describe the evaluation and treatment of pneumonia and tuberculosis. (EPA 1-4)

Considerations This is a 5-year-old boy with worsening cough and fever. The most important initial goal in managing this patient is to ensure adequacy of the ABCs (maintaining the airway, controlling the breathing, and ensuring adequate circulation). A patient with pneumonia may present with varying degrees of respiratory compromise. Evaluate for signs of distress, such as tachypnea, retractions (suprasternal, intercostal, subcostal), and in older children, the ability to speak. A pulse oximeter can be used to check oxygenation. In severe cases, respiratory failure may be imminent, necessitating intubation and mechanical ventilation. The patient with pneumonia and sepsis may have evidence of circulatory failure (septic shock) and require vigorous fluid resuscitation. After the basics of resuscitation have been achieved, further evaluation and management can be initiated.

15_Toy-Pediatrics_Case14_p139-150.indd 140

12/07/21 2:53 PM

SECTION II: CLINICAL CASES

141

APPROACH TO: Pneumonia and Tuberculosis DEFINITIONS BRONCHIAL BREATH SOUNDS: Normal breath sounds heard over the proximal airway; they can indicate consolidations when heard over the lung fields due to increased transmission of airway sounds. CRACKLES/RALES: Synonymous terms referring to inspiratory sounds associated with alveolar fluid and debris. Can be further divided into fine (like wood burning, pouring milk on rice cereal) and coarse (like rubbing hair next to the ear, bubbles). Usually heard in pneumonia, but can also be associated with other causes of fluid in the lung (heart failure, pulmonary edema). PARAPNEUMONIC EFFUSION/EMPYEMA: Pleural effusion as a result of infection or inflammatory reaction associated with bacterial pneumonia or pulmonary abscess. Empyema generally refers to grossly purulent fluid, whereas parapneumonic effusion refers to nonpurulent fluid. May be associated with chest pain, dyspnea, or fever. Larger effusions require drainage in addition to intravenous antibiotics. PLEURAL EFFUSION: Fluid accumulation in the pleural space; may be associated with chest pain or dyspnea; can be transudate or exudate depending on results of fluid analysis for protein and lactate dehydrogenase (Light’s criteria); origins include cardiovascular (heart failure), infectious (mycobacterial pneumonia), and malignant (lymphoma). PLEURAL RUB: Inspiratory and expiratory “rubbing” or scratching breath sounds heard when inflamed visceral and parietal pleurae come together. PULSE OXIMETRY: Noninvasive estimation of arterial oxyhemoglobin concentration (SPO2) using select wavelengths of light. STACCATO COUGH: Coughing spells with quiet intervals, often heard in pertussis and chlamydial pneumonia. VESICULAR BREATH SOUNDS: Normal breath sounds heard over the lung fields. WHEEZING: Classically described as a musical, expiratory sound associated with airway obstruction. Not everything that wheezes is asthma. For example, mucus plugs or foreign bodies can also cause wheezing.

CLINICAL APPROACH TO PNEUMONIA Epidemiology Community-acquired pneumonia is one of the most common reasons for hospitalization, with approximately 125,000 pediatric hospital admissions per year,

15_Toy-Pediatrics_Case14_p139-150.indd 141

12/07/21 2:53 PM

142

CASE FILES: PEDIATRICS

especially affecting children younger than age 2 years. After elderly patients, the second highest rate of influenza-related hospitalizations is in children younger than 5 years of age.

Pathophysiology Pneumonia is a type of lower respiratory tract infection (LRTI) that is primarily diagnosed clinically and supported radiographically. Although the pediatric patient with pneumonia may have traditional findings (fever, cough, tachypnea, toxicity), others may present with few signs, depending on the organism involved and the patient’s age and health status. Pneumonia typically begins with organism acquisition via inhalation of infected droplets or contact with a contaminated surface. Depending on the organism, spread to distal airways occurs over varying intervals. Bacterial infection typically progresses rapidly over a few days; viral pneumonia may develop more gradually. With infection progression, an inflammatory cascade ensues with airways affected by humoral and cellular mediators. The resulting milieu adversely affects ventilation-perfusion, and respiratory symptoms develop. LRTI occurs more frequently in the fall and winter and with greater frequency in younger patients, especially those in group environments (large households, daycare facilities, elementary schools). When all age groups are considered, approximately 60% of pediatric pneumonias are bacterial in origin, with pneumococcus topping the list. Viruses (respiratory syncytial virus [RSV], adenovirus, influenza, parainfluenza, enteric cytopathic human orphan [ECHO] virus, coxsackievirus, human metapneumovirus) run a close second. Routine vaccinations, including PCV13 and influenza, have been shown to reduce hospitalization rates. Bacterial and Viral Pathogens.  Common bacterial and viral pathogens and associations by age group are provided in the following lists. Neonates (less than 3 weeks): šš

Group B Streptococcus: most common cause of early-onset neonatal sepsis

šš

Escherichia coli: most common cause of mortality in early-onset neonatal sepsis

šš

Other Enterobacteriaceae (eg, Klebsiella, Salmonella)

šš

šš

Streptococcus pneumoniae: known as pneumococcus; associated with neonatal meningitis, complicated pneumonia Haemophilus influenzae: nontypeable is the most common, but consider for type B in the unimmunized

šš

Staphylococcus aureus: complicated pneumonia; nosocomial infection

šš

Listeria monocytogenes: neonatal meningitis; can be transferred transplacentally

šš

šš

Herpes simplex virus (HSV): most concerning and prevalent viral pneumonia in the first few days of life; treat with acyclovir Other viruses (eg, adenovirus, enterovirus, parainfluenza, RSV, rhinovirus, human metapneumovirus)

15_Toy-Pediatrics_Case14_p139-150.indd 142

12/07/21 2:53 PM

SECTION II: CLINICAL CASES

143

Infants aged 3 weeks to children aged 4 years: šš

RSV: most common cause of bronchiolitis; seen more often in the winter

šš

Other viruses (as listed earlier)

šš

S. pneumoniae

šš

H. influenzae

šš

Chlamydia trachomatis: afebrile infant presenting with staccato cough and tachypnea; may present past the first month of life, especially if a history of conjunctivitis or maternal history of chlamydia; can have eosinophilia, bilateral infiltrates on chest radiograph

Children aged 5 years and older: šš

Mycoplasma pneumoniae: schools, dormitories

šš

S. pneumoniae

šš

Chlamydophila pneumoniae: adolescent patients

šš

H. influenzae

šš

Viruses (influenza, adenovirus)

šš

Legionella pneumophila: exposure to stagnant water, air coolers; causes Legionnaires’ disease

Fungal Pathogens.  Fungal infections are considered in patients who live in certain geographical areas or for patients with risk factors, such as immunosuppression or concomitant respiratory disorders (eg, asthma). šš

šš

šš

šš

šš

Histoplasma capsulatum: Mississippi and Ohio River Valleys, spelunking (exposure to bat feces), farms (exposure to bird feces); causes hilar lymphadenopathy, nodules Blastomyces dermatitidis: overlaps with Histoplasma (Mississippi and Ohio River Valleys), but also seen in mid-Atlantic seaboard states; large, single, broad-based budding yeasts; purulent sputum production, mass-like infiltrates Coccidioides immitis: southwestern United States, causes erythema nodosum and erythema multiforme Aspergillus species: refractory asthma, “fungal ball” on chest radiography; can lead to allergic bronchopulmonary aspergillosis: type I and III hypersensitivity reaction Pneumocystis jirovecii: immunocompromised patients (eg, HIV)

Special Considerations.  Special considerations and other commonly tested organisms are outlined in the following list. šš

Intensive care unit patients: consider Pseudomonas, Candida species, coagulasenegative staphylococci, especially in those who are intubated, have central lines, or have indwelling catheters

15_Toy-Pediatrics_Case14_p139-150.indd 143

12/07/21 2:53 PM

144

šš

šš

šš

CASE FILES: PEDIATRICS

Cystic fibrosis: consider Pseudomonas, Aspergillus (see earlier fungal causes of pneumonia) Exposure to infected animals: consider Coxiella burnetii (sheep, goats, cattle; Q fever: flu-like symptoms), Bacillus anthracis (cattle; anthrax), Chlamydophila psittaci (birds; psittacosis), Francisella tularensis (birds; psittacosis), Pasteurella multocida (dogs, cats) Viruses: Consider cytomegalovirus (can present with retinitis), varicella-zoster virus (typical skin findings)

Clinical Presentation Signs and Symptoms.  The pneumonia process may produce few findings or may present with increased work of breathing (eg, nasal flaring, accessory muscle use). Tachypnea is a sensitive indicator of pneumonia. Associated symptoms may include malaise, headache, abdominal pain, nausea, or emesis. Toxicity can develop, especially in bacterial pneumonia. Fever is not a constant finding. Subtle temperature instability may be noted in neonatal pneumonia. Clinically, pneumonia can be associated with decreased or abnormal breathing (rales, wheezing). Hypoxia can be seen. Pneumonia complications may be identified by finding localized decreased breath sounds (parapneumonic effusion) or rubs (pleuritis). Imaging.  Radiographic findings in LRTI may be limited, nonexistent, or lag the clinical symptoms, especially in the dehydrated patient. Chest radiography is not always needed for diagnosis in the outpatient treatment setting. However, if a patient is being admitted, typically two-view chest radiographs characterize the pneumonia process and any potential complications. Findings may include single or multilobar consolidation (pneumococcal or staphylococcal pneumonia), air trapping with a flattened diaphragm (viral pneumonia with bronchospasm), or perihilar lymphadenopathy (mycobacterial pneumonia). Alternatively, an interstitial pattern may predominate (mycoplasmal pneumonia). Pleural effusion and abscess formation are more consistent with bacterial infection. Ultrasound is becoming more common in evaluating consolidations and pleural effusions, but its sensitivity remains user dependent. Laboratory Findings.  Identifying an organism in pediatric pneumonia may prove difficult; causative organisms are identified in only 40% to 80% of cases. Nucleic acid PCR amplification of secretions from a nasal swab or wash often is performed to confirm a viral etiology. In the outpatient setting with an otherwise well child, studies such as nasopharyngeal cultures (poor sensitivity and specificity), sputum samples (difficulty obtaining specimens in young patients), blood cultures, CBC, and acute phase reactants (erythrocyte sedimentation rate, C-reactive protein, procalcitonin) are not necessary to guide treatment. For the admitted patient, and depending on the patient’s clinical status, a combination of these labs may be obtained.

Treatment Treatment for pneumonia varies by patient age and the suspected etiology. In the newborn, broad-spectrum antimicrobials (ampicillin with either gentamicin or

15_Toy-Pediatrics_Case14_p139-150.indd 144

12/07/21 2:53 PM

SECTION II: CLINICAL CASES

145

cefotaxime) are indicated. In the winter months when RSV and other respiratory viruses predominate, patients with nasal and chest congestion, increased work of breathing, wheezing, and hypoxemia may require admission for observation, hydration, oxygen, and bronchodilator therapies. In older children, antibacterial coverage should be considered if the clinical scenario, examination, or radiographic findings suggest bacterial infection. In children over the age of 5, antibiotics are directed toward Mycoplasma and typical bacteria, as listed earlier.

CLINICAL APPROACH TO TUBERCULOSIS Epidemiology Although the rate of new tuberculosis (TB) cases in the United States has decreased recently, the incidence of disease is still disproportionately high in certain ethnic, racial, and geographic communities. Hispanic and Asian youths, and recent immigrants, in particular, are at higher risk. Treatment of Mycobacterium tuberculosis has become more problematic; multidrug resistance is increasingly seen.

Clinical Presentation Signs and Symptoms.  Patients may present with symptoms ranging from a traditional cough, bloody sputum, fever, and weight loss to subtle or nonspecific symptoms. Some children will simply be more tired, play less, and be less active; others will only experience weight loss. Those who are HIV infected especially may have nonspecific symptoms. If the symptoms, especially cough, have persisted for more than 2 weeks, then TB should be considered. Extrapulmonary disease is common in children. Physical examination may reveal adenitis, especially in the cervical area. Often the lung examination has subtle or no abnormal findings. The chest x-ray remains the most important initial imaging tool; the most common abnormality is lymphadenopathy, especially with asymmetry. Laboratory Testing.  A positive purified protein derivative (PPD), also known as a positive tuberculin skin test (TST), is the traditional test described to diagnose TB. An interferon-gamma release assay (IGRA) can be used in place of the TST and preferentially is used for pregnant patients, patients who received bacillus Calmette-Guérin (BCG), and serial assessment of those infected and treated. Examples of IGRA studies include the Quantiferon Gold In-Tube (QFT-GIT) and the T-SPOT.TB (T-Spot) tests. Both screen whole blood for M. tuberculosis proteins not found in BCG but may cross-react with proteins in nontuberculous mycobacteria (Mycobacterium kansasii, Mycobacterium bovis) and cause falsepositive results. Sensitivity and specificity for both are about 90% in children. A positive TST or IGRA requires follow-up with a thorough physical examination and chest x-ray. Sources for acid-fast bacilli for stain and culture (depending on the age of the patient) include sputum samples, first-morning gastric aspirates, cerebrospinal fluid, bronchial washes or biopsy obtained through bronchoscopy, and empyema fluid analysis or pleural biopsy if surgical intervention is required.

15_Toy-Pediatrics_Case14_p139-150.indd 145

12/07/21 2:53 PM

146

CASE FILES: PEDIATRICS

Treatment Standard antituberculous therapy includes isoniazid, rifampin, and pyrazinamide. For possible drug-resistant organisms, ethambutol can be added temporarily. The typical antibiotic course consists of an initial phase of approximately 2 months’ duration on three or four medications, followed by a continuation phase of 4 to 7 months on isoniazid and rifampin. Therapy for 9 to 12 months is recommended for central nervous system or disseminated TB. Ultimately, total therapy duration is dependent upon the extent of imaging abnormalities, resistance patterns, and results of follow-up sputum samples in the age-appropriate patient. Directly observed therapy (DOT) is advised.

CASE CORRELATION šš

See Case 4 (Sepsis and Group B Streptococcal Infections), Case 6 (Neonatal Herpes Simplex Virus Infection), Case 7 (Esophageal Atresia), Case  8 (Transient Tachypnea of the Newborn), Case 10 (Failure to Thrive), and Case 13 (Sickle Cell Disease).

COMPREHENSION QUESTIONS 14.1 A 6-week-old boy, born by vaginal delivery after an uncomplicated term gestation, has experienced cough and “fast breathing” for 2 days. His mother relates that he has a 1-week history of nasal congestion and watery eye discharge, but he has had no fever or change in appetite. He has a temperature of 99.4 °F (37.4 °C) and a respiratory rate of 44 breaths per minute. He has nasal congestion, clear rhinorrhea, erythematous conjunctivae bilaterally, and watery, right eye discharge. His lungs demonstrate scattered crackles without wheezes. Which of the following is the most likely pathogen? A. Chlamydia trachomatis B. Listeria monocytogenes C. Respiratory syncytial virus (RSV) D. Rhinovirus E. Streptococcus pneumoniae

15_Toy-Pediatrics_Case14_p139-150.indd 146

12/07/21 2:53 PM

SECTION II: CLINICAL CASES

147

14.2 A 2-year-old girl has increased work of breathing. Her father notes she has had cough and subjective fever over the past 3 days. She has been complaining of abdominal pain and has experienced one episode of posttussive emesis but no diarrhea. Her immunizations are current, and she is otherwise healthy. Her temperature is 102 °F (38.9 °C). She is somnolent but easily aroused. Respirations are 28 breaths per minute, and her examination is remarkable for decreased breath sounds at the left base posteriorly with prominent crackles. Which of the following acute interventions is the next best step in your evaluation? A. B. C. D. E.

Blood culture Chest radiography Pulse oximetry Sputum culture Viral nasal swab

14.3 You are evaluating a previously healthy 8-year-old boy with subjective fever, sore throat, and cough over the past week. He has had no rhinorrhea, emesis, or diarrhea; his appetite is unchanged. His immunizations are current. His weight was at the 25th percentile on his examination 6 months ago. Today, he is noted to be at the 10th percentile for weight. He is afebrile, with clear nares and posterior oropharynx, and a normal respiratory effort. He has bilateral cervical and right supraclavicular lymphadenopathy. Chest auscultation is notable for diminished breath sounds at the left base. Beyond obtaining a chest radiograph, which of the following is the best next step in your evaluation? A. Rapid strep throat swab B. Viral nasal swab C. Purified protein derivative (PPD) placement D. Lymph node biopsy E. Bordetella pertussis direct fluorescent antibody (DFA) stain testing 14.4 A 15-year-old boy presents to your clinic as a new patient having arrived from Indiana after he completed proctoring at a 2-month rural summer camp. He reports 2 weeks of fever, cough, chest pain, increased fatigue, and headaches. His immunizations are current. He has a temperature of 101 °F (38.3 °C) but otherwise normal vital signs, and his physical examination reveals bibasilar rales. Chest x-ray was notable for enlarged hilar lymph nodes and interstitial infiltrates. Which of the following is the most sensitive test in diagnosing the most likely causative organism? A. Urine antigen test B. Serum antigen test C. Complement fixation D. Viral PCR E. Blood culture

15_Toy-Pediatrics_Case14_p139-150.indd 147

12/07/21 2:53 PM

148

CASE FILES: PEDIATRICS

ANSWERS 14.1 A. C. trachomatis. Cough and increased respiratory effort in an afebrile infant with eye discharge are consistent with Chlamydia. Transmission typically occurs during vaginal delivery. Approximately 25% of infants born to mothers with Chlamydia trachomatis develop conjunctivitis; about half of these develop pneumonia. Most infants present with respiratory infection in the second month of life, but symptoms can be seen as early as the second week. Inner eyelid swabs are sent for PCR, and oral erythromycin or sulfisoxazole (older than 2 months) is given for 2 weeks for either conjunctivitis or pneumonia. L. monocytogenes (answer B) usually presents within 24 hours of birth as sepsis or at 14 days of life as meningitis. RSV (answer C) usually affects babies with nasal congestion, cough, tachypnea, and fever. S. pneumoniae (answer E) can affect infants between ages 3 weeks and 3 months but usually does not cause conjunctivitis. 14.2 C. Pulse oximetry. Tachypnea and lethargy are prominent in this patient with clinical pneumonia. Pulse oximetry should urgently be performed to assess oxygenation and ascertain whether oxygen is required. Other diagnostic testing can be performed after the patient is noted to be stable. Sputum culturing (answer D) is reasonable for an older patient who can produce sputum, but an adequate and diagnostically useful specimen can only be obtained from a 2-year-old by endotracheal aspirate or bronchoscopy. In this otherwise healthy toddler for whom concerns for atypical pneumonia are high, invasive maneuvers are not indicated. Viruses (RSV and adenovirus) are prominent at this age; one might consider performing a nasal swab (answer E) viral PCR, but ensuring oxygenation is still the priority. Abdominal pain, as noted in this question, can be seen as a presenting symptom in pneumonia, probably because of irritation of the diaphragm by pulmonary infection. 14.3 C. PPD placement. The scenario is typical for pediatric TB. Neck and perihilar or mediastinal lymphadenopathy and pulmonary or extrapulmonary manifestations can occur, with miliary disease and meningitis being more common in infants and younger children. Fever, weight loss, and lower respiratory tract signs and symptoms (possible left pleural effusion in this patient) are archetypal TB findings. A PPD should be placed or interferon-gamma release assay (IGRA) drawn, and consideration should be given to hospitalizing this patient in negative-pressure isolation for further evaluation beyond initial screening (pleurocentesis, bronchoalveolar lavage, gastric aspirates) and possible antituberculous treatment. Strep throat (answer A) and pertussis (answer E) usually do not present with lung findings. Lymph node biopsy (answer D) is more invasive, and if the PPD is positive, a biopsy would not be required. 14.4 A. Urine antigen test. This patient most likely has acute pulmonary histoplasmosis, caused by H. capsulatum. He has recently moved from an endemic region (the Midwest) and has been potentially exposed to contaminated soil (such as when hiking). Typical symptoms of acute pulmonary histoplasmosis

15_Toy-Pediatrics_Case14_p139-150.indd 148

12/07/21 2:53 PM

SECTION II: CLINICAL CASES

149

are flu-like, including fevers, chills, nonproductive cough, and malaise. The most sensitive test for detecting H. capsulatum in acute pulmonary infections is urine antigen detection. Serum antigen testing (answer B) is less sensitive than urine antigen testing. Although viral PCR (answer D) could be useful in diagnosing influenza, the time of year makes flu less likely. Complement fixation (answer C) is useful for diagnosing coccidioidomycosis. Blood cultures (answer E) would not be helpful, as it is unlikely that the patient has a bacterial infection.

CLINICAL PEARLS »»

The etiology of pneumonia varies according to the patient’s age. Neonates have the greatest risk of group B Streptococcus, toddlers are more likely to have RSV, and adolescents usually contract Mycoplasma.

»»

Historical clues, including travel history and exposures, may help define an atypical pathogen in lower respiratory tract infections.

»»

Efforts in TB management should be directed toward isolating an organism and obtaining sensitivities, thus allowing selection of the optimal antituberculous regimen.

»»

The child with tracheoesophageal atresia will have recurrent episodes of aspiration resulting in pneumonia.

»»

In the first hours of life, transient tachypnea of the newborn may result in increased respiratory rate and streakiness on the radiograph; the condition typically self-resolves in the first 2 days of life but occasionally may be confused with neonatal pneumonia.

REFERENCES American Academy of Pediatrics. Histoplasmosis. In: Kimberlin DW, Brady MT, Jackson MA, Long SS, eds. Red Book: 2018 Report of the Committee on Infectious Diseases. Washington, DC: American Academy of Pediatrics; 2018:449-453. American Academy of Pediatrics. Tuberculosis. In: Kimberlin DW, Brady MT, Jackson MA, Long SS, eds. Red Book: 2018 Report of the Committee on Infectious Diseases. Washington, DC: American Academy of Pediatrics; 2018:829-853. Bickley LS, Szilagyi PG, Hoffman RM. The thorax and lungs. In: Bates’ Guide to Physical Examination and History Taking. 21st ed. Philadelphia, PA: Wolters Kluwer; 2017:303-342. Bradley JS, Byington CL, Shah SS, et al. The management of community-acquired pneumonia in infants and children older than 3 months of age: clinical practice guidelines by the Pediatric Infectious Diseases Society and the Infectious Diseases Society of America. Clin Infect Dis. 2011;53:e25-e76. Federico MJ, Baker CD, Deboer EM, et al. Respiratory tract and mediastinum. In: Hay WW Jr, Levin MJ, Deterding RR, Abzug MJ, eds. Current Diagnosis & Treatment: Pediatrics. 24th ed. New York, NY: McGraw Hill; 2018. Haslam DB. Epidemiology of infections. In: Kleigman RM, St. Geme JW, Blum NJ, et al, eds. Nelson Textbook of Pediatrics. 21st ed. Philadelphia, PA: Elsevier; 2020:996-1005.

15_Toy-Pediatrics_Case14_p139-150.indd 149

12/07/21 2:53 PM

150

CASE FILES: PEDIATRICS

Jones K, Yoff M, Sockrider MM, Cateletto M. Respiratory disorders. In: Hannaman RA. MedStudy: Pediatrics Review Core Curriculum. 7th ed. Colorado Springs, CO; 2015:13-1-13-32. Kelly MS, Sandora TJ. Community-acquired pneumonia. In: Kleigman RM, St. Geme JW, Blum NJ, et al, eds. Nelson Textbook of Pediatrics. 21st ed. Philadelphia, PA: Elsevier; 2020:2266-2274. Moscona A, Murrell MT, Horga M, Burroughs M. Respiratory infections. In: Katz SL, Hotez PJ, Gerson AA, eds. Krugman’s Infectious Diseases of Children. 11th ed. Philadelphia, PA: Mosby; 2005:493-524. The University of Texas Health Science Center at Tyler Heartland National TB Center. Tuberculosis at a Glance: A Reference for Practitioners on Basic Tuberculosis Information. https://dph.georgia.gov/ sites/dph.georgia.gov/files/TB-TBataGlance_heartland.pdf. Accessed March 8, 2020.

15_Toy-Pediatrics_Case14_p139-150.indd 150

12/07/21 2:53 PM

CASE 15 A mother brings her 2-year-old son to the clinic for a 3-day history of having about 1 tablespoon of bright red blood in his diaper with each stool and spots of red blood when she wipes him after his stooling. He has not had abdominal pain, but for the past week, he has sometimes cried with stooling and now cries when she wipes him. The mother denies fever, vomiting, and diarrhea. She reports he previously had two soft bowel movements per day, but over the past month, he has been having one hard and bulky bowel movement per day. The mother and the child’s father are his only caregivers, and she notes that the change occurred after she started “potty training.” He acts afraid of the toilet and avoids it. He is eating his regular diet of grilled cheese sandwiches with 20 oz of milk and 20 oz of a fruit-flavored beverage each day. On examination, his heart rate is 112 beats per minute, temperature is 98.4 °F (36.9 °C), and he is playing on the examination table with his toy cars. His height and weight are in the 90th percentile for his age. On palpation of his abdomen, he has no tenderness, guarding, or hepatosplenomegaly. On inspection of the rectal area, you find a 7-mm linear split in the posterior midline traversing from the anocutaneous junction to the dentate line. A fecal occult blood test (FOBT) is positive. ▶▶ ▶▶

What is the most likely diagnosis? What is the best management for this condition?

16_Toy-Pediatrics_Case15_p151-158.indd 151

12/07/21 3:19 PM

152

CASE FILES: PEDIATRICS

ANSWERS TO CASE 15: Rectal Bleeding Summary: A 2-year-old boy presents with šš

Rectal bleeding

šš

Painful defecation following a period of constipation and stool-withholding behavior

šš

Normal physical examination except for a fissure in the perianal region

Most likely diagnosis: Anal fissure, one of the most common causes of rectal bleeding in children of all ages. Best management: Begin dietary changes and a stool softener to ameliorate constipation. Parents should minimize foods known to be constipating (such as dairy products), increase water intake, and avoid bulking agents (such as fiber). For other causes of constipation, fiber is important. Oral polyethylene glycol (PEG) 3350 is the most commonly used stool softener for children because it is an odorless, flavorless powder that can be dissolved in any liquid. Suppositories should be avoided because they will further traumatize the fissured skin. Application of petrolatum and gentle wiping should be performed after each stool until the skin no longer bleeds.

ANALYSIS Objectives 1. Know the differential diagnosis for rectal bleeding at various ages. (EPA 2) 2. Know how to manage rectal bleeding. (EPA 4, 12) 3. Be familiar with methods of investigating the cause of bleeding. (EPA 1, 3)

Considerations Anal fissures are one of the most common causes of rectal bleeding in pediatrics. It is a benign condition that usually results from constipation; large or dried hard stool can cause splitting of the skin. A cycle can then ensue in which the child avoids stooling due to the pain at the fissure site during defecation, which leads to accumulation of bulkier stool. Although the diagnosis of an anal fissure is made by physical examination, certain historical details suggest the child is prone to their development, such as resisting stooling (an infant’s extending their legs), finding encopresis in an older child, or noting the patient may be struggling to stool (grunting or taking long periods in the bathroom to stool). The constipation and these accompanying behaviors may precede the bleeding by weeks to months and usually are provoked by a change in either the child’s diet (such as beginning more solid foods) or when the child no longer has a designated time to spend for stooling (such as school entry). The bleeding that occurs with a fissure will be small in volume and may be associated with pain during or following defecation and with wiping the perianal area.

16_Toy-Pediatrics_Case15_p151-158.indd 152

12/07/21 3:19 PM

SECTION II: CLINICAL CASES

153

APPROACH TO: Rectal Bleeding DEFINITIONS FECAL OCCULT BLOOD TEST (FOBT): A small amount of stool is placed on a test strip and mixed with a reagent containing guaiacum (usually called guaiac). In the presence of hemoglobin, a blue color appears. HEMATOCHEZIA: Blood in the stool that is red or maroon colored; may sometimes be referred to as bright red blood per rectum (BRBPR). LOWER GASTROINTESTINAL (GI) TRACT BLEEDING: Bleeding within any part of the intestine that is distal to the ligament of Treitz, which encompasses the small bowel beginning with the jejunum and extending to the rectum. MECKEL DIVERTICULUM: A 3- to 6-cm pouch off the ileum that is a remnant of the omphalomesenteric duct. It is often lined with an endothelium that can secrete acid similar to gastric mucosa, causing ulceration of the adjacent ileal mucosa. It causes half of the lower GI bleeds in children aged 2 years or older. It can have a chronic presentation with occult blood detected in the stool by FOBT, or it can present with acute large-volume hematochezia and a child in shock.

CLINICAL APPROACH Pathophysiology Lower GI tract bleeding most often presents with hematochezia. Distinguishing whether the cause of the bleeding is serious or benign begins with the history. The patient should be asked to quantify the amount of blood as well as its distribution, such as if the stool is mixed with blood streaks or is brick-colored, if the toilet water was stained red, or if there was blood with wiping. If the patient’s stooling is supervised, then the parent should also provide the history. Information should also be obtained about the baseline and recent stooling pattern, such as the frequency, size, texture, time needed to stool, ease of stooling, and presence of encopresis. Accompanying symptoms of fever, weight loss, pain, and diarrhea would suggest inflammatory bowel disease in a school-age child or adolescent, especially if a positive family history of autoimmune-mediated disease is found. Fever, myalgias, vomiting, diarrhea, and recent antibiotics or contact with a potential contaminated food or water source can identify infectious enteritis or colitis. Breastfed infants or those on formula, including soy-based milk, who exhibit respiratory distress, vomiting with feeds, and failure to thrive may exhibit symptoms of milk protein allergy. Painless rectal bleeding with a history of normal stooling and no associated symptoms could be caused by a juvenile polyp or a Meckel diverticulum. Juvenile polyps, one of the most common causes of rectal bleeding in children aged 1 to 7 years old, are benign and will cause bleeding when dislodged by the passage of stool. Common etiologies of lower GI bleeding are listed in Table 15–1.

16_Toy-Pediatrics_Case15_p151-158.indd 153

12/07/21 3:19 PM

154

CASE FILES: PEDIATRICS

Table 15–1  •  COMMON ETIOLOGIES OF LOWER GASTROINTESTINAL BLEEDING IN CHILDREN Age Group

Most Severe Bleeding

Less Severe Bleeding

Infants

Hirschsprung disease Stress ulcers Necrotizing enterocolitis Vitamin K deficiency and other coagulopathies Volvulus

Anal fissure Milk protein allergy Viral infection leading to diarrhea

Children

Gastritis Intussusception Liver dysfunction Meckel diverticulum Varices Vasculitides Volvulus

Esophagitis Anal fissures Hemorrhoids Infectious enterocolitis Juvenile polyp

Adolescents

Inflammatory bowel disease (eg, ulcerative colitis and Crohn disease) Meckel diverticulum NSAID use Vasculitides

Anal fissures Hemorrhoids Infectious enterocolitis

NSAID, nonsteroidal anti-inflammatory drug.

Physical Examination Physical examination begins with the vital signs and the patient’s general appearance. Tachycardia and diaphoresis may be present before hematochezia appears. Hypotension or lethargy is a late and ominous finding, indicating the patient is in shock. Signs of pain may be seen when the bowel is ischemic, such as with intussusception or volvulus. The conjunctiva and oral mucosa may not reflect pallor with acute bleeding. Hepatosplenomegaly, petechiae, or purpura would indicate a coagulopathy as the underlying cause of the rectal bleeding. Palpation may elicit abdominal pain in areas where inflammation is present, or if constipation is present, hard stool may be palpable. As part of the abdominal examination, the perirectal skin is inspected. Findings might include a fissure, skin tag, fistula, or hemorrhoid. Rectal examination is useful because hard or impacted stool and benign polyps can be identified. In addition, stool can be obtained for FOBT; some red-pigmented food products, such as beets, gelatin, or beverages, can pass unchanged through the GI tract and mimic blood, but the stool will have a negative FOBT.

Laboratory Workup With chronic blood loss, a low hemoglobin and hematocrit are accompanied by a low mean corpuscular volume (MCV), indicating iron deficiency anemia. An infant suspected of sensitivity to cow’s milk should be transitioned to either hydrolyzed or amino acid–based formulas. Stool studies, erythrocyte sedimentation rate (ESR), and C-reactive protein should be sent if a history of fever or diarrhea is found.

16_Toy-Pediatrics_Case15_p151-158.indd 154

12/07/21 3:19 PM

SECTION II: CLINICAL CASES

155

Laboratory evaluation for contributing coagulopathic conditions should be performed, which includes measurement of platelets, prothrombin time (PT), activated partial thromboplastin time (APTT), liver enzymes, serum albumin level, and creatinine. Blood urea nitrogen (BUN) levels may be elevated due to urea being produced from hemoglobin breakdown in the GI tract.

Imaging A single-view supine, frontal view radiograph of the abdomen (kidneys, ureter, and bladder [KUB]) may show an obstructive pattern that could signal intussusception or volvulus and the need for urgent surgical consultation. The KUB can be normal in these conditions, so air-contrast enema or ultrasound should follow if these lifethreatening causes are still suspected. When the child is well-appearing, the blood loss is a small quantity, and the expected course of the underlying cause of GI bleeding is benign, no laboratory investigation or imaging may be necessary. However, if the bleeding continues to recur or worsens, a technetium-99m (Tc99m) scan may be indicated to identify a Meckel diverticulum.

Treatment Management of the rectal bleeding is determined by the patient’s examination. If the vital signs are abnormal or if the child appears ill or is exhibiting pain, then stabilization and transfer to an emergency department are the first steps in management. Similarly, a large-volume bleed or increasing bleeding may warrant urgent evaluation to identify the cause and to initiate monitoring of hemoglobin or hematocrit. In such situations, intravascular volume is initially restored with isotonic saline; packed red blood cells may be needed. Consultation with a pediatric gastroenterologist and colonoscopy are usually part of the management of lower GI bleeding when inflammatory bowel disease is suspected or if the source of the bleeding cannot be identified.

CASE CORRELATION šš

See Case 28 (Bacterial Enteritis) and Case 53 (Inflammatory Bowel Disease).

COMPREHENSION QUESTIONS 15.1 A 5-year-old boy is brought to clinic by his mother for evaluation of blood in his stool. Yesterday, the boy told his mother that he saw “red stuff ” on his stool. When the mother visualized the stool, it appeared to have “threads of blood” over the surface. She states that he has anal fissures. The provider believes that the cause of the bleeding is not due to anal fissures. Which of the following findings would support this hypothesis? A. The boy has a 1-month history of encopresis. B. The child’s height and weight are at the 75th percentile for his age. C. The boy’s conjunctivae, gingiva, and nail beds are pale. D. The child affirms that it has been painful to wipe after stooling. E. Her son prefers to eat the school lunch, consisting of pizza and chocolate milk, rather than a homemade sandwich with water.

16_Toy-Pediatrics_Case15_p151-158.indd 155

12/07/21 3:19 PM

156

CASE FILES: PEDIATRICS

15.2 A 2-year-old girl presents to the emergency department with her second episode of bloody stool. The mother has brought in the pull up, which is filled with about one-fourth cup of brick-colored stool. The first episode of hematochezia occurred 6 months ago, but the stools returned to normal within 2 days. The mother denies constipation, diarrhea, and fever. On physical examination, the child is awake and alert, her heart rate is 150 beats per minute, she has no tenderness on palpation of the abdomen, and the rectal examination is normal except that her fecal occult blood test (FOBT) is positive. What would be the next best steps in management? A. Prescribe a stool softener and have the child follow-up with her pediatrician the next day. B. Reassure the mother that the cause of bleeding is a benign polyp and no treatment is needed. C. Inquire about any ibuprofen use and prescribe omeprazole. D. Administer an intravenous (IV) normal saline bolus and order a technetium-99m (Tc99m) scan. E. Ask the mother about any family history of Crohn disease or ulcerative colitis, and order an erythrocyte sedimentation rate (ESR). 15.3 A 14-year-old girl reports she has intermittent bouts of blood in her stools that last 2 days. Once the bleeding stops, it does not recur for about 2 weeks. She denies fever, abdominal pain, and diarrhea. She affirms that she has constipation. On physical examination, you see an anal fissure. You counsel her on how to take PEG 3350 and how to apply petrolatum to the site of the fissure. What else should you discuss with her in order to prevent a recurrence of the fissure? A. She should use a suppository if she feels constipated again. B. She should increase her dietary fiber intake. C. She should return to the clinic for further evaluation if the fissure recurs, despite resolution of the constipation. D. She should decrease the number of times she goes to defecate. E. She should apply triple-antibiotic ointment to the fissure.

16_Toy-Pediatrics_Case15_p151-158.indd 156

12/07/21 3:19 PM

SECTION II: CLINICAL CASES

157

15.4 A 15-month-old boy is brought to the emergency department by his mother for increasing bloody diarrhea over the past 2 days. She reports he was febrile 3 days ago with several episodes of emesis that had resolved by yesterday. In the past 24 hours, he has had more than 10 episodes of diarrhea, with the most recent appearing to consist of only blood with no identifiable stool mixed with it. On physical examination, he is somnolent. Which of the following is the most important part of his immediate management? A. A bolus of 5% dextrose in normal saline (D5NS) B. Measurement of prothrombin time (PT) and activated partial thromboplastin time (APTT) C. Computed tomography (CT) imaging of the abdomen D. Intranasal naloxone E. IV antibiotics

ANSWERS 15.1 C. The boy’s conjunctivae, gingiva, and nail beds are pale. Bleeding from an anal fissure is not chronic, and the amount of hematochezia is small. The physical exam findings suggest significant anemia. A hemoglobin that is low enough to cause pallor of the mucosal membranes is not produced from an anal fissure. The other answer choices (answer A, history of encopresis; answer B, height and weight in the 75th percentile; and answer D, painful wiping after stooling) are consistent with anal fissures. 15.2 D. Administer an IV normal saline bolus and order a Tc99m scan. This child is tachycardic, and the amount of GI bleeding is significant. She requires her intravascular volume to be restored; thus, IV normal saline should be administered and vital signs monitored. Given her age, the most likely cause of her painless rectal bleeding is Meckel diverticulum. The other answer choices (answer A, stool softener and follow-up with pediatrician; answer B, reassurance and no treatment; answer C, inquire about ibuprofen and prescribe omeprazole; and answer E, ask about family history of Crohn or ulcerative colitis and order ESR) do not address the ABCs (airway, breathing, and circulation), the keys to preventing further hemorrhage and shock. 15.3 C. She should return to the clinic for further evaluation if the fissure recurs, despite resolution of the constipation. A suppository (answer A), increased fiber (answer B), and stool withholding (answer D) will increase the stool bulk and are likely to cause a fissure. The etiology for her fissure formation is not infectious (answer E). If she no longer has constipation but the fissure recurs, then Crohn disease may be the cause and she should have further evaluation. 15.4 A. A bolus of D5NS. Given the child’s somnolence, immediate restoration of intravascular volume is needed. Isotonic saline should be given until volume is restored. Based on the hemoglobin and clotting studies (answer B), he may need red blood cells or plasma, but this is not the main concern at present.

16_Toy-Pediatrics_Case15_p151-158.indd 157

12/07/21 3:19 PM

158

CASE FILES: PEDIATRICS

His clinical presentation could represent intussusception, and if seen on imaging, surgical consultation is needed. Ultrasound is preferred rather than CT imaging (answer C) in children. Naloxone (answer D) is not warranted because there is no indication of opioid intoxication. IV antibiotics (answer E) are not indicated because there is no evidence of systemic infection.

CLINICAL PEARLS »»

Anal fissures are the most common cause of hematochezia in pediatrics.

»»

Tachycardia is the first indication that the rate or volume of bleeding is significant and warrants stabilization with isotonic saline and transfer to a hospital.

»»

If the patient is lethargic or ill-appearing and has abdominal pain, emergent laboratory and radiographic evaluation is indicated.

»»

Lower GI bleeding with a benign etiology identified on physical examination does not require any laboratory or radiographic studies.

REFERENCES Aprahamian CJ, Mortellaro VE. Omphalo-mesenteric duct remnants. In: Kline MW, Blaney SM, Giardino AP, et al, eds. Rudolph’s Pediatrics. 23rd ed. New York, NY: McGraw Hill; 2018:1764-1766. Garner EF, Beierle EA. Intussusception. In: Kline MW, Blaney SM, Giardino AP, et al, eds. Rudolph’s Pediatrics. 23rd ed. New York, NY: McGraw Hill; 2018:1769-1771. Niedich GA, Cole SR. Gastrointestinal bleeding. Pediatr Rev. 2014;35:243-254. Noel RJ. Upper and lower gastrointestinal bleeding. In: Kline MW, Blaney SM, Giardino AP, et al, eds. Rudolph’s Pediatrics. 23rd ed. New York, NY: McGraw Hill; 2018:1714-1717. Shanti CM. Anal fissure. In: Kleigman RM, St. Geme JW, Blum NJ, et al, eds. Nelson Textbook of Pediatrics. 21st ed. Philadelphia, PA: Elsevier; 2020:2059.

16_Toy-Pediatrics_Case15_p151-158.indd 158

12/07/21 3:19 PM

CASE 16 A 3-year-old child presents with the complaint of right ear pain for 1 day. The patient’s mother says that 3 days prior he developed a cough and yellow nasal discharge. One day prior, he began complaining of right ear pain and had a temperature of 101.5 °F (38.6 °C) at that time. He has had no nausea, vomiting, diarrhea, or rash. His urine output has been good; his solid intake is slightly decreased, but he is drinking well. She reports that he is fussy at night but is acting normal throughout the day. His mother notes that his pain is minimally improved with acetaminophen and ibuprofen. ▶▶ ▶▶

What is the most likely diagnosis? What is the best therapy?

17_Toy-Pediatrics_Case16_p159-168.indd 159

29/07/21 8:13 PM

160

CASE FILES: PEDIATRICS

ANSWERS TO CASE 16: Acute Otitis Media Summary: A 3-year-old boy presents with šš

Ear pain of 1 day’s duration

šš

Cough and yellow nasal discharge that began 3 days prior

šš

Fever

šš

Fussiness at night

šš

No nausea, vomiting, diarrhea, or rash

šš

Good urine output and liquid intake; slightly decreased solid intake

Most likely diagnosis: Acute otitis media (AOM). Best therapy: Oral antibiotics.

ANALYSIS Objectives 1. Be familiar with the epidemiology of otitis media (OM) in children. (EPA 12) 2. Understand the treatment of this condition. (EPA 4) 3. Learn the consequences of severe infection. (EPA 10, 12)

Considerations Otitis media is high on the differential diagnosis for this child with a recent upper respiratory infection (URI) and ear pain. The diagnosis can be confirmed by pneumatic otoscopy, and treatment can be started. A “telephone diagnosis” should be avoided, as the patient should be assessed to see if antibiotic therapy is necessary or if other constitutional signs such as meningitis are present. Figure 16–1 illustrates the anatomy of the middle ear.

17_Toy-Pediatrics_Case16_p159-168.indd 160

29/07/21 8:13 PM

161

SECTION II: CLINICAL CASES

Crura of stapes Incus

Footplate of stapes in oval window

Malleus

Semicircular canals

Tympanic membrane (eardrum)

Facial nerve Vestibular nerve

Auricle (pinna) External acoustic meatus (ear canal) Middle ear Round window

Internal acoustic meatus Cochlear nerve Cochlea Eustachian tube

Parotid gland

Internal jugular vein

Figure 16–1.  Anatomy of the middle ear. (Modified with permission, from Rudolph CD, Rudolph AM, Hostetter MK, et al. Rudolph’s Pediatrics. 21st ed. 2003. Copyright © McGraw Hill LLC. All rights reserved.)

APPROACH TO: Acute Otitis Media DEFINITIONS ACUTE OTITIS MEDIA (AOM): A condition of otalgia (ear pain), fever, and other symptoms along with findings of a red, opaque, poorly moving, bulging tympanic membrane (TM). MYRINGOTOMY AND PLACEMENT OF PRESSURE EQUALIZATION (PE) TUBES: A surgical procedure involving TM incision and placement of PE tubes (tiny plastic or metal tubes anchored into the TM) to ventilate the middle ear and help prevent reaccumulation of middle ear fluid. OTITIS MEDIA WITH EFFUSION: A condition in which fluid collects behind the TM but without signs and symptoms of AOM. Sometimes also called serous OM. PNEUMATIC OTOSCOPY: The process of obtaining a tight ear canal seal with a speculum and then applying slight positive and negative pressure with a rubber bulb to verify TM mobility. TYMPANOCENTESIS: A minor surgical procedure in which a small incision is made into the TM to drain pus and fluid from the middle ear space. This procedure rarely is done in the primary care office; rather, it is usually done by a specialist. TYMPANOMETRY: An examination that measures the transfer of acoustic energy at varying levels of ear canal pressures, which will reflect TM mobility.

17_Toy-Pediatrics_Case16_p159-168.indd 161

29/07/21 8:13 PM

162

CASE FILES: PEDIATRICS

CLINICAL APPROACH Pathophysiology Otitis media is a common childhood diagnosis. Common bacterial pathogens include Streptococcus pneumoniae, nontypeable Haemophilus influenzae, and Moraxella catarrhalis. Since introduction of the pneumococcal vaccine, nontypeable H. influenzae has become the most isolated bacterial pathogen in children with AOM. Other organisms (Staphylococcus aureus, Escherichia coli, Klebsiella pneumoniae, and Pseudomonas aeruginosa) are seen in neonates and patients with immune deficiencies. Viruses can cause AOM, and in many cases, the etiology is unknown.

Clinical Presentation Acute OM is suspected in a child with fever (usually less than 104 °F [40 °C]), ear pain (often nocturnal, awakening child from sleep), and generalized malaise. Systemic symptoms may include anorexia, nausea, vomiting, irritability, crying at night, diarrhea, and headache. Children are more susceptible to developing AOM due to the anatomy of the Eustachian tubes positioned more horizontally than in adolescents and adults, leading to fluid accumulating in the middle ear more easily. A preceding viral URI leads to inflammation and decreased drainage of middle ear fluid, as well as to development of infection in the middle ear. The cornerstone of diagnosis is the physical examination findings on pneumatic otoscopy of a red or opaque, bulging TM with middle ear effusion and decreased mobility by either pneumatic otoscopy and/or tympanometry. The TM may be opaque with pus behind it, the middle ear landmarks may be obscured, and if the TM has ruptured, pus may be seen in the ear canal. Normal landmarks are shown in Figure 16–2.

Short process malleus

Incus

Manubrium

Round window

Light reflex Umbo

Figure 16–2.  The tympanic membrane. (Recreated with permission, from Rudolph CD, Rudolph AM, Hostetter MK, et al. Rudolph’s Pediatrics. 21st ed. 2003. Copyright © McGraw Hill LLC. All rights reserved.)

17_Toy-Pediatrics_Case16_p159-168.indd 162

29/07/21 8:13 PM

SECTION II: CLINICAL CASES

163

Treatment Watchful Waiting.  In some situations, especially in a child older than 6 months with mild symptoms (ie, mild otalgia for less than 48 hours, temperature below 39 °C), a “watchful waiting” period of a few days may be indicated because many AOM cases self-resolve. Numerous studies have shown that only about one-third of children with evidence of AOM initially observed for a period had persistent or worsening symptoms that required rescue antibiotics. Alternatively, an effective approach is providing a “safety net prescription” to be filled if symptoms do not improve in 48 hours. This has also led to reduced antibiotic use and has been accepted by caregivers. Ensuring close medical follow-up is paramount if the choice is made to withhold antibiotics and to observe children with mild AOM. Pharmacotherapy.  Should antibiotics be deemed necessary, and depending on a community’s bacterial resistance patterns, amoxicillin at doses up to 80 to 90 mg/kg/d for 7 to 10 days is often the initial treatment. If the child has received amoxicillin in the previous 30 days, has a history of recurrent AOM unresponsive to amoxicillin, or has concurrent purulent conjunctivitis (likely due to nontypeable H. influenzae), an antibiotic with beta-lactamase coverage is warranted. In the child begun on amoxicillin who demonstrates clinical failure after 3 treatment days, a change to amoxicillin-clavulanate, cefuroxime axetil, cefdinir, azithromycin, ceftriaxone, or tympanocentesis is considered. Adjuvant therapies (analgesics or antipyretics) often are indicated, but other measures (antihistamines, decongestants, and corticosteroids) are ineffective. Recently, the sugar supplement xylitol administered as chewing gum, lozenges, or syrup has been shown to reduce the recurrence of AOM, but adherence is sometimes a challenge. Myringotomy With PE Tubes.  After an AOM episode, middle ear fluid can persist for up to several months. If hearing is normal, middle ear effusion often is treated with observation; some practitioners treat with antibiotics. When the fluid does not resolve or recurrent episodes of AOM occur (defined as three or more episodes in the previous 6 months or four or more episodes in the previous year with one episode in the previous 6 months), especially if hearing loss is noted, myringotomy with PE tubes is often implemented.

Complications Rare but serious AOM complications include mastoiditis, temporal bone osteomyelitis, facial nerve paralysis, epidural and subdural abscess formation, meningitis, lateral sinus thrombosis, and otitic hydrocephalus (evidence of increased intracranial pressure with OM). An AOM patient whose clinical course is unusual or prolonged should be evaluated for one of these conditions.

CASE CORRELATION šš

See Case 10 (Failure to Thrive).

17_Toy-Pediatrics_Case16_p159-168.indd 163

29/07/21 8:13 PM

164

CASE FILES: PEDIATRICS

COMPREHENSION QUESTIONS 16.1 An 11-month-old boy presents with fever, ear pain, and purulent discharge from both eyes. On examination, bilateral, erythematous, bulging TMs and purulent conjunctivitis are noted. Based on his symptoms and examination, amoxicillin-clavulanate is prescribed. The mother asks what she could do to prevent future ear infections. Which of the following is not one of the current recommendations to reduce the incidence of otitis media? A. Pneumococcal vaccine B. Influenza vaccine C. Xylitol D. Eliminating exposure to tobacco smoke E. Breastfeeding 16.2 Three days after beginning oral amoxicillin therapy for OM, a 4-year-old boy has continued fever, ear pain, and swelling with redness behind his ear. His ear lobe is pushed superiorly and laterally; he has tenderness posterior to the ear. He seems to be doing well otherwise. Which of the following is the most appropriate course of action? A. Change to oral cefdinir B. Myringotomy and parenteral antibiotics C. Nuclear scan of the head D. Topical steroids E. Tympanocentesis 16.3 A 5-year-old girl developed high fever, ear pain, and vomiting a week ago. She was diagnosed with OM and started on amoxicillin-clavulanate. On the third day of this medication, she continued with findings of OM, fever, and pain. She received ceftriaxone intramuscularly and switched to oral cefuroxime. Now, 48 hours later, she has fever, pain, and no improvement in her OM; otherwise, she is doing well. Which of the following is the most logical next step in her management? A. Addition of intranasal topical steroids to the oral cefuroxime B. Adenoidectomy C. Addition of otic drops to current antibiotic regimen D. Oral trimethoprim-sulfamethoxazole E. Tympanocentesis and culture of middle ear fluid

17_Toy-Pediatrics_Case16_p159-168.indd 164

29/07/21 8:13 PM

SECTION II: CLINICAL CASES

165

16.4 A 1-month-old boy has a fever of 102.7 °F (39.3 °C), is irritable, has diarrhea, and has not been eating well. On examination, he has an immobile, dull, and red TM that has pus behind it. Which of the following is the most appropriate course of action? A. Admission to the hospital with complete sepsis evaluation B. Intramuscular ceftriaxone and close outpatient follow-up C. Oral amoxicillin-clavulanate D. Oral cefuroxime E. High-dose oral amoxicillin

ANSWERS 16.1 C. Xylitol. Although some research shows xylitol use five times a day in gum or syrup can reduce AOM, adherence issues make it an ineffective therapy. With the pneumococcal vaccine (answer A), a 2016 Cochrane review showed the overall incidence of AOM has been reduced by about 6.5%. Because as many as two-thirds of children who have influenza have AOM, studies have demonstrated the influenza vaccine (answer B) has a 30% to 55% efficacy of preventing AOM during respiratory season. Breastfeeding (answer E) for at least 4 to 6 months reduces episodes of AOM and recurrent AOM. In addition to other benefits, elimination of passive tobacco smoke exposure (answer D) reduces AOM in infancy. 16.2 B. Myringotomy and parenteral antibiotics. The child has mastoiditis, a clinical diagnosis that can require computed tomography (CT) scan confirmation. Treatment includes myringotomy, fluid culture, and parenteral antibiotics. Surgical drainage of the mastoid air cells may be needed if improvement is not seen in 24 to 48 hours. Change of antibiotics (answer A) would not help without surgical drainage. Nuclear scan (answer C) is not as effective as CT imaging. Topical steroids (answer D) are not the appropriate therapy. Tympanocentesis (answer E) would be a treatment for severe otitis media but not for mastoiditis. 16.3 E. Tympanocentesis and culture of middle ear fluid. After failing several antibiotic regimens, tympanocentesis and culture of the middle ear fluid are indicated to assess for an atypical pathogen or resistant organism. Intranasal steroids (answer A) would be helpful for allergic rhinitis but not refractory OM. Adenoidectomy (answer B) is not indicated at this time. Otic drops (answer C) are helpful for otitis externa but not for refractory OM. Oral trimethoprim-sulfamethoxazole (answer D) would not be the best choice because the patient has already been on two appropriate antibiotic regimens. 16.4 A. Admission to the hospital with complete sepsis evaluation. Very young children with OM (especially if irritable, lethargic, or with decreased intake) are at higher risk for bacteremia or other serious infection. Hospitalization and parenteral antibiotics often are needed. Answer C (oral amoxicillin-clavulanate), answer D (oral cefuroxime), and answer E (high-dose oral amoxicillin) offer

17_Toy-Pediatrics_Case16_p159-168.indd 165

29/07/21 8:13 PM

166

CASE FILES: PEDIATRICS

oral antibiotics, which will likely not be effective enough to address the infection. Similarly, answer B (intramuscular ceftriaxone and outpatient follow-up) does not offer enough support to this 1-month-old child with these signs and symptoms.

CLINICAL PEARLS »»

The most common bacterial pathogens causing otitis media (OM) are S. pneumoniae, nontypeable H. influenzae, and M. catarrhalis.

»»

Examination findings of OM include a red, bulging tympanic membrane (TM) that does not move well with pneumatic otoscopy, an opaque TM with pus behind it, and obscured middle-ear landmarks; if the TM has ruptured, pus is usually seen in the ear canal.

»»

Initial treatment of OM often includes amoxicillin (depending on local bacterial resistance patterns). If a clinical failure is seen on day 3, a tympanocentesis or a change to amoxicillin-clavulanate, cefuroxime axetil, or ceftriaxone is indicated.

»»

Administration of the pneumococcal and influenza vaccines, tobacco smoke avoidance, and increase in breastfeeding reduce the incidence of acute otitis media (AOM).

»»

Complications of AOM are rare but include mastoiditis, temporal bone osteomyelitis, facial nerve palsy, epidural and subdural abscess formation, meningitis, lateral sinus thrombosis, and otitic hydrocephalus.

»»

Patients with failure to thrive who also present with chronic or recurrent OM need a thorough evaluation. An anatomic abnormality or immune deficiency may be a contributing factor.

REFERENCES Azarpazhooh A, Lawrence HP, Shah PS. Xylitol for preventing acute otitis media in children up to 12 years of age. Cochrane Database Syst Rev. 2016;8:CD007095. Chonmaitree T. Infection of the middle ear. In: Kline MW, Blaney SM, Giardino AP, et al, eds. Rudolph’s Pediatrics. 23rd ed. New York, NY: McGraw Hill; 2018:1147-1154. Haddad J, Dodhia SN. External otitis (otitis externa). In: Kleigman RM, St. Geme JW, Blum NJ, et al, eds. Nelson Textbook of Pediatrics. 21st ed. Philadelphia, PA: Elsevier; 2020:3414-3417. Kaur R, Morris M, Pichichero ME. Epidemiology of acute otitis media in the postpneumococcal conjugate vaccine era. Pediatrics. 2017;140(3):e20170101. Kerschner JE, Preciado D. Otitis media. In: Kleigman RM, St. Geme JW, Blum NJ, et al, eds. Nelson Textbook of Pediatrics. 21st ed. Philadelphia, PA: Elsevier; 2020:3418-3431. Kline MW. Mastoiditis. In: McMillan JA, Feigin RD, DeAngelis CD, Jones MD, eds. Oski’s Pediatrics: Principles and Practice. 4th ed. Philadelphia, PA: Lippincott Williams & Wilkins; 2006:1501-1502. Kline MW. Otitis externa. In: McMillan JA, Feigin RD, DeAngelis CD, Jones MD, eds. Oski’s Pediatrics: Principles and Practice. 4th ed. Philadelphia, PA: Lippincott Williams & Wilkins; 2006:1496-1497.

17_Toy-Pediatrics_Case16_p159-168.indd 166

29/07/21 8:13 PM

SECTION II: CLINICAL CASES

167

Lieberthal AS, Carroll AE, Chonmaitree T, et al. Diagnosis and management of acute otitis media. Pediatrics. 2013;131:e964-e999. Liu YC, Greinwald JH. Otitis externa. In: Kline MW, Blaney SM, Giardino AP, et al, eds. Rudolph’s Pediatrics. 23rd ed. New York, NY: McGraw Hill; 2018:1612-1613. Schwarzwald H, Kline MW. Otitis media. In: McMillan JA, Feigin RD, DeAngelis CD, Jones MD, eds. Oski’s Pediatrics: Principles and Practice. 4th ed. Philadelphia, PA: Lippincott Williams & Wilkins; 2006:1497-1500. Siegel RM, Bien J, Lichtenstein P, et al. A safety-net antibiotic prescription for otitis media: the effects of a PBRN study on patients and practitioners. Clin Pediatr (Phila). 2006;45(6):518-524. Sugita G, Hotomi M, Sugita R, et al. Genetic characteristics of Haemophilus influenzae and Streptococcus pneumoniae isolated from children with conjunctivitis-otitis media syndrome. J Infect Chemother. 2014;20(8):493-497. Yoon PJ, Scholes MA, Friedman NR. Ear, nose, and throat. In: Hay WW Jr, Levin MJ, Deterding RR, Abzug MJ, eds. Current Diagnosis & Treatment: Pediatrics. 24th ed. New York, NY: McGraw Hill; 2018.

17_Toy-Pediatrics_Case16_p159-168.indd 167

29/07/21 8:13 PM

This page intentionally left blank

CASE 17 A 16-year-old girl with no significant past medical history presents to the emergency department (ED) with lower, cramping abdominal pain. The pain began 3 days prior and has gradually worsened. It improves with acetaminophen and is worsened by sudden movements, especially coitus. The patient has had two male sexual partners over the past 10 months and sometimes uses condoms. In the past day, she has vomited twice and has developed a thin, white, foulsmelling vaginal discharge. She denies dysuria and diarrhea. On examination, she is ill-appearing with a temperature of 101.3 °F (38.5 °C), and she has moderate tenderness to palpation in the lower abdomen without rebound or guarding. Pelvic examination reveals discharge in the vaginal vault, bilateral adnexal tenderness, and cervical motion tenderness. A urine pregnancy test is negative. Wet mount microscopy of the vaginal discharge shows numerous white blood cells, no trichomonads, and no clue cells. Potassium hydroxide (KOH) applied to the discharge shows no pseudohyphae, and the whiff test is negative. A sample of the discharge is collected and sent for nucleic acid amplification testing (NAAT) for Neisseria gonorrhoeae and Chlamydia trachomatis. ▶▶ ▶▶ ▶▶

What is the most likely diagnosis? What is the best management for this illness? What are the possible complications of this condition?

18_Toy-Pediatrics_Case17_p169-176.indd 169

29/07/21 8:13 PM

170

CASE FILES: PEDIATRICS

ANSWERS TO CASE 17: Pelvic Inflammatory Disease Summary: A 16-year-old girl presents with šš

History of sexual activity with inconsistent condom use

šš

Fever, lower abdominal pain, and mucopurulent cervical discharge

šš

Additional findings of pelvic inflammatory disease (PID) on abdominopelvic exam

šš

Negative testing for Candida vaginitis and bacterial vaginosis

Most likely diagnosis: PID. Best management: Empiric antibiotics should be initiated for a high clinical suspicion for PID. Of note, this patient’s urine pregnancy test was negative; treatment for PID is altered in the pregnant female. If the patient is ill-appearing, febrile, or unable to maintain oral intake, hospitalization for intravenous (IV) antibiotics is indicated. Possible complications: Sepsis, pelvic peritonitis, tubo-ovarian abscess, and perihepatitis (Fitz-Hugh–Curtis syndrome) are short-term complications. Ectopic pregnancy, chronic pelvic pain, and infertility secondary to scarring are possible long-term complications.

ANALYSIS Objectives 1. Understand the clinical presentation and diagnostic criteria of PID (Figure 17–1). (EPA 1, 2) 2. Recognize which patients require different PID treatment regimens. (EPA 4) 3. List possible complications of PID. (EPA 10, 12)

Considerations PID is an infection of the female upper reproductive tract, which consists of the ovaries, fallopian tubes, uterus, and pelvic peritoneum. It is usually caused by an

Female with any sexual activity Lower abdominal pain OR Pelvic pain

PLUS

Cervical motion OR Uterine OR Adnexal tenderness

PLUS

No other identified etiology for symptoms

PID

Figure 17–1.  Clinical diagnosis of pelvic inflammatory disease (PID).

18_Toy-Pediatrics_Case17_p169-176.indd 170

29/07/21 8:13 PM

SECTION II: CLINICAL CASES

171

ascending infection from the vagina or cervix (lower female genital tract). PID is considered a sexually transmitted infection (STI) and most commonly presents in the adolescent pediatric population. The usual causative microorganisms are N. gonorrhoeae and C. trachomatis; however, other organisms that are found in the vaginal flora, such as anaerobes, Gardnerella vaginalis, gram-negative enterics, and group B Streptococcus, may also play a role in PID.

APPROACH TO: Pelvic Inflammatory Disease DEFINITIONS CHANDELIER SIGN: Cervical motion tenderness on pelvic exam. CLUE CELLS: Large vaginal squamous epithelial cells seen on wet mount microscopy of vaginal fluid that appear to have bacteria, usually gram-negative cocci and rods, “stuck” on their surface; indicate bacterial vaginosis. FITZ-HUGH–CURTIS SYNDROME: Complication of PID, also known as perihepatitis, in which inflammation of the liver capsule and adhesion formation occurs; this is due to spread of infection from the upper genital tract into the adjacent peritoneal space and presents as right upper quadrant pain. WHIFF TEST: Fishy odor upon application of KOH to vaginal discharge; this is due to the release of amines from anaerobes and generally indicates bacterial vaginosis.

CLINICAL APPROACH Pathophysiology PID is a bacterial infection that affects the upper genital tract: uterus, fallopian tubes, and/or ovaries. The pathogens typically originate in the endocervix, such as C. trachomatis or N. gonorrhoeae; they ascend the genital tract to the endometrium and then to the tubes.

Clinical Presentation Signs and Symptoms.  The clinical presentation of PID is varied and often consists of mild or nonspecific symptoms. Abdominal or pelvic pain may be described as a discomfort, poorly localized, and intermittent. Fever may not be present. Common accompanying symptoms are vaginal discharge, abnormal vaginal bleeding (nonmenstrual or excess menstrual bleeding), and dyspareunia. PID as the cause of abdominal or pelvic pain is considered in any adolescent female, especially if she discloses sexual activity. A pelvic examination is indicated, along with the abdominal examination, to further identify PID features or findings that would indicate another diagnosis. The most specific sign of PID is bilateral adnexal tenderness, uterine tenderness, or

18_Toy-Pediatrics_Case17_p169-176.indd 171

29/07/21 8:13 PM

172

CASE FILES: PEDIATRICS

cervical motion tenderness (often referred to as the “chandelier sign”). Discharge in the vaginal vault or at the cervical os often indicates vaginitis or cervicitis. Wet mount microscopy of the discharge can be performed to identify other diagnoses, such as bacterial vaginosis, Candida vaginitis, or trichomoniasis. However, the clinician must still note any findings of PID, as these organisms may be present along with PID. Laboratory Results.  A urine pregnancy test should be performed because ectopic pregnancy is a surgical emergency and can present with symptoms similar to PID. Furthermore, PID in a pregnant female requires hospitalization for parenteral therapy. Urinalysis can identify a urinary tract infection and pyelonephritis. Urine or a cervical swab should be sent for chlamydia and gonorrhea NAAT. Treatment should not be withheld to await NAAT results if the clinical suspicion for PID is present; positive results are not always present in PID because additional organisms may be the cause. Additionally, short- and long-term complications increase in untreated PID, regardless of the level of severity of the patient’s initial presentation.

Treatment Current guidelines from the Centers for Disease Control and Prevention (CDC) recommend that sexually active adolescent females presenting with lower abdominal pain and meeting minimum criteria for a clinical diagnosis of PID be empirically treated. As most infections are a result of N. gonorrhoeae and C. trachomatis, empiric treatment against these organisms is the focus of therapy. If patients are able to tolerate oral antibiotic therapy, they can be treated in the outpatient setting with one dose of an intramuscular long-acting cephalosporin (usually ceftriaxone), plus a 2-week course of oral doxycycline. Metronidazole can be added to the treatment regimen for anaerobic therapy. Patients who are pregnant, are ill-appearing on examination, are unable to tolerate oral therapy, or are suffering from complications of untreated or poorly treated PID require hospitalization and parenteral therapy.

Complications Infection may progress to formation of a tubo-ovarian abscess, spread into the peritoneum resulting in peritonitis and adhesion formation, including spread into the liver capsule (Fitz-Hugh–Curtis syndrome), or lead to sepsis. Therefore, every patient with PID must be reevaluated within 72 hours to confirm clinical response to therapy; those who don’t improve should be re-evaluated or hospitalized. The patient should be counseled that her partner will need treatment for chlamydia and gonorrhea. Long-term complications of PID occur due to irreversible scarring of the upper reproductive tract. These complications include increased risk of ectopic pregnancy, chronic pelvic pain, or infertility.

CASE CORRELATION šš

See Case 28 (Bacterial Enteritis).

18_Toy-Pediatrics_Case17_p169-176.indd 172

29/07/21 8:13 PM

SECTION II: CLINICAL CASES

173

COMPREHENSION QUESTIONS 17.1 A previously healthy adolescent girl presents to clinic with a week of lowgrade fevers and mild suprapubic pain. She has had intermittent nausea and vomiting over the past week. Her last menstrual period was 3 weeks ago. She has recently become sexually active with her boyfriend and uses condoms 75% of the time. She denies dysuria and hematuria. Physical examination reveals mild bilateral adnexal tenderness. What is the next best step in management of this patient? A. Obtain a urine pregnancy test B. Initiate oral antibiotic therapy for suspected pelvic inflammatory disease (PID) C. Counsel the patient on safe sex practices and discuss possible contraceptive methods D. Admit the patient for intravenous (IV) antibiotic therapy E. Order nucleic acid amplification testing (NAAT) and await results 17.2 The patient in Question 17.1 presents for follow-up 72 hours later with newonset right upper quadrant (RUQ) pain. She states that her lower abdominal pain is somewhat improved, but she has had fever of 102 °F. On examination, she has severe RUQ tenderness to palpation with mild guarding and no rebound tenderness. What is the most likely etiology of her worsening symptoms? A. Ovarian torsion B. Ectopic pregnancy C. Tubo-ovarian abscess D. Fitz-Hugh–Curtis syndrome E. Appendicitis 17.3 A 17-year-old woman who has no past medical history presents to the clinic with a complaint of vaginal discharge that began 3 days ago and is light green in color. She has also had intermittent lower abdominal pain radiating to the back and 1 day of vaginal bleeding. She denies fever. On further history, she reports that she has been sexually active with her boyfriend for the past 3 months. She reports discomfort of the lower abdomen during palpation and with bimanual exam. Urine pregnancy testing is negative. Urinalysis is negative for pyuria, blood, and nitrite. Gonorrhea and chlamydial testing are pending. What is the best treatment for the patient at this time? A. Ciprofloxacin B. Ceftriaxone C. Metronidazole D. Clindamycin and amoxicillin-clavulanic acid E. Ceftriaxone and doxycycline

18_Toy-Pediatrics_Case17_p169-176.indd 173

29/07/21 8:13 PM

174

CASE FILES: PEDIATRICS

17.4 A 16-year-old girl is diagnosed with PID and requires hospital admission for IV antibiotics. After 24 hours, she is no longer vomiting, and her abdominal pain has improved. The NAAT for chlamydia and gonorrhea is negative. She appears medically ready for discharge from the hospital. Which of the following statements should be reviewed with her? A. She tested negative for sexually transmitted infections (STIs), so she does not have PID. B. Her boyfriend will not require treatment because her tests are negative. C. She must attend a follow-up appointment with her primary care provider to reexamine her and confirm continued response to treatment. D. She does not need to worry about contraception because she cannot have children in the future. E. As long as she takes most of the antibiotics, she will recover.

ANSWERS 17.1 A. Obtain a urine pregnancy test. The patient is a sexually active adolescent female with inconsistent condom use. Although her symptoms indicate PID, pregnancy must be ruled out prior to initiating therapy. The patient has no peritoneal signs upon presentation, and inpatient admission for IV antibiotic therapy (answer D) is unnecessary. After obtaining the pregnancy test, intramuscular ceftriaxone can be administered and the patient can be prescribed oral doxycycline and metronidazole (answer B) to complete outpatient treatment. Counseling (answer C) is very important but not as important as addressing the chief complaint. NAAT (answer E) is also very important, but assessment of possible pregnancy is of primary importance. 17.2 D. Fitz-Hugh–Curtis syndrome. The patient has developed peritoneal signs and severe RUQ tenderness, which are concerning for perihepatitis or FitzHugh–Curtis syndrome. She now requires hospitalization. Tubo-ovarian abscess (answer C) is also a complication of PID, but it does not typically present with RUQ findings. Ovarian torsion (answer A) usually presents as colicky lower abdominal pain rather than RUQ pain. Both ectopic pregnancy (answer B) and appendicitis (answer E) are associated with lower abdominal pain. 17.3 E.  Ceftriaxone and doxycycline. PID requires empiric therapy; the recommended regimen in the outpatient setting for a nonpregnant female is ceftriaxone and doxycycline. The other antibiotic combinations (answer A, ciprofloxacin; answer B, ceftriaxone; answer C, metronidazole; and answer D, clindamycin and amoxicillin-clavulanic acid) are not sufficient for the treatment of acute PID. 17.4 C. She must attend a follow-up appointment with her primary care provider to reexamine her and confirm continued response to treatment. All patients diagnosed with PID are reevaluated in 72 hours to ensure clinical improvement.

18_Toy-Pediatrics_Case17_p169-176.indd 174

29/07/21 8:13 PM

SECTION II: CLINICAL CASES

175

The diagnosis of PID does not require microbiologic confirmation of chlamydia or gonorrhea (answer A). Her boyfriend requires empiric treatment for gonorrhea and chlamydia (answer B) because she has been diagnosed with PID, regardless of her test results. Infertility (answer D) is a potential complication of PID, but when counseling adolescents, preventing unplanned pregnancy and preventing acquisition of genital infections should be emphasized. Complications are more likely to occur in poorly treated PID, so she should be counseled on the necessity of taking all antibiotic doses to complete the treatment (answer E).

CLINICAL PEARLS »»

Sexually active adolescent females presenting with symptoms of pelvic inflammatory disease (PID) should receive empiric therapy; outpatient therapy in the nonpregnant female consists of an intramuscular cephalosporin and tetracycline.

»»

Clinicians should maintain a high clinical suspicion for PID because complications from untreated disease cause serious morbidity.

»»

Any patient diagnosed clinically with PID must be reevaluated in 72 hours, regardless of the outcome of STI testing results.

»»

The clinical presentation of PID is cervical motion tenderness and adnexal tenderness.

»»

Complications of PID include ectopic pregnancy, infertility, and chronic pelvic pain.

REFERENCES Burnstein GR. Sexually transmitted infections. In: Kleigman RM, St. Geme JW, Blum NJ, et al, eds. Nelson Textbook of Pediatrics. 21st ed. Philadelphia, PA: Elsevier; 2020:1081-1091. Centers for Disease Control and Prevention. Pelvic inflammatory disease. https://www.cdc.gov/std/ tg2015/pid.htm. Accessed February 23, 2020. Pelvic inflammatory disease. In: Kimberlin DW, Brady MT, Jackson MA, et al, eds. Red Book: 2018-2021 Report of the Committee on Infectious Disease. 31st ed. Elk Grove Village, IL: American Academy of Pediatrics; 2018:614-619.

18_Toy-Pediatrics_Case17_p169-176.indd 175

29/07/21 8:13 PM

This page intentionally left blank

CASE 18 A 2½-year-old boy comes to your clinic for the first time with a history of fever and increasing “wet” cough for 8 days. His mother reports that he has been diagnosed with asthma and has an albuterol inhaler to use for wheezing or cough. Since 6 months of age, he has had several similar episodes of “wet” cough and fever, which were diagnosed as bronchitis or pneumonia, and each time, he improved when treated with antibiotics and albuterol. However, over the past year, these episodes have become more frequent and the cough now occurs almost daily. Sometimes the mother sees him expectorate the sputum, which is thick and purulent. He has daily nasal congestion, for which she uses saline  and bulb suction. She is able to remove some thick yellow discharge, but the symptoms mainly improve when he is treated with antibiotics. He is not in daycare and has no tobacco exposure. She is concerned that his frequent illnesses are causing him to be “small for his age.” The mother notes his stools are malodorous, and since initiating potty training, she has observed that his stools float and sometimes appear to have drops of oil on them. On physical examination, the patient is a moderately ill-appearing child whose height and weight are at the third percentile for his age. His temperature is 101 °F (38.3 °C), and respiratory rate is 32 breaths per minute and oxygen saturation of 96%. He is breathing with his mouth open. Over the upper lung fields, he has crackles and rhonchi, along with a few expiratory wheezes over all lung fields. He has no heart murmur; S1 and S2 are normal. His fingers show clubbing. You obtain a chest radiograph, which shows linear opacities in a parallel tram-track configuration in the upper lobes with some ring-shaped opacities; the radiologist interprets the findings as bronchiectasis. ▶▶ ▶▶

What is the most likely diagnosis? What is the next step in evaluation?

19_Toy-Pediatrics_Case18_p177-184.indd 177

29/07/21 8:11 PM

178

CASE FILES: PEDIATRICS

ANSWERS TO CASE 18: Cystic Fibrosis Summary: A 2½-year-old boy presents with šš

Recurrent sinopulmonary infections

šš

Daily nasal congestion

šš

Steatorrhea

šš

Bronchiectasis shown on chest x-ray

šš

Failure to thrive possibly due to the recurrent illness and steatorrhea

Most likely diagnosis: Cystic fibrosis (CF). Next step in evaluation: Obtain a sweat chloride test.

ANALYSIS Objectives 1. Know the historical clues and physical signs to distinguish CF from more common conditions. (EPA 1, 2) 2. Know how to accurately diagnose CF. (EPA 1, 3) 3. Have a basic understanding of the complications of CF and its treatment. (EPA 4, 9, 10)

Considerations Several causes for bronchiectasis exist, with asthma and infection being the most common. However, the child’s poor growth and clubbing suggest a more diffuse pulmonary disease or a medical condition in which other organ systems are involved. The symptoms of bronchiectasis typically improve with antibiotics. Rhinosinusitis caused by viral respiratory pathogens should improve within 10 days; otherwise, a bacterial etiology is assumed and treatment with antibiotics is indicated. Recurrent bacterial sinusitis, however, is not typical for a child this age unless an underlying disorder is found.

APPROACH TO: Cystic Fibrosis DEFINITIONS BRONCHIECTASIS: Condition in which a bronchus or bronchi remain dilated after an infection or obstruction.

19_Toy-Pediatrics_Case18_p177-184.indd 178

29/07/21 8:11 PM

SECTION II: CLINICAL CASES

179

CLUBBING: Increase in the angle between the nail and nail base of 180 degrees or greater and softening of the nail base to palpation. Although the condition can be familial, clubbing is uncommon in children, usually indicating chronic pulmonary, hepatic, cardiac, or gastrointestinal disease. CYSTIC FIBROSIS (CF): An autosomal recessive disorder that involves a defect in chloride channels leading to excess loss of sodium chloride and water, with resultant accumulation of thick mucus in the lumina of the respiratory and gastrointestinal tracts.

CLINICAL APPROACH Pathophysiology CF affects all ethnicities, with the highest prevalence in Caucasians. CF shows autosomal recessive inheritance; it is caused by an abnormal CFTR (cystic fibrosis transmembrane conductance regulator) protein. The most common mutation causing the abnormal protein is known as delta F508. The altered CFTR protein allows excess loss of sodium and chloride. The loss from the eccrine glands of the skin causes a hyponatremic, hypochloremic alkalosis. With excess loss of sodium chloride from the respiratory tract, mucus thickens and obstructs airways, leading to bronchiectasis in most CF patients by the age of 5 years. These children will have cough and wheezing that mimic asthma and bronchiolitis. Bacteria then proliferate in the inspissated mucus and damaged respiratory cilia, resulting in pneumonia. Pseudomonas aeruginosa is a particular pathogen that affects older children with CF. Lung function is lost with the destruction from recurrent inflammation, obstruction, and infection. The upper respiratory tract is also involved, and the findings of pansinusitis and nasal polyps, which are rarely seen in healthy children, often signal an underlying disorder such as CF.

Clinical Presentation CF commonly manifests as a gastrointestinal disorder, particularly with loss of pancreatic function. Exocrine function is usually lost, resulting in frequent passage of oily, malodorous, and floating stools, which can eventually lead to malnutrition and failure to thrive. The resulting fat-soluble vitamin deficiencies may manifest as peripheral neuropathy and hemolytic anemia (vitamin E), night blindness (vitamin A), or mucosal bleeding (vitamin K). In 15% to 20% of neonates with CF, meconium ileus occurs in the first 1 to 2 days of life. In this condition, meconium becomes inspissated in the ileum and the infant will not pass stool; abdominal distention and emesis follow. Intestinal perforation can occur without prompt intervention and treatment. A variety of age-related clinical presentations (Table 18–1) prompt an evaluation for CF. The diagnosis is made by an abnormal sweat chloride concentration of 60 mEq/L or greater or identification of two CF-causing mutations along with a family history of CF, a positive newborn screening test, or at least one characteristic phenotypic feature of CF (eg, exocrine pancreatic insufficiency, sweat salt loss syndrome, male infertility, or chronic obstructive pulmonary disease).

19_Toy-Pediatrics_Case18_p177-184.indd 179

29/07/21 8:11 PM

180

CASE FILES: PEDIATRICS

Table 18-1  •  AGE-RELATED CLINICAL PRESENTATIONS OF CYSTIC FIBROSIS Early Childhood

Adolescence/Adulthood

Usually asymptomatic

Sinusitis Nasal polyps Cough Clubbing Bronchiectasis Allergic bronchopulmonary aspergillosis

Nasal polyps Cough Clubbing Bronchiectasis Allergic bronchopulmonary aspergillosis Respiratory failure

Meconium ileus intestinal atresia

Distal intestinal obstruction syndrome Constipation Intussusception Rectal prolapse Gastroesophageal reflux

Distal intestinal obstruction syndrome Constipation Intussusception Rectal prolapse Gastroesophageal reflux Intestinal malignancy

Pancreatic insufficiency

Pancreatic insufficiency Pancreatitis Cystic fibrosis–related diabetes

Cystic fibrosis–related diabetes Pancreatitis Pancreatic insufficiency Pancreatic cancer

Hepatic steatosis Billary fibrosis Billary cirrhosis Portal hypertension

Hepatic steatosis Billary fibrosis Billary cirrhosis Portal hypertension Liver failure Biliary tract cancer

Failure to thrive

Failure to thrive Malnutrition

Failure to thrive

Other

Nutrition

Hepatobiliary Tract

Pancreas

Gastrointestinal Tract

Airway

Infancy

Delayed puberty Arthritis Infertility Osteoporosis

Reproduced with permission, from Kline MW, ed. Rudolph’s Pediatrics, 23rd ed. 2018. Copyright © McGraw Hill LLC. All rights reserved. https://accesspediatrics.mhmedical.com.

Treatment Long-term management of CF patients is best coordinated by experienced pediatric pulmonary specialists. Bronchodilators, inhaled corticosteroids, antibiotics, airway clearance, and inhaled recombinant human DNAse are used to minimize airway reactivity, infections, and secretions. Ivacaftor, a CFTR potentiator, has gained US Food and Drug Administration approval for patients who are age 2 or older with at least one G551D gene mutation. Optimal nutrition is dependent on pancreatic enzyme replacement and vitamin supplements. The prognosis varies depending on

19_Toy-Pediatrics_Case18_p177-184.indd 180

29/07/21 8:11 PM

SECTION II: CLINICAL CASES

181

disease severity, and most patients reach adolescence or adulthood. Mean survival for persons with CF is 40 years; respiratory disease accounts for the majority of deaths.

CASE CORRELATION šš

See also Case 10 (Failure to Thrive), Case 14 (Pneumonia and Tuberculosis), Case 16 (Acute Otitis Media), and Case 20 (Asthma Exacerbation).

COMPREHENSION QUESTIONS 18.1 An 18-month-old girl is seen in the clinic for cough and fever. Her weight is in the third percentile. The mother reports she is concerned her daughter is around “toxic mold” because she has had five to six prior episodes of bronchitis since they moved to a new apartment when the child was 6 months of age. She states that albuterol and an antibiotic have always been given for treatment, with symptoms typically resolving in 2 weeks. A chest radiograph is obtained. What finding on the radiograph would prompt you to perform a sweat chloride test? A. An enlarged cardiac silhouette B. Absent thymus C. Bronchiectasis D. Dextrocardia E. Hilar lymphadenopathy 18.2 A 7-year-old girl is admitted to the hospital in respiratory distress due to pneumonia. This is her third admission in the past 6 months. At this time, you suspect cystic fibrosis (CF) and order a sputum culture. Which organism would be most consistent with a diagnosis of CF? A. Streptococcus pneumoniae B. Mycobacterium tuberculosis C. Pseudomonas aeruginosa D. Bacillus cereus E. Haemophilus influenzae 18.3 A 10-year-old Caucasian boy has a history of recurrent sinusitis and multiple episodes of pneumonia. You suspect CF and order a sweat chloride test. The sweat electrolyte test result is within the normal range. What is your next step in management? A. Perform DNA testing for CFTR gene mutations B. Perform a pH probe test for gastroesophageal reflux C. Refer to a pulmonologist D. Reassure parents that he does not have CF E. Place him on a high-calorie, high-protein diet

19_Toy-Pediatrics_Case18_p177-184.indd 181

29/07/21 8:11 PM

182

CASE FILES: PEDIATRICS

18.4 A 3-month-old infant is admitted to the hospital with her third episode of lobar pneumonia and wheezing. Findings that would increase your suspicions for CF and prompt sweat chloride testing include all of the following EXCEPT: A. Hyponatremia, hypochloremia, and metabolic alkalosis B. Failure to thrive C. Lymphocytosis D. Digital clubbing E. Oily-appearing stools

ANSWERS 18.1 C. Bronchiectasis. Bronchiectasis occurs as a sequela to impaired mucus clearance combined with inflammation and injury to the bronchial walls. CF is the most common noninfectious and chronic cause of this finding; sweat chloride testing is a standard for diagnosis. The other findings (answer A, enlarged cardiac silhouette; answer B, absent thymus; answer D, dextrocardia; and answer E, hilar lymphadenopathy) are not characteristics of CF. 18.2 C. Pseudomonas aeruginosa. The presence of P. aeruginosa on a sputum sample strongly suggests the diagnosis of CF. Patients with CF have a high prevalence of colonization with P. aeruginosa, Staphylococcus aureus, and Burkholderia cepacia. The innate defenses of the airway epithelium cells of CF patients may be compromised, making them unable to fight these organisms. The other organisms listed as answer choices (answer A, Streptococcus pneumoniae; answer B, Mycobacterium tuberculosis; answer D, Bacillus cereus; and answer E, Haemophilus influenzae) are not as specific for CF. 18.3 A. Perform DNA testing for CFTR gene mutations. This child has recurrent upper and lower respiratory tract infections, suggesting CF. The sweat chloride test can yield falsely low values, so the next step would be to perform DNA testing to identify any of the common CFTR mutations. Negative sweat chloride test results do not exclude CF. DNA testing is the next step when there is clinical suspicion of CF and is more appropriate than investigating gastroesophageal reflux (answer B), referring to a pulmonologist (answer C), or reassuring the parents that the patient does not have CR (answer D). 18.4 C.  Lymphocytosis. Infants with CF will lose excess amounts of sodium chloride in their sweat, resulting in a hyponatremic, hypochloremic metabolic alkalosis (answer A). Malabsorption of fats and protein due to pancreatic exocrine insufficiency usually presents as steatorrhea (answer E); malabsorption is a major cause of morbidity for patients with CF and can result in failure to thrive (answer B). Lymphocytosis can occur with viral infections or pertussis but is not a hallmark of CF.

19_Toy-Pediatrics_Case18_p177-184.indd 182

29/07/21 8:11 PM

SECTION II: CLINICAL CASES

183

CLINICAL PEARLS »»

Cystic fibrosis (CF) is an autosomal recessive disorder that involves a defect in chloride channels, leading to excess loss of sodium chloride and water with resultant accumulation of thick mucus in the lumina of the respiratory and gastrointestinal tracts.

»»

Extrapulmonary signs and symptoms, such as digital clubbing, recurrent sinusitis, failure to thrive, fat malabsorption, and a history of meconium ileus are clues to the diagnosis of CF.

»»

A negative sweat chloride test result does not exclude CF.

»»

CF is diagnosed by two positive sweat chloride tests and/or DNA testing detecting two gene mutations known to cause CF.

REFERENCES Egan ME, Schechter MS, Voynow JA. Cystic fibrosis. In: Kleigman RM, St. Geme JW, Blum NJ, et al, eds. Nelson Textbook of Pediatrics. 21st ed. Philadelphia, PA: Elsevier; 2020:2282-2297. Saddi V, Ooi CY, Jaffe A. Cystic fibrosis. In: Kline MW, Blaney SM, Giardino AP, et al, eds. Rudolph’s Pediatrics. 23rd ed. New York, NY: McGraw Hill; 2018:2444-2457.

19_Toy-Pediatrics_Case18_p177-184.indd 183

29/07/21 8:11 PM

This page intentionally left blank

CASE 19 A mother brings her previously healthy 6-year-old son to your clinic because he has been limping and complaining of left leg and knee pain for 1 week. He has no recent trauma, and his past medical history is unremarkable. His physical examination reveals a temperature of 100  °F (37.8  °C) orally with no lower extremity swelling, misalignment, or weakness. He has tenderness over the right knee, hepatosplenomegaly, and petechiae on his cheeks and chest. ▶▶ ▶▶

What is the most likely diagnosis? What should be the next step in evaluation?

20_Toy-Pediatrics_Case19_p185-192.indd 185

29/07/21 8:16 PM

186

CASE FILES: PEDIATRICS

ANSWERS TO CASE 19: Acute Lymphoblastic Leukemia Summary: A 6-year-old boy presents with šš

A 1-week history of leg pain and limping

šš

Low-grade fever

šš

Hepatosplenomegaly

šš

Petechiae on his face and chest

Most likely diagnosis: Acute lymphoblastic leukemia (ALL). Next step in evaluation: Complete blood count (CBC) with platelets and differential.

ANALYSIS Objectives 1. Describe the clinical manifestations of ALL. (EPA 1) 2. Describe the laboratory and radiologic tests used in diagnosing ALL. (EPA 3) 3. Know the treatment plan for a child with newly diagnosed ALL. (EPA 4, 8, 9) 4. Understand the long-term survival and follow-up issues for children with ALL. (EPA 3, 4, 12)

Considerations This patient has several manifestations of ALL, including leg and joint pain, fever, petechiae, and hepatosplenomegaly. Most of the signs and symptoms of ALL result either from replacement of normal bone marrow components with clonal proliferation of a single lymphoblast that has undergone malignant transformation or from infiltrates of extramedullary sites by these malignant lymphoid cells. Rapid diagnosis and referral to a pediatric cancer center can increase survival.

APPROACH TO: Acute Lymphoblastic Leukemia DEFINITIONS EXTRAMEDULLARY: Areas of the body outside the bone marrow. GRANULOCYTOPENIA: A reduction in total circulating leukocytes. LYMPHOBLAST: A large, primitive, undifferentiated precursor cell not normally seen in the peripheral circulation.

20_Toy-Pediatrics_Case19_p185-192.indd 186

29/07/21 8:16 PM

SECTION II: CLINICAL CASES

187

PANCYTOPENIA: A reduction in circulating erythrocytes, leukocytes, and platelets. THROMBOCYTOPENIA: A reduction in circulating platelets.

CLINICAL APPROACH Pathophysiology ALL is the most common childhood cancer, with an incidence of 3.4 cases per 100,000 children, accounting for approximately 25% of all pediatric malignancies. ALL affects the lymphoid cell line and comprises approximately 75% of leukemia cases in children. Acute myeloblastic leukemia (AML) affects the myeloid cell line (granulocytes, monocytes, and possibly erythrocytes or megakaryocytes) and comprises approximately 20% of childhood leukemia. The clinical manifestations of AML and ALL are similar. In the United States, childhood ALL has a peak incidence at age 2 to 5 years and occurs more frequently in boys. Children with certain genetic conditions, such as Down syndrome, Fanconi anemia, Bloom syndrome, Li-Fraumeni syndrome, and ataxia-telangiectasia, have an increased risk of ALL.

Clinical Presentation and Differential Diagnosis Signs and Symptoms.  ALL is often called the “great imitator” because of its nonspecific symptoms, including anorexia, irritability, lethargy, pallor, bleeding, petechiae, leg and joint pain, lymphadenopathy, and fever. A physical examination includes evaluating the child’s general appearance and energy level and vital signs (note if antipyretics taken), as well as assessing for the presence of bleeding, bruising, petechiae, pallor, pain upon palpating bones or joints, and hepatosplenomegaly. Laboratory Values and Imaging.  At the time of diagnosis, children may present with high, normal, or low white blood cell (WBC) counts. It is important to note that almost half of the children with newly diagnosed leukemia have total leukocyte counts less than 10,000 cells/mm3. Leukemic blasts may not be seen in the peripheral blood smear. Therefore, the diagnosis of leukemia is established by examination of bone marrow, most commonly aspirated from the posterior iliac crest. A normal marrow contains less than 5% blasts; a minimum of 25% blasts confirms the diagnosis. Approximately two-thirds of children with ALL have leukemic cell karyotypic abnormalities, including changes in chromosome number (ie, hypodiploidy or hyperdiploidy) or chromosome structure (translocation, deletions, inversions). A lumbar puncture should also be performed to examine the central nervous system (CNS) for early leukemic involvement; blasts in the cerebrospinal fluid are associated with a worse prognosis and therefore may change the treatment regimen. A chest radiograph should be ordered to detect a mediastinal mass. At the time of diagnosis, the disease may be further characterized through flow cytometry, cytochemistry, and cytogenetics. Differential Diagnosis.  The differential diagnosis includes the following diseases: šš

Immune thrombocytopenic purpura is a common cause of bruising and petechiae because of low platelet levels. However, anemia, leukocyte disturbances, and hepatosplenomegaly are absent.

20_Toy-Pediatrics_Case19_p185-192.indd 187

29/07/21 8:16 PM

188

šš

šš

šš

šš

šš

CASE FILES: PEDIATRICS

Aplastic anemia causes pancytopenia and fever. Lymphadenopathy, arthralgias, bone pain, and hepatosplenomegaly are unusual findings. Children with infectious mononucleosis (caused by Epstein-Barr virus [EBV]) or other acute viral illnesses may present with fever, malaise, adenopathy, splenomegaly, and lymphocytosis. Atypical lymphocytes resembling leukemic lymphoblasts are characteristic of these viral illnesses. Leukemoid reactions may be observed in bacterial sepsis, pertussis, acute hemolysis, granulomatous disease, and vasculitis. The leukemoid reaction resolves as the underlying disease is treated. Children with ALL who present with fever, arthralgias, arthritis, or a limp frequently are diagnosed initially with juvenile idiopathic arthritis ( JIA). Anemia, leukocytosis, and mild splenomegaly may also be seen in JIA, causing even more confusion. A bone marrow examination may be required to differentiate ALL from other diagnoses. Children with solid tumors, such as neuroblastoma, rhabdomyosarcoma, Ewing sarcoma, or retinoblastoma, can occasionally present with pancytopenia due to infiltration of the marrow by malignant cells. These tumor cells usually are found in clumps in the normal marrow but occasionally replace the marrow completely.

Treatment Risk Stratification.  Pediatric ALL patients are classified as standard or high risk at the time of diagnosis. A variety of markers can help gauge prognosis. Children 1 to 9 years of age with WBC counts less than 50,000 cells/mm3 and without adverse cytogenetic features are considered standard risk. Children older than 10 years or those with higher WBC counts are considered higher risk and have an overall worse prognosis. Infants also have a worse prognosis, and up to 60% of infants with ALL have the t(4;11) translocation, which is correlated with a poor outcome. Another translocation with a poor outcome is t(9;22) (Philadelphia chromosome) in patients with pre–B-cell ALL. The classification of patients into standard- or high-risk ALL is important because it determines the intensity of the initial chemotherapy (induction chemotherapy). If a patient has high-risk ALL, a more aggressive induction chemotherapy regimen is initiated. Chemotherapy.  Combination chemotherapy is the principal therapy. The therapy involves several phases: induction, consolidation, intensification, and maintenance. šš

šš

Induction therapy, which typically is composed of prednisone, vincristine, and L-asparaginase, produces remission within 4 weeks in approximately 98% of children with standard-risk ALL. Intrathecal therapy (with or without craniospinal irradiation) has decreased the incidence of CNS leukemia as a primary site of relapse from 50% to approximately 3% to 6%. Consolidation and intensification treatment, aimed at further reducing residual leukemia, delivers multiple chemotherapies in a relatively short period of time.

20_Toy-Pediatrics_Case19_p185-192.indd 188

29/07/21 8:16 PM

SECTION II: CLINICAL CASES

šš

189

Maintenance therapy with methotrexate and 6-mercaptopurine, vincristine, and prednisone is given for 2 to 3 years to prevent relapse; therapy is discontinued for children who remain in complete remission for 2 to 3 years.

Prognosis The 5-year survival rate for childhood ALL has steadily improved over the past 40  years and now is 90%. Treatment response is key for determining the overall outcome. After induction chemotherapy, patients are evaluated for the presence of minimal residual disease (MRD). If a patient has more than 0.01% of leukemic cells in the bone marrow, their MRD status is considered to be positive, and therefore, they are considered to be at a high risk of relapse. Depending on the response to therapy, allogeneic hematopoietic stem cell transplantation may be recommended after the first complete remission. In general, girls have a better prognosis. African American and Hispanic populations historically have lower remission and higher relapse rates, although newer studies suggest this might be due to factors other than race. The karyotypes of leukemic cells have diagnostic, prognostic, and therapeutic significance. Patients with hyperdiploidy generally have a more favorable prognosis; those with hypodiploidy and pseudodiploidy do less well. With improving cancer survival rates, childhood cancer survivors require unique considerations in their long-term care. Patients require live vaccines missed during their treatment, and they may need revaccination for previously received vaccines. Late effects of cancer treatment include myriad issues, such as cardiotoxicity, infertility, peripheral neuropathy, neurodevelopmental problems, seizures, endocrine disturbances, dental problems, chronic kidney disease, and secondary malignancies. Patients require long-term follow-up to monitor for relapse and late effects of cancer treatment.

CASE CORRELATION šš

See Case 10 (Failure to Thrive) and Case 13 (Sickle Cell Disease).

COMPREHENSION QUESTIONS 19.1 A mother brings her 3-year-old son with Down syndrome to the clinic because his gums have been bleeding for 1 week. She reports that he has been less energetic than usual. Examination reveals that the child has an oral temperature of 100 °F (37.8 °C), pallor, splenomegaly, gingival bleeding, and bruises on the lower extremities. Which of the following is most likely? A. Aplastic anemia B. Immune thrombocytopenic purpura (ITP) C. Leukemia D. Leukemoid reaction E. Megaloblastic anemia

20_Toy-Pediatrics_Case19_p185-192.indd 189

29/07/21 8:16 PM

190

CASE FILES: PEDIATRICS

19.2 A father brings to the clinic his 6-year-old son who currently is undergoing induction chemotherapy for ALL. The school will not allow the child to register until his immunizations are up to date. Which of the following is the best course of action? A. Call the school nurse or principal to inform him or her that this child should not receive immunizations while he is taking chemotherapy. B. Update all immunizations except for measles, mumps, and rubella (MMR) and varicella. C. Update all immunizations except for oral polio vaccine. D. Update all immunizations. E. Call the school nurse or principal to inform him or her that this child will never receive immunizations because of the alteration in his immune system. 19.3 A mother arrives at the clinic with her 4-year-old daughter, who began complaining of right knee pain 2 weeks ago. The patient is limping slightly, is fatigued, and has had a fever of 100.4 °F (38 °C) over the past week. Which of the following laboratory tests is most important? A. Antinuclear antibodies B. Complete blood count (CBC) with differential and platelets C. Epstein-Barr virus (EBV) titer D. Rheumatoid factor E. Sedimentation rate 19.4 Two weeks after a viral syndrome, a 2-year-old boy develops bruising and generalized petechiae that is more prominent over the legs. He has neither hepatosplenomegaly nor lymph node enlargement. Laboratory testing reveals a normal hemoglobin, hematocrit, and WBC count and differential. The platelet count is 15,000/mm3. Which of the following is the most likely diagnosis? A. Acute lymphoblastic leukemia (ALL) B. Aplastic anemia C. ITP D. Thrombotic thrombocytopenic purpura (TTP) E. von Willebrand disease

ANSWERS 19.1 C. Leukemia. A high susceptibility to leukemia is associated with certain heritable diseases (eg, Bloom syndrome, Fanconi syndrome, ataxia-telangiectasia, neurofibromatosis) and chromosomal disorders such as Down syndrome and Klinefelter syndrome. Children with Down syndrome have a 10- to 15-fold

20_Toy-Pediatrics_Case19_p185-192.indd 190

29/07/21 8:16 PM

SECTION II: CLINICAL CASES

191

increased risk for developing leukemia and should have routine screening performed at well-child checks. Siblings of an ALL patient have a two- to four-fold increased risk for ALL, and the monozygotic twin of a child who develops ALL in the first year of life has a more than 70% chance of also developing ALL. A few cases of ALL are associated with P53 gene aberrations. Overall, these genetic links account for a small number of total ALL cases. ALL is more likely than the other answer choices (answer A, aplastic anemia; answer B, ITP; answer D, leukemoid reaction; and answer E, megaloblastic anemia). 19.2 A. Call the school nurse or principal to inform him or her that this child should not receive immunizations while he is taking chemotherapy. Live virus vaccines are contraindicated for the child with ALL (and all members of the household) during chemotherapy and for at least 6 months after completion of treatment. Although the viruses in the vaccine are attenuated, immunosuppression from treatment can be profound, and viral disease can result. Immunizations without live virus (diphtheria, tetanus, inactivated poliovirus vaccine, hepatitis A and B) are not absolutely contraindicated in this case, but the immunosuppression with chemotherapy often inhibits adequate antibody responses. 19.3 B. CBC with differential and platelets. This child has symptoms consistent with both juvenile idiopathic arthritis ( JIA) and leukemia. The CBC with differential and platelets is the best initial screening test. The leukocyte and platelet counts are normal to increased in JIA, and no blast cells are present. Frequently, blast cells are found on the peripheral smear with ALL. The child in the question ultimately may require a bone marrow aspiration. Leukemia should first be ruled out prior to assessing for rheumatologic conditions (answer A, antinuclear antibodies; answer D, rheumatoid factor; and answer E, sedimentation rate) or viral illness (answer C, EBV titer). 19.4 C. ITP. ITP is common in children. In most cases, a preceding viral infection can be documented. The platelet count frequently is less than 20,000/ mm3, but other laboratory test results are normal. Bone marrow aspiration may be normal or may show an increase in megakaryocytes, although this usually is not required to make the diagnosis. Treatment consists of observation or possibly intravenous immunoglobulin (IVIG), intravenous anti-D (in Rh-positive patients), immunosuppressives, or steroids. The history must be reviewed for other possible causes of thrombocytopenia, including recent MMR vaccination, drug ingestion, and human immunodeficiency virus (HIV). ALL (answer A) is associated with bone marrow suppression of multiple cell lines and not only the platelet count. Aplastic anemia (answer B) is associated with only low red blood cell counts. TTP (answer D) presents as the pentad of fever, neurological dysfunction, renal insufficiency, thrombocytopenia, and microangiopathic hemolytic anemia. Von Willebrand disease (answer E) is associated with a normal platelet count and a qualitative problem with platelet adhesion.

20_Toy-Pediatrics_Case19_p185-192.indd 191

29/07/21 8:16 PM

192

CASE FILES: PEDIATRICS

CLINICAL PEARLS »»

Leukemias are the most common childhood cancers, and acute lymphoblastic leukemia (ALL) represents approximately 75% of all leukemia cases in children.

»»

ALL has a peak incidence at the age of 2 to 5 years, and boys are affected more frequently.

»»

ALL is often called the “great imitator” because of its nonspecific symptoms of anorexia, irritability, lethargy, pallor, bleeding, petechiae, leg and joint pain, and fever.

»»

Combination chemotherapy is the principal therapy for childhood ALL. Induction therapy produces remission within 4 weeks in approximately 98% of children with average-risk ALL.

»»

Any chronic condition (such as malignancy) may result in failure to thrive.

»»

The hematologic finding of sickle cell disease typically is isolated to the red blood cell, whereas that of leukemia affects all cell lines. Both may present with pallor and bone pain.

REFERENCES Borowitz MJ, Devidas M, Hunger SP, et al. Clinical significance of minimal residual disease in childhood acute lymphoblastic leukemia and its relationship to other prognostic factors: a Children’s Oncology Group study. Blood. 2008;111:5477-5485. de Guzman MM, Wallace CA, Cabral, DA, et al. Juvenile idiopathic arthritis. In: Kline MW, Blaney SM, Giardino AP, et al, eds. Rudolph’s Pediatrics. 23rd ed. New York, NY: McGraw Hill; 2018:927-933. Hunger SP, Teachey DT, Grupp S, et al. Childhood leukemia. In: Niederhuber JE, Armitage JO, Kastan MB, et al, eds. Abeloff ’s Clinical Oncology. 6th ed. Philadelphia, PA: Elsevier; 2020:1748-1764. Rytting M, Choroszy M, Petropoulous D, Chan K. Acute leukemia. In: Chan K, Raney R, eds. MD Anderson Cancer Care Series Pediatric Oncology. New York, NY: Springer; 2005:1-17. Sartain SE, Despotovic JM. Immune thrombocytopenia. In: Kline MW, Blaney SM, Giardino AP, et al, eds. Rudolph’s Pediatrics. 23rd ed. New York, NY: McGraw Hill; 2018:1961-1964. Tasian SK, Raetz EA. Acute lymphoblastic leukemia. In: Kline MW, Blaney SM, Giardino AP, et al, eds. Rudolph’s Pediatrics. 23rd ed. New York, NY: McGraw Hill; 2018:2018-2025. Tubergen DT, Bleyer A, Ritchey AK, Friehling E. The leukemias. In: Kleigman RM, St. Geme JW, Blum NJ, et al, eds. Nelson Textbook of Pediatrics. 21st ed. Philadelphia, PA: Elsevier; 2020:2649-2656.

20_Toy-Pediatrics_Case19_p185-192.indd 192

29/07/21 8:16 PM

CASE 20 A 10-year-old boy in respiratory distress arrives to the emergency department (ED) with his mother. She reports that the patient developed nasal congestion and a sore throat 24 hours prior and then a cough a few hours previously. Over the past 2 hours, he has complained of chest pain and has been breathing rapidly. His mother administered a unit dose of albuterol via the nebulizer and then a second dose 5 minutes later. He showed no improvement. She reports that he has had two similar episodes in the past year. Your examination reveals an afebrile boy with a respiratory rate of 40 breaths per minute, oxygen saturation of 88%, and a heart rate of 130 beats per minute. You note that his radial pulse becomes weak in amplitude with inspiration. His blood pressure is normal, but his capillary refill is sluggish at 4 to 6 seconds. He appears drowsy and is using accessory chest muscles to breathe. You hear faint inspiratory wheezes and no breath sounds during expiration. ▶▶ ▶▶ ▶▶

What is the most likely diagnosis? What is the next step in management? What other medical history should be obtained?

21_Toy-Pediatrics_Case20_p193-202.indd 193

29/07/21 8:22 PM

194

CASE FILES: PEDIATRICS

ANSWERS TO CASE 20: Asthma Exacerbation Summary: A 10-year-old boy presents with šš

Tachypnea, pulsus paradoxus, and use of accessory muscles of breathing

šš

Inspiratory wheezing and absent expiratory breath sounds

šš

Delayed capillary refill and drowsiness

šš

Oxygen saturation of 88%

šš

History of two similar episodes in the past year

Most likely diagnosis: Asthma exacerbation. Next step in management: Treating this patient’s respiratory distress is of immediate concern. Initial management includes administration of oxygen, an inhaled short-acting beta-agonist (SABA), and a systemic dose of prednisone. Intravenous (IV) administration of fluids and medications is indicated for a patient with this degree of distress. Medical history: After or simultaneously with initial stabilization, information on his asthma medications, triggers, and frequency and severity of previous exacerbations, especially how many exacerbations have needed hospitalization or intensive care unit admission, should be obtained.

ANALYSIS Objectives 1. Know how to diagnose asthma. (EPA 1-3) 2. Know the acute management of an asthma exacerbation. (EPA 4, 10) 3. Know how to classify asthma severity and the management of each level. (EPA 4, 10)

Considerations This child’s history of ED visits for respiratory difficulty, along with his acute symptoms, point to asthma as the most likely diagnosis. Less likely conditions include anaphylaxis, cystic fibrosis, foreign-body aspiration, and heart failure. The National Heart, Lung, and Blood Institute (NHLBI) asthma guidelines suggest that this child’s exacerbation is severe and requires immediate, intensive treatment. His drowsiness is of particular concern, indicating impending respiratory failure; his respiratory and circulatory status must be assessed frequently. The paucity of wheezes results from severe airway obstruction and reduced air movement; thus, he has no breath sounds during expiration. Wheezing is likely to increase when therapy allows more air movement, indicating symptomatic improvement.

21_Toy-Pediatrics_Case20_p193-202.indd 194

29/07/21 8:22 PM

SECTION II: CLINICAL CASES

195

APPROACH TO: Asthma Exacerbation DEFINITIONS ASTHMA: The diagnosis when: (1) episodic symptoms of airflow obstruction are present; (2) airflow obstruction is at least partially reversible; and (3) alternative diagnoses are excluded. ASTHMA EXACERBATION: Characterized by the triad of acute, progressively worsening bronchoconstriction, airway inflammation, and mucus plugging. PULSUS PARADOXUS: A fall in systolic blood pressure that varies more widely than normal between inspiration and expiration; the amplitude of the pulse decreases or disappears when airway obstruction is severe. A variance of greater than 10 mm Hg between inspiration and expiration suggests severe obstructive airway disease, pericardial tamponade, or constrictive pericarditis. SPIROMETRY: A test of pulmonary function that can generally be performed in children aged 5 years or older. For patients with asthma, this test demonstrates airflow obstruction and reversibility and can be used to determine an individual’s response to treatment. The most common measure demonstrating airflow obstruction is forced expiratory volume in 1 second (FEV1).

CLINICAL APPROACH Epidemiology The prevalence of asthma in children in the United States is 6.2 million, and the median age at onset is 4 years. Atopy and a family history of asthma are strong risk factors for its development, as is respiratory infection early in life; between 40% and 50% of children with respiratory syncytial virus (RSV) bronchiolitis later develop asthma. More than half of children with asthma have symptom resolution by young adulthood. Heavy exposure to pollution, allergens, or cigarette smoke makes resolution less likely.

Pathophysiology Airway inflammation in asthma is caused by mast cell activation. An immediate immunoglobulin (Ig) E response to environmental triggers occurs within several minutes and includes vasodilation, increased vascular permeability, smooth-muscle constriction, and mucus secretion. Symptoms result from these changes and may include wheezing, cough (especially worse at night), difficulty breathing, or chest tightness. These symptoms are triggered by dust mites, animal dander, cigarette smoke, pollution, weather changes, pollen, upper respiratory infections, or exercise (particularly when performed in a cold environment). Several hours after this acute response, a late-phase reaction (LPR) begins. The LPR is characterized by infiltration of inflammatory cells into the airway parenchyma; it is responsible for the chronic inflammation seen in asthma and can lead

21_Toy-Pediatrics_Case20_p193-202.indd 195

29/07/21 8:22 PM

196

CASE FILES: PEDIATRICS

Figure 20–1.  Acute asthma exacerbation with hyperexpansion of the lungs and flattened diaphragms on the chest X-ray. (Reproduced with permission, from Susan D. John, MD).

to recurrence of bronchoconstriction through a mechanism more complex than smooth-muscle constriction. Airway hyperresponsiveness and the accompanying symptoms may persist for weeks after the LPR.

Clinical Presentation Physical examination findings that suggest asthma are wheezing, hyperexpansion of the thorax, or a prolonged phase of forced exhalation. The presence of other atopic conditions such as allergic rhinitis, nasal polyps, or atopic dermatitis may be supportive of the diagnosis. Spirometry should be performed, if possible, whenever a diagnosis of asthma is considered. A chest radiograph is not a required study but can help exclude other diagnoses, such as heart failure or, in toddlers, foreign-body aspiration. Chest x-ray usually shows no infiltrates but may reveal hyper-inflated lungs and flattened diaphragms (see Figure 20-1). Nonspecific findings of hyperinflation, flattened diaphragms, or increased bronchial wall markings may be the only abnormalities seen with asthma. Other investigations, such as sweat chloride testing, may be needed to exclude other obstructive diseases of the small airways, such as cystic fibrosis. Viral bronchiolitis is the most commonly occurring disease of the small airways and usually does not respond to conventional asthma therapy.

Treatment Classification.  Asthma management involves classifying the baseline disease severity and identifying and minimizing exposure to triggers. Classification is made based on spirometry and the patient’s symptoms over the prior 2 to 4 weeks. The features that are assessed are the frequency of nighttime symptoms, how often the rescue medication is needed, and how much symptoms limit daily activities (Table 20–1). Severity is defined as either intermittent or persistent; persistent asthma is further divided into mild, moderate, or severe. Exacerbations of asthma can occur with any level of severity. Pharmacotherapy for the child’s asthma symptoms

21_Toy-Pediatrics_Case20_p193-202.indd 196

29/07/21 8:22 PM

197

SECTION II: CLINICAL CASES

Table 20–1  •  CLASSIFICATION OF ASTHMA SEVERITY AND ITS TREATMENT IN SCHOOL-AGE CHILDREN Classification of Asthma Severity (5-11 years of age) Components of Severity

Persistent Intermittent

Mild

Moderate

Severe

Daytime symptoms

Less than or equal to 2 d/wk

Greater than 2 d/wk (but not daily)

Daily

Throughout the day

Nighttime awakenings

Less than or equal to 2×/ month

3-4×/mo

Greater than 1×/wk (but not nightly)

Nightly

Short-acting beta2-agonist use for symptom control (excludes using it for prevention of EIB)

Less than or equal to 2 d/wk

Greater than 2 d/wk (but not daily)

Daily

Several times per day

Interference with activity

None

Minor limitation

Some limitation

Extremely limited

Lung function

Normal FEV1 between exacerbations

FEV1 greater than 80% of predicted

FEV1 equal to 60%-80% of predicted

FEV1 less than 60% of predicted

FEV1 greater than 80% predicted

FEV1/FVC greater than 80%

FEV1/FVC equal to 75%-80%

FEV1/FVC less than 75%

FEV1/FVC greater than 85% Number of exacerbations requiring systemic steroids

0-1/y

Greater than or equal to 2/y

Therapy

Short-acting beta-agonist (SABA) prn

Add low-dose ICS

Low-dose ICS + montelukast or medium-dose ICS

Refer to pulmonologist Mediumdose ICS + long-acting beta-agonist

Abbreviations: d, day; EIB, exercise-induced bronchoconstriction; FEV1, forced expiratory volume in 1 second; FCV, forced vital capacity; ICS, inhaled corticosteroid; mo, month; prn, as needed; wk, week; y, year.

follows American Academy of Pediatrics guidelines (https://www.aap.org/en-us/ Documents/medicalhome_resources_keypointsforasthma.pdf ). Pharmacotherapy.  Pharmacotherapy for asthma includes quick-relief medications for the acute symptoms and exacerbations, as well as long-term controller medications. SABAs (ie, albuterol, levalbuterol) rapidly reverse bronchoconstriction via beta-2-receptors on bronchial smooth muscle cells; they do not significantly inhibit the LPR. These agents also can be used immediately prior to exercise or

21_Toy-Pediatrics_Case20_p193-202.indd 197

29/07/21 8:22 PM

198

CASE FILES: PEDIATRICS

exposure to allergens to minimize the acute asthmatic response. Common side effects include tachycardia and muscle tremor. When these medications are delivered through inhalation routes (nebulizer or inhaler), increased levels of drug are delivered to the lungs, and toxicity is decreased. When inhalers are used, a reservoir device (“spacer”) is used to maximize the amount of medication delivered to the lungs. Patients must not overrely on short-acting inhalers because this practice is associated with death in severe asthma attacks. Anticholinergics (ie, ipratropium) may be useful in the acute management of asthma exacerbation but are of little value in chronic therapy; they work by inhibiting the vagal reflex of smooth muscles and give additive benefit to SABAs. The most potent anti-inflammatory drugs are corticosteroids, which are useful for acute exacerbations (oral or IV dexamethasone, prednisone, or prednisolone) and for chronic therapy (inhaled corticosteroids). They block the LPR and reduce hyperresponsive airways. The inhaled route is best for long-term or chronic therapy so that adverse effects on bone mineral density, growth, and immune function are minimized, while maximal amounts of the drug can be delivered to the lungs. Other long-term controller medications include mast cell stabilizers (cromolyn) and leukotriene receptor inhibitors (montelukast), which act by reducing the immune response to allergen exposure. They become effective after 2 to 4 weeks of therapy. For patients in whom IgE-mediated asthma has been diagnosed, specific anti-IgE medications are available and are administered by a subspecialist expert in the care of pediatric asthma.

CASE CORRELATION šš

See also Case 7 (Esophageal Atresia), Case 10 (Failure to Thrive), Case 14 (Pneumonia and Tuberculosis), and Case 18 (Cystic Fibrosis).

COMPREHENSION QUESTIONS 20.1 A 12-year-old asthmatic girl presents to the emergency department (ED) with tachypnea, intercostal retractions, perioral cyanosis, and minimal wheezing. Oxygen, inhaled albuterol, and IV prednisone are provided. Upon reassessment, wheezing increases in all fields, and the child’s color has improved. Which of the following is the most likely explanation for these findings? A. The girl is not having an asthma attack. B. The girl is not responding to the albuterol, and her symptoms are worsening. C. The girl is responding to the albuterol, and her symptoms are improving. D. The girl did not receive enough albuterol. E. The albuterol was inadvertently left out of the inhalation treatment, and the girl received only saline.

21_Toy-Pediatrics_Case20_p193-202.indd 198

29/07/21 8:22 PM

SECTION II: CLINICAL CASES

199

20.2 Which of the following is not an essential medication for acute asthma exacerbations? A. Montelukast B. Albuterol C. Prednisone D. Ipratropium E. Levalbuterol 20.3 A 6-year-old girl comes to clinic with her mother for a refill of her albuterol inhaler. She has been using the albuterol 4 days per week, and about once per week her mother has heard her coughing at night, which requires inhaler use. The girl reports she gets tired more quickly at recess compared to her friends. What information do you need to tell the mother? A. A chest radiograph is needed B. The dose of albuterol needs to be increased C. A long-acting beta-adrenergic agonist is needed D. A change in her medication to levalbuterol is needed E. Her child has mild persistent asthma and needs a daily controller medication 20.4 A 15-year-old boy with asthma uses his albuterol inhaler shortly after he mows the lawn because of a mild feeling of chest “tightness.” He later returns home early from dinner at a friend’s house when he has a sudden onset of wheezing, cough, and chest pain. Which of the following is the most likely explanation for these circumstances? A. He likely aspirated a piece of grass B. His albuterol inhaler must be empty C. His albuterol inhaler must be outdated D. He is having a late-phase reaction (LPR) E. He has been exposed to a new allergen that is more irritating than grass

ANSWERS 20.1 C. The girl is responding to the albuterol, and her symptoms are improving. This child presented in severe respiratory distress. Her improved color indicates reversible symptoms, confirming the diagnosis of asthma (answer A). Increased wheezing is auscultated after albuterol treatment because lung areas previously obstructed are now opening, allowing additional airflow. Less experienced examiners may misinterpret the initial lack of air movement as “clear” breath sounds, further delaying appropriate medical management. Thus, she is improving and not worsening (answer B), responding to the treatment including albuterol (answers D and E).

21_Toy-Pediatrics_Case20_p193-202.indd 199

29/07/21 8:22 PM

200

CASE FILES: PEDIATRICS

20.2 A. Montelukast. Montelukast is a leukotriene receptor antagonist that is often used as part of the treatment for asthma associated with allergic rhinitis; it takes weeks to demonstrate effectiveness. The other medications (answer B, albuterol; answer C, prednisone; answer D, ipratropium; and answer E, levalbuterol) are used in acute asthma exacerbations. 20.3 E. Her child has mild persistent asthma and needs a daily controller medication. It is essential to define the severity of every patient’s asthma to determine the appropriate management. This patient’s frequency of daily inhaler use, nighttime symptoms, and level of limitation from symptoms classify her as having persistent asthma; a controller medication (low-dose inhaled corticosteroid) is indicated. Changing the dose (answer B) or type (answers  C and D) of beta-adrenergic agonist will not improve her symptoms and is not appropriate management for her severity of asthma. Chest radiographs (answer A) are not used to define asthma severity and would not be indicated in this scenario. 20.4 D. He is having an LPR. An LPR typically occurs 2 to 4 hours after an initial wheezing episode. It is caused by accumulation of inflammatory and immune cells including eosinophils, basophils, neutrophils, and helper T cells in the airway. This is a much more plausible explanation than the inhaler being empty (answer B) or outdated (answer C). With aspiration (answer A), the symptoms would be immediate.

CLINICAL PEARLS »»

Asthma is characterized by episodic airflow obstruction that is at least partially reversible and may manifest as cough, dyspnea with exertion, chest pain, or wheezing.

»»

Acute asthma symptoms are managed with short-acting beta-adrenergic agonists that rapidly reverse the bronchoconstriction.

»»

The late-phase reaction begins 2 to 4 hours after allergen exposure and is responsible for the chronic inflammation seen in asthma.

»»

Acute and long-term management of asthma is guided by the level of severity and the characteristics of any accompanying exacerbations.

»»

Differentiating asthma from pneumonia requires a thorough personal and family history, careful physical examination, and selected testing such as chest radiographs. In some cases, viral pneumonia may be a trigger for asthma.

»»

Difficult-to-control or atypical presentations of asthma, especially in a child with failure to thrive, should prompt a consideration of tracheoesophageal atresia or cystic fibrosis.

21_Toy-Pediatrics_Case20_p193-202.indd 200

29/07/21 8:22 PM

SECTION II: CLINICAL CASES

201

REFERENCES American Academy of Pediatrics. Key points for asthma guideline implementation. https://www.aap. org/en-us/Documents/medicalhome_resources_keypointsforasthma.pdf. Accessed on February 23, 2020. Centers for Disease Control and Prevention. Asthma’s impact on the nation. https://www.cdc.gov/ asthma/nhis/2017/table3-1.htm. Accessed on February 23, 2020. Hershey GKK. Childhood asthma. In: Kline MW, Blaney SM, Giardino AP, et al, eds. Rudolph’s Pediatrics. 23rd ed. New York, NY: McGraw Hill; 2018:2425-2440. Liu AH, Covar RA, Spahn JD, Sicherer SH. Childhood asthma. In: Kleigman RM, St. Geme JW, Blum NJ, et al, eds. Nelson Textbook of Pediatrics. 21st ed. Philadelphia, PA: Elsevier; 2020:1186-1209. National Heart, Lung, and Blood Institute. National Asthma Education and Prevention Program, Expert Panel Report 3: guidelines for the diagnosis and management of asthma, 2007. http://www .nhlbi.nih.gov/guidelines/asthma/asthsumm.pdf. Accessed on February 23, 2020.

21_Toy-Pediatrics_Case20_p193-202.indd 201

29/07/21 8:22 PM

This page intentionally left blank

CASE 21 A 3-month-old boy is discovered not breathing in his crib this morning. Cardiopulmonary resuscitation (CPR) is begun by the parents and is continued by paramedics en route to the hospital. Attempts to try to revive the child in the emergency department (ED) begin, but he is pronounced dead after 60 minutes of resuscitation. You review the history with the family and examine the child, but you are unable to detect a cause of death. ▶▶ ▶▶ ▶▶

How should you manage this situation in the ED? What is the most likely diagnosis? What is the next step?

22_Toy-Pediatrics_Case21_p203-210.indd 203

29/07/21 1:06 PM

204

CASE FILES: PEDIATRICS

ANSWERS TO CASE 21: Sudden Infant Death Syndrome Summary: A 3-month-old boy presents with šš

Absent breathing discovered by his parents this morning

šš

Pronouncement of death after extensive CPR efforts

šš

No detectable cause of death

Management in the ED: Tell the boy’s parents that despite everyone’s best efforts, their son has died. Ask the parents if they would like you to call a friend, family member, religious leader, or other support person. Provide them with a quiet room where they can be left alone. Most likely diagnosis: Sudden infant death syndrome (SIDS) is the most likely diagnosis, assuming that the parents’ story is true. Infanticide must be considered, as well as the possibility of an underlying congenital or metabolic disorder. Next step: Discuss with the parents that a routine protocol is followed after an unexplained infant death. A coroner will perform an autopsy, and police investigators will examine the parents’ home for clues related to the death. Emphasize that these measures can help to bring closure for the family and may yield important information for preventing future child deaths should the couple have more children.

ANALYSIS Objectives 1. Verbalize the definition of SIDS. (EPA 12) 2. Describe the factors that are associated with SIDS. (EPA 1, 2) 3. Apply the counseling principles in clinical scenarios involving parents about SIDS risk–reducing measures. (EPA 12)

Considerations SIDS is one of the most tragic and frustrating medical diagnoses. When the family is in the ED, other possible causes of death (eg, child abuse or inherited disorders) cannot be excluded. The health care team’s role is to remain objective about these other possibilities yet sympathetic to the parents’ grieving. Meticulous documentation of the history and physical examination findings is imperative.

22_Toy-Pediatrics_Case21_p203-210.indd 204

29/07/21 1:06 PM

SECTION II: CLINICAL CASES

205

APPROACH TO: Sudden Infant Death Syndrome DEFINITIONS APNEA: Cessation of breathing for at least 20 seconds that may be accompanied by bradycardia or cyanosis. BRIEF RESOLVED UNEXPLAINED EVENT (BRUE): An abrupt and brief episode of change in an infant’s breathing, tone, color, or mental status that is observed by a caregiver. The episode lasts less than 1 minute, and the infant must return to baseline afterward. A thorough history and physical examination will not identify an alternate explanation for the symptoms. SUDDEN INFANT DEATH SYNDROME (SIDS): The sudden death of an infant that cannot be explained by results of a postmortem examination, death scene investigation, or historical information.

CLINICAL APPROACH Epidemiology SIDS is the most common cause of death in infants between the ages of 1 week and 1 year. The majority of SIDS deaths occur before 6 months of age. No cause of SIDS has been identified. Epidemiologic studies suggest that the following are independent SIDS risk factors: prone or side sleep position, sleeping on a soft surface, bed sharing, pre- and postnatal exposure to tobacco smoke, parental illicit drugs or alcohol, overheating, late or no prenatal care, and prematurity or low birth weight. The incidence of SIDS has decreased dramatically in areas with public education campaigns targeted at limiting prone sleep positioning. Breastfeeding, immunizations, and pacifier use seem to reduce the risk of SIDS; to promote successful breastfeeding, the introduction of a pacifier is not recommended until the baby is 3 to 4 weeks of age. Home monitors and products marketed to reduce SIDS have no role in protection.

Investigation of Unexpected Infant Death The investigation of the unexpected infant death includes a clinical history, a postmortem examination, and a death scene investigation. In some infants, autopsy reveals mild pulmonary edema and scattered intrathoracic petechiae; these findings are supportive but not diagnostic of SIDS. Sudden unexpected infant deaths (SUIDs) may have an explainable cause such as a congenital condition (cardiac arrhythmia, structural heart disease, metabolic disorder) or an acquired condition (infection, intentional trauma, or accidental such as asphyxiation). It can also have an unexplainable cause, of which SIDS is the most common.

22_Toy-Pediatrics_Case21_p203-210.indd 205

29/07/21 1:06 PM

206

CASE FILES: PEDIATRICS

Brief Resolved Unexplained Event An infant who has experienced a BRUE should further be classified as lower risk or higher risk. The higher-risk infant may have another condition as the cause of the BRUE, such as a cardiac, respiratory, central nervous system (CNS), metabolic, infectious, or gastrointestinal (GI) disorder, and will have different management than a lower-risk infant. Laboratory studies for a patient experiencing a BRUE often include a complete blood count (leukocytosis could suggest an infectious etiology), as well as testing for respiratory syncytial virus, pertussis, or other respiratory tract pathogens. A comprehensive metabolic panel may uncover a metabolic etiology. A report of feeding difficulties or emesis should lead to consideration of upper GI studies such as imaging, swallowing evaluation, and pH probe. In contrast, unusual posturing or movements should prompt an electroencephalogram. If a murmur is detected, an electrocardiogram, chest x-ray, or echocardiogram may be considered to look for arrhythmias, such as prolonged QT syndrome, or a structural cardiac defect.

CASE CORRELATION šš

See Case 38 (Child Abuse).

COMPREHENSION QUESTIONS 21.1 Which of the following features makes sudden infant death syndrome (SIDS) the likely cause of a sudden death? A. An infant found with a bulging fontanelle and facial bruise B. An 18-month-old girl who had a prior sibling that at 1 year of age also died suddenly and unexpectedly C. A 5-month-old infant with dysmorphic features and an enlarged heart found on postmortem examination D. A 3-month-old boy whose parents smoke in the home but were using a high-efficiency particulate air (HEPA) purifier in his room and a baby monitor, placing him on his side to sleep to avoid formula aspiration, and using a special foam wedge pillow to keep him in that position E. All the above features make SIDS a likely cause of a sudden death.

22_Toy-Pediatrics_Case21_p203-210.indd 206

29/07/21 1:06 PM

SECTION II: CLINICAL CASES

207

21.2 A mother presents to the ED with her 6-month-old daughter late at night after she noticed her breathing rapidly for about 30 seconds, then stop breathing for 30 seconds, and then become limp, pale, and unresponsive. The mother attempted to give mouth-to-mouth breaths for a few seconds, and her daughter then began to cry and her breathing and appearance normalized. Your next best step is which of the following? A. Perform a thorough history and physical examination; if all are normal, the infant is diagnosed with BRUE and additional evaluation will be guided by the presence of risk factors. B. Reassure the mother that her daughter does not need to be evaluated because she did not require chest compressions, and thus, she can be discharged from the ED. C. Perform food allergy testing due to concern for anaphylaxis. D. Prescribe an antibiotic in case the infant is starting to develop pneumonia. E. Tell the mother this was a near-SIDS event and instruct her to buy a home apnea monitor. 21.3 You are counseling parents of a newborn about prevention of SIDS. Which of the following statements about ways to reduce SIDS is accurate? A. Infants should sleep in the same bed as the parent or on the parent’s chest so they can be closely monitored for apnea. B. Infants should sleep on their back on a firm mattress with no accompanying soft bedding or objects, including no devices advertised to maintain the sleep position. C. Pacifiers should be avoided because they can obstruct the baby’s airflow during respiration. D. Keep the infant dressed in several layers and covered with a heavy blanket. E. Infants should be given acetaminophen before their scheduled vaccines in order to prevent an undetected febrile seizure and resulting SIDS. 21.4 The investigation of an unexpected infant death includes a history, a postmortem examination, and which of the following? A. DNA studies B. Maternal drug screen C. Analysis of parental electrocardiograms D. A death scene investigation E. Stool studies

22_Toy-Pediatrics_Case21_p203-210.indd 207

29/07/21 1:06 PM

208

CASE FILES: PEDIATRICS

ANSWERS 21.1 D. A 3-month-old boy whose parents smoke in the home but were using a highefficiency particulate arrestance (HEPA) air purifier in his room and a baby monitor, placing him on his side to sleep to avoid formula aspiration, and using a special foam wedge pillow to keep him in that position. Unfortunately, no specific product has been proven to reduce the risk of SIDS. The child’s age and gender are the main features that support SIDS as a possible cause of death. Exposure to tobacco smoke is also a risk factor. Physical signs of trauma (answer A) make intentional injury and infanticide the most likely causes of death, whereas anatomic defects (answer C) make cardiac or metabolic causes likely. The occurrence of a genetic susceptibility to SIDS within a family (answer B) is exceedingly rare, and SIDS cannot be the diagnosis if the patient is not an infant (eg, older than 12 months). 21.2 A. Perform a thorough history and physical examination; if all are normal, the infant is diagnosed with BRUE and additional evaluation will be guided by the presence of risk factors. The infant’s change in breathing and mental status that was observed by the caregiver needs further evaluation. If the history and physical examination do not identify the cause, then BRUE is the diagnosis. Further diagnostic studies will be needed if the infant is a high-risk patient. Reassurance (answer B) is not appropriate and can lead to mortality. There is no evidence of a food allergy (answer C) or pneumonia (answer D) in this case. This event is unlikely to be caused by SIDS (answer E) because most episodes of SIDS have no type of event beforehand. 21.3 B. Infants should sleep on their back on a firm mattress with no accompanying soft bedding or objects, including no devices advertised to maintain the sleep position. The decline in SIDS has been attributed to the change in sleep position emphasizing infants be placed on their back. Other protective factors include pacifier use (answer C) and vaccination. Risk factors associated with SIDS include bed sharing (answer A), prone sleep position, and overheating (answer D). Acetaminophen (answer E) does not prevent febrile reactions from vaccination and may reduce the immunogenic reaction from the vaccine. 21.4 D. A death scene investigation. A death scene investigation is crucial to rule out trauma, both intentional and accidental. The other studies (answer A, DNA studies; answer B, maternal drug screen; answer C, parental electrocardiograms; and answer E, stool studies) are possible adjuncts and may be indicated depending on other findings.

22_Toy-Pediatrics_Case21_p203-210.indd 208

29/07/21 1:06 PM

SECTION II: CLINICAL CASES

209

CLINICAL PEARLS »»

SIDS is a diagnosis of exclusion assigned only after the postmortem investigation, postnatal history, and crime scene investigation fail to yield another explanation.

»»

Prone sleep position, bed sharing, overheating, and exposure to cigarette smoke are significant preventable risk factors for SIDS.

»»

Breastfeeding, immunizations, and pacifier use have been identified as factors to reduce the risk of SIDS.

»»

An infant who has experienced a BRUE should further be classified as lower risk or higher risk.

»»

High-risk BRUE is often caused by a cardiac, respiratory, central nervous system, metabolic, infectious, or gastrointestinal disorder and will have different management than a lower-risk infant.

REFERENCES AAP Task Force on Sudden Infant Death Syndrome. SIDS and other sleep-related infant deaths: updated 2016 recommendations for a safe infant sleeping environment. Pediatrics. 2016;138:e20162938. Hauck FR, Carlin RF, Moon RY, Hunt CE. Sudden infant death syndrome. In: Kleigman RM, St. Geme JW, Blum NJ, et al, eds. Nelson Textbook of Pediatrics. 21st ed. Philadelphia, PA: Elsevier; 2020:2167-2176. Tieder JS, Bonkowsky JL, Etzel RA, et al. Brief resolved unexplained events (formerly apparent life-threatening events) and evaluation of lower-risk infants. Pediatrics. 2016;137:e20160590.

22_Toy-Pediatrics_Case21_p203-210.indd 209

29/07/21 1:06 PM

This page intentionally left blank

CASE 22 A 2-month-old girl presents to her pediatrician’s office for her well-child check. She was born vaginally at 35 weeks’ gestation after an uncomplicated pregnancy. She was discharged at 3 days of age and has had no significant medical issues. The baby was noted to have a murmur at birth; the mother reports she was told, “it should be watched.” Since the previous checkup, her mother has noticed that her daughter is not breastfeeding as well as usual and becomes sweaty while eating. On examination, growth parameters show her weight to be at the 24th percentile (previously 78th percentile at 2 weeks), heart rate is 158 beats per minute, and respiratory rate is 58 breaths per minute. She has nasal flaring and subcostal retractions. Auscultation of the lungs reveals diffuse crackles. She has bounding pulses and a grade 3/6 continuous machine-like murmur at the left infraclavicular area. ▶▶ ▶▶ ▶▶

What is the most likely diagnosis? What are the possible complications of this condition? What is the most important diagnostic test?

23_Toy-Pediatrics_Case22_p211-218.indd 211

26/07/21 12:53 PM

212

CASE FILES: PEDIATRICS

ANSWERS TO CASE 22: Patent Ductus Arteriosus With Heart Failure Summary: A 2-month-old girl presents with šš

History of vaginal birth at 35 weeks’ gestation after an uncomplicated pregnancy

šš

Difficulty breastfeeding and sweating while eating

šš

Poor growth and signs of respiratory distress

šš

Bounding pulses

šš

A continuous machine-like murmur at the left infraclavicular space

Most likely diagnosis: Patent ductus arteriosus (PDA) that has progressed to heart failure. Possible complications: Eisenmenger syndrome. Most important diagnostic test: Transthoracic echocardiography.

ANALYSIS Objectives 1. Discuss the pathophysiology and diagnostic features of PDA. (EPA 1, 2, 3) 2. Describe the medical treatment of PDA. (EPA 4) 3. Recognize the complications of PDA. (EPA 10, 12)

Considerations This patient has signs of heart failure likely due to an undiagnosed PDA. A continuous machine-like murmur is the classic description for a PDA and is best heard at the left infraclavicular region. Small PDAs can be benign and asymptomatic, but this patient likely had an undiagnosed moderate to severe PDA that then resulted in heart failure. Symptoms of heart failure in infants include poor feeding and growth, diaphoresis with feeding, and respiratory distress (tachypnea, retractions, or nasal flaring).

APPROACH TO: Patent Ductus Arteriosus DEFINITIONS EISENMENGER SYNDROME: A complication of acyanotic heart conditions in which increased pulmonary circulation from left-to-right shunting results in pulmonary arterial hypertension. The increased right-sided heart pressure ultimately reverses flow to right-to-left shunting, which then is associated with cyanosis.

23_Toy-Pediatrics_Case22_p211-218.indd 212

26/07/21 12:53 PM

SECTION II: CLINICAL CASES

213

It can occur in congenital heart conditions such as ventricular septal defects, atrioventricular canal lesions, and PDA. HEART FAILURE: A syndrome in which the heart is not able to sustain adequate cardiac output as a result of poor systolic or diastolic function, resulting in respiratory distress, poor feeding and growing, and poor systemic perfusion. LEFT-TO-RIGHT SHUNT: A condition in which blood flow will favor movement from the higher-pressure systemic circuit (left) to the lower-pressure venouspulmonary circuit (right) through an open atrial or ventricular wall defect. Usually, no cyanosis is associated with left-to-right shunting, unlike with right-to-left shunting, in which cyanosis may be noted. PATENT DUCTUS ARTERIOSUS (PDA): A structure that connects the aorta and pulmonary arteries to provide physiologic right-to-left shunting in utero. If it persists beyond the neonatal period, it can result in pathologic left-to-right shunting.

CLINICAL APPROACH Pathophysiology Congenital cardiac defects are first categorized according to the presence or absence of cyanosis, which can be determined via pulse oximetry and a complete history and physical examination. Other data helpful in making this diagnosis include a chest x-ray (CXR), which shows evidence of increased, normal, or decreased pulmonary vascular markings, and an electrocardiogram (ECG), which provides voltage data to determine if atrial or ventricular hypertrophy is present. The majority of acyanotic lesions result in increased volume load to the right heart due to increased flow from the systemic circulation to the pulmonary circulation (the so-called left-to-right shunt). If untreated, this increased flow to the right heart can result in increased pulmonary vascular pressure, reversal of blood flow across the defect (creating a right-to-left shunt), and ultimately clinical cyanosis, which is a condition called Eisenmenger syndrome. Such acyanotic lesions include ventricular septal defect (VSD), atrial septal defect (ASD), and PDA. Other forms of acyanotic defects cause changes in pressure due to obstruction of blood flow; this group includes pulmonic and aortic valve stenosis and coarctation of the aorta.

Patent Ductus Arteriosus Epidemiology.  PDA most commonly occurs in preterm infants. Risk factors for PDA are prematurity, trisomy 21, and congenital rubella syndrome. Pathophysiology.  In utero, a fetus is dependent on oxygen supply from the placenta and does not rely on their own lungs. Thus, blood flow mostly bypasses the fetal lungs and is supplied to the rest of the body via the ductus arteriosus. A ductus arteriosus is a normal, physiologic right-to-left shunt that connects flow from the pulmonary arteries to the aortic arch, allowing for flow away from the high-pressure pulmonary circulation to the lower-pressure systemic circulation (Figure 22–1). Once the newborn initiates breathing at birth, the resistance in the pulmonary

23_Toy-Pediatrics_Case22_p211-218.indd 213

26/07/21 12:53 PM

214

CASE FILES: PEDIATRICS

A

B

Figure 22–1.  Blood flow in the normal heart (A) and in the heart with patent ductus arteriosus (B). (Reproduced with permission, from National Heart, Lung, and Blood Institute. Congenital Heart Defects. 2021. National Institutes of Health; U.S. Department of Health and Human Services. https:// www.nhlbi.nih.gov.)

circulation drops due to an increased pulmonary arterial oxygenation and a drop in prostaglandin E2 levels, which drives the ductus arteriosus to vasoconstrict significantly and eventually close within the first few days of life. Ductus closure in term infants usually occurs within 10 to 15 hours of birth and almost always by 2 days of age. However, in about 1 in 2000 patients, the ductus arteriosus does not close, creating pathologic left-to-right shunting. Imaging and Treatment.  An echocardiogram is the most important diagnostic test. It can visualize the PDA and can identify other cardiac pathology related to shunting, valvular issues, and atrial and ventricular size and function. Treatment for PDA ranges from supportive care and monitoring for select groups of low-risk patients to medical management with nonselective cyclooxygenase inhibitors, including indomethacin and ibuprofen, to surgical closure via ligation or percutaneous closure. Complications.  Complications of a PDA include the risk of infectious endocarditis, heart failure, pulmonary hypertension, and resultant cyanosis. For some neonates with cyanotic heart conditions (eg, pulmonary atresia, hypoplastic left heart syndrome, or coarctation of the aorta), the PDA is life-saving. These conditions are ductus dependent, and administration of prostaglandin E1 is necessary for prevention of PDA closure.

Ventricular Septal Defects Epidemiology and Pathophysiology.  VSDs are the most common heart lesions in children, accounting for 25% of all congenital heart defects and affecting 3 to 6 of

23_Toy-Pediatrics_Case22_p211-218.indd 214

26/07/21 12:53 PM

SECTION II: CLINICAL CASES

215

every 1000 live term births. Most VSDs are small, with left-to-right shunting, and are located in the membranous portion of the ventricular septum. Clinical Presentation.  Children with small VSDs (less than 5 mm) usually are asymptomatic, and a harsh, holosystolic murmur is heard at the left lower sternal border. The murmur of a large VSD may be less harsh because of the absence of a significant pressure gradient across the defect. Large lesions can result in dyspnea, feeding difficulties, growth failure, and diaphoresis, and they may lead to recurrent infections and cardiac failure. Infants with large VSDs generally are not cyanotic, but they may become dusky during feeding or have protracted crying. A VSD may not be detected on examination in the first few weeks of life because of high rightsided pressures, but it may become audible as pulmonary vascular resistance drops and left-to-right shunting of blood increases across the defect. In children with significant VSDs, CXR shows cardiomegaly with an enlarged left atrium and ventricle and increased pulmonary vascular markings. The ECG shows biventricular hypertrophy. The ECG and CXR are usually normal with a small VSD.

Atrial Septal Defects Children with small ASDs usually have no symptoms, but large defects may cause mild growth failure, frequent upper respiratory tract infections, and exercise intolerance. Physical examination findings include a fixed, split-second heart sound that does not vary with respiration, as well as a systolic murmur at the left upper and midsternal borders caused by high-volume blood flow from the right ventricle into the normal pulmonary artery; the murmur is not blood flowing across the ASD itself. A lower left sternal border diastolic murmur produced by increased flow across the tricuspid valve may be present. The CXR reveals an enlarged right atrium, right ventricle, and pulmonary artery and increased pulmonary vascularity. ECG shows right ventricular hypertrophy and sometimes right-axis deviation. ASDs are well tolerated during childhood but can lead to pulmonary hypertension in adulthood or atrial arrhythmias from atrial enlargement.

Coarctation of the Aorta Coarctation of the aorta is a narrowing of the aorta, and although it can occur anywhere along its path from the transverse arch to the iliac bifurcation, it most commonly occurs just below the origin of the left subclavian artery. If the narrowing is severe enough, symptoms may appear in the neonatal period. Classic findings are weak or absent pulses in the lower extremities, along with lower systolic blood pressure and pulse oximetry in the legs compared to the arms. Coarctation can be ductus dependent. Symptoms of the coarctation may then not appear until the ductus begins to close, at which point loss of lower extremity perfusion, severe acidosis, and cardiovascular collapse occur. An infusion of prostaglandin E would be required to maintain the patent ductus until definitive repair could be done. Echocardiogram with color Doppler can usually identify the site of the coarctation. Otherwise, computed tomography, magnetic resonance imaging, or cardiac catheterization is needed. Table 22–1 provides a summary of the four congenital heart defects discussed in this text.

23_Toy-Pediatrics_Case22_p211-218.indd 215

26/07/21 12:53 PM

216

CASE FILES: PEDIATRICS

Table 22–1  •  ACYANOTIC HEART LESIONS Lesion

Physical Exam

CXR

ECG

VSD

•  Holosystolic murmur at left lower sternal border

•  Normal if a small shunt •  Cardiomegaly •  Enlarged left atrium and left ventricle •  Increased pulmonary vascular markings

•  Normal if a small shunt •  Biventricular hypertrophy •  Left axis deviation

ASD

•  Fixed, widely split S2

•  Enlarged right ventricle •  Right ventricular •  Prominent pulmonary hypertrophy arteries •  Right axis deviation •  Increased pulmonary vascular markings

PDA

•  Widened pulse pressure •  Continuous machinerylike murmur at second intercostal space

•  Increased pulmonary vascular markings

•  Biventricular hypertrophy •  Right axis deviation

Coarctation of aorta

•  Systolic murmur in the left axilla

•  Cardiomegaly •  Normal to increased pulmonary vascularity

•  Right ventricular hypertrophy •  Right bundle branch block •  Right axis deviation

Abbreviations: ASD, atrial septal defect; CXR, chest x-ray; ECG, electrocardiogram; PDA, patent ductus arteriosus; VSD, ventricular septal defect.

CASE CORRELATION šš

See Case 2 (Infant of a Diabetic Mother), Case 7 (Esophageal Atresia), Case 10 (Failure to Thrive), and Case 23 (Cyanotic Congenital Heart Disease).

COMPREHENSION QUESTIONS 22.1 A 3-month-old boy arrives in the emergency department with tachypnea and retractions. His mother reports he has not been feeding well for the past week. On auscultation, you note fine crackles and wheezes. His heart rate is 190 beats per minute, and a 2/6 harsh holosystolic murmur is identified along the left sternal border. Chest x-ray (CXR) shows cardiomegaly and increased pulmonary vascular markings. Which of the following defects is the likely cause of his symptoms? A. Pneumonia B. Patent ductus arteriosus (PDA) C. Small ventricular septal defect (VSD) D. Large VSD E. Small atrial septal defect (ASD)

23_Toy-Pediatrics_Case22_p211-218.indd 216

26/07/21 12:53 PM

SECTION II: CLINICAL CASES

217

22.2 A premature infant is born at 32 weeks’ gestation to a mother who recently immigrated and had no prenatal care. The infant is noted to have bilateral cataracts and a continuous machine-like murmur on physical examination. Furthermore, he fails his newborn hearing screen. The patient’s mother most likely had which of the following infections during the first trimester of her pregnancy? A. Cytomegalovirus B. Rubella C. Varicella D. Toxoplasmosis E. Herpes simplex virus 22.3 A previously healthy term infant is being seen in the emergency department because the parents state that he suddenly developed respiratory distress about 2 hours ago. He is 3 days old and the product of a normal vaginal delivery. The patient has pronounced nasal flaring and intercostal retractions. An echocardiogram reveals coarctation of the aorta. Which of the following is the most appropriate treatment for immediate stabilization of this infant? A. Digoxin B. Furosemide C. Albuterol D. Racemic epinephrine E. Prostaglandin therapy

ANSWERS 22.1 D. Large VSD. This infant is exhibiting congestive heart failure. Based on the characteristics of the accompanying murmur and CXR, the most likely etiology is a large VSD. Pneumonia (answer A) would present with cough, fever, and no cardiac murmur. PDA (answer B) would present with a continuous, machine-like murmur. A small VSD (answer C) or small ASD (answer E) would likely be asymptomatic. 22.2 B. Rubella. The infant has a murmur characteristic of a PDA. This finding, in addition to cataracts and deafness, points toward congenital rubella syndrome. Most infants with the syndrome are infected during the first trimester of pregnancy. Congenital cytomegalovirus (answer A) presents with microcephaly, periventricular calcifications, deafness, and, not as frequently, cardiac anomalies. Varicella (answer C) is not common and can cause microcephaly, Horner syndrome, and limb abnormalities. Toxoplasmosis (answer D) leads to intracranial calcifications, chorioretinitis, and, not as often, cardiac defects. Herpes simplex virus (answer E) usually infects the infant through the birth process and leads to encephalitis.

23_Toy-Pediatrics_Case22_p211-218.indd 217

26/07/21 12:53 PM

218

CASE FILES: PEDIATRICS

22.3 E. Prostaglandin therapy. This infant’s symptoms started when his ductus arteriosus began to close. A continuous intravenous infusion of prostaglandin will keep the ductus open and allow blood flow to reach past the area of coarctation and perfuse the lower portion of the body. Surgery provides definitive repair of the coarctation. Digoxin (answer A) would not be effective since this problem is not due to a cardiac contractility or conduction problem. Furosemide (answer  B) would not help because there is no evidence of cardiac failure. Neither albuterol (answer  C) nor racemic epinephrine (answer D) would be effective because there is no bronchospasm associated with this condition.

CLINICAL PEARLS »»

Acyanotic cardiac lesions include ventricular septal defect (VSD), atrial septal defect (ASD), and patent ductus arteriosus (PDA).

»»

The ductus arteriosus is a structure that connects the aorta and pulmonary arteries to provide physiologic right-to-left shunting in utero, but if it persists beyond the newborn period, pathologic left-to-right shunting may result.

»»

PDA is associated with maternal rubella and can be seen more frequently in infants with certain genetic conditions such as trisomy 21.

»»

Left-to-right shunts in acyanotic heart diseases eventually can reverse direction (right-to-left) and cause cyanosis if pulmonary hypertension develops (Eisenmenger syndrome).

»»

VSD is the most common congenital cardiac defect and can cause heart failure.

»»

ASD is often asymptomatic, but large defects may lead to growth failure, exercise intolerance, and a fixed, split S2 heart sound.

»»

Coarctation of the aorta is associated with hypertension, absent femoral pulses, and lower extremity ischemia.

REFERENCES Backes CH, Kovalchin C, Rivera BK, Smith CV. Patent ductus arteriosus: from bench to bedside. In: Kline MW, Blaney SM, Giardino AP, et al, eds. Rudolph’s Pediatrics. 23rd ed. New York, NY: McGraw Hill; 2018:270-277. Benitz WE, Committee on Fetus and Newborn. Patent ductus arteriosus in preterm infants. Pediatrics. 2016;137(1):e20153730. Bernstein D. Evaluation and screening of the infant or child with congenital heart disease. In: Kleigman RM, St. Geme JW, Blum NJ, et al, eds. Nelson Textbook of Pediatrics. 21st ed. Philadelphia, PA: Elsevier; 2020:2371-2373. Schneider DJ, Moore JM. Patent ductus arteriosus. Circulation. 2006;114:1873-1882.

23_Toy-Pediatrics_Case22_p211-218.indd 218

26/07/21 12:53 PM

CASE 23 A 1-week-old boy presents to the emergency department with a 5-day history of progressively increased work of breathing and poor feeding. He is diaphoretic, has a respiratory rate of 68 breaths per minute with subcostal retractions, pulse of 190 beats per minute, oxygen saturation of 85%, and a widened pulse pressure. His gingiva and buccal mucosa appear blue. Rales are heard over both lung fields. An early systolic click followed by a systolic ejection murmur at the left sternal border and a single S2 are heard. The precordium is hyperdynamic, and the femoral pulses are bounding. The liver span is enlarged. Chest radiograph shows cardiomegaly and increased pulmonary vascularity. Emergent echocardiography shows a single arterial cardiac outflow tract that is receiving blood from both ventricles. ▶▶ ▶▶

What is the most likely diagnosis? What is the best management for this condition?

24_Toy-Pediatrics_Case23_p219-228.indd 219

28/07/21 7:09 AM

220

CASE FILES: PEDIATRICS

ANSWERS TO CASE 23: Truncus Arteriosus Summary: A 1-week-old boy presents with šš

šš

Signs of congestive heart failure (tachypnea, dyspnea, diaphoresis, cardiomegaly, pulmonary vascular congestion, and hepatomegaly) Central cyanosis and hypoxia along with features suggesting a cardiac malformation (a murmur, a systolic click, and a single S2)

Most likely diagnosis: Truncus arteriosus, a type of cyanotic congenital heart disease (CHD). Best management: Medications to treat the congestive heart failure include diuretics to reduce systemic, pulmonary, and venous congestion; angiotensin-converting enzyme (ACE) inhibitors to reduce afterload; and inotropes (digoxin, dopamine, dobutamine) to increase contractility. Ultimately, surgical correction is needed. Delay of surgery beyond the first few months of life can increase the likelihood of pulmonary vascular disease, and many centers now perform neonatal repair at the time of diagnosis.

ANALYSIS Objectives 1. Know the major types of cyanotic CHD and their most common clinical presentations. (EPA 1, 2) 2. Understand the management of cyanotic CHD. (EPA 4)

Considerations This infant has truncus arteriosus, an uncommon cardiac defect, in which a single arterial trunk arises from the normally formed ventricles. The single trunk typically straddles a defect in the outlet portion of the intraventricular septum, and a single valve at the outflow tract is found (Figure 23–1). The main pulmonary artery and ascending aorta arise from the common arterial trunk. Cyanosis results from the mixing of the systemic and pulmonary blood flow that occurs across the ventricular septal defect and within the single arterial trunk. The condition can be complicated by the presence of an interrupted aortic arch and aberrant coronary arteries. When the pulmonary resistance drops below the systemic pressure, increased blood flow to the pulmonary system occurs, leading to pulmonary congestion, increased myocardial work, and subsequent heart failure.

24_Toy-Pediatrics_Case23_p219-228.indd 220

28/07/21 7:09 AM

SECTION II: CLINICAL CASES

221

Figure 23–1.  Schematic drawing of circulation of various cardiac defects: (A) normal circulation, (B) tetralogy of Fallot, (C) pulmonary atresia, (D) tricuspid atresia, (E) transposition of the great arteries, (F) truncus arteriosus. Black arrows indicate deoxygenated blood, cross-hatched arrows indicate mixed blood, and white arrows indicate oxygenated blood. Abbreviations: LA, left atrium; LV, left ventricle; RA, right atrium; RV, right ventricle.

APPROACH TO: Cyanotic Congenital Heart Disease DEFINITIONS CONOTRUNCAL HEART DEFECTS: Malformations of the cardiac outflow tracts (aorta and main pulmonary artery). Common types include truncus arteriosus, tetralogy of Fallot (TOF), pulmonary atresia, and interrupted aortic arch. These are commonly seen in chromosome 22q11.2 deletions such as DiGeorge syndrome.

24_Toy-Pediatrics_Case23_p219-228.indd 221

28/07/21 7:09 AM

222

CASE FILES: PEDIATRICS

CYANOSIS: Bluish discoloration of the skin and mucous membranes caused by insufficient saturation of the blood with oxygen. Peripheral cyanosis (acrocyanosis) is common in neonates and involves the extremities only; it may be normal. Central cyanosis is always abnormal and is seen on the tongue, gingiva, and buccal mucosa. DUCTUS-DEPENDENT LESIONS: Cardiac defects that are incompatible with life in the absence of a patent ductus arteriosus (PDA). RIGHT-TO-LEFT CARDIAC SHUNT: Abnormal flow of blood across a cardiac defect from the right side of the heart containing deoxygenated blood to the left side of the heart where it is then pumped into the systemic circulation. These lesions result in cyanosis.

CLINICAL APPROACH Pathophysiology Cyanotic CHD mainly comprises cardiac anomalies that either allow deoxygenated blood to bypass the lungs and enter the systemic circulation or allow deoxygenated blood to mix with blood that has already been oxygenated as it enters the systemic circulation. Mixing occurs via an accompanying atrial or ventricular septal defect. However, in the anomalies where the right ventricle does not connect to the pulmonary circulation, a PDA is needed to conduct the deoxygenated blood to the lungs. These cardiac lesions are termed ductal dependent and often manifest earlier than the anomalies that are ductus independent because ductal closure normally occurs on the first or second day of life in term infants. The distinction is important because intravenous prostaglandin E1 is given to keep the ductus open in ductal-dependent lesions and allows for infant stabilization prior to more definitive surgical correction. Ductal-dependent lesions include tricuspid atresia, pulmonary valve atresia, severe pulmonary valve stenosis, TOF if the accompanying pulmonary stenosis is severe, some forms of total anomalous pulmonary venous return (TAPVR), and transposition of the great arteries without ventricular inversion (D-TGA). Hypoplastic left heart syndrome (HLHS) is also a type of ductus-dependent cyanotic CHD, but cyanosis is due to decreased systemic perfusion rather than mixing of deoxygenated and oxygenated blood.

Clinical Presentation Signs and Symptoms.  Cyanosis may not be visible unless oxygen saturation is 85% or less, so pulse oximetry is used to identify its presence. Clubbing will not be encountered until the hypoxia has been present for several months. Any infant with tachypnea, tachycardia, poor feeding, or an abnormal cardiac examination should have pulse oximetry performed. Measurement should be done on the tissues that are perfused by the portion of the aorta that is proximal to the ductus (the right hand or an ear lobe), as well as on the tissues that are perfused by the portion of the aorta that is distal to the ductus (the lower extremity). If a difference of more than 3% to 5% is found, then a right-to-left shunt across the ductus may be present.

24_Toy-Pediatrics_Case23_p219-228.indd 222

28/07/21 7:09 AM

SECTION II: CLINICAL CASES

223

Further evaluation is then required. Several states have mandated that standard newborn care include pre- and postductal pulse oximetry in neonates 24 hours after birth to screen for cyanotic CHD prior to hospital discharge. Heart Sounds.  Many of the cyanotic heart lesions have a nonspecific murmur caused by the accompanying septal defect or PDA. The exception is severe pulmonary valve stenosis, with its systolic ejection murmur located at the upper left sternal border. Abnormal heart sounds may be found in some of the cyanotic CHD lesions: a single S2 occurs with pulmonary valve atresia or truncus arteriosus; an early systolic ejection click is heard with pulmonary stenosis or truncus arteriosus. Diagnostic Testing.  On chest radiograph, cyanotic CHD is usually characterized by increased pulmonary vascularity and cardiomegaly. Only an atretic tricuspid valve, atretic pulmonary valve, or TOF (with its pulmonic valve stenosis) will have decreased pulmonary vascularity. Some distinctive radiographic appearances of specific anomalies are described (Table 23–1). The right ventricle hypertrophy in TOF causes the apical shadow of the heart to point upward, creating a “boot” or “wooden shoe” shape. D-TGA is described as “an egg on a string” because the reversed pulmonary artery and aorta give a narrow mediastinal vascular shadow. TAPVR can appear as a “snowman,” which is created by the round supracardiac shadow of a dilated innominate vein and vena cava that are receiving venous blood flow from the body as well as from the pulmonary veins. This finding is usually not seen until after the neonatal period because the prominent thymus of the neonate obscures it.

Table 23–1  •  TYPICAL RADIOGRAPHIC FINDINGS OF COMMON HEART LESIONS Heart Anomaly

Radiographic Appearance

Tetralogy of Fallot

“Boot-shaped” heart and decreased pulmonary vascularity

Pulmonary atresia (with intact ventricular septum)

Decreased pulmonary vascularity

Tricuspid atresia (with normally related great vessels)

Decreased pulmonary vascularity

Epstein anomaly

Heart size may be normal to massive, with normal or decreased pulmonary vascularity

Transposition of the great arteries

“Egg-on-a-string” (narrow mediastinum) with normal to increased pulmonary vascularity

Truncus arteriosus

Cardiomegaly and increased pulmonary vascularity

Total anomalous pulmonary venous return

“Snowman” (supracardiac shadow caused by anomalous pulmonary veins entering the innominate vein and persistent left superior vena cava) and increased pulmonary vascularity

Hypoplastic left heart syndrome

Cardiomegaly and increased pulmonary vascularity

24_Toy-Pediatrics_Case23_p219-228.indd 223

28/07/21 7:09 AM

224

CASE FILES: PEDIATRICS

Electrocardiogram (ECG) can be normal, and many abnormal findings are nonspecific. Right ventricular hypertrophy can be normal for a neonate, and biventricular hypertrophy may accompany a ventricular septal defect. However, tricuspid valve atresia may be distinguished by ECG because of its left axis deviation, biatrial enlargement, and absent right ventricle markings (ie, no R waves in leads V1-V3). Echocardiography is needed for definitive diagnosis of the specific cardiac lesion in any of these conditions.

Treatment Treatment of the infant with cyanotic heart disease depends on the specific lesions that are present. Stabilization is the essential first step and usually involves medications. If the lesion is ductal dependent, prostaglandin E1 infusion should be started (prostaglandin E1 is normally produced by the ductus of a developing fetus and helps to keep the ductus open). Other measures include creating an atrial septum via cardiac catheterization, known as atrial septostomy, which is used for TGA before definitive surgery occurs. Surgical management is often performed once the infant is stabilized. Initially, palliative procedures, such as the creation of an aortic pulmonary shunt for tricuspid atresia or TOF, may be required. Complete surgical repair occurs later when the infant has grown. Long-term prognosis varies. After complete surgical repair, 90% of patients with TOF survive to adulthood; however, HLHS remains the most common fatal congenital heart defect.

CASE CORRELATION šš

See Case 2 (Infant of a Diabetic Mother), Case 7 (Esophageal Atresia), Case 10 (Failure to Thrive), and Case 22 (Patent Ductus Arteriosus).

COMPREHENSION QUESTIONS 23.1 A 15-hour-old neonate appears dusky. He is tachycardic and tachypneic, but the lung fields are clear to auscultation, and he has no murmur. His oxygen saturation is 82% and does not improve with administration of 90% oxygen via oxygen hood. A chest radiograph shows slightly increased pulmonary vascularity and a narrow mediastinum. Initial management of this infant’s condition should include which of the following? A. Perform electrocardiogram (ECG) and increase the administered oxygen concentration to 100% B. Perform echocardiogram and begin prostaglandin E1 infusion C. Administer furosemide and repeat the chest radiograph D. Perform a karyotype and begin dopamine infusion E. Perform ECG and administer packed red blood cells

24_Toy-Pediatrics_Case23_p219-228.indd 224

28/07/21 7:09 AM

SECTION II: CLINICAL CASES

225

23.2 A 30-hour-old term infant has been feeding normally and has a normal physical examination, and her mother is being discharged. The nurse notifies you that the infant’s routine pre- and postductal pulse oximetry readings are 99% on the right hand and 91% on the right foot. Over the past 4 hours, he has repeated the measurements twice without change. When can you tell the mother that her infant does not have a congenital heart defect? A. If the pulse oximetry readings on the left foot are 93% B. If the pulse oximetry readings on all extremities at 48 hours of life are 100% C. If an echocardiogram is normal D. If the pulse oximetry readings do not improve after an infusion of prostaglandin E1 is started E. If a chest radiograph and ECG are normal 23.3 Which of the following statements about cyanotic congenital heart disease (CHD) is true? A. Newborns will always manifest symptoms before discharge from the hospital B. Clubbing is usually the first finding of cyanotic CHD C. Cyanotic CHD cannot be present unless an infant appears cyanotic D. Infants with cyanotic CHD may require treatment with diuretics and angiotensin coverting enzyme (ACE) inhibitors E. Emergent surgical repair is the first step in management for all cyanotic heart defects in the neonatal period 23.4 Which of the following statements about truncus arteriosus is true? A. A PDA is required for survival B. A “snowman” appearance on chest radiograph is commonly seen C. Characteristic ECG findings allow diagnosis D. Symptoms may not be present until after 2 weeks of life E. Balloon septostomy is the indicated treatment

ANSWERS 23.1 B. Perform echocardiogram and begin prostaglandin E1 infusion. This neonate has transposition of the great arteries (TGA), and as the ductus arteriosus begins to close, he will decompensate further. Urgent echocardiogram can confirm the diagnosis. Prostaglandin E1 should be started immediately to maintain the patency of the ductus arteriosus and to stabilize his condition. Increasing the oxygen concentration (answer A) or administering furosemide (answer C), dopamine (answer D), or packed red blood cells (answer E) will not improve his oxygenation if the ductus closes and no conduit for the oxygenated blood to reach the systemic circulation exists. An ECG is often normal or has nonspecific findings that do not identify the heart defect that is present.

24_Toy-Pediatrics_Case23_p219-228.indd 225

28/07/21 7:09 AM

226

CASE FILES: PEDIATRICS

23.2 C. If an echocardiogram is normal. This infant has evidence of a ductal shunt that should be investigated further because it can signal the presence of a cardiac defect. A postductal pulse oximetry measurement obtained from either lower extremity that is more than 3% to 5% lower than the preductal measurement is indicative of a ductal shunt. A normal echocardiogram would indicate that the infant does not have a congenital heart defect. A normal chest radiograph, a normal ECG, and the absence of a ductal shunt do not exclude the presence of a serious congenital cardiac defect—for example, truncus arteriosus (answer E). Prostaglandin infusion (answer D) would only improve systemic oxygenation if the infant had a ductal-dependent lesion and was undergoing closure of the ductus arteriosus. Pulse oximetry readings (answers A and B) are helpful screens but not diagnostic studies. 23.3 D. Infants with cyanotic CHD may require treatment with diuretics and ACE inhibitors. Pharmacologic management usually is the first step in stabilizing the infant presenting with heart failure. The presence of a heart defect may not be apparent until the ductus closes or the pulmonary vascular resistance falls, leading to heart failure (answer A). Clubbing (answer B) requires months to develop. ECG findings in CHD are usually nonspecific. Surgical intervention (answer E) is best performed once the infant is stabilized, and some defects require palliative procedures until the infant has grown sufficiently to undergo complete repair. Cyanosis (answer C) is not a requirement for the diagnosis of a cyanotic CHD. 23.4 D. Symptoms may not be present until after 2 weeks of life. Infants with truncus arteriosus may not show symptoms until the pulmonary vascular resistance drops and creates increased pulmonary blood flow with accompanying symptoms of heart failure. This condition is diagnosed by echocardiography because the ECG findings (answer C) are nonspecific; similar findings would be seen with many other cardiac defects. A snowman (answer B) appearance on chest radiography is found with total anomalous pulmonary venous return (TAPVR). Treatment for truncus arteriosus is surgical repair (not balloon septostomy, as in answer E, or maintaining a PDA, as in answer A).

CLINICAL PEARLS »»

Pulse oximetry is a valuable tool for the detection of cyanotic congenital heart defects (CHD).

»»

Lesions of CHD that are incompatible with life except in the presence of a patent ductus arteriosus (PDA) are termed “ductal dependent.”

»»

Infusion of prostaglandin E1 can stabilize infants with ductal-dependent lesions until more definitive surgical correction can be attempted.

»»

Truncus arteriosus can present with heart failure and requires initial pharmacologic stabilization prior to undergoing early surgical repair.

24_Toy-Pediatrics_Case23_p219-228.indd 226

28/07/21 7:09 AM

SECTION II: CLINICAL CASES

227

REFERENCES Bernstein D. Cyanotic congenital heart disease: lesions associated with decreased pulmonary blood flow. In: Kleigman RM, St. Geme JW, Blum NJ, et al, eds. Nelson Textbook of Pediatrics. 21st ed. Philadelphia, PA: Elsevier; 2020:2396-2407. Bernstein D. Cyanotic congenital heart disease: lesions associated with increased pulmonary blood flow. In: Kleigman RM, St. Geme JW, Blum NJ, et al, eds. Nelson Textbook of Pediatrics. 21st ed. Philadelphia, PA: Elsevier; 2020:2407-2420. Cabrera AG, Hoffman JIE. Congenital heart disease. In: Kline MW, Blaney SM, Giardino AP, et al, eds. Rudolph’s Pediatrics. 23rd ed. New York, NY: McGraw Hill; 2018:2227-2262. Jaggers J, Cole CR. Truncus arteriosus. In: Ungerleider RM, Nelson K, Cooper DS, Meliones J, Jacobs J, eds. Critical Heart Disease in Infants and Children. 3rd ed. Philadelphia, PA: Elsevier; 2019:661-669. Masarone D, Valente F, Rubino M, et al. Pediatric heart failure: a practical guide to diagnosis and management. Pediatr Neonatol. 2017;58:303-312. Teitel DF. Neonate and infant with cardiovascular disease. In: Kline MW, Blaney SM, Giardino AP, et al, eds. Rudolph’s Pediatrics. 23rd ed. New York, NY: McGraw Hill; 2018:2213-2227.

24_Toy-Pediatrics_Case23_p219-228.indd 227

28/07/21 7:09 AM

This page intentionally left blank

CASE 24 A 6-year-old unimmunized girl is seen for an initial illness visit in the pediatrician’s office. She has a 5-day history of fever, cough, rhinorrhea, and red, watery eyes. Her mom noted a red raised rash on her face that has spread to her neck, chest, back, arms, and legs. The rash began as small, separate spots that have coalesced to large blotches, particularly over the chest, back, and abdomen. She has missed the past 5 days of school due to the fever. On examination, her temperature is 102.2 °F (39 °C), and she appears ill but is in no acute distress. She has injected conjunctivae, small white papules with bluish-white centers on the buccal mucosa, and a blanching, erythematous, palpable rash over her chest, abdomen, and back. Pulmonary exam shows crackles of the right lower lung zone. A complete blood count (CBC) shows a white blood cell count of 3100 cells/mm3, hemoglobin of 15 mg/dL, and platelets of 102,000/mm3. ▶▶ ▶▶

What is the most likely diagnosis? What are the possible complications of this condition?

25_Toy-Pediatrics_Case24_p229-236.indd 229

10/06/21 1:41 AM

230

CASE FILES: PEDIATRICS

ANSWERS TO CASE 24: Measles Summary: A 6-year-old girl presents with šš šš

An acute history of fever, malaise, cough, conjunctivitis, and rhinorrhea An erythematous palpable rash that spares the palms and soles and appears to be clearing in the face

šš

Lung exam with rales of right lower lung zone

šš

Mild leukopenia and thrombocytopenia

šš

Medical history significant for incomplete immunizations

Most likely diagnosis: Measles (also known as rubeola) with secondary bacterial pneumonia. Possible complications: Measles has many possible complications. Secondary infection can occur as the measles virus infects T cells and dendritic cells, impairing antigen presentation and accessory T-cell function; this results in a transient immunosuppression. The most common of these secondary infections include diarrhea, otitis media, respiratory tract infections, and pneumonia. Neurologic complications are among the most feared and include acute measles-induced encephalitis, acute disseminated encephalomyelitis (ADEM), and subacute sclerosing panencephalitis (SSPE). Respiratory disease and encephalitis are the most common lethal complications. Other complications include myocarditis, pericarditis, keratitis, and corneal ulcerations.

ANALYSIS Objectives 1. Describe the clinical presentation, physical examination findings, and course of measles. (EPA 1, 3) 2. Apply the strategies for the diagnosis and treatment of measles, including proper prevention techniques. (EPA 3, 4) 3. Recognize the complications of measles. (EPA 10, 12)

Considerations Measles is a highly contagious disease that requires prompt recognition to ensure adequate isolation and treatment. It follows a typical course in most cases that is clearly divided into four stages: incubation, prodrome, exanthema, and recovery (convalescence). In the postvaccine era, a rise in measles cases due to vaccine refusal has been noted. What was once an essentially eliminated disease in the developed world has become a resurgent problem that is preventable with adherence to vaccination recommendations.

25_Toy-Pediatrics_Case24_p229-236.indd 230

10/06/21 1:41 AM

SECTION II: CLINICAL CASES

231

APPROACH TO: Measles DEFINITIONS ACUTE DISSEMINATED ENCEPHALOMYELITIS (ADEM): An acute demyelinating disease that occurs in 1 of 1000 measles cases during the recovery phase, typically 2 to 3 weeks after the exanthem. It is characterized by fever, headache, neck stiffness, and neurologic changes. More serious signs include seizures, ataxia, loss of bowel and bladder function, coma, or death (up to 20% of cases). This condition is thought to be a postinfectious autoimmune response. ACUTE MEASLES-INDUCED ENCEPHALITIS: A similar syndrome to ADEM, although typically with a lesser degree of signs and symptoms. It typically occurs within a few days of the rash and resolves more quickly than ADEM. EXANTHEM: A widespread rash typically associated with toxins, microorganisms, drug reactions, or autoimmune conditions. MMR VACCINE: A live-attenuated vaccine against measles, mumps, and rubella. This vaccine typically is given in a two-dose series, the first at 12 to 15 months of age and the second at 4 to 6 years of age. PRODROME: A period of early signs or symptoms occurring before more diagnostically specific signs or symptoms manifest. SUBACUTE SCLEROSING PANENCEPHALITIS (SSPE): SSPE is a rare, progressive, fatal degenerative neurologic condition thought to be due to chronic measles infection in the central nervous system. It typically occurs 7 to 10 years after measles infection and manifests with insidious onset of personality and behavior changes, lethargy, and difficulty in school. Over a period of weeks to years, worsening dementia will develop along with myoclonic jerks. Within a few months, further neurologic deterioration occurs with progression to flaccidity or decorticate rigidity, autonomic dysfunction, loss of myoclonus, and total vegetative state. Death is the eventual result in all cases of SSPE.

CLINICAL APPROACH Pathophysiology Measles is a resurging viral illness that, although generally self-limited, can lead to serious complications with lifelong sequelae and even death. It is predominantly seen in unvaccinated children, although adults and vaccinated people have been known to become ill. In recent years, it has become more prevalent due to waning vaccination rates and declining herd immunity, as well as increased travel throughout the world. When measles is suspected, especially in areas of high prevalence or during an outbreak, the patient should be isolated and provided with a mask to prevent the spread of infectious droplets. Health care personnel should exercise

25_Toy-Pediatrics_Case24_p229-236.indd 231

10/06/21 1:41 AM

232

CASE FILES: PEDIATRICS

standard and airborne precautions. Once the diagnosis is suspected and/or confirmed, the local health care department should be notified. The diagnosis of measles requires a high index of suspicion, given the effectiveness of the vaccine in preventing infection. The differential diagnosis of measles is broad, depending on the stage of infection. During the prodrome phase, measles can be mistaken for common respiratory viruses of childhood, such as rhinovirus, parainfluenza, influenza, adenovirus, and respiratory syncytial virus. Dengue fever must also be considered if the history supports it. During the exanthem phase, multiple viral infections may be considered: roseola, varicella, erythema infectiosum, enterovirus, and rubella. Group A Streptococcus infections can manifest with a rash (toxic shock syndrome or scarlet fever). Kawasaki disease, immunoglobulin (Ig) A vasculitis, and disseminated meningococcemia must also be considered.

Clinical Presentation Clinical Stages.  Classic measles infection consists of four clinical stages: incubation, prodrome, exanthema, and recovery (convalescence). šš

šš

šš

šš

Incubation (asymptomatic) begins with virus entry into the respiratory mucosa or conjunctivae, followed by local replication, lymphatic spread, and viremia. The incubation period lasts 6 to 21 days prior to symptoms, but patients are contagious 5 days before the rash appears to 4 days afterward. The prodrome phase begins with development of high fever (usually exceeding 103  °F), malaise, and anorexia. Cough, conjunctivitis, and coryza then develop and worsen prior to the exanthem phase. Koplik spots, which are 1- to 3-mm bluish-white spots on the buccal mucosa, are specific for measles and may appear in this stage (Figure 24–1). The exanthem typically begins 2 to 4 days after the initial prodrome symptoms. The rash begins at the hairline as scattered erythematous spots that coalesce to a raised blotchy lesion that then progresses in a cephalocaudal pattern; the rash spares the palms and soles. Laboratory examination may reveal leukopenia and thrombocytopenia. Many of the prodromal symptoms intensify during the early part of the exanthem stage. The most severe symptoms coincide with the worst of the rash, typically around 48 hours after appearance, and gradually fade with the rash in a cephalocaudal pattern. The exanthem stage typically lasts 6 to 7 days. Recovery: Following resolution of the rash, patients enter the recovery phase, which may be marked by persistent cough for up to 2 weeks but otherwise general resolution of symptoms.

Laboratory Values.  To establish the diagnosis, laboratory testing must show evidence of infection. Typical assays include a serum test for measles IgM, a nasopharyngeal or throat swab for viral culture, and/or a urine sample for viral culture. Reverse transcription polymerase chain reaction (RT-PCR) for measles RNA is also useful but may not be available in many areas. Finally, if the diagnosis is unclear, especially in the setting of prior exposure or vaccination, acute and convalescent

25_Toy-Pediatrics_Case24_p229-236.indd 232

10/06/21 1:41 AM

SECTION II: CLINICAL CASES

233

A

B Figure 24–1.  (A) Measles with exanthem. Erythematous macules, first appearing on the face and neck where they become confluent, spreading to the trunk and arms in 2 to 3 days where they remain discrete. In contrast, rubella also first appears initially on the face but spreads to the trunk in 1 day. Koplik spots on the buccal mucosa are also present. Erythematous papules become confluent on the face on the fourth day. (B) Measles with Koplik spots. Red papules on buccal mucosa opposite premolars prior to appearance of exanthema. (A: Reproduced with permission, from Wolff K, Johnson R, Saavedra AP, et al., eds. Fitzpatrick’s Color Atlas and Synopsis of Clinical Dermatology. 8th ed. 2017. Copyright © McGraw Hill LLC. All rights reserved. https://accessmedicine.mhmedical.com. B: Reproduced with permission, from Heinz F. Eichenwald. 2021. Centers for Disease Control and Prevention. https://phil.cdc.gov/.)

25_Toy-Pediatrics_Case24_p229-236.indd 233

10/06/21 1:41 AM

234

CASE FILES: PEDIATRICS

titers of serum measles IgG can be obtained, with a four-fold increase in antimeasles antibody titer being diagnostic of infection.

Treatment Treatment of measles is supportive. Patients may exhibit hypovolemia due to diarrhea, poor oral intake, and increased insensible losses from frequent fevers; volume resuscitation and maintenance are essential. Antipyretics for persistent fever and sore throat may help improve patient comfort as well as oral intake, but pediatric patients should not be given aspirin due to the risk of Reye syndrome. Treatment of secondary infections (eg, pneumonia, otitis media, cellulitis) consists of antibiotics tailored to the most likely pathogens. Vitamin A deficiency has been linked to higher measles complication rates, and measles has been shown to precipitate acute vitamin A deficiency. Therefore, the World Health Organization recommends treatment with vitamin A for 2 days for all patients infected with measles. No specific antiviral therapy is approved to treat measles. The most effective treatment is prevention through vaccination.

CASE CORRELATION šš

See Case 1 (Infant Rashes), Case 6 (Neonatal Herpes Simplex Virus Infection), and Case 26 (Stevens-Johnson Syndrome).

COMPREHENSION QUESTIONS 24.1 For children, which of the following is the appropriate two-dose measles, mumps, and rubella (MMR) vaccine schedule? A. 2-4 months, 15-18 months B. 12-15 months, 2-3 years C. 15-18 months, 4-6 years D. 12-15 months, 4-6 years E. 12-15 months, 3-4 years 24.2 A 5-year-old girl suspected of having measles is seen in the pediatrician’s office. Two days prior, a rash was noted on her face that spread to her body and extremities, along with other symptoms of fever, cough, coryza, and conjunctivitis. After laboratory samples are obtained, her mother asks for advice on preventing the spread of illness. Apart from hand hygiene, covering her mouth and nose with a mask, and notifying the health department, the pediatrician recommends quarantine at home for which duration? A. Until the rash completely resolves B. For 7 days following onset of the rash C. For 4 days following onset of the rash D. No isolation is needed until the diagnosis is confirmed E. For 7 days following onset of the fever

25_Toy-Pediatrics_Case24_p229-236.indd 234

10/06/21 1:41 AM

SECTION II: CLINICAL CASES

235

24.3 The parents of an asymptomatic 6-month-old girl are notified by the local health department that they were exposed to measles at a restaurant 5 days prior. The parents are the only other members of the household and are fully vaccinated. Assuming she remains asymptomatic, for how long should the child be quarantined at home? A. For 21 days after exposure B. For 14 days after exposure C. For 7 days after exposure D. No quarantine is necessary because she is not ill 5 days after exposure E. Until she sees a clinician and laboratory testing is negative 24.4 Which of the following is the most serious complication of measles infection? A. Acute disseminated encephalomyelitis (ADEM) B. Acute measles-induced encephalitis C. Subacute sclerosing panencephalitis (SSPE) D. Secondary bacterial pneumonia E. Mild hypovolemia due to diarrhea and decreased oral intake

ANSWERS 24.1 D. 12-15 months, 4-6 years. The initial MMR vaccine is given at 12 to 15 months and is followed by the second dose at 4 to 6 years. The other vaccine schedules listed (answers A, B, C, and E) are incorrect. 24.2 C. For 4 days following onset of the rash. Patients are contagious 5 days before the onset of rash until 4 days (not 7 days, as in answer B) after the onset of rash. This patient has had a rash for 2  days; she should be quarantined at home for 2 additional days. 24.3 A. For 21 days after exposure. The girl should remain in quarantine at home for 21 days after exposure (incubation period is 6-21 days), not 7 days (answer  C) or 14 days (answer B). Despite negative laboratory testing, a patient may still become ill following exposure during the incubation period; laboratory testing does not rule out infection during the incubation period (answer E). 24.4 C. SSPE. SSPE is the most serious complication of measles infection; it is incurable and invariably fatal. It typically occurs 7 to 10 years after measles infection and has no treatment. ADEM (answer A) is an acute autoimmune reaction to infection that is serious in some cases, but it can be treated. Acute measles-induced encephalitis (answer B) is a serious complication of measles but typically occurs quickly following infection and resolves rapidly, with only supportive therapy required. Mild hypovolemia (answer E) is common in measles and can be treated with intravenous fluids until the illness resolves.

25_Toy-Pediatrics_Case24_p229-236.indd 235

10/06/21 1:41 AM

236

CASE FILES: PEDIATRICS

CLINICAL PEARLS »»

Measles is a serious viral infection that is more common in recent years with waning vaccination rates and increased global travel.

»»

Measles infection consists of four well-described phases: incubation, prodrome, exanthem, and recovery, each with a fairly predictable course.

»»

Treatment of measles is supportive, with quarantine being especially important to preventing the spread of illness in undervaccinated populations; vaccination is the most effective way to prevent infection.

»»

Serious complications of measles infection include secondary bacterial infections and neurologic disorders, the most serious of which is subacute sclerosing panencephalitis.

REFERENCES American Academy of Pediatrics. Measles. In: Kimberlin DW, Brady MT, Jackson MA, Long SS, eds. Red Book: 2018 Report of the Committee on Infectious Diseases. 31st ed. Elk Grove, IL: American Academy of Pediatrics; 2018:537-550. Centers for Disease Control and Prevention. Measles (rubeola). https://www.cdc.gov/measles/hcp/ index.html. Accessed on February 28, 2020. Centers for Disease Control and Prevention. Measles, mumps, and rubella (MMR) vaccination: what everyone should know. htpps://www.cdc.gov/vaccines/vpd/mmr/public/index.html. Accessed on March 1, 2020. Gans H, Maldonado YA. Measles: clinical manifestations, diagnosis, treatment, and prevention. Hirsch MS, Kaplan SL, eds. UpToDate. Waltham, MA: UpToDate Inc. https://www.uptodate.com. Accessed on February 28, 2020.

25_Toy-Pediatrics_Case24_p229-236.indd 236

10/06/21 1:41 AM

CASE 25 A 3-year-old boy arrives to the emergency department (ED) after having suffered a seizure. The family reports that they had moved to Baltimore, Maryland, from the Midwest 3 months ago. The child was the product of a normal pregnancy and delivery, and he had experienced no medical problems until the move. The parents report that he has developed emotional lability, abdominal pain, “achy bones,” and intermittent vomiting and constipation. They initially attributed his behavior to the move and to the chaos in their house, which is being extensively renovated. ▶▶ ▶▶ ▶▶

What is the most likely diagnosis? What is the best test to diagnose this condition? What is the best therapy?

26_Toy-Pediatrics_Case25_p237-246.indd 237

07/07/21 12:11 PM

238

CASE FILES: PEDIATRICS

ANSWERS TO CASE 25: Lead Ingestion (Microcytic Anemia) Summary: A 3-year-old boy presents with šš

Seizures, neurologic changes, and abdominal complaints

šš

History of normal pregnancy and delivery

šš

No medical complaints until moving to a home undergoing extensive renovation

Most likely diagnosis: Lead toxicity. Best diagnostic test: Blood lead level (BLL). Best therapy: Remove child from lead source and initiate chelation therapy.

ANALYSIS Objectives 1. Understand the signs, symptoms, and treatment of lead poisoning. (EPA 1, 4) 2. Be familiar with the environmental sources of lead. (EPA 2, 12) 3. Understand the sources of other environmental exposures (heavy metals) that can cause anemia. (EPA 2, 12)

Considerations This child is demonstrating signs and symptoms of lead poisoning. In addition, the patient is in the age range (below 5 years) in which normal toddler activities contribute to an overall greater risk of lead exposure and toxicity. He may have been exposed to lead-containing dust in the renovation environment, or he may have displayed pica (the eating of nonfood substances such as paint chips, dirt, or clay). Therapy can be initiated immediately while awaiting the BLL. During the evaluation and treatment, other children in the home must be screened for elevated lead levels given their shared exposure risk. Lead exposure sources vary across the United States. In the northeastern United States, older homes undergoing renovation are a common source of exposure. Leaded paint is far less common in other parts of the country. A complete investigation includes a travel history and an accounting of lead exposures through activities such as stained glass work, home renovation, welding, radiator repair, furniture refinishing, or pottery glazing.

26_Toy-Pediatrics_Case25_p237-246.indd 238

07/07/21 12:11 PM

SECTION II: CLINICAL CASES

239

APPROACH TO: Lead Ingestion DEFINITIONS CHELATING AGENT: A soluble compound that binds a metal ion (in this case lead) so that the new complex is excreted in the urine. PLUMBISM: Alternate name for lead poisoning.

CLINICAL APPROACH Epidemiology The incidence of lead poisoning in the United States has decreased dramatically over the past few decades because of regulatory interventions by the federal, state, and local governments. Previous sources (gasoline, foods, beverage cans) have been eliminated; lead-containing paint in older homes is now the major source. One study showed that up to 15% of children living in homes built prior to 1950 had BLLs equal to or exceeding 5 mcg/dL, compared to only 2.1% of children living in homes built after 1978. Less common sources include foodstuffs from countries where regulations are not strict, traditional ethnic remedies, soil, water (especially important in formula-fed infants), glazed pottery, ingestion of leaded items (jewelry, fishing equipment), exposure through burning of lead-containing batteries, or exposure through hobbies involving lead smelting. Several lines of toys were recalled by the US Consumer Product Safety Commission in 2010 when they were found to be coated with lead-based paint. Although the prevalence of elevated BLLs has declined by 84% in the past 20 years, the local prevalence can differ by 10-fold between communities. Children younger than 5 years are at a greater risk for lead toxicity and its sequelae because of increased gastrointestinal absorption, more frequent hand-tomouth activity, and a susceptible, developing central nervous system.

Pathophysiology The exact mechanism whereby lead causes toxicity is unclear, although the leading theory is that lead increases generation of reactive oxygen species and interferes with calcium and zinc in some of the key enzyme functions of the body. Lead especially has a strong affinity for red blood cells.

Clinical Presentation The signs and symptoms of lead exposure vary from none (especially at lower lead levels) to those listed in this case. However, symptoms may be seen at low BLLs, and a child with very high BLLs occasionally may be asymptomatic. Anorexia, hyperirritability, altered sleep pattern, and decreased play are commonly seen. Developmental regression, especially with speech, may also be present. Abdominal complaints (occasional vomiting, intermittent pain, and constipation) are sometimes noted. Persistent vomiting, ataxia, altered consciousness, coma, and seizures are signs of encephalopathy. Permanent, long-term consequences include learning and

26_Toy-Pediatrics_Case25_p237-246.indd 239

07/07/21 12:11 PM

240

CASE FILES: PEDIATRICS

cognitive deficits and aggressive behavior; with less lead in the environment and decreasing average lead levels, these subtler findings are now more common than acute lead encephalopathy. The BLL is the diagnostic test of choice and demonstrates recent ingestion; however, a significant amount of lead is stored in other tissue, most notably bone. BLL, then, does not accurately reflect total body lead load. Other findings (free erythrocyte protoporphyrin, basophilic stippling, glycosuria, hypophosphatemia, long bone “lead lines,” and gastrointestinal tract radiopaque flecks) in symptomatic patients are less specific.

Treatment Treatment varies depending on the BLL and the patient’s symptoms. Admission to the hospital, stabilization, and chelation are appropriate for symptomatic patients. Therapy for asymptomatic patients could involve investigation of the child’s environment, outpatient chelation, or immediate hospitalization (Table 25–1). Close contact with local health agencies is important; they usually are charged with ensuring that the child’s environment is lead free. Chelation.  Chelation in an asymptomatic child may consist of intramuscular calcium disodium ethylenediaminetetraacetic acid (CaEDTA) or, more commonly, oral meso-2,3-dimercaptosuccinic acid (DMSA, succimer). Hospitalized symptomatic patients are often treated with 2,3-dimercaptopropanol (British anti-Lewisite [BAL]) and CaEDTA. Fluid balance is challenging; urine output must be maintained because CaEDTA is renally excreted, but encephalopathy may be exacerbated with overhydration. Newer research has cast doubt on the utility of chelation therapy in asymptomatic children with lead levels less than 45 mcg/dL. Lead levels do decrease acutely with chelation therapy, but affected children do not show improvement in longterm cognitive testing. The most recent literature suggests that no “safe” lead level exists; even lead levels less than 5 mcg/dL have been shown to have a deleterious impact on neurocognitive development. This evidence places further importance upon the primary prevention of lead exposure in children. Currently, the Centers for Disease Control and Prevention (CDC) identifies children with lead levels above 5 mcg/dL as having an elevated lead level and requiring further investigation. Screening.  No direct evidence shows that targeted BLL screening actually improves clinical outcomes in patients who are asymptomatic. Universal BLL screening is not recommended, and BLL screening should be reserved for children identified to be at higher risk for lead toxicity based on location, history, and questionnaires. In 2005, the American Academy of Pediatrics and CDC recommended screening based on each community’s prevalence of housing built before 1960. Furthermore, screening beyond 36 months of age is not recommended unless high-risk factors are identified. Questionnaires to assess the risk of lead exposure query the age of the home or day care center, the possibility of exposure to high-lead environments (eg, battery recycling plant, lead smelter), or environments in which others (eg, siblings, playmates) with elevated BLLs have been identified.

26_Toy-Pediatrics_Case25_p237-246.indd 240

07/07/21 12:11 PM

26_Toy-Pediatrics_Case25_p237-246.indd 241

Table 25–1  •  SUMMARY OF RECOMMENDATIONS FOR CHILDREN WITH CONFIRMED (VENOUS) ELEVATED BLOOD LEAD LEVELS Blood Lead Level (mcg/dL) 10-14

15-19

20-44

45-69

70 and over

Lead education •  Dietary •  Environmental Follow-up blood lead monitoring

Lead education •  Dietary •  Environmental Follow-up blood lead monitoring Proceed according to actions for 20-44 mcg/dL if: •  A follow-up blood lead level is in this range at least 3 mo after initial venous test Or •  Blood lead levels increase

Lead education •  Dietary •  Environmental Follow-up blood lead monitoring Complete history and physical examination Laboratory work •  Hemoglobin or hematocrit •  Iron status Environmental investigation Lead hazard reduction Neurodevelopmental monitoring Abdominal x-ray (if particulate lead ingestion is suspected) with bowel decontamination if indicated

Lead education •  Dietary •  Environmental Follow-up blood lead monitoring Complete history and physical examination Complete neurological examination Laboratory work •  Hemoglobin or hematocrit •  Iron status •  Free erythrocyte protoporphyrin (FPP) or zinc protoporphyrin (ZPP) Environmental investigation Lead hazard reduction Neurodevelopmental monitoring Abdominal x-ray with bowel decontamination if indicated Chelation therapy

Hospitalize and commence chelation therapy Proceed according to actions for 45-69 mcg/dL

241

07/07/21 12:11 PM

The following actions are NOT recommended at any blood lead level: •  Searching for gingival lead lines •  Testing of neurophysiologic function •  Evaluation of renal function (except during chelation with EDTA) •  Testing of hair, teeth, or fingernails for lead •  Radiographic imaging of long bones •  X-ray fluorescence of long bones Abbreviation: EDTA, ethylenediaminetetraacetic acid. Reproduced from the Centers for Disease Control and Prevention, http://www.cdc.gov.

242

CASE FILES: PEDIATRICS

CASE CORRELATION šš

See also Case 10 (Failure to Thrive).

COMPREHENSION QUESTIONS 25.1 A developmentally normal 2-year-old child is in your inner city clinic for a well-child checkup. As part of the visit, you obtain a blood lead level (BLL) and a hemoglobin level in accordance with your state’s Medicaid screening guidelines. The following week, the state laboratory calls your clinic to report that the child’s BLL is 14 mcg/dL. Appropriate management of this level should include which of the following actions? A. Initiate chelation therapy B. Perform long bone radiographs C. Reassure the parents that no action is required D. Repeat the BLL in 1 to 3 months E. Report to the local health department for environmental investigation 25.2 While evaluating the family in Question 25.1, you discovered a 3-year-old sibling with a BLL of 50 mcg/dL. You reported the case to the local authorities and initiated chelation therapy. All lead sources in the home have since been removed (verified by dust wipe samples), and the parents do not work in occupations prone to lead exposure. After a course of outpatient chelation therapy, the 3-year-old’s BLL dropped to 5 mcg/dL. Today, however, the child’s 3-month follow-up BLL is 15 mcg/dL. At this point, appropriate management includes which of the following actions? A. Initiate a course of inpatient parenteral chelation therapy B. Perform long bone radiographs C. Reassure the parents and repeat a BLL in 1 to 3 months D. Recommend the family to move to another home E. Repeat a course of outpatient chelation therapy

26_Toy-Pediatrics_Case25_p237-246.indd 242

07/07/21 12:11 PM

SECTION II: CLINICAL CASES

243

25.3 A term newborn infant is admitted to the neonatal intensive care unit after having a seizure. Your examination reveals a microcephalic infant with low birth weight who does not respond to sound. In your discussions with the family, you discover this is the parents’ first child. They recount odd symptoms that have developed in both of them in the past few months, including fine tremors in their upper extremities and blurry vision. They also note that they both can no longer smell their food and that it “tastes funny.” The mother notes that she has had trouble walking straight in the past few weeks, but she attributes that to her pregnancy. Which of the following environmental toxins is most likely to have caused these findings? A. Inorganic arsenic salts B. Lead C. Methyl mercury D. Orellanine E. Polychlorinated biphenyls 25.4 A mother brings her 2-year-old son to your clinic for a well-child checkup. He was lost to follow-up in early infancy but has come to clinic today to reestablish care. She reports that he has been overall well except that he seems to have less energy lately. He is a very picky eater and mostly drinks whole milk all day. The mother wants your advice about keeping her son from his new favorite hobby, eating dirt. What is the most appropriate next step? A. HEADSS (home, education, activities/employment, drugs, suicidality, and sex) examination B. Hemoglobin level and lead exposure questionnaire C. Hemoglobin level and BLL D. Cholesterol screening E. Reassurance

ANSWERS 25.1 D. Repeat the BLL in 1 to 3 months. The patient’s lead screen is mildly elevated. Appropriate management includes educating the parents about potential lead exposures in the environment as well as in the diet. A repeat level should be performed in 1 to 3 months. Chelation therapy (answer A) is currently advised for patients with a BLL of 45 mcg/dL and above. Environmental investigation (answer E) is recommended in patients with a BLL of 20 mcg/dL and above or if levels remain elevated despite educational efforts. Long bone radiographs (answer B) are not recommended at any BLL. Reassurance alone (answer C) does not provide the proper assessment and possible treatment.

26_Toy-Pediatrics_Case25_p237-246.indd 243

07/07/21 12:11 PM

244

CASE FILES: PEDIATRICS

25.2 C. Reassure the parents and repeat a BLL in 1 to 3 months. In this case, reassurance is appropriate. Lead is deposited in bone, and chelation does not remove all lead from the body. After chelation is complete, lead levels tend to rise again; the source is thought to be the redistribution of lead stored in bone. Immediate repeat chelation (answers A and E) is only recommended if the BLL rebounds to 45 mcg/dL or higher. Moving to another home (answer D) is not necessary, assuming the health department successfully remediated their current home. Long bone radiographs (answer B) are not recommended at any BLL. 25.3 C. Methyl mercury. Infants exposed in utero to methyl mercury may display low birth weight, microcephaly, and seizures. They also display significant developmental delay and can have vision and hearing impairments. Symptoms in children and adults include ataxia, tremor, dysarthria, memory loss, altered sensorium (including vision, hearing, smell, and taste), dementia, and ultimately death. Acute ingestion of arsenic (answer  A) causes severe gastrointestinal symptoms; chronic exposure causes skin lesions and can cause peripheral neuropathy and encephalopathy. Maternal lead poisoning (answer B) can lead to low birth weight and neurodevelopmental problems, but not to the degree of mercury toxicity. Orellanine (answer D) is a toxin found in the Cortinarius species of mushroom that causes nausea, vomiting, and diarrhea; renal toxicity may occur several days later. Polychlorinated biphenyls (PCBs) (answer  E) cross the placenta and accumulate in breast milk; exposure in utero is thought to cause behavioral problems in later life. 25.4 B. Hemoglobin level and lead exposure questionnaire. This patient is at high risk for iron deficiency anemia given his insufficient dietary intake and symptoms of pica and lethargy. Screening with a blood hemoglobin level is indicated. BLL screening (answer C) should be performed in patients with identifiable risk factors such as living in pre-1950 homes, parents with occupational lead exposure, or recent immigration. This history should be elucidated from the family either by interview or questionnaire, with subsequent BLL if indicated. A HEADSS examination (answer A) is more appropriate for a preteen or teen patient. Cholesterol testing (answer D) is indicated for a family history of obesity or known inherited cholesterol abnormalities.

26_Toy-Pediatrics_Case25_p237-246.indd 244

07/07/21 12:11 PM

SECTION II: CLINICAL CASES

245

CLINICAL PEARLS »»

Lead-containing paint in older homes is the major source of lead exposure in the United States.

»»

Behavioral signs of lead toxicity include hyperirritability, altered sleep patterns, decreased play activity, loss of developmental milestones (especially speech), and altered state of consciousness.

»»

Physical symptoms of lead toxicity include vomiting, intermittent abdominal pain, constipation, ataxia, coma, and seizures.

»»

Chelation therapy in an asymptomatic child with elevated lead levels consists of intramuscular calcium disodium ethylenediaminetetraacetic acid (CaEDTA) or oral meso-2,3-dimercaptosuccinic acid (succimer). Hospitalized patients with symptomatic disease are often treated with 2,3-dimercaptopropanol (BAL) and CaEDTA.

»»

Blood lead levels (BLL) is the diagnostic test of choice in evaluating lead poisoning.

»»

BLL’s can sometimes rebound after chelation therapy and is not necessarily indicative of further ingestion.

»»

“Lead lines” can sometimes be seen on the x-rays of long bones of affecting patients, but are not recommended as a diagnostic test.

REFERENCES Advisory Committee on Childhood Lead Poisoning Prevention (ACCLPP). Recommendations for blood lead screening of young children enrolled in Medicaid: targeting a group at high risk. MMWR Recomm Rep. 2000;49(RR-14):1-13. American Academy of Pediatrics. Lead exposure in children: prevention, detection, and management. Pediatrics. 2005;116:1036-1046. American Academy of Pediatrics. Prevention of childhood lead toxicity. Council on Environmental Health. Pediatrics. 2016;138:e20161493. American Academy of Pediatrics. Screening for elevated lead levels in childhood and pregnancy: an updated summary of evidence for the US Preventative Services Task Force. Pediatrics. 2006;118:e1867-e1895. American Academy of Pediatrics. Trends in blood lead levels and blood lead testing among US children aged 1 to 5 years, 1988-2004. Pediatrics. 2009;123:e376-e385. Centers for Disease Control and Prevention (CDC) Advisory Committee on Childhood Lead Poisoning Prevention. Interpreting and managing blood lead levels